Fix: enlèvement de la correction dans les sujets
continuous-integration/drone/push Build is passing Details

This commit is contained in:
Bertrand Benjamin 2021-09-27 10:26:51 +02:00
parent e7559f8cbb
commit 656613e364
67 changed files with 1718 additions and 1848 deletions

View File

@ -17,17 +17,17 @@
Détailler les calculs suivants et donner le résultat sous la forme d'une fraction irréductible.
\begin{multicols}{3}
\begin{enumerate}[label={\Alph*=}]
\item $\dfrac{9}{5} + \dfrac{8}{5}$
\item $\dfrac{2}{9} + 3$
\item $\dfrac{2}{3} + \dfrac{2}{3}$
\item $\dfrac{5}{8} + 9$
\item $\dfrac{4}{5} + \dfrac{10}{25}$
\item $\dfrac{6}{3} + \dfrac{10}{7}$
\item $\dfrac{3}{2} + \dfrac{9}{2}$
\item $\dfrac{5}{7} + \dfrac{4}{9}$
\item $\dfrac{3}{8} \times 2$
\item $\dfrac{1}{2} \times \dfrac{9}{2}$
\item $\dfrac{4}{10} \times 1$
\item $\dfrac{9}{4} \times \dfrac{8}{4}$
\item $\dfrac{- 2}{7} \times \dfrac{- 8}{70}$
\item $\dfrac{\dfrac{1}{4}}{\dfrac{9}{7}}$
\item $\dfrac{- 10}{6} \times \dfrac{- 5}{18}$
\item $\dfrac{\dfrac{9}{2}}{\dfrac{4}{7}}$
\end{enumerate}
\end{multicols}
\end{exercise}
@ -35,15 +35,15 @@
\begin{solution}
\begin{enumerate}[label={\Alph*=}]
\item $\dfrac{9}{5} + \dfrac{8}{5}=\dfrac{9 + 8}{5}=\dfrac{17}{5} = \dfrac{17}{5}$
\item $\dfrac{2}{9} + 3=\dfrac{2}{9} + \dfrac{3}{1}=\dfrac{2}{9} + \dfrac{3 \times 9}{1 \times 9}=\dfrac{2}{9} + \dfrac{27}{9}=\dfrac{2 + 27}{9}=\dfrac{29}{9} = \dfrac{29}{9}$
\item $\dfrac{4}{5} + \dfrac{10}{25}=\dfrac{4 \times 5}{5 \times 5} + \dfrac{10}{25}=\dfrac{20}{25} + \dfrac{10}{25}=\dfrac{20 + 10}{25}=\dfrac{30}{25} = \dfrac{6}{5}$
\item $\dfrac{6}{3} + \dfrac{10}{7}=\dfrac{6 \times 7}{3 \times 7} + \dfrac{10 \times 3}{7 \times 3}=\dfrac{42}{21} + \dfrac{30}{21}=\dfrac{42 + 30}{21}=\dfrac{72}{21} = \dfrac{24}{7}$
\item $\dfrac{2}{3} + \dfrac{2}{3}=\dfrac{2 + 2}{3}=\dfrac{4}{3} = \dfrac{4}{3}$
\item $\dfrac{5}{8} + 9=\dfrac{5}{8} + \dfrac{9}{1}=\dfrac{5}{8} + \dfrac{9 \times 8}{1 \times 8}=\dfrac{5}{8} + \dfrac{72}{8}=\dfrac{5 + 72}{8}=\dfrac{77}{8} = \dfrac{77}{8}$
\item $\dfrac{3}{2} + \dfrac{9}{2}=\dfrac{3 + 9}{2}=\dfrac{12}{2} = 6$
\item $\dfrac{5}{7} + \dfrac{4}{9}=\dfrac{5 \times 9}{7 \times 9} + \dfrac{4 \times 7}{9 \times 7}=\dfrac{45}{63} + \dfrac{28}{63}=\dfrac{45 + 28}{63}=\dfrac{73}{63} = \dfrac{73}{63}$
\item $\dfrac{3}{8} \times 2=\dfrac{3 \times 2}{8}=\dfrac{6}{8} = \dfrac{3}{4}$
\item $\dfrac{1}{2} \times \dfrac{9}{2}=\dfrac{1 \times 9}{2 \times 2}=\dfrac{9}{4} = \dfrac{9}{4}$
\item $\dfrac{- 2}{7} \times \dfrac{- 8}{70}=\dfrac{- 2(- 8)}{7 \times 70}=\dfrac{16}{490} = \dfrac{8}{245}$
\item $\dfrac{\dfrac{1}{4}}{\dfrac{9}{7}}=\dfrac{1}{4} \times \dfrac{7}{9}=\dfrac{1 \times 7}{4 \times 9}=\dfrac{7}{36} = \dfrac{7}{36}$
\item $\dfrac{4}{10} \times 1=\dfrac{4}{10} = \dfrac{2}{5}$
\item $\dfrac{9}{4} \times \dfrac{8}{4}=\dfrac{9 \times 8}{4 \times 4}=\dfrac{72}{16} = \dfrac{9}{2}$
\item $\dfrac{- 10}{6} \times \dfrac{- 5}{18}=\dfrac{- 10(- 5)}{6 \times 18}=\dfrac{50}{108} = \dfrac{25}{54}$
\item $\dfrac{\dfrac{9}{2}}{\dfrac{4}{7}}=\dfrac{9}{2} \times \dfrac{7}{4}=\dfrac{9 \times 7}{2 \times 4}=\dfrac{63}{8} = \dfrac{63}{8}$
\end{enumerate}
\end{solution}
@ -52,10 +52,10 @@
Le radar a pris des photos pendant l'été:
\begin{itemize}
\item en juin, il y a eu 54 photos prises dont 25 ratées.
\item en juillet, il y a eu 39 photos réussies et 37 ratées.
\item en août, il y a eu 58 photos dont une proportion de 0.26 de photos ratées.
\item en septembre, il y a eu 9 photos ratées, ce qui correspondait à 15.52\% des photos prises.
\item en juin, il y a eu 43 photos prises dont 20 ratées.
\item en juillet, il y a eu 32 photos réussies et 34 ratées.
\item en août, il y a eu 60 photos dont une proportion de 0.17 de photos ratées.
\item en septembre, il y a eu 10 photos ratées, ce qui correspondait à 20.0\% des photos prises.
\end{itemize}
\begin{enumerate}
@ -87,37 +87,35 @@
\hline
& Juin & Juillet & Août & Septembre & Total\\
\hline
Réussies & 29 & 39 & 43 & 49 & 160\\
Réussies & 23 & 32 & 50 & 40 & 145\\
\hline
Ratées & 25 & 37 & 15 & 9 & 86\\
Ratées & 20 & 34 & 10 & 10 & 74\\
\hline
Total & 54 & 76 & 58 & 58 & 246\\
Total & 43 & 66 & 60 & 50 & 219\\
\hline
\end{tabular}
\end{center}
\item Proportion de photos réussies
\[
\frac{160}{246} = 0.65 = 65\%
\frac{145}{219} = 0.66 = 66\%
\]
\item
\begin{itemize}
\item De juin à juillet
\[
\frac{37 - 25}{25} = \frac{12}{25} = 0.48 = 48\%
\frac{34 - 20}{20} = \frac{14}{20} = 0.7 = 70\%
\]
\item De juillet à août
\[
\frac{15 - 37}{37} = \frac{-22}{37} = -0.59 = -59\%
\frac{10 - 34}{34} = \frac{-24}{34} = -0.71 = -70\%
\]
\item De août à septembre
\[
\frac{9 - 15}{15} = \frac{-6}{15} = -0.4 = -40\%
\frac{10 - 10}{10} = \frac{0}{10} = 0.0 = 0\%
\]
\end{itemize}
\end{enumerate}
\end{solution}
\printsolutionstype{exercise}
\end{document}

View File

@ -17,17 +17,17 @@
Détailler les calculs suivants et donner le résultat sous la forme d'une fraction irréductible.
\begin{multicols}{3}
\begin{enumerate}[label={\Alph*=}]
\item $\dfrac{2}{9} + \dfrac{2}{9}$
\item $\dfrac{9}{6} + 9$
\item $\dfrac{10}{2} + \dfrac{4}{2}$
\item $\dfrac{7}{2} + 7$
\item $\dfrac{9}{4} + \dfrac{8}{12}$
\item $\dfrac{1}{7} + \dfrac{6}{8}$
\item $\dfrac{9}{10} + \dfrac{7}{10}$
\item $\dfrac{5}{10} + \dfrac{7}{10}$
\item $\dfrac{6}{10} \times 4$
\item $\dfrac{3}{4} \times \dfrac{2}{4}$
\item $\dfrac{4}{9} \times 7$
\item $\dfrac{4}{7} \times \dfrac{6}{7}$
\item $\dfrac{- 6}{10} \times \dfrac{6}{30}$
\item $\dfrac{\dfrac{7}{3}}{\dfrac{4}{3}}$
\item $\dfrac{- 2}{7} \times \dfrac{2}{42}$
\item $\dfrac{\dfrac{9}{10}}{\dfrac{6}{4}}$
\end{enumerate}
\end{multicols}
\end{exercise}
@ -35,15 +35,15 @@
\begin{solution}
\begin{enumerate}[label={\Alph*=}]
\item $\dfrac{2}{9} + \dfrac{2}{9}=\dfrac{2 + 2}{9}=\dfrac{4}{9} = \dfrac{4}{9}$
\item $\dfrac{9}{6} + 9=\dfrac{9}{6} + \dfrac{9}{1}=\dfrac{9}{6} + \dfrac{9 \times 6}{1 \times 6}=\dfrac{9}{6} + \dfrac{54}{6}=\dfrac{9 + 54}{6}=\dfrac{63}{6} = \dfrac{21}{2}$
\item $\dfrac{9}{4} + \dfrac{8}{12}=\dfrac{9 \times 3}{4 \times 3} + \dfrac{8}{12}=\dfrac{27}{12} + \dfrac{8}{12}=\dfrac{27 + 8}{12}=\dfrac{35}{12} = \dfrac{35}{12}$
\item $\dfrac{1}{7} + \dfrac{6}{8}=\dfrac{1 \times 8}{7 \times 8} + \dfrac{6 \times 7}{8 \times 7}=\dfrac{8}{56} + \dfrac{42}{56}=\dfrac{8 + 42}{56}=\dfrac{50}{56} = \dfrac{25}{28}$
\item $\dfrac{10}{2} + \dfrac{4}{2}=\dfrac{10 + 4}{2}=\dfrac{14}{2} = 7$
\item $\dfrac{7}{2} + 7=\dfrac{7}{2} + \dfrac{7}{1}=\dfrac{7}{2} + \dfrac{7 \times 2}{1 \times 2}=\dfrac{7}{2} + \dfrac{14}{2}=\dfrac{7 + 14}{2}=\dfrac{21}{2} = \dfrac{21}{2}$
\item $\dfrac{9}{10} + \dfrac{7}{10}=\dfrac{9 + 7}{10}=\dfrac{16}{10} = \dfrac{8}{5}$
\item $\dfrac{5}{10} + \dfrac{7}{10}=\dfrac{5 + 7}{10}=\dfrac{12}{10} = \dfrac{6}{5}$
\item $\dfrac{6}{10} \times 4=\dfrac{6 \times 4}{10}=\dfrac{24}{10} = \dfrac{12}{5}$
\item $\dfrac{3}{4} \times \dfrac{2}{4}=\dfrac{3 \times 2}{4 \times 4}=\dfrac{6}{16} = \dfrac{3}{8}$
\item $\dfrac{- 6}{10} \times \dfrac{6}{30}=\dfrac{- 6 \times 6}{10 \times 30}=\dfrac{- 36}{300} = \dfrac{- 3}{25}$
\item $\dfrac{\dfrac{7}{3}}{\dfrac{4}{3}}=\dfrac{7}{3} \times \dfrac{3}{4}=\dfrac{7 \times 3}{3 \times 4}=\dfrac{21}{12} = \dfrac{7}{4}$
\item $\dfrac{4}{9} \times 7=\dfrac{4 \times 7}{9}=\dfrac{28}{9} = \dfrac{28}{9}$
\item $\dfrac{4}{7} \times \dfrac{6}{7}=\dfrac{4 \times 6}{7 \times 7}=\dfrac{24}{49} = \dfrac{24}{49}$
\item $\dfrac{- 2}{7} \times \dfrac{2}{42}=\dfrac{- 2 \times 2}{7 \times 42}=\dfrac{- 4}{294} = \dfrac{- 2}{147}$
\item $\dfrac{\dfrac{9}{10}}{\dfrac{6}{4}}=\dfrac{9}{10} \times \dfrac{4}{6}=\dfrac{9 \times 4}{10 \times 6}=\dfrac{36}{60} = \dfrac{3}{5}$
\end{enumerate}
\end{solution}
@ -52,10 +52,10 @@
Le radar a pris des photos pendant l'été:
\begin{itemize}
\item en juin, il y a eu 52 photos prises dont 25 ratées.
\item en juillet, il y a eu 49 photos réussies et 36 ratées.
\item en août, il y a eu 60 photos dont une proportion de 0.22 de photos ratées.
\item en septembre, il y a eu 5 photos ratées, ce qui correspondait à 9.62\% des photos prises.
\item en juin, il y a eu 52 photos prises dont 26 ratées.
\item en juillet, il y a eu 30 photos réussies et 40 ratées.
\item en août, il y a eu 61 photos dont une proportion de 0.21 de photos ratées.
\item en septembre, il y a eu 9 photos ratées, ce qui correspondait à 17.65\% des photos prises.
\end{itemize}
\begin{enumerate}
@ -87,37 +87,35 @@
\hline
& Juin & Juillet & Août & Septembre & Total\\
\hline
Réussies & 27 & 49 & 47 & 47 & 170\\
Réussies & 26 & 30 & 48 & 42 & 146\\
\hline
Ratées & 25 & 36 & 13 & 5 & 79\\
Ratées & 26 & 40 & 13 & 9 & 88\\
\hline
Total & 52 & 85 & 60 & 52 & 249\\
Total & 52 & 70 & 61 & 51 & 234\\
\hline
\end{tabular}
\end{center}
\item Proportion de photos réussies
\[
\frac{170}{249} = 0.68 = 68\%
\frac{146}{234} = 0.62 = 62\%
\]
\item
\begin{itemize}
\item De juin à juillet
\[
\frac{36 - 25}{25} = \frac{11}{25} = 0.44 = 44\%
\frac{40 - 26}{26} = \frac{14}{26} = 0.54 = 53\%
\]
\item De juillet à août
\[
\frac{13 - 36}{36} = \frac{-23}{36} = -0.64 = -63\%
\frac{13 - 40}{40} = \frac{-27}{40} = -0.68 = -67\%
\]
\item De août à septembre
\[
\frac{5 - 13}{13} = \frac{-8}{13} = -0.62 = -61\%
\frac{9 - 13}{13} = \frac{-4}{13} = -0.31 = -30\%
\]
\end{itemize}
\end{enumerate}
\end{solution}
\printsolutionstype{exercise}
\end{document}

View File

@ -17,17 +17,17 @@
Détailler les calculs suivants et donner le résultat sous la forme d'une fraction irréductible.
\begin{multicols}{3}
\begin{enumerate}[label={\Alph*=}]
\item $\dfrac{1}{10} + \dfrac{2}{10}$
\item $\dfrac{4}{6} + 5$
\item $\dfrac{7}{5} + \dfrac{6}{5}$
\item $\dfrac{7}{6} + 4$
\item $\dfrac{10}{3} + \dfrac{5}{27}$
\item $\dfrac{1}{10} + \dfrac{2}{9}$
\item $\dfrac{9}{4} + \dfrac{6}{36}$
\item $\dfrac{2}{8} + \dfrac{7}{8}$
\item $\dfrac{5}{9} \times 9$
\item $\dfrac{8}{7} \times \dfrac{9}{7}$
\item $\dfrac{1}{4} \times 8$
\item $\dfrac{3}{7} \times \dfrac{10}{7}$
\item $\dfrac{- 6}{7} \times \dfrac{10}{28}$
\item $\dfrac{\dfrac{1}{7}}{\dfrac{1}{9}}$
\item $\dfrac{- 8}{3} \times \dfrac{- 3}{12}$
\item $\dfrac{\dfrac{1}{5}}{\dfrac{4}{5}}$
\end{enumerate}
\end{multicols}
\end{exercise}
@ -35,15 +35,15 @@
\begin{solution}
\begin{enumerate}[label={\Alph*=}]
\item $\dfrac{1}{10} + \dfrac{2}{10}=\dfrac{1 + 2}{10}=\dfrac{3}{10} = \dfrac{3}{10}$
\item $\dfrac{4}{6} + 5=\dfrac{4}{6} + \dfrac{5}{1}=\dfrac{4}{6} + \dfrac{5 \times 6}{1 \times 6}=\dfrac{4}{6} + \dfrac{30}{6}=\dfrac{4 + 30}{6}=\dfrac{34}{6} = \dfrac{17}{3}$
\item $\dfrac{10}{3} + \dfrac{5}{27}=\dfrac{10 \times 9}{3 \times 9} + \dfrac{5}{27}=\dfrac{90}{27} + \dfrac{5}{27}=\dfrac{90 + 5}{27}=\dfrac{95}{27} = \dfrac{95}{27}$
\item $\dfrac{1}{10} + \dfrac{2}{9}=\dfrac{1 \times 9}{10 \times 9} + \dfrac{2 \times 10}{9 \times 10}=\dfrac{9}{90} + \dfrac{20}{90}=\dfrac{9 + 20}{90}=\dfrac{29}{90} = \dfrac{29}{90}$
\item $\dfrac{7}{5} + \dfrac{6}{5}=\dfrac{7 + 6}{5}=\dfrac{13}{5} = \dfrac{13}{5}$
\item $\dfrac{7}{6} + 4=\dfrac{7}{6} + \dfrac{4}{1}=\dfrac{7}{6} + \dfrac{4 \times 6}{1 \times 6}=\dfrac{7}{6} + \dfrac{24}{6}=\dfrac{7 + 24}{6}=\dfrac{31}{6} = \dfrac{31}{6}$
\item $\dfrac{9}{4} + \dfrac{6}{36}=\dfrac{9 \times 9}{4 \times 9} + \dfrac{6}{36}=\dfrac{81}{36} + \dfrac{6}{36}=\dfrac{81 + 6}{36}=\dfrac{87}{36} = \dfrac{29}{12}$
\item $\dfrac{2}{8} + \dfrac{7}{8}=\dfrac{2 + 7}{8}=\dfrac{9}{8} = \dfrac{9}{8}$
\item $\dfrac{5}{9} \times 9=\dfrac{5 \times 9}{9}=\dfrac{45}{9} = 5$
\item $\dfrac{8}{7} \times \dfrac{9}{7}=\dfrac{8 \times 9}{7 \times 7}=\dfrac{72}{49} = \dfrac{72}{49}$
\item $\dfrac{- 6}{7} \times \dfrac{10}{28}=\dfrac{- 6 \times 10}{7 \times 28}=\dfrac{- 60}{196} = \dfrac{- 15}{49}$
\item $\dfrac{\dfrac{1}{7}}{\dfrac{1}{9}}=\dfrac{1}{7} \times \dfrac{9}{1}=\dfrac{1 \times 9}{7 \times 1}=\dfrac{9}{7} = \dfrac{9}{7}$
\item $\dfrac{1}{4} \times 8=\dfrac{1 \times 8}{4}=\dfrac{8}{4} = 2$
\item $\dfrac{3}{7} \times \dfrac{10}{7}=\dfrac{3 \times 10}{7 \times 7}=\dfrac{30}{49} = \dfrac{30}{49}$
\item $\dfrac{- 8}{3} \times \dfrac{- 3}{12}=\dfrac{- 8(- 3)}{3 \times 12}=\dfrac{24}{36} = \dfrac{2}{3}$
\item $\dfrac{\dfrac{1}{5}}{\dfrac{4}{5}}=\dfrac{1}{5} \times \dfrac{5}{4}=\dfrac{1 \times 5}{5 \times 4}=\dfrac{5}{20} = \dfrac{1}{4}$
\end{enumerate}
\end{solution}
@ -52,10 +52,10 @@
Le radar a pris des photos pendant l'été:
\begin{itemize}
\item en juin, il y a eu 55 photos prises dont 27 ratées.
\item en juillet, il y a eu 35 photos réussies et 45 ratées.
\item en août, il y a eu 66 photos dont une proportion de 0.29 de photos ratées.
\item en septembre, il y a eu 10 photos ratées, ce qui correspondait à 19.61\% des photos prises.
\item en juin, il y a eu 47 photos prises dont 25 ratées.
\item en juillet, il y a eu 45 photos réussies et 31 ratées.
\item en août, il y a eu 65 photos dont une proportion de 0.31 de photos ratées.
\item en septembre, il y a eu 11 photos ratées, ce qui correspondait à 18.97\% des photos prises.
\end{itemize}
\begin{enumerate}
@ -87,37 +87,35 @@
\hline
& Juin & Juillet & Août & Septembre & Total\\
\hline
Réussies & 28 & 35 & 47 & 41 & 151\\
Réussies & 22 & 45 & 45 & 47 & 159\\
\hline
Ratées & 27 & 45 & 19 & 10 & 101\\
Ratées & 25 & 31 & 20 & 11 & 87\\
\hline
Total & 55 & 80 & 66 & 51 & 252\\
Total & 47 & 76 & 65 & 58 & 246\\
\hline
\end{tabular}
\end{center}
\item Proportion de photos réussies
\[
\frac{151}{252} = 0.6 = 59\%
\frac{159}{246} = 0.65 = 64\%
\]
\item
\begin{itemize}
\item De juin à juillet
\[
\frac{45 - 27}{27} = \frac{18}{27} = 0.67 = 66\%
\frac{31 - 25}{25} = \frac{6}{25} = 0.24 = 24\%
\]
\item De juillet à août
\[
\frac{19 - 45}{45} = \frac{-26}{45} = -0.58 = -57\%
\frac{20 - 31}{31} = \frac{-11}{31} = -0.35 = -35\%
\]
\item De août à septembre
\[
\frac{10 - 19}{19} = \frac{-9}{19} = -0.47 = -47\%
\frac{11 - 20}{20} = \frac{-9}{20} = -0.45 = -45\%
\]
\end{itemize}
\end{enumerate}
\end{solution}
\printsolutionstype{exercise}
\end{document}

View File

@ -17,17 +17,17 @@
Détailler les calculs suivants et donner le résultat sous la forme d'une fraction irréductible.
\begin{multicols}{3}
\begin{enumerate}[label={\Alph*=}]
\item $\dfrac{10}{4} + \dfrac{10}{4}$
\item $\dfrac{4}{5} + 7$
\item $\dfrac{6}{4} + \dfrac{8}{4}$
\item $\dfrac{10}{3} + 3$
\item $\dfrac{8}{6} + \dfrac{2}{30}$
\item $\dfrac{7}{2} + \dfrac{3}{2}$
\item $\dfrac{6}{4} + \dfrac{5}{40}$
\item $\dfrac{4}{8} + \dfrac{7}{9}$
\item $\dfrac{8}{2} \times 3$
\item $\dfrac{1}{9} \times \dfrac{5}{9}$
\item $\dfrac{3}{2} \times 3$
\item $\dfrac{10}{9} \times \dfrac{1}{9}$
\item $\dfrac{3}{9} \times \dfrac{1}{18}$
\item $\dfrac{\dfrac{6}{7}}{\dfrac{4}{3}}$
\item $\dfrac{3}{10} \times \dfrac{- 8}{100}$
\item $\dfrac{\dfrac{2}{6}}{\dfrac{10}{7}}$
\end{enumerate}
\end{multicols}
\end{exercise}
@ -35,15 +35,15 @@
\begin{solution}
\begin{enumerate}[label={\Alph*=}]
\item $\dfrac{10}{4} + \dfrac{10}{4}=\dfrac{10 + 10}{4}=\dfrac{20}{4} = 5$
\item $\dfrac{4}{5} + 7=\dfrac{4}{5} + \dfrac{7}{1}=\dfrac{4}{5} + \dfrac{7 \times 5}{1 \times 5}=\dfrac{4}{5} + \dfrac{35}{5}=\dfrac{4 + 35}{5}=\dfrac{39}{5} = \dfrac{39}{5}$
\item $\dfrac{8}{6} + \dfrac{2}{30}=\dfrac{8 \times 5}{6 \times 5} + \dfrac{2}{30}=\dfrac{40}{30} + \dfrac{2}{30}=\dfrac{40 + 2}{30}=\dfrac{42}{30} = \dfrac{7}{5}$
\item $\dfrac{7}{2} + \dfrac{3}{2}=\dfrac{7 + 3}{2}=\dfrac{10}{2} = 5$
\item $\dfrac{6}{4} + \dfrac{8}{4}=\dfrac{6 + 8}{4}=\dfrac{14}{4} = \dfrac{7}{2}$
\item $\dfrac{10}{3} + 3=\dfrac{10}{3} + \dfrac{3}{1}=\dfrac{10}{3} + \dfrac{3 \times 3}{1 \times 3}=\dfrac{10}{3} + \dfrac{9}{3}=\dfrac{10 + 9}{3}=\dfrac{19}{3} = \dfrac{19}{3}$
\item $\dfrac{6}{4} + \dfrac{5}{40}=\dfrac{6 \times 10}{4 \times 10} + \dfrac{5}{40}=\dfrac{60}{40} + \dfrac{5}{40}=\dfrac{60 + 5}{40}=\dfrac{65}{40} = \dfrac{13}{8}$
\item $\dfrac{4}{8} + \dfrac{7}{9}=\dfrac{4 \times 9}{8 \times 9} + \dfrac{7 \times 8}{9 \times 8}=\dfrac{36}{72} + \dfrac{56}{72}=\dfrac{36 + 56}{72}=\dfrac{92}{72} = \dfrac{23}{18}$
\item $\dfrac{8}{2} \times 3=\dfrac{8 \times 3}{2}=\dfrac{24}{2} = 12$
\item $\dfrac{1}{9} \times \dfrac{5}{9}=\dfrac{1 \times 5}{9 \times 9}=\dfrac{5}{81} = \dfrac{5}{81}$
\item $\dfrac{3}{9} \times \dfrac{1}{18}=\dfrac{3 \times 1}{9 \times 18}=\dfrac{3}{162} = \dfrac{1}{54}$
\item $\dfrac{\dfrac{6}{7}}{\dfrac{4}{3}}=\dfrac{6}{7} \times \dfrac{3}{4}=\dfrac{6 \times 3}{7 \times 4}=\dfrac{18}{28} = \dfrac{9}{14}$
\item $\dfrac{3}{2} \times 3=\dfrac{3 \times 3}{2}=\dfrac{9}{2} = \dfrac{9}{2}$
\item $\dfrac{10}{9} \times \dfrac{1}{9}=\dfrac{10 \times 1}{9 \times 9}=\dfrac{10}{81} = \dfrac{10}{81}$
\item $\dfrac{3}{10} \times \dfrac{- 8}{100}=\dfrac{3(- 8)}{10 \times 100}=\dfrac{- 24}{1000} = \dfrac{- 3}{125}$
\item $\dfrac{\dfrac{2}{6}}{\dfrac{10}{7}}=\dfrac{2}{6} \times \dfrac{7}{10}=\dfrac{2 \times 7}{6 \times 10}=\dfrac{14}{60} = \dfrac{7}{30}$
\end{enumerate}
\end{solution}
@ -52,10 +52,10 @@
Le radar a pris des photos pendant l'été:
\begin{itemize}
\item en juin, il y a eu 42 photos prises dont 20 ratées.
\item en juillet, il y a eu 30 photos réussies et 36 ratées.
\item en août, il y a eu 57 photos dont une proportion de 0.21 de photos ratées.
\item en septembre, il y a eu 7 photos ratées, ce qui correspondait à 14.58\% des photos prises.
\item en juin, il y a eu 50 photos prises dont 29 ratées.
\item en juillet, il y a eu 43 photos réussies et 39 ratées.
\item en août, il y a eu 52 photos dont une proportion de 0.23 de photos ratées.
\item en septembre, il y a eu 7 photos ratées, ce qui correspondait à 14.89\% des photos prises.
\end{itemize}
\begin{enumerate}
@ -87,27 +87,27 @@
\hline
& Juin & Juillet & Août & Septembre & Total\\
\hline
Réussies & 22 & 30 & 45 & 41 & 138\\
Réussies & 21 & 43 & 40 & 40 & 144\\
\hline
Ratées & 20 & 36 & 12 & 7 & 75\\
Ratées & 29 & 39 & 12 & 7 & 87\\
\hline
Total & 42 & 66 & 57 & 48 & 213\\
Total & 50 & 82 & 52 & 47 & 231\\
\hline
\end{tabular}
\end{center}
\item Proportion de photos réussies
\[
\frac{138}{213} = 0.65 = 64\%
\frac{144}{231} = 0.62 = 62\%
\]
\item
\begin{itemize}
\item De juin à juillet
\[
\frac{36 - 20}{20} = \frac{16}{20} = 0.8 = 80\%
\frac{39 - 29}{29} = \frac{10}{29} = 0.34 = 34\%
\]
\item De juillet à août
\[
\frac{12 - 36}{36} = \frac{-24}{36} = -0.67 = -66\%
\frac{12 - 39}{39} = \frac{-27}{39} = -0.69 = -69\%
\]
\item De août à septembre
\[
@ -118,6 +118,4 @@
\end{solution}
\printsolutionstype{exercise}
\end{document}

View File

@ -17,17 +17,17 @@
Détailler les calculs suivants et donner le résultat sous la forme d'une fraction irréductible.
\begin{multicols}{3}
\begin{enumerate}[label={\Alph*=}]
\item $\dfrac{9}{7} + \dfrac{4}{7}$
\item $\dfrac{1}{3} + 7$
\item $\dfrac{7}{10} + \dfrac{1}{10}$
\item $\dfrac{1}{5} + 2$
\item $\dfrac{9}{2} + \dfrac{4}{10}$
\item $\dfrac{1}{6} + \dfrac{9}{10}$
\item $\dfrac{9}{10} + \dfrac{2}{50}$
\item $\dfrac{2}{6} + \dfrac{9}{2}$
\item $\dfrac{1}{7} \times 10$
\item $\dfrac{10}{2} \times \dfrac{8}{2}$
\item $\dfrac{7}{10} \times 2$
\item $\dfrac{8}{3} \times \dfrac{5}{3}$
\item $\dfrac{- 10}{2} \times \dfrac{- 4}{18}$
\item $\dfrac{\dfrac{6}{3}}{\dfrac{4}{10}}$
\item $\dfrac{8}{9} \times \dfrac{7}{36}$
\item $\dfrac{\dfrac{1}{6}}{\dfrac{9}{4}}$
\end{enumerate}
\end{multicols}
\end{exercise}
@ -35,15 +35,15 @@
\begin{solution}
\begin{enumerate}[label={\Alph*=}]
\item $\dfrac{9}{7} + \dfrac{4}{7}=\dfrac{9 + 4}{7}=\dfrac{13}{7} = \dfrac{13}{7}$
\item $\dfrac{1}{3} + 7=\dfrac{1}{3} + \dfrac{7}{1}=\dfrac{1}{3} + \dfrac{7 \times 3}{1 \times 3}=\dfrac{1}{3} + \dfrac{21}{3}=\dfrac{1 + 21}{3}=\dfrac{22}{3} = \dfrac{22}{3}$
\item $\dfrac{9}{2} + \dfrac{4}{10}=\dfrac{9 \times 5}{2 \times 5} + \dfrac{4}{10}=\dfrac{45}{10} + \dfrac{4}{10}=\dfrac{45 + 4}{10}=\dfrac{49}{10} = \dfrac{49}{10}$
\item $\dfrac{1}{6} + \dfrac{9}{10}=\dfrac{1 \times 5}{6 \times 5} + \dfrac{9 \times 3}{10 \times 3}=\dfrac{5}{30} + \dfrac{27}{30}=\dfrac{5 + 27}{30}=\dfrac{32}{30} = \dfrac{16}{15}$
\item $\dfrac{7}{10} + \dfrac{1}{10}=\dfrac{7 + 1}{10}=\dfrac{8}{10} = \dfrac{4}{5}$
\item $\dfrac{1}{5} + 2=\dfrac{1}{5} + \dfrac{2}{1}=\dfrac{1}{5} + \dfrac{2 \times 5}{1 \times 5}=\dfrac{1}{5} + \dfrac{10}{5}=\dfrac{1 + 10}{5}=\dfrac{11}{5} = \dfrac{11}{5}$
\item $\dfrac{9}{10} + \dfrac{2}{50}=\dfrac{9 \times 5}{10 \times 5} + \dfrac{2}{50}=\dfrac{45}{50} + \dfrac{2}{50}=\dfrac{45 + 2}{50}=\dfrac{47}{50} = \dfrac{47}{50}$
\item $\dfrac{2}{6} + \dfrac{9}{2}=\dfrac{2}{6} + \dfrac{9 \times 3}{2 \times 3}=\dfrac{2}{6} + \dfrac{27}{6}=\dfrac{2 + 27}{6}=\dfrac{29}{6} = \dfrac{29}{6}$
\item $\dfrac{1}{7} \times 10=\dfrac{1 \times 10}{7}=\dfrac{10}{7} = \dfrac{10}{7}$
\item $\dfrac{10}{2} \times \dfrac{8}{2}=\dfrac{10 \times 8}{2 \times 2}=\dfrac{80}{4} = 20$
\item $\dfrac{- 10}{2} \times \dfrac{- 4}{18}=\dfrac{- 10(- 4)}{2 \times 18}=\dfrac{40}{36} = \dfrac{10}{9}$
\item $\dfrac{\dfrac{6}{3}}{\dfrac{4}{10}}=\dfrac{6}{3} \times \dfrac{10}{4}=\dfrac{6 \times 10}{3 \times 4}=\dfrac{60}{12} = 5$
\item $\dfrac{7}{10} \times 2=\dfrac{7 \times 2}{10}=\dfrac{14}{10} = \dfrac{7}{5}$
\item $\dfrac{8}{3} \times \dfrac{5}{3}=\dfrac{8 \times 5}{3 \times 3}=\dfrac{40}{9} = \dfrac{40}{9}$
\item $\dfrac{8}{9} \times \dfrac{7}{36}=\dfrac{8 \times 7}{9 \times 36}=\dfrac{56}{324} = \dfrac{14}{81}$
\item $\dfrac{\dfrac{1}{6}}{\dfrac{9}{4}}=\dfrac{1}{6} \times \dfrac{4}{9}=\dfrac{1 \times 4}{6 \times 9}=\dfrac{4}{54} = \dfrac{2}{27}$
\end{enumerate}
\end{solution}
@ -52,10 +52,10 @@
Le radar a pris des photos pendant l'été:
\begin{itemize}
\item en juin, il y a eu 42 photos prises dont 22 ratées.
\item en juillet, il y a eu 44 photos réussies et 32 ratées.
\item en août, il y a eu 54 photos dont une proportion de 0.26 de photos ratées.
\item en septembre, il y a eu 10 photos ratées, ce qui correspondait à 18.87\% des photos prises.
\item en juin, il y a eu 51 photos prises dont 22 ratées.
\item en juillet, il y a eu 48 photos réussies et 43 ratées.
\item en août, il y a eu 62 photos dont une proportion de 0.21 de photos ratées.
\item en septembre, il y a eu 14 photos ratées, ce qui correspondait à 24.56\% des photos prises.
\end{itemize}
\begin{enumerate}
@ -87,37 +87,35 @@
\hline
& Juin & Juillet & Août & Septembre & Total\\
\hline
Réussies & 20 & 44 & 40 & 43 & 147\\
Réussies & 29 & 48 & 49 & 43 & 169\\
\hline
Ratées & 22 & 32 & 14 & 10 & 78\\
Ratées & 22 & 43 & 13 & 14 & 92\\
\hline
Total & 42 & 76 & 54 & 53 & 225\\
Total & 51 & 91 & 62 & 57 & 261\\
\hline
\end{tabular}
\end{center}
\item Proportion de photos réussies
\[
\frac{147}{225} = 0.65 = 65\%
\frac{169}{261} = 0.65 = 64\%
\]
\item
\begin{itemize}
\item De juin à juillet
\[
\frac{32 - 22}{22} = \frac{10}{22} = 0.45 = 45\%
\frac{43 - 22}{22} = \frac{21}{22} = 0.95 = 95\%
\]
\item De juillet à août
\[
\frac{14 - 32}{32} = \frac{-18}{32} = -0.56 = -56\%
\frac{13 - 43}{43} = \frac{-30}{43} = -0.7 = -69\%
\]
\item De août à septembre
\[
\frac{10 - 14}{14} = \frac{-4}{14} = -0.29 = -28\%
\frac{14 - 13}{13} = \frac{1}{13} = 0.08 = 7\%
\]
\end{itemize}
\end{enumerate}
\end{solution}
\printsolutionstype{exercise}
\end{document}

View File

@ -17,17 +17,17 @@
Détailler les calculs suivants et donner le résultat sous la forme d'une fraction irréductible.
\begin{multicols}{3}
\begin{enumerate}[label={\Alph*=}]
\item $\dfrac{1}{3} + \dfrac{9}{3}$
\item $\dfrac{5}{2} + 2$
\item $\dfrac{4}{7} + \dfrac{10}{7}$
\item $\dfrac{5}{6} + 6$
\item $\dfrac{10}{2} + \dfrac{6}{2}$
\item $\dfrac{5}{6} + \dfrac{4}{9}$
\item $\dfrac{8}{6} + \dfrac{1}{6}$
\item $\dfrac{10}{7} + \dfrac{10}{3}$
\item $\dfrac{6}{8} \times 5$
\item $\dfrac{5}{2} \times \dfrac{5}{2}$
\item $\dfrac{2}{4} \times 7$
\item $\dfrac{4}{10} \times \dfrac{2}{10}$
\item $\dfrac{6}{2} \times \dfrac{- 4}{10}$
\item $\dfrac{\dfrac{7}{9}}{\dfrac{7}{2}}$
\item $\dfrac{7}{2} \times \dfrac{- 6}{20}$
\item $\dfrac{\dfrac{5}{10}}{\dfrac{6}{9}}$
\end{enumerate}
\end{multicols}
\end{exercise}
@ -35,15 +35,15 @@
\begin{solution}
\begin{enumerate}[label={\Alph*=}]
\item $\dfrac{1}{3} + \dfrac{9}{3}=\dfrac{1 + 9}{3}=\dfrac{10}{3} = \dfrac{10}{3}$
\item $\dfrac{5}{2} + 2=\dfrac{5}{2} + \dfrac{2}{1}=\dfrac{5}{2} + \dfrac{2 \times 2}{1 \times 2}=\dfrac{5}{2} + \dfrac{4}{2}=\dfrac{5 + 4}{2}=\dfrac{9}{2} = \dfrac{9}{2}$
\item $\dfrac{10}{2} + \dfrac{6}{2}=\dfrac{10 + 6}{2}=\dfrac{16}{2} = 8$
\item $\dfrac{5}{6} + \dfrac{4}{9}=\dfrac{5 \times 3}{6 \times 3} + \dfrac{4 \times 2}{9 \times 2}=\dfrac{15}{18} + \dfrac{8}{18}=\dfrac{15 + 8}{18}=\dfrac{23}{18} = \dfrac{23}{18}$
\item $\dfrac{4}{7} + \dfrac{10}{7}=\dfrac{4 + 10}{7}=\dfrac{14}{7} = 2$
\item $\dfrac{5}{6} + 6=\dfrac{5}{6} + \dfrac{6}{1}=\dfrac{5}{6} + \dfrac{6 \times 6}{1 \times 6}=\dfrac{5}{6} + \dfrac{36}{6}=\dfrac{5 + 36}{6}=\dfrac{41}{6} = \dfrac{41}{6}$
\item $\dfrac{8}{6} + \dfrac{1}{6}=\dfrac{8 + 1}{6}=\dfrac{9}{6} = \dfrac{3}{2}$
\item $\dfrac{10}{7} + \dfrac{10}{3}=\dfrac{10 \times 3}{7 \times 3} + \dfrac{10 \times 7}{3 \times 7}=\dfrac{30}{21} + \dfrac{70}{21}=\dfrac{30 + 70}{21}=\dfrac{100}{21} = \dfrac{100}{21}$
\item $\dfrac{6}{8} \times 5=\dfrac{6 \times 5}{8}=\dfrac{30}{8} = \dfrac{15}{4}$
\item $\dfrac{5}{2} \times \dfrac{5}{2}=\dfrac{5 \times 5}{2 \times 2}=\dfrac{25}{4} = \dfrac{25}{4}$
\item $\dfrac{6}{2} \times \dfrac{- 4}{10}=\dfrac{6(- 4)}{2 \times 10}=\dfrac{- 24}{20} = \dfrac{- 6}{5}$
\item $\dfrac{\dfrac{7}{9}}{\dfrac{7}{2}}=\dfrac{7}{9} \times \dfrac{2}{7}=\dfrac{7 \times 2}{9 \times 7}=\dfrac{14}{63} = \dfrac{2}{9}$
\item $\dfrac{2}{4} \times 7=\dfrac{2 \times 7}{4}=\dfrac{14}{4} = \dfrac{7}{2}$
\item $\dfrac{4}{10} \times \dfrac{2}{10}=\dfrac{4 \times 2}{10 \times 10}=\dfrac{8}{100} = \dfrac{2}{25}$
\item $\dfrac{7}{2} \times \dfrac{- 6}{20}=\dfrac{7(- 6)}{2 \times 20}=\dfrac{- 42}{40} = \dfrac{- 21}{20}$
\item $\dfrac{\dfrac{5}{10}}{\dfrac{6}{9}}=\dfrac{5}{10} \times \dfrac{9}{6}=\dfrac{5 \times 9}{10 \times 6}=\dfrac{45}{60} = \dfrac{3}{4}$
\end{enumerate}
\end{solution}
@ -52,10 +52,10 @@
Le radar a pris des photos pendant l'été:
\begin{itemize}
\item en juin, il y a eu 51 photos prises dont 25 ratées.
\item en juillet, il y a eu 37 photos réussies et 32 ratées.
\item en août, il y a eu 65 photos dont une proportion de 0.26 de photos ratées.
\item en septembre, il y a eu 8 photos ratées, ce qui correspondait à 14.55\% des photos prises.
\item en juin, il y a eu 47 photos prises dont 22 ratées.
\item en juillet, il y a eu 43 photos réussies et 30 ratées.
\item en août, il y a eu 66 photos dont une proportion de 0.3 de photos ratées.
\item en septembre, il y a eu 8 photos ratées, ce qui correspondait à 16.67\% des photos prises.
\end{itemize}
\begin{enumerate}
@ -87,37 +87,35 @@
\hline
& Juin & Juillet & Août & Septembre & Total\\
\hline
Réussies & 26 & 37 & 48 & 47 & 158\\
Réussies & 25 & 43 & 46 & 40 & 154\\
\hline
Ratées & 25 & 32 & 17 & 8 & 82\\
Ratées & 22 & 30 & 20 & 8 & 80\\
\hline
Total & 51 & 69 & 65 & 55 & 240\\
Total & 47 & 73 & 66 & 48 & 234\\
\hline
\end{tabular}
\end{center}
\item Proportion de photos réussies
\[
\frac{158}{240} = 0.66 = 65\%
\frac{154}{234} = 0.66 = 65\%
\]
\item
\begin{itemize}
\item De juin à juillet
\[
\frac{32 - 25}{25} = \frac{7}{25} = 0.28 = 28\%
\frac{30 - 22}{22} = \frac{8}{22} = 0.36 = 36\%
\]
\item De juillet à août
\[
\frac{17 - 32}{32} = \frac{-15}{32} = -0.47 = -46\%
\frac{20 - 30}{30} = \frac{-10}{30} = -0.33 = -33\%
\]
\item De août à septembre
\[
\frac{8 - 17}{17} = \frac{-9}{17} = -0.53 = -52\%
\frac{8 - 20}{20} = \frac{-12}{20} = -0.6 = -60\%
\]
\end{itemize}
\end{enumerate}
\end{solution}
\printsolutionstype{exercise}
\end{document}

View File

@ -17,17 +17,17 @@
Détailler les calculs suivants et donner le résultat sous la forme d'une fraction irréductible.
\begin{multicols}{3}
\begin{enumerate}[label={\Alph*=}]
\item $\dfrac{4}{6} + \dfrac{1}{6}$
\item $\dfrac{6}{7} + 6$
\item $\dfrac{5}{6} + \dfrac{4}{6}$
\item $\dfrac{3}{6} + 7$
\item $\dfrac{5}{4} + \dfrac{6}{24}$
\item $\dfrac{3}{6} + \dfrac{6}{8}$
\item $\dfrac{6}{7} + \dfrac{1}{63}$
\item $\dfrac{6}{3} + \dfrac{1}{3}$
\item $\dfrac{3}{7} \times 7$
\item $\dfrac{8}{4} \times \dfrac{5}{4}$
\item $\dfrac{9}{4} \times 1$
\item $\dfrac{3}{4} \times \dfrac{2}{4}$
\item $\dfrac{4}{10} \times \dfrac{- 5}{50}$
\item $\dfrac{\dfrac{10}{9}}{\dfrac{5}{7}}$
\item $\dfrac{7}{3} \times \dfrac{- 10}{21}$
\item $\dfrac{\dfrac{2}{9}}{\dfrac{2}{4}}$
\end{enumerate}
\end{multicols}
\end{exercise}
@ -35,15 +35,15 @@
\begin{solution}
\begin{enumerate}[label={\Alph*=}]
\item $\dfrac{4}{6} + \dfrac{1}{6}=\dfrac{4 + 1}{6}=\dfrac{5}{6} = \dfrac{5}{6}$
\item $\dfrac{6}{7} + 6=\dfrac{6}{7} + \dfrac{6}{1}=\dfrac{6}{7} + \dfrac{6 \times 7}{1 \times 7}=\dfrac{6}{7} + \dfrac{42}{7}=\dfrac{6 + 42}{7}=\dfrac{48}{7} = \dfrac{48}{7}$
\item $\dfrac{5}{4} + \dfrac{6}{24}=\dfrac{5 \times 6}{4 \times 6} + \dfrac{6}{24}=\dfrac{30}{24} + \dfrac{6}{24}=\dfrac{30 + 6}{24}=\dfrac{36}{24} = \dfrac{3}{2}$
\item $\dfrac{3}{6} + \dfrac{6}{8}=\dfrac{3 \times 4}{6 \times 4} + \dfrac{6 \times 3}{8 \times 3}=\dfrac{12}{24} + \dfrac{18}{24}=\dfrac{12 + 18}{24}=\dfrac{30}{24} = \dfrac{5}{4}$
\item $\dfrac{5}{6} + \dfrac{4}{6}=\dfrac{5 + 4}{6}=\dfrac{9}{6} = \dfrac{3}{2}$
\item $\dfrac{3}{6} + 7=\dfrac{3}{6} + \dfrac{7}{1}=\dfrac{3}{6} + \dfrac{7 \times 6}{1 \times 6}=\dfrac{3}{6} + \dfrac{42}{6}=\dfrac{3 + 42}{6}=\dfrac{45}{6} = \dfrac{15}{2}$
\item $\dfrac{6}{7} + \dfrac{1}{63}=\dfrac{6 \times 9}{7 \times 9} + \dfrac{1}{63}=\dfrac{54}{63} + \dfrac{1}{63}=\dfrac{54 + 1}{63}=\dfrac{55}{63} = \dfrac{55}{63}$
\item $\dfrac{6}{3} + \dfrac{1}{3}=\dfrac{6 + 1}{3}=\dfrac{7}{3} = \dfrac{7}{3}$
\item $\dfrac{3}{7} \times 7=\dfrac{3 \times 7}{7}=\dfrac{21}{7} = 3$
\item $\dfrac{8}{4} \times \dfrac{5}{4}=\dfrac{8 \times 5}{4 \times 4}=\dfrac{40}{16} = \dfrac{5}{2}$
\item $\dfrac{4}{10} \times \dfrac{- 5}{50}=\dfrac{4(- 5)}{10 \times 50}=\dfrac{- 20}{500} = \dfrac{- 1}{25}$
\item $\dfrac{\dfrac{10}{9}}{\dfrac{5}{7}}=\dfrac{10}{9} \times \dfrac{7}{5}=\dfrac{10 \times 7}{9 \times 5}=\dfrac{70}{45} = \dfrac{14}{9}$
\item $\dfrac{9}{4} \times 1=\dfrac{9}{4} = \dfrac{9}{4}$
\item $\dfrac{3}{4} \times \dfrac{2}{4}=\dfrac{3 \times 2}{4 \times 4}=\dfrac{6}{16} = \dfrac{3}{8}$
\item $\dfrac{7}{3} \times \dfrac{- 10}{21}=\dfrac{7(- 10)}{3 \times 21}=\dfrac{- 70}{63} = \dfrac{- 10}{9}$
\item $\dfrac{\dfrac{2}{9}}{\dfrac{2}{4}}=\dfrac{2}{9} \times \dfrac{4}{2}=\dfrac{2 \times 4}{9 \times 2}=\dfrac{8}{18} = \dfrac{4}{9}$
\end{enumerate}
\end{solution}
@ -52,10 +52,10 @@
Le radar a pris des photos pendant l'été:
\begin{itemize}
\item en juin, il y a eu 50 photos prises dont 20 ratées.
\item en juillet, il y a eu 47 photos réussies et 44 ratées.
\item en août, il y a eu 53 photos dont une proportion de 0.25 de photos ratées.
\item en septembre, il y a eu 9 photos ratées, ce qui correspondait à 16.98\% des photos prises.
\item en juin, il y a eu 51 photos prises dont 22 ratées.
\item en juillet, il y a eu 48 photos réussies et 49 ratées.
\item en août, il y a eu 62 photos dont une proportion de 0.29 de photos ratées.
\item en septembre, il y a eu 15 photos ratées, ce qui correspondait à 23.08\% des photos prises.
\end{itemize}
\begin{enumerate}
@ -87,37 +87,35 @@
\hline
& Juin & Juillet & Août & Septembre & Total\\
\hline
Réussies & 30 & 47 & 40 & 44 & 161\\
Réussies & 29 & 48 & 44 & 50 & 171\\
\hline
Ratées & 20 & 44 & 13 & 9 & 86\\
Ratées & 22 & 49 & 18 & 15 & 104\\
\hline
Total & 50 & 91 & 53 & 53 & 247\\
Total & 51 & 97 & 62 & 65 & 275\\
\hline
\end{tabular}
\end{center}
\item Proportion de photos réussies
\[
\frac{161}{247} = 0.65 = 65\%
\frac{171}{275} = 0.62 = 62\%
\]
\item
\begin{itemize}
\item De juin à juillet
\[
\frac{44 - 20}{20} = \frac{24}{20} = 1.2 = 120\%
\frac{49 - 22}{22} = \frac{27}{22} = 1.23 = 122\%
\]
\item De juillet à août
\[
\frac{13 - 44}{44} = \frac{-31}{44} = -0.7 = -70\%
\frac{18 - 49}{49} = \frac{-31}{49} = -0.63 = -63\%
\]
\item De août à septembre
\[
\frac{9 - 13}{13} = \frac{-4}{13} = -0.31 = -30\%
\frac{15 - 18}{18} = \frac{-3}{18} = -0.17 = -16\%
\]
\end{itemize}
\end{enumerate}
\end{solution}
\printsolutionstype{exercise}
\end{document}

View File

@ -17,17 +17,17 @@
Détailler les calculs suivants et donner le résultat sous la forme d'une fraction irréductible.
\begin{multicols}{3}
\begin{enumerate}[label={\Alph*=}]
\item $\dfrac{3}{7} + \dfrac{3}{7}$
\item $\dfrac{8}{3} + 9$
\item $\dfrac{6}{5} + \dfrac{3}{5}$
\item $\dfrac{7}{2} + 5$
\item $\dfrac{1}{4} + \dfrac{3}{28}$
\item $\dfrac{4}{7} + \dfrac{6}{2}$
\item $\dfrac{5}{4} + \dfrac{2}{32}$
\item $\dfrac{10}{2} + \dfrac{10}{9}$
\item $\dfrac{9}{4} \times 8$
\item $\dfrac{5}{7} \times \dfrac{3}{7}$
\item $\dfrac{10}{3} \times 4$
\item $\dfrac{3}{2} \times \dfrac{7}{2}$
\item $\dfrac{- 10}{8} \times \dfrac{- 2}{40}$
\item $\dfrac{\dfrac{4}{9}}{\dfrac{6}{9}}$
\item $\dfrac{- 10}{3} \times \dfrac{8}{12}$
\item $\dfrac{\dfrac{9}{3}}{\dfrac{1}{4}}$
\end{enumerate}
\end{multicols}
\end{exercise}
@ -35,15 +35,15 @@
\begin{solution}
\begin{enumerate}[label={\Alph*=}]
\item $\dfrac{3}{7} + \dfrac{3}{7}=\dfrac{3 + 3}{7}=\dfrac{6}{7} = \dfrac{6}{7}$
\item $\dfrac{8}{3} + 9=\dfrac{8}{3} + \dfrac{9}{1}=\dfrac{8}{3} + \dfrac{9 \times 3}{1 \times 3}=\dfrac{8}{3} + \dfrac{27}{3}=\dfrac{8 + 27}{3}=\dfrac{35}{3} = \dfrac{35}{3}$
\item $\dfrac{1}{4} + \dfrac{3}{28}=\dfrac{1 \times 7}{4 \times 7} + \dfrac{3}{28}=\dfrac{7}{28} + \dfrac{3}{28}=\dfrac{7 + 3}{28}=\dfrac{10}{28} = \dfrac{5}{14}$
\item $\dfrac{4}{7} + \dfrac{6}{2}=\dfrac{4 \times 2}{7 \times 2} + \dfrac{6 \times 7}{2 \times 7}=\dfrac{8}{14} + \dfrac{42}{14}=\dfrac{8 + 42}{14}=\dfrac{50}{14} = \dfrac{25}{7}$
\item $\dfrac{6}{5} + \dfrac{3}{5}=\dfrac{6 + 3}{5}=\dfrac{9}{5} = \dfrac{9}{5}$
\item $\dfrac{7}{2} + 5=\dfrac{7}{2} + \dfrac{5}{1}=\dfrac{7}{2} + \dfrac{5 \times 2}{1 \times 2}=\dfrac{7}{2} + \dfrac{10}{2}=\dfrac{7 + 10}{2}=\dfrac{17}{2} = \dfrac{17}{2}$
\item $\dfrac{5}{4} + \dfrac{2}{32}=\dfrac{5 \times 8}{4 \times 8} + \dfrac{2}{32}=\dfrac{40}{32} + \dfrac{2}{32}=\dfrac{40 + 2}{32}=\dfrac{42}{32} = \dfrac{21}{16}$
\item $\dfrac{10}{2} + \dfrac{10}{9}=\dfrac{10 \times 9}{2 \times 9} + \dfrac{10 \times 2}{9 \times 2}=\dfrac{90}{18} + \dfrac{20}{18}=\dfrac{90 + 20}{18}=\dfrac{110}{18} = \dfrac{55}{9}$
\item $\dfrac{9}{4} \times 8=\dfrac{9 \times 8}{4}=\dfrac{72}{4} = 18$
\item $\dfrac{5}{7} \times \dfrac{3}{7}=\dfrac{5 \times 3}{7 \times 7}=\dfrac{15}{49} = \dfrac{15}{49}$
\item $\dfrac{- 10}{8} \times \dfrac{- 2}{40}=\dfrac{- 10(- 2)}{8 \times 40}=\dfrac{20}{320} = \dfrac{1}{16}$
\item $\dfrac{\dfrac{4}{9}}{\dfrac{6}{9}}=\dfrac{4}{9} \times \dfrac{9}{6}=\dfrac{4 \times 9}{9 \times 6}=\dfrac{36}{54} = \dfrac{2}{3}$
\item $\dfrac{10}{3} \times 4=\dfrac{10 \times 4}{3}=\dfrac{40}{3} = \dfrac{40}{3}$
\item $\dfrac{3}{2} \times \dfrac{7}{2}=\dfrac{3 \times 7}{2 \times 2}=\dfrac{21}{4} = \dfrac{21}{4}$
\item $\dfrac{- 10}{3} \times \dfrac{8}{12}=\dfrac{- 10 \times 8}{3 \times 12}=\dfrac{- 80}{36} = \dfrac{- 20}{9}$
\item $\dfrac{\dfrac{9}{3}}{\dfrac{1}{4}}=\dfrac{9}{3} \times \dfrac{4}{1}=\dfrac{9 \times 4}{3 \times 1}=\dfrac{36}{3} = 12$
\end{enumerate}
\end{solution}
@ -52,10 +52,10 @@
Le radar a pris des photos pendant l'été:
\begin{itemize}
\item en juin, il y a eu 45 photos prises dont 21 ratées.
\item en juillet, il y a eu 39 photos réussies et 30 ratées.
\item en août, il y a eu 54 photos dont une proportion de 0.19 de photos ratées.
\item en septembre, il y a eu 7 photos ratées, ce qui correspondait à 14.0\% des photos prises.
\item en juin, il y a eu 47 photos prises dont 23 ratées.
\item en juillet, il y a eu 34 photos réussies et 47 ratées.
\item en août, il y a eu 67 photos dont une proportion de 0.3 de photos ratées.
\item en septembre, il y a eu 12 photos ratées, ce qui correspondait à 22.22\% des photos prises.
\end{itemize}
\begin{enumerate}
@ -87,37 +87,35 @@
\hline
& Juin & Juillet & Août & Septembre & Total\\
\hline
Réussies & 24 & 39 & 44 & 43 & 150\\
Réussies & 24 & 34 & 47 & 42 & 147\\
\hline
Ratées & 21 & 30 & 10 & 7 & 68\\
Ratées & 23 & 47 & 20 & 12 & 102\\
\hline
Total & 45 & 69 & 54 & 50 & 218\\
Total & 47 & 81 & 67 & 54 & 249\\
\hline
\end{tabular}
\end{center}
\item Proportion de photos réussies
\[
\frac{150}{218} = 0.69 = 68\%
\frac{147}{249} = 0.59 = 59\%
\]
\item
\begin{itemize}
\item De juin à juillet
\[
\frac{30 - 21}{21} = \frac{9}{21} = 0.43 = 42\%
\frac{47 - 23}{23} = \frac{24}{23} = 1.04 = 104\%
\]
\item De juillet à août
\[
\frac{10 - 30}{30} = \frac{-20}{30} = -0.67 = -66\%
\frac{20 - 47}{47} = \frac{-27}{47} = -0.57 = -57\%
\]
\item De août à septembre
\[
\frac{7 - 10}{10} = \frac{-3}{10} = -0.3 = -30\%
\frac{12 - 20}{20} = \frac{-8}{20} = -0.4 = -40\%
\]
\end{itemize}
\end{enumerate}
\end{solution}
\printsolutionstype{exercise}
\end{document}

View File

@ -17,17 +17,17 @@
Détailler les calculs suivants et donner le résultat sous la forme d'une fraction irréductible.
\begin{multicols}{3}
\begin{enumerate}[label={\Alph*=}]
\item $\dfrac{5}{4} + \dfrac{7}{4}$
\item $\dfrac{2}{4} + 5$
\item $\dfrac{3}{4} + \dfrac{7}{4}$
\item $\dfrac{4}{3} + 7$
\item $\dfrac{2}{5} + \dfrac{9}{30}$
\item $\dfrac{1}{7} + \dfrac{4}{6}$
\item $\dfrac{5}{4} + \dfrac{3}{20}$
\item $\dfrac{6}{9} + \dfrac{7}{5}$
\item $\dfrac{8}{9} \times 3$
\item $\dfrac{2}{5} \times \dfrac{7}{5}$
\item $\dfrac{4}{10} \times 5$
\item $\dfrac{1}{9} \times \dfrac{4}{9}$
\item $\dfrac{4}{7} \times \dfrac{- 2}{70}$
\item $\dfrac{\dfrac{5}{4}}{\dfrac{1}{3}}$
\item $\dfrac{- 5}{5} \times \dfrac{7}{45}$
\item $\dfrac{\dfrac{1}{2}}{\dfrac{10}{9}}$
\end{enumerate}
\end{multicols}
\end{exercise}
@ -35,15 +35,15 @@
\begin{solution}
\begin{enumerate}[label={\Alph*=}]
\item $\dfrac{5}{4} + \dfrac{7}{4}=\dfrac{5 + 7}{4}=\dfrac{12}{4} = 3$
\item $\dfrac{2}{4} + 5=\dfrac{2}{4} + \dfrac{5}{1}=\dfrac{2}{4} + \dfrac{5 \times 4}{1 \times 4}=\dfrac{2}{4} + \dfrac{20}{4}=\dfrac{2 + 20}{4}=\dfrac{22}{4} = \dfrac{11}{2}$
\item $\dfrac{2}{5} + \dfrac{9}{30}=\dfrac{2 \times 6}{5 \times 6} + \dfrac{9}{30}=\dfrac{12}{30} + \dfrac{9}{30}=\dfrac{12 + 9}{30}=\dfrac{21}{30} = \dfrac{7}{10}$
\item $\dfrac{1}{7} + \dfrac{4}{6}=\dfrac{1 \times 6}{7 \times 6} + \dfrac{4 \times 7}{6 \times 7}=\dfrac{6}{42} + \dfrac{28}{42}=\dfrac{6 + 28}{42}=\dfrac{34}{42} = \dfrac{17}{21}$
\item $\dfrac{3}{4} + \dfrac{7}{4}=\dfrac{3 + 7}{4}=\dfrac{10}{4} = \dfrac{5}{2}$
\item $\dfrac{4}{3} + 7=\dfrac{4}{3} + \dfrac{7}{1}=\dfrac{4}{3} + \dfrac{7 \times 3}{1 \times 3}=\dfrac{4}{3} + \dfrac{21}{3}=\dfrac{4 + 21}{3}=\dfrac{25}{3} = \dfrac{25}{3}$
\item $\dfrac{5}{4} + \dfrac{3}{20}=\dfrac{5 \times 5}{4 \times 5} + \dfrac{3}{20}=\dfrac{25}{20} + \dfrac{3}{20}=\dfrac{25 + 3}{20}=\dfrac{28}{20} = \dfrac{7}{5}$
\item $\dfrac{6}{9} + \dfrac{7}{5}=\dfrac{6 \times 5}{9 \times 5} + \dfrac{7 \times 9}{5 \times 9}=\dfrac{30}{45} + \dfrac{63}{45}=\dfrac{30 + 63}{45}=\dfrac{93}{45} = \dfrac{31}{15}$
\item $\dfrac{8}{9} \times 3=\dfrac{8 \times 3}{9}=\dfrac{24}{9} = \dfrac{8}{3}$
\item $\dfrac{2}{5} \times \dfrac{7}{5}=\dfrac{2 \times 7}{5 \times 5}=\dfrac{14}{25} = \dfrac{14}{25}$
\item $\dfrac{4}{7} \times \dfrac{- 2}{70}=\dfrac{4(- 2)}{7 \times 70}=\dfrac{- 8}{490} = \dfrac{- 4}{245}$
\item $\dfrac{\dfrac{5}{4}}{\dfrac{1}{3}}=\dfrac{5}{4} \times \dfrac{3}{1}=\dfrac{5 \times 3}{4 \times 1}=\dfrac{15}{4} = \dfrac{15}{4}$
\item $\dfrac{4}{10} \times 5=\dfrac{4 \times 5}{10}=\dfrac{20}{10} = 2$
\item $\dfrac{1}{9} \times \dfrac{4}{9}=\dfrac{1 \times 4}{9 \times 9}=\dfrac{4}{81} = \dfrac{4}{81}$
\item $\dfrac{- 5}{5} \times \dfrac{7}{45}=\dfrac{- 5 \times 7}{5 \times 45}=\dfrac{- 35}{225} = \dfrac{- 7}{45}$
\item $\dfrac{\dfrac{1}{2}}{\dfrac{10}{9}}=\dfrac{1}{2} \times \dfrac{9}{10}=\dfrac{1 \times 9}{2 \times 10}=\dfrac{9}{20} = \dfrac{9}{20}$
\end{enumerate}
\end{solution}
@ -52,10 +52,10 @@
Le radar a pris des photos pendant l'été:
\begin{itemize}
\item en juin, il y a eu 51 photos prises dont 24 ratées.
\item en juillet, il y a eu 42 photos réussies et 39 ratées.
\item en août, il y a eu 55 photos dont une proportion de 0.25 de photos ratées.
\item en septembre, il y a eu 14 photos ratées, ce qui correspondait à 25.0\% des photos prises.
\item en juin, il y a eu 52 photos prises dont 24 ratées.
\item en juillet, il y a eu 47 photos réussies et 31 ratées.
\item en août, il y a eu 63 photos dont une proportion de 0.21 de photos ratées.
\item en septembre, il y a eu 14 photos ratées, ce qui correspondait à 23.73\% des photos prises.
\end{itemize}
\begin{enumerate}
@ -87,37 +87,35 @@
\hline
& Juin & Juillet & Août & Septembre & Total\\
\hline
Réussies & 27 & 42 & 41 & 42 & 152\\
Réussies & 28 & 47 & 50 & 45 & 170\\
\hline
Ratées & 24 & 39 & 14 & 14 & 91\\
Ratées & 24 & 31 & 13 & 14 & 82\\
\hline
Total & 51 & 81 & 55 & 56 & 243\\
Total & 52 & 78 & 63 & 59 & 252\\
\hline
\end{tabular}
\end{center}
\item Proportion de photos réussies
\[
\frac{152}{243} = 0.63 = 62\%
\frac{170}{252} = 0.67 = 67\%
\]
\item
\begin{itemize}
\item De juin à juillet
\[
\frac{39 - 24}{24} = \frac{15}{24} = 0.62 = 62\%
\frac{31 - 24}{24} = \frac{7}{24} = 0.29 = 29\%
\]
\item De juillet à août
\[
\frac{14 - 39}{39} = \frac{-25}{39} = -0.64 = -64\%
\frac{13 - 31}{31} = \frac{-18}{31} = -0.58 = -58\%
\]
\item De août à septembre
\[
\frac{14 - 14}{14} = \frac{0}{14} = 0.0 = 0\%
\frac{14 - 13}{13} = \frac{1}{13} = 0.08 = 7\%
\]
\end{itemize}
\end{enumerate}
\end{solution}
\printsolutionstype{exercise}
\end{document}

View File

@ -17,17 +17,17 @@
Détailler les calculs suivants et donner le résultat sous la forme d'une fraction irréductible.
\begin{multicols}{3}
\begin{enumerate}[label={\Alph*=}]
\item $\dfrac{7}{2} + \dfrac{8}{2}$
\item $\dfrac{8}{6} + 5$
\item $\dfrac{9}{5} + \dfrac{9}{5}$
\item $\dfrac{4}{5} + 7$
\item $\dfrac{10}{6} + \dfrac{9}{48}$
\item $\dfrac{9}{7} + \dfrac{6}{2}$
\item $\dfrac{2}{9} + \dfrac{2}{63}$
\item $\dfrac{2}{6} + \dfrac{2}{4}$
\item $\dfrac{2}{6} \times 6$
\item $\dfrac{1}{7} \times \dfrac{3}{7}$
\item $\dfrac{2}{4} \times 5$
\item $\dfrac{6}{8} \times \dfrac{5}{8}$
\item $\dfrac{- 7}{7} \times \dfrac{2}{35}$
\item $\dfrac{\dfrac{10}{8}}{\dfrac{6}{2}}$
\item $\dfrac{- 9}{7} \times \dfrac{- 1}{42}$
\item $\dfrac{\dfrac{6}{9}}{\dfrac{7}{6}}$
\end{enumerate}
\end{multicols}
\end{exercise}
@ -35,15 +35,15 @@
\begin{solution}
\begin{enumerate}[label={\Alph*=}]
\item $\dfrac{7}{2} + \dfrac{8}{2}=\dfrac{7 + 8}{2}=\dfrac{15}{2} = \dfrac{15}{2}$
\item $\dfrac{8}{6} + 5=\dfrac{8}{6} + \dfrac{5}{1}=\dfrac{8}{6} + \dfrac{5 \times 6}{1 \times 6}=\dfrac{8}{6} + \dfrac{30}{6}=\dfrac{8 + 30}{6}=\dfrac{38}{6} = \dfrac{19}{3}$
\item $\dfrac{10}{6} + \dfrac{9}{48}=\dfrac{10 \times 8}{6 \times 8} + \dfrac{9}{48}=\dfrac{80}{48} + \dfrac{9}{48}=\dfrac{80 + 9}{48}=\dfrac{89}{48} = \dfrac{89}{48}$
\item $\dfrac{9}{7} + \dfrac{6}{2}=\dfrac{9 \times 2}{7 \times 2} + \dfrac{6 \times 7}{2 \times 7}=\dfrac{18}{14} + \dfrac{42}{14}=\dfrac{18 + 42}{14}=\dfrac{60}{14} = \dfrac{30}{7}$
\item $\dfrac{9}{5} + \dfrac{9}{5}=\dfrac{9 + 9}{5}=\dfrac{18}{5} = \dfrac{18}{5}$
\item $\dfrac{4}{5} + 7=\dfrac{4}{5} + \dfrac{7}{1}=\dfrac{4}{5} + \dfrac{7 \times 5}{1 \times 5}=\dfrac{4}{5} + \dfrac{35}{5}=\dfrac{4 + 35}{5}=\dfrac{39}{5} = \dfrac{39}{5}$
\item $\dfrac{2}{9} + \dfrac{2}{63}=\dfrac{2 \times 7}{9 \times 7} + \dfrac{2}{63}=\dfrac{14}{63} + \dfrac{2}{63}=\dfrac{14 + 2}{63}=\dfrac{16}{63} = \dfrac{16}{63}$
\item $\dfrac{2}{6} + \dfrac{2}{4}=\dfrac{2 \times 2}{6 \times 2} + \dfrac{2 \times 3}{4 \times 3}=\dfrac{4}{12} + \dfrac{6}{12}=\dfrac{4 + 6}{12}=\dfrac{10}{12} = \dfrac{5}{6}$
\item $\dfrac{2}{6} \times 6=\dfrac{2 \times 6}{6}=\dfrac{12}{6} = 2$
\item $\dfrac{1}{7} \times \dfrac{3}{7}=\dfrac{1 \times 3}{7 \times 7}=\dfrac{3}{49} = \dfrac{3}{49}$
\item $\dfrac{- 7}{7} \times \dfrac{2}{35}=\dfrac{- 7 \times 2}{7 \times 35}=\dfrac{- 14}{245} = \dfrac{- 2}{35}$
\item $\dfrac{\dfrac{10}{8}}{\dfrac{6}{2}}=\dfrac{10}{8} \times \dfrac{2}{6}=\dfrac{10 \times 2}{8 \times 6}=\dfrac{20}{48} = \dfrac{5}{12}$
\item $\dfrac{2}{4} \times 5=\dfrac{2 \times 5}{4}=\dfrac{10}{4} = \dfrac{5}{2}$
\item $\dfrac{6}{8} \times \dfrac{5}{8}=\dfrac{6 \times 5}{8 \times 8}=\dfrac{30}{64} = \dfrac{15}{32}$
\item $\dfrac{- 9}{7} \times \dfrac{- 1}{42}=\dfrac{- 9(- 1)}{7 \times 42}=\dfrac{9}{294} = \dfrac{3}{98}$
\item $\dfrac{\dfrac{6}{9}}{\dfrac{7}{6}}=\dfrac{6}{9} \times \dfrac{6}{7}=\dfrac{6 \times 6}{9 \times 7}=\dfrac{36}{63} = \dfrac{4}{7}$
\end{enumerate}
\end{solution}
@ -52,10 +52,10 @@
Le radar a pris des photos pendant l'été:
\begin{itemize}
\item en juin, il y a eu 50 photos prises dont 26 ratées.
\item en juillet, il y a eu 43 photos réussies et 32 ratées.
\item en août, il y a eu 61 photos dont une proportion de 0.2 de photos ratées.
\item en septembre, il y a eu 5 photos ratées, ce qui correspondait à 9.26\% des photos prises.
\item en juin, il y a eu 46 photos prises dont 25 ratées.
\item en juillet, il y a eu 35 photos réussies et 44 ratées.
\item en août, il y a eu 52 photos dont une proportion de 0.21 de photos ratées.
\item en septembre, il y a eu 9 photos ratées, ce qui correspondait à 17.31\% des photos prises.
\end{itemize}
\begin{enumerate}
@ -87,37 +87,35 @@
\hline
& Juin & Juillet & Août & Septembre & Total\\
\hline
Réussies & 24 & 43 & 49 & 49 & 165\\
Réussies & 21 & 35 & 41 & 43 & 140\\
\hline
Ratées & 26 & 32 & 12 & 5 & 75\\
Ratées & 25 & 44 & 11 & 9 & 89\\
\hline
Total & 50 & 75 & 61 & 54 & 240\\
Total & 46 & 79 & 52 & 52 & 229\\
\hline
\end{tabular}
\end{center}
\item Proportion de photos réussies
\[
\frac{165}{240} = 0.69 = 68\%
\frac{140}{229} = 0.61 = 61\%
\]
\item
\begin{itemize}
\item De juin à juillet
\[
\frac{32 - 26}{26} = \frac{6}{26} = 0.23 = 23\%
\frac{44 - 25}{25} = \frac{19}{25} = 0.76 = 76\%
\]
\item De juillet à août
\[
\frac{12 - 32}{32} = \frac{-20}{32} = -0.62 = -62\%
\frac{11 - 44}{44} = \frac{-33}{44} = -0.75 = -75\%
\]
\item De août à septembre
\[
\frac{5 - 12}{12} = \frac{-7}{12} = -0.58 = -58\%
\frac{9 - 11}{11} = \frac{-2}{11} = -0.18 = -18\%
\]
\end{itemize}
\end{enumerate}
\end{solution}
\printsolutionstype{exercise}
\end{document}

View File

@ -17,17 +17,17 @@
Détailler les calculs suivants et donner le résultat sous la forme d'une fraction irréductible.
\begin{multicols}{3}
\begin{enumerate}[label={\Alph*=}]
\item $\dfrac{10}{5} + \dfrac{2}{5}$
\item $\dfrac{8}{9} + 6$
\item $\dfrac{10}{2} + \dfrac{3}{2}$
\item $\dfrac{6}{9} + 7$
\item $\dfrac{3}{4} + \dfrac{1}{24}$
\item $\dfrac{2}{4} + \dfrac{1}{2}$
\item $\dfrac{3}{9} + \dfrac{8}{45}$
\item $\dfrac{2}{3} + \dfrac{10}{4}$
\item $\dfrac{2}{4} \times 1$
\item $\dfrac{1}{5} \times \dfrac{1}{5}$
\item $\dfrac{5}{6} \times 9$
\item $\dfrac{6}{10} \times \dfrac{5}{10}$
\item $\dfrac{- 2}{10} \times \dfrac{- 5}{20}$
\item $\dfrac{\dfrac{4}{7}}{\dfrac{7}{4}}$
\item $\dfrac{- 6}{9} \times \dfrac{- 4}{90}$
\item $\dfrac{\dfrac{6}{7}}{\dfrac{1}{8}}$
\end{enumerate}
\end{multicols}
\end{exercise}
@ -35,15 +35,15 @@
\begin{solution}
\begin{enumerate}[label={\Alph*=}]
\item $\dfrac{10}{5} + \dfrac{2}{5}=\dfrac{10 + 2}{5}=\dfrac{12}{5} = \dfrac{12}{5}$
\item $\dfrac{8}{9} + 6=\dfrac{8}{9} + \dfrac{6}{1}=\dfrac{8}{9} + \dfrac{6 \times 9}{1 \times 9}=\dfrac{8}{9} + \dfrac{54}{9}=\dfrac{8 + 54}{9}=\dfrac{62}{9} = \dfrac{62}{9}$
\item $\dfrac{3}{4} + \dfrac{1}{24}=\dfrac{3 \times 6}{4 \times 6} + \dfrac{1}{24}=\dfrac{18}{24} + \dfrac{1}{24}=\dfrac{18 + 1}{24}=\dfrac{19}{24} = \dfrac{19}{24}$
\item $\dfrac{2}{4} + \dfrac{1}{2}=\dfrac{2}{4} + \dfrac{1 \times 2}{2 \times 2}=\dfrac{2}{4} + \dfrac{2}{4}=\dfrac{2 + 2}{4}=\dfrac{4}{4} = 1$
\item $\dfrac{10}{2} + \dfrac{3}{2}=\dfrac{10 + 3}{2}=\dfrac{13}{2} = \dfrac{13}{2}$
\item $\dfrac{6}{9} + 7=\dfrac{6}{9} + \dfrac{7}{1}=\dfrac{6}{9} + \dfrac{7 \times 9}{1 \times 9}=\dfrac{6}{9} + \dfrac{63}{9}=\dfrac{6 + 63}{9}=\dfrac{69}{9} = \dfrac{23}{3}$
\item $\dfrac{3}{9} + \dfrac{8}{45}=\dfrac{3 \times 5}{9 \times 5} + \dfrac{8}{45}=\dfrac{15}{45} + \dfrac{8}{45}=\dfrac{15 + 8}{45}=\dfrac{23}{45} = \dfrac{23}{45}$
\item $\dfrac{2}{3} + \dfrac{10}{4}=\dfrac{2 \times 4}{3 \times 4} + \dfrac{10 \times 3}{4 \times 3}=\dfrac{8}{12} + \dfrac{30}{12}=\dfrac{8 + 30}{12}=\dfrac{38}{12} = \dfrac{19}{6}$
\item $\dfrac{2}{4} \times 1=\dfrac{2}{4} = \dfrac{1}{2}$
\item $\dfrac{1}{5} \times \dfrac{1}{5}=\dfrac{1 \times 1}{5 \times 5}=\dfrac{1}{25} = \dfrac{1}{25}$
\item $\dfrac{- 2}{10} \times \dfrac{- 5}{20}=\dfrac{- 2(- 5)}{10 \times 20}=\dfrac{10}{200} = \dfrac{1}{20}$
\item $\dfrac{\dfrac{4}{7}}{\dfrac{7}{4}}=\dfrac{4}{7} \times \dfrac{4}{7}=\dfrac{4 \times 4}{7 \times 7}=\dfrac{16}{49} = \dfrac{16}{49}$
\item $\dfrac{5}{6} \times 9=\dfrac{5 \times 9}{6}=\dfrac{45}{6} = \dfrac{15}{2}$
\item $\dfrac{6}{10} \times \dfrac{5}{10}=\dfrac{6 \times 5}{10 \times 10}=\dfrac{30}{100} = \dfrac{3}{10}$
\item $\dfrac{- 6}{9} \times \dfrac{- 4}{90}=\dfrac{- 6(- 4)}{9 \times 90}=\dfrac{24}{810} = \dfrac{4}{135}$
\item $\dfrac{\dfrac{6}{7}}{\dfrac{1}{8}}=\dfrac{6}{7} \times \dfrac{8}{1}=\dfrac{6 \times 8}{7 \times 1}=\dfrac{48}{7} = \dfrac{48}{7}$
\end{enumerate}
\end{solution}
@ -52,10 +52,10 @@
Le radar a pris des photos pendant l'été:
\begin{itemize}
\item en juin, il y a eu 52 photos prises dont 28 ratées.
\item en juillet, il y a eu 39 photos réussies et 36 ratées.
\item en août, il y a eu 51 photos dont une proportion de 0.2 de photos ratées.
\item en septembre, il y a eu 5 photos ratées, ce qui correspondait à 10.42\% des photos prises.
\item en juin, il y a eu 50 photos prises dont 27 ratées.
\item en juillet, il y a eu 46 photos réussies et 35 ratées.
\item en août, il y a eu 65 photos dont une proportion de 0.29 de photos ratées.
\item en septembre, il y a eu 5 photos ratées, ce qui correspondait à 9.43\% des photos prises.
\end{itemize}
\begin{enumerate}
@ -87,37 +87,35 @@
\hline
& Juin & Juillet & Août & Septembre & Total\\
\hline
Réussies & 24 & 39 & 41 & 43 & 147\\
Réussies & 23 & 46 & 46 & 48 & 163\\
\hline
Ratées & 28 & 36 & 10 & 5 & 79\\
Ratées & 27 & 35 & 19 & 5 & 86\\
\hline
Total & 52 & 75 & 51 & 48 & 226\\
Total & 50 & 81 & 65 & 53 & 249\\
\hline
\end{tabular}
\end{center}
\item Proportion de photos réussies
\[
\frac{147}{226} = 0.65 = 65\%
\frac{163}{249} = 0.65 = 65\%
\]
\item
\begin{itemize}
\item De juin à juillet
\[
\frac{36 - 28}{28} = \frac{8}{28} = 0.29 = 28\%
\frac{35 - 27}{27} = \frac{8}{27} = 0.3 = 29\%
\]
\item De juillet à août
\[
\frac{10 - 36}{36} = \frac{-26}{36} = -0.72 = -72\%
\frac{19 - 35}{35} = \frac{-16}{35} = -0.46 = -45\%
\]
\item De août à septembre
\[
\frac{5 - 10}{10} = \frac{-5}{10} = -0.5 = -50\%
\frac{5 - 19}{19} = \frac{-14}{19} = -0.74 = -73\%
\]
\end{itemize}
\end{enumerate}
\end{solution}
\printsolutionstype{exercise}
\end{document}

View File

@ -17,17 +17,17 @@
Détailler les calculs suivants et donner le résultat sous la forme d'une fraction irréductible.
\begin{multicols}{3}
\begin{enumerate}[label={\Alph*=}]
\item $\dfrac{6}{8} + \dfrac{7}{8}$
\item $\dfrac{9}{5} + 8$
\item $\dfrac{3}{8} + \dfrac{7}{8}$
\item $\dfrac{1}{2} + 2$
\item $\dfrac{2}{5} + \dfrac{10}{35}$
\item $\dfrac{6}{3} + \dfrac{8}{6}$
\item $\dfrac{4}{9} + \dfrac{7}{81}$
\item $\dfrac{6}{8} + \dfrac{9}{6}$
\item $\dfrac{7}{4} \times 8$
\item $\dfrac{4}{8} \times \dfrac{5}{8}$
\item $\dfrac{9}{4} \times 5$
\item $\dfrac{8}{2} \times \dfrac{10}{2}$
\item $\dfrac{- 6}{2} \times \dfrac{- 2}{10}$
\item $\dfrac{\dfrac{3}{10}}{\dfrac{8}{10}}$
\item $\dfrac{- 4}{7} \times \dfrac{9}{49}$
\item $\dfrac{\dfrac{4}{7}}{\dfrac{5}{7}}$
\end{enumerate}
\end{multicols}
\end{exercise}
@ -35,15 +35,15 @@
\begin{solution}
\begin{enumerate}[label={\Alph*=}]
\item $\dfrac{6}{8} + \dfrac{7}{8}=\dfrac{6 + 7}{8}=\dfrac{13}{8} = \dfrac{13}{8}$
\item $\dfrac{9}{5} + 8=\dfrac{9}{5} + \dfrac{8}{1}=\dfrac{9}{5} + \dfrac{8 \times 5}{1 \times 5}=\dfrac{9}{5} + \dfrac{40}{5}=\dfrac{9 + 40}{5}=\dfrac{49}{5} = \dfrac{49}{5}$
\item $\dfrac{2}{5} + \dfrac{10}{35}=\dfrac{2 \times 7}{5 \times 7} + \dfrac{10}{35}=\dfrac{14}{35} + \dfrac{10}{35}=\dfrac{14 + 10}{35}=\dfrac{24}{35} = \dfrac{24}{35}$
\item $\dfrac{6}{3} + \dfrac{8}{6}=\dfrac{6 \times 2}{3 \times 2} + \dfrac{8}{6}=\dfrac{12}{6} + \dfrac{8}{6}=\dfrac{12 + 8}{6}=\dfrac{20}{6} = \dfrac{10}{3}$
\item $\dfrac{3}{8} + \dfrac{7}{8}=\dfrac{3 + 7}{8}=\dfrac{10}{8} = \dfrac{5}{4}$
\item $\dfrac{1}{2} + 2=\dfrac{1}{2} + \dfrac{2}{1}=\dfrac{1}{2} + \dfrac{2 \times 2}{1 \times 2}=\dfrac{1}{2} + \dfrac{4}{2}=\dfrac{1 + 4}{2}=\dfrac{5}{2} = \dfrac{5}{2}$
\item $\dfrac{4}{9} + \dfrac{7}{81}=\dfrac{4 \times 9}{9 \times 9} + \dfrac{7}{81}=\dfrac{36}{81} + \dfrac{7}{81}=\dfrac{36 + 7}{81}=\dfrac{43}{81} = \dfrac{43}{81}$
\item $\dfrac{6}{8} + \dfrac{9}{6}=\dfrac{6 \times 3}{8 \times 3} + \dfrac{9 \times 4}{6 \times 4}=\dfrac{18}{24} + \dfrac{36}{24}=\dfrac{18 + 36}{24}=\dfrac{54}{24} = \dfrac{9}{4}$
\item $\dfrac{7}{4} \times 8=\dfrac{7 \times 8}{4}=\dfrac{56}{4} = 14$
\item $\dfrac{4}{8} \times \dfrac{5}{8}=\dfrac{4 \times 5}{8 \times 8}=\dfrac{20}{64} = \dfrac{5}{16}$
\item $\dfrac{- 6}{2} \times \dfrac{- 2}{10}=\dfrac{- 6(- 2)}{2 \times 10}=\dfrac{12}{20} = \dfrac{3}{5}$
\item $\dfrac{\dfrac{3}{10}}{\dfrac{8}{10}}=\dfrac{3}{10} \times \dfrac{10}{8}=\dfrac{3 \times 10}{10 \times 8}=\dfrac{30}{80} = \dfrac{3}{8}$
\item $\dfrac{9}{4} \times 5=\dfrac{9 \times 5}{4}=\dfrac{45}{4} = \dfrac{45}{4}$
\item $\dfrac{8}{2} \times \dfrac{10}{2}=\dfrac{8 \times 10}{2 \times 2}=\dfrac{80}{4} = 20$
\item $\dfrac{- 4}{7} \times \dfrac{9}{49}=\dfrac{- 4 \times 9}{7 \times 49}=\dfrac{- 36}{343} = \dfrac{- 36}{343}$
\item $\dfrac{\dfrac{4}{7}}{\dfrac{5}{7}}=\dfrac{4}{7} \times \dfrac{7}{5}=\dfrac{4 \times 7}{7 \times 5}=\dfrac{28}{35} = \dfrac{4}{5}$
\end{enumerate}
\end{solution}
@ -52,10 +52,10 @@
Le radar a pris des photos pendant l'été:
\begin{itemize}
\item en juin, il y a eu 45 photos prises dont 20 ratées.
\item en juillet, il y a eu 39 photos réussies et 50 ratées.
\item en août, il y a eu 60 photos dont une proportion de 0.23 de photos ratées.
\item en septembre, il y a eu 15 photos ratées, ce qui correspondait à 26.32\% des photos prises.
\item en juin, il y a eu 49 photos prises dont 24 ratées.
\item en juillet, il y a eu 35 photos réussies et 34 ratées.
\item en août, il y a eu 58 photos dont une proportion de 0.29 de photos ratées.
\item en septembre, il y a eu 6 photos ratées, ce qui correspondait à 11.76\% des photos prises.
\end{itemize}
\begin{enumerate}
@ -87,37 +87,35 @@
\hline
& Juin & Juillet & Août & Septembre & Total\\
\hline
Réussies & 25 & 39 & 46 & 42 & 152\\
Réussies & 25 & 35 & 41 & 45 & 146\\
\hline
Ratées & 20 & 50 & 14 & 15 & 99\\
Ratées & 24 & 34 & 17 & 6 & 81\\
\hline
Total & 45 & 89 & 60 & 57 & 251\\
Total & 49 & 69 & 58 & 51 & 227\\
\hline
\end{tabular}
\end{center}
\item Proportion de photos réussies
\[
\frac{152}{251} = 0.61 = 60\%
\frac{146}{227} = 0.64 = 64\%
\]
\item
\begin{itemize}
\item De juin à juillet
\[
\frac{50 - 20}{20} = \frac{30}{20} = 1.5 = 150\%
\frac{34 - 24}{24} = \frac{10}{24} = 0.42 = 41\%
\]
\item De juillet à août
\[
\frac{14 - 50}{50} = \frac{-36}{50} = -0.72 = -72\%
\frac{17 - 34}{34} = \frac{-17}{34} = -0.5 = -50\%
\]
\item De août à septembre
\[
\frac{15 - 14}{14} = \frac{1}{14} = 0.07 = 7\%
\frac{6 - 17}{17} = \frac{-11}{17} = -0.65 = -64\%
\]
\end{itemize}
\end{enumerate}
\end{solution}
\printsolutionstype{exercise}
\end{document}

View File

@ -17,17 +17,17 @@
Détailler les calculs suivants et donner le résultat sous la forme d'une fraction irréductible.
\begin{multicols}{3}
\begin{enumerate}[label={\Alph*=}]
\item $\dfrac{1}{2} + \dfrac{8}{2}$
\item $\dfrac{3}{8} + 5$
\item $\dfrac{9}{5} + \dfrac{3}{5}$
\item $\dfrac{1}{5} + 6$
\item $\dfrac{9}{7} + \dfrac{8}{63}$
\item $\dfrac{2}{10} + \dfrac{7}{10}$
\item $\dfrac{6}{3} + \dfrac{1}{27}$
\item $\dfrac{5}{7} + \dfrac{2}{6}$
\item $\dfrac{8}{2} \times 10$
\item $\dfrac{9}{2} \times \dfrac{8}{2}$
\item $\dfrac{8}{6} \times 1$
\item $\dfrac{7}{9} \times \dfrac{4}{9}$
\item $\dfrac{- 9}{7} \times \dfrac{8}{14}$
\item $\dfrac{\dfrac{1}{7}}{\dfrac{9}{4}}$
\item $\dfrac{9}{5} \times \dfrac{- 9}{10}$
\item $\dfrac{\dfrac{1}{5}}{\dfrac{2}{10}}$
\end{enumerate}
\end{multicols}
\end{exercise}
@ -35,15 +35,15 @@
\begin{solution}
\begin{enumerate}[label={\Alph*=}]
\item $\dfrac{1}{2} + \dfrac{8}{2}=\dfrac{1 + 8}{2}=\dfrac{9}{2} = \dfrac{9}{2}$
\item $\dfrac{3}{8} + 5=\dfrac{3}{8} + \dfrac{5}{1}=\dfrac{3}{8} + \dfrac{5 \times 8}{1 \times 8}=\dfrac{3}{8} + \dfrac{40}{8}=\dfrac{3 + 40}{8}=\dfrac{43}{8} = \dfrac{43}{8}$
\item $\dfrac{9}{7} + \dfrac{8}{63}=\dfrac{9 \times 9}{7 \times 9} + \dfrac{8}{63}=\dfrac{81}{63} + \dfrac{8}{63}=\dfrac{81 + 8}{63}=\dfrac{89}{63} = \dfrac{89}{63}$
\item $\dfrac{2}{10} + \dfrac{7}{10}=\dfrac{2 + 7}{10}=\dfrac{9}{10} = \dfrac{9}{10}$
\item $\dfrac{9}{5} + \dfrac{3}{5}=\dfrac{9 + 3}{5}=\dfrac{12}{5} = \dfrac{12}{5}$
\item $\dfrac{1}{5} + 6=\dfrac{1}{5} + \dfrac{6}{1}=\dfrac{1}{5} + \dfrac{6 \times 5}{1 \times 5}=\dfrac{1}{5} + \dfrac{30}{5}=\dfrac{1 + 30}{5}=\dfrac{31}{5} = \dfrac{31}{5}$
\item $\dfrac{6}{3} + \dfrac{1}{27}=\dfrac{6 \times 9}{3 \times 9} + \dfrac{1}{27}=\dfrac{54}{27} + \dfrac{1}{27}=\dfrac{54 + 1}{27}=\dfrac{55}{27} = \dfrac{55}{27}$
\item $\dfrac{5}{7} + \dfrac{2}{6}=\dfrac{5 \times 6}{7 \times 6} + \dfrac{2 \times 7}{6 \times 7}=\dfrac{30}{42} + \dfrac{14}{42}=\dfrac{30 + 14}{42}=\dfrac{44}{42} = \dfrac{22}{21}$
\item $\dfrac{8}{2} \times 10=\dfrac{8 \times 10}{2}=\dfrac{80}{2} = 40$
\item $\dfrac{9}{2} \times \dfrac{8}{2}=\dfrac{9 \times 8}{2 \times 2}=\dfrac{72}{4} = 18$
\item $\dfrac{- 9}{7} \times \dfrac{8}{14}=\dfrac{- 9 \times 8}{7 \times 14}=\dfrac{- 72}{98} = \dfrac{- 36}{49}$
\item $\dfrac{\dfrac{1}{7}}{\dfrac{9}{4}}=\dfrac{1}{7} \times \dfrac{4}{9}=\dfrac{1 \times 4}{7 \times 9}=\dfrac{4}{63} = \dfrac{4}{63}$
\item $\dfrac{8}{6} \times 1=\dfrac{8}{6} = \dfrac{4}{3}$
\item $\dfrac{7}{9} \times \dfrac{4}{9}=\dfrac{7 \times 4}{9 \times 9}=\dfrac{28}{81} = \dfrac{28}{81}$
\item $\dfrac{9}{5} \times \dfrac{- 9}{10}=\dfrac{9(- 9)}{5 \times 10}=\dfrac{- 81}{50} = \dfrac{- 81}{50}$
\item $\dfrac{\dfrac{1}{5}}{\dfrac{2}{10}}=\dfrac{1}{5} \times \dfrac{10}{2}=\dfrac{1 \times 10}{5 \times 2}=\dfrac{10}{10} = 1$
\end{enumerate}
\end{solution}
@ -52,10 +52,10 @@
Le radar a pris des photos pendant l'été:
\begin{itemize}
\item en juin, il y a eu 46 photos prises dont 25 ratées.
\item en juillet, il y a eu 45 photos réussies et 33 ratées.
\item en août, il y a eu 66 photos dont une proportion de 0.24 de photos ratées.
\item en septembre, il y a eu 13 photos ratées, ce qui correspondait à 23.21\% des photos prises.
\item en juin, il y a eu 47 photos prises dont 22 ratées.
\item en juillet, il y a eu 32 photos réussies et 30 ratées.
\item en août, il y a eu 69 photos dont une proportion de 0.28 de photos ratées.
\item en septembre, il y a eu 13 photos ratées, ce qui correspondait à 24.07\% des photos prises.
\end{itemize}
\begin{enumerate}
@ -87,37 +87,35 @@
\hline
& Juin & Juillet & Août & Septembre & Total\\
\hline
Réussies & 21 & 45 & 50 & 43 & 159\\
Réussies & 25 & 32 & 50 & 41 & 148\\
\hline
Ratées & 25 & 33 & 16 & 13 & 87\\
Ratées & 22 & 30 & 19 & 13 & 84\\
\hline
Total & 46 & 78 & 66 & 56 & 246\\
Total & 47 & 62 & 69 & 54 & 232\\
\hline
\end{tabular}
\end{center}
\item Proportion de photos réussies
\[
\frac{159}{246} = 0.65 = 64\%
\frac{148}{232} = 0.64 = 63\%
\]
\item
\begin{itemize}
\item De juin à juillet
\[
\frac{33 - 25}{25} = \frac{8}{25} = 0.32 = 32\%
\frac{30 - 22}{22} = \frac{8}{22} = 0.36 = 36\%
\]
\item De juillet à août
\[
\frac{16 - 33}{33} = \frac{-17}{33} = -0.52 = -51\%
\frac{19 - 30}{30} = \frac{-11}{30} = -0.37 = -36\%
\]
\item De août à septembre
\[
\frac{13 - 16}{16} = \frac{-3}{16} = -0.19 = -18\%
\frac{13 - 19}{19} = \frac{-6}{19} = -0.32 = -31\%
\]
\end{itemize}
\end{enumerate}
\end{solution}
\printsolutionstype{exercise}
\end{document}

View File

@ -17,17 +17,17 @@
Détailler les calculs suivants et donner le résultat sous la forme d'une fraction irréductible.
\begin{multicols}{3}
\begin{enumerate}[label={\Alph*=}]
\item $\dfrac{3}{6} + \dfrac{7}{6}$
\item $\dfrac{2}{7} + 3$
\item $\dfrac{8}{4} + \dfrac{5}{4}$
\item $\dfrac{8}{5} + 5$
\item $\dfrac{6}{2} + \dfrac{4}{14}$
\item $\dfrac{4}{8} + \dfrac{2}{3}$
\item $\dfrac{9}{7} + \dfrac{8}{14}$
\item $\dfrac{9}{6} + \dfrac{5}{6}$
\item $\dfrac{4}{8} \times 8$
\item $\dfrac{4}{2} \times \dfrac{7}{2}$
\item $\dfrac{10}{9} \times 4$
\item $\dfrac{9}{4} \times \dfrac{5}{4}$
\item $\dfrac{10}{8} \times \dfrac{- 2}{64}$
\item $\dfrac{\dfrac{4}{6}}{\dfrac{7}{9}}$
\item $\dfrac{- 8}{9} \times \dfrac{- 6}{27}$
\item $\dfrac{\dfrac{9}{8}}{\dfrac{3}{9}}$
\end{enumerate}
\end{multicols}
\end{exercise}
@ -35,15 +35,15 @@
\begin{solution}
\begin{enumerate}[label={\Alph*=}]
\item $\dfrac{3}{6} + \dfrac{7}{6}=\dfrac{3 + 7}{6}=\dfrac{10}{6} = \dfrac{5}{3}$
\item $\dfrac{2}{7} + 3=\dfrac{2}{7} + \dfrac{3}{1}=\dfrac{2}{7} + \dfrac{3 \times 7}{1 \times 7}=\dfrac{2}{7} + \dfrac{21}{7}=\dfrac{2 + 21}{7}=\dfrac{23}{7} = \dfrac{23}{7}$
\item $\dfrac{6}{2} + \dfrac{4}{14}=\dfrac{6 \times 7}{2 \times 7} + \dfrac{4}{14}=\dfrac{42}{14} + \dfrac{4}{14}=\dfrac{42 + 4}{14}=\dfrac{46}{14} = \dfrac{23}{7}$
\item $\dfrac{4}{8} + \dfrac{2}{3}=\dfrac{4 \times 3}{8 \times 3} + \dfrac{2 \times 8}{3 \times 8}=\dfrac{12}{24} + \dfrac{16}{24}=\dfrac{12 + 16}{24}=\dfrac{28}{24} = \dfrac{7}{6}$
\item $\dfrac{8}{4} + \dfrac{5}{4}=\dfrac{8 + 5}{4}=\dfrac{13}{4} = \dfrac{13}{4}$
\item $\dfrac{8}{5} + 5=\dfrac{8}{5} + \dfrac{5}{1}=\dfrac{8}{5} + \dfrac{5 \times 5}{1 \times 5}=\dfrac{8}{5} + \dfrac{25}{5}=\dfrac{8 + 25}{5}=\dfrac{33}{5} = \dfrac{33}{5}$
\item $\dfrac{9}{7} + \dfrac{8}{14}=\dfrac{9 \times 2}{7 \times 2} + \dfrac{8}{14}=\dfrac{18}{14} + \dfrac{8}{14}=\dfrac{18 + 8}{14}=\dfrac{26}{14} = \dfrac{13}{7}$
\item $\dfrac{9}{6} + \dfrac{5}{6}=\dfrac{9 + 5}{6}=\dfrac{14}{6} = \dfrac{7}{3}$
\item $\dfrac{4}{8} \times 8=\dfrac{4 \times 8}{8}=\dfrac{32}{8} = 4$
\item $\dfrac{4}{2} \times \dfrac{7}{2}=\dfrac{4 \times 7}{2 \times 2}=\dfrac{28}{4} = 7$
\item $\dfrac{10}{8} \times \dfrac{- 2}{64}=\dfrac{10(- 2)}{8 \times 64}=\dfrac{- 20}{512} = \dfrac{- 5}{128}$
\item $\dfrac{\dfrac{4}{6}}{\dfrac{7}{9}}=\dfrac{4}{6} \times \dfrac{9}{7}=\dfrac{4 \times 9}{6 \times 7}=\dfrac{36}{42} = \dfrac{6}{7}$
\item $\dfrac{10}{9} \times 4=\dfrac{10 \times 4}{9}=\dfrac{40}{9} = \dfrac{40}{9}$
\item $\dfrac{9}{4} \times \dfrac{5}{4}=\dfrac{9 \times 5}{4 \times 4}=\dfrac{45}{16} = \dfrac{45}{16}$
\item $\dfrac{- 8}{9} \times \dfrac{- 6}{27}=\dfrac{- 8(- 6)}{9 \times 27}=\dfrac{48}{243} = \dfrac{16}{81}$
\item $\dfrac{\dfrac{9}{8}}{\dfrac{3}{9}}=\dfrac{9}{8} \times \dfrac{9}{3}=\dfrac{9 \times 9}{8 \times 3}=\dfrac{81}{24} = \dfrac{27}{8}$
\end{enumerate}
\end{solution}
@ -52,10 +52,10 @@
Le radar a pris des photos pendant l'été:
\begin{itemize}
\item en juin, il y a eu 58 photos prises dont 30 ratées.
\item en juillet, il y a eu 34 photos réussies et 33 ratées.
\item en août, il y a eu 57 photos dont une proportion de 0.19 de photos ratées.
\item en septembre, il y a eu 10 photos ratées, ce qui correspondait à 18.52\% des photos prises.
\item en juin, il y a eu 45 photos prises dont 20 ratées.
\item en juillet, il y a eu 30 photos réussies et 30 ratées.
\item en août, il y a eu 64 photos dont une proportion de 0.27 de photos ratées.
\item en septembre, il y a eu 15 photos ratées, ce qui correspondait à 26.79\% des photos prises.
\end{itemize}
\begin{enumerate}
@ -87,37 +87,35 @@
\hline
& Juin & Juillet & Août & Septembre & Total\\
\hline
Réussies & 28 & 34 & 46 & 44 & 152\\
Réussies & 25 & 30 & 47 & 41 & 143\\
\hline
Ratées & 30 & 33 & 11 & 10 & 84\\
Ratées & 20 & 30 & 17 & 15 & 82\\
\hline
Total & 58 & 67 & 57 & 54 & 236\\
Total & 45 & 60 & 64 & 56 & 225\\
\hline
\end{tabular}
\end{center}
\item Proportion de photos réussies
\[
\frac{152}{236} = 0.64 = 64\%
\frac{143}{225} = 0.64 = 63\%
\]
\item
\begin{itemize}
\item De juin à juillet
\[
\frac{33 - 30}{30} = \frac{3}{30} = 0.1 = 10\%
\frac{30 - 20}{20} = \frac{10}{20} = 0.5 = 50\%
\]
\item De juillet à août
\[
\frac{11 - 33}{33} = \frac{-22}{33} = -0.67 = -66\%
\frac{17 - 30}{30} = \frac{-13}{30} = -0.43 = -43\%
\]
\item De août à septembre
\[
\frac{10 - 11}{11} = \frac{-1}{11} = -0.09 = -9\%
\frac{15 - 17}{17} = \frac{-2}{17} = -0.12 = -11\%
\]
\end{itemize}
\end{enumerate}
\end{solution}
\printsolutionstype{exercise}
\end{document}

View File

@ -17,17 +17,17 @@
Détailler les calculs suivants et donner le résultat sous la forme d'une fraction irréductible.
\begin{multicols}{3}
\begin{enumerate}[label={\Alph*=}]
\item $\dfrac{7}{8} + \dfrac{5}{8}$
\item $\dfrac{1}{5} + 6$
\item $\dfrac{10}{5} + \dfrac{4}{5}$
\item $\dfrac{1}{3} + 6$
\item $\dfrac{3}{7} + \dfrac{4}{35}$
\item $\dfrac{1}{3} + \dfrac{8}{10}$
\item $\dfrac{1}{2} + \dfrac{4}{4}$
\item $\dfrac{8}{3} + \dfrac{6}{9}$
\item $\dfrac{2}{9} \times 6$
\item $\dfrac{7}{5} \times \dfrac{7}{5}$
\item $\dfrac{10}{6} \times 2$
\item $\dfrac{3}{2} \times \dfrac{7}{2}$
\item $\dfrac{- 7}{4} \times \dfrac{- 4}{12}$
\item $\dfrac{\dfrac{8}{7}}{\dfrac{3}{7}}$
\item $\dfrac{- 2}{6} \times \dfrac{10}{36}$
\item $\dfrac{\dfrac{2}{9}}{\dfrac{5}{10}}$
\end{enumerate}
\end{multicols}
\end{exercise}
@ -35,15 +35,15 @@
\begin{solution}
\begin{enumerate}[label={\Alph*=}]
\item $\dfrac{7}{8} + \dfrac{5}{8}=\dfrac{7 + 5}{8}=\dfrac{12}{8} = \dfrac{3}{2}$
\item $\dfrac{1}{5} + 6=\dfrac{1}{5} + \dfrac{6}{1}=\dfrac{1}{5} + \dfrac{6 \times 5}{1 \times 5}=\dfrac{1}{5} + \dfrac{30}{5}=\dfrac{1 + 30}{5}=\dfrac{31}{5} = \dfrac{31}{5}$
\item $\dfrac{3}{7} + \dfrac{4}{35}=\dfrac{3 \times 5}{7 \times 5} + \dfrac{4}{35}=\dfrac{15}{35} + \dfrac{4}{35}=\dfrac{15 + 4}{35}=\dfrac{19}{35} = \dfrac{19}{35}$
\item $\dfrac{1}{3} + \dfrac{8}{10}=\dfrac{1 \times 10}{3 \times 10} + \dfrac{8 \times 3}{10 \times 3}=\dfrac{10}{30} + \dfrac{24}{30}=\dfrac{10 + 24}{30}=\dfrac{34}{30} = \dfrac{17}{15}$
\item $\dfrac{10}{5} + \dfrac{4}{5}=\dfrac{10 + 4}{5}=\dfrac{14}{5} = \dfrac{14}{5}$
\item $\dfrac{1}{3} + 6=\dfrac{1}{3} + \dfrac{6}{1}=\dfrac{1}{3} + \dfrac{6 \times 3}{1 \times 3}=\dfrac{1}{3} + \dfrac{18}{3}=\dfrac{1 + 18}{3}=\dfrac{19}{3} = \dfrac{19}{3}$
\item $\dfrac{1}{2} + \dfrac{4}{4}=\dfrac{1 \times 2}{2 \times 2} + \dfrac{4}{4}=\dfrac{2}{4} + \dfrac{4}{4}=\dfrac{2 + 4}{4}=\dfrac{6}{4} = \dfrac{3}{2}$
\item $\dfrac{8}{3} + \dfrac{6}{9}=\dfrac{8 \times 3}{3 \times 3} + \dfrac{6}{9}=\dfrac{24}{9} + \dfrac{6}{9}=\dfrac{24 + 6}{9}=\dfrac{30}{9} = \dfrac{10}{3}$
\item $\dfrac{2}{9} \times 6=\dfrac{2 \times 6}{9}=\dfrac{12}{9} = \dfrac{4}{3}$
\item $\dfrac{7}{5} \times \dfrac{7}{5}=\dfrac{7 \times 7}{5 \times 5}=\dfrac{49}{25} = \dfrac{49}{25}$
\item $\dfrac{- 7}{4} \times \dfrac{- 4}{12}=\dfrac{- 7(- 4)}{4 \times 12}=\dfrac{28}{48} = \dfrac{7}{12}$
\item $\dfrac{\dfrac{8}{7}}{\dfrac{3}{7}}=\dfrac{8}{7} \times \dfrac{7}{3}=\dfrac{8 \times 7}{7 \times 3}=\dfrac{56}{21} = \dfrac{8}{3}$
\item $\dfrac{10}{6} \times 2=\dfrac{10 \times 2}{6}=\dfrac{20}{6} = \dfrac{10}{3}$
\item $\dfrac{3}{2} \times \dfrac{7}{2}=\dfrac{3 \times 7}{2 \times 2}=\dfrac{21}{4} = \dfrac{21}{4}$
\item $\dfrac{- 2}{6} \times \dfrac{10}{36}=\dfrac{- 2 \times 10}{6 \times 36}=\dfrac{- 20}{216} = \dfrac{- 5}{54}$
\item $\dfrac{\dfrac{2}{9}}{\dfrac{5}{10}}=\dfrac{2}{9} \times \dfrac{10}{5}=\dfrac{2 \times 10}{9 \times 5}=\dfrac{20}{45} = \dfrac{4}{9}$
\end{enumerate}
\end{solution}
@ -52,10 +52,10 @@
Le radar a pris des photos pendant l'été:
\begin{itemize}
\item en juin, il y a eu 46 photos prises dont 21 ratées.
\item en juillet, il y a eu 48 photos réussies et 45 ratées.
\item en août, il y a eu 60 photos dont une proportion de 0.23 de photos ratées.
\item en septembre, il y a eu 10 photos ratées, ce qui correspondait à 16.67\% des photos prises.
\item en juin, il y a eu 48 photos prises dont 22 ratées.
\item en juillet, il y a eu 40 photos réussies et 31 ratées.
\item en août, il y a eu 59 photos dont une proportion de 0.19 de photos ratées.
\item en septembre, il y a eu 6 photos ratées, ce qui correspondait à 11.11\% des photos prises.
\end{itemize}
\begin{enumerate}
@ -87,37 +87,35 @@
\hline
& Juin & Juillet & Août & Septembre & Total\\
\hline
Réussies & 25 & 48 & 46 & 50 & 169\\
Réussies & 26 & 40 & 48 & 48 & 162\\
\hline
Ratées & 21 & 45 & 14 & 10 & 90\\
Ratées & 22 & 31 & 11 & 6 & 70\\
\hline
Total & 46 & 93 & 60 & 60 & 259\\
Total & 48 & 71 & 59 & 54 & 232\\
\hline
\end{tabular}
\end{center}
\item Proportion de photos réussies
\[
\frac{169}{259} = 0.65 = 65\%
\frac{162}{232} = 0.7 = 69\%
\]
\item
\begin{itemize}
\item De juin à juillet
\[
\frac{45 - 21}{21} = \frac{24}{21} = 1.14 = 114\%
\frac{31 - 22}{22} = \frac{9}{22} = 0.41 = 40\%
\]
\item De juillet à août
\[
\frac{14 - 45}{45} = \frac{-31}{45} = -0.69 = -68\%
\frac{11 - 31}{31} = \frac{-20}{31} = -0.65 = -64\%
\]
\item De août à septembre
\[
\frac{10 - 14}{14} = \frac{-4}{14} = -0.29 = -28\%
\frac{6 - 11}{11} = \frac{-5}{11} = -0.45 = -45\%
\]
\end{itemize}
\end{enumerate}
\end{solution}
\printsolutionstype{exercise}
\end{document}

View File

@ -17,17 +17,17 @@
Détailler les calculs suivants et donner le résultat sous la forme d'une fraction irréductible.
\begin{multicols}{3}
\begin{enumerate}[label={\Alph*=}]
\item $\dfrac{5}{4} + \dfrac{6}{4}$
\item $\dfrac{7}{4} + 6$
\item $\dfrac{1}{2} + \dfrac{3}{2}$
\item $\dfrac{4}{6} + 4$
\item $\dfrac{8}{4} + \dfrac{9}{20}$
\item $\dfrac{3}{10} + \dfrac{3}{7}$
\item $\dfrac{6}{5} + \dfrac{7}{10}$
\item $\dfrac{6}{2} + \dfrac{4}{10}$
\item $\dfrac{1}{4} \times 2$
\item $\dfrac{3}{9} \times \dfrac{10}{9}$
\item $\dfrac{6}{2} \times 7$
\item $\dfrac{5}{8} \times \dfrac{4}{8}$
\item $\dfrac{6}{2} \times \dfrac{4}{10}$
\item $\dfrac{\dfrac{8}{9}}{\dfrac{9}{5}}$
\item $\dfrac{- 6}{5} \times \dfrac{4}{15}$
\item $\dfrac{\dfrac{10}{8}}{\dfrac{3}{6}}$
\end{enumerate}
\end{multicols}
\end{exercise}
@ -35,15 +35,15 @@
\begin{solution}
\begin{enumerate}[label={\Alph*=}]
\item $\dfrac{5}{4} + \dfrac{6}{4}=\dfrac{5 + 6}{4}=\dfrac{11}{4} = \dfrac{11}{4}$
\item $\dfrac{7}{4} + 6=\dfrac{7}{4} + \dfrac{6}{1}=\dfrac{7}{4} + \dfrac{6 \times 4}{1 \times 4}=\dfrac{7}{4} + \dfrac{24}{4}=\dfrac{7 + 24}{4}=\dfrac{31}{4} = \dfrac{31}{4}$
\item $\dfrac{8}{4} + \dfrac{9}{20}=\dfrac{8 \times 5}{4 \times 5} + \dfrac{9}{20}=\dfrac{40}{20} + \dfrac{9}{20}=\dfrac{40 + 9}{20}=\dfrac{49}{20} = \dfrac{49}{20}$
\item $\dfrac{3}{10} + \dfrac{3}{7}=\dfrac{3 \times 7}{10 \times 7} + \dfrac{3 \times 10}{7 \times 10}=\dfrac{21}{70} + \dfrac{30}{70}=\dfrac{21 + 30}{70}=\dfrac{51}{70} = \dfrac{51}{70}$
\item $\dfrac{1}{2} + \dfrac{3}{2}=\dfrac{1 + 3}{2}=\dfrac{4}{2} = 2$
\item $\dfrac{4}{6} + 4=\dfrac{4}{6} + \dfrac{4}{1}=\dfrac{4}{6} + \dfrac{4 \times 6}{1 \times 6}=\dfrac{4}{6} + \dfrac{24}{6}=\dfrac{4 + 24}{6}=\dfrac{28}{6} = \dfrac{14}{3}$
\item $\dfrac{6}{5} + \dfrac{7}{10}=\dfrac{6 \times 2}{5 \times 2} + \dfrac{7}{10}=\dfrac{12}{10} + \dfrac{7}{10}=\dfrac{12 + 7}{10}=\dfrac{19}{10} = \dfrac{19}{10}$
\item $\dfrac{6}{2} + \dfrac{4}{10}=\dfrac{6 \times 5}{2 \times 5} + \dfrac{4}{10}=\dfrac{30}{10} + \dfrac{4}{10}=\dfrac{30 + 4}{10}=\dfrac{34}{10} = \dfrac{17}{5}$
\item $\dfrac{1}{4} \times 2=\dfrac{1 \times 2}{4}=\dfrac{2}{4} = \dfrac{1}{2}$
\item $\dfrac{3}{9} \times \dfrac{10}{9}=\dfrac{3 \times 10}{9 \times 9}=\dfrac{30}{81} = \dfrac{10}{27}$
\item $\dfrac{6}{2} \times \dfrac{4}{10}=\dfrac{6 \times 4}{2 \times 10}=\dfrac{24}{20} = \dfrac{6}{5}$
\item $\dfrac{\dfrac{8}{9}}{\dfrac{9}{5}}=\dfrac{8}{9} \times \dfrac{5}{9}=\dfrac{8 \times 5}{9 \times 9}=\dfrac{40}{81} = \dfrac{40}{81}$
\item $\dfrac{6}{2} \times 7=\dfrac{6 \times 7}{2}=\dfrac{42}{2} = 21$
\item $\dfrac{5}{8} \times \dfrac{4}{8}=\dfrac{5 \times 4}{8 \times 8}=\dfrac{20}{64} = \dfrac{5}{16}$
\item $\dfrac{- 6}{5} \times \dfrac{4}{15}=\dfrac{- 6 \times 4}{5 \times 15}=\dfrac{- 24}{75} = \dfrac{- 8}{25}$
\item $\dfrac{\dfrac{10}{8}}{\dfrac{3}{6}}=\dfrac{10}{8} \times \dfrac{6}{3}=\dfrac{10 \times 6}{8 \times 3}=\dfrac{60}{24} = \dfrac{5}{2}$
\end{enumerate}
\end{solution}
@ -52,10 +52,10 @@
Le radar a pris des photos pendant l'été:
\begin{itemize}
\item en juin, il y a eu 55 photos prises dont 27 ratées.
\item en juillet, il y a eu 31 photos réussies et 42 ratées.
\item en août, il y a eu 59 photos dont une proportion de 0.2 de photos ratées.
\item en septembre, il y a eu 11 photos ratées, ce qui correspondait à 21.57\% des photos prises.
\item en juin, il y a eu 45 photos prises dont 22 ratées.
\item en juillet, il y a eu 34 photos réussies et 48 ratées.
\item en août, il y a eu 61 photos dont une proportion de 0.26 de photos ratées.
\item en septembre, il y a eu 12 photos ratées, ce qui correspondait à 22.64\% des photos prises.
\end{itemize}
\begin{enumerate}
@ -87,37 +87,35 @@
\hline
& Juin & Juillet & Août & Septembre & Total\\
\hline
Réussies & 28 & 31 & 47 & 40 & 146\\
Réussies & 23 & 34 & 45 & 41 & 143\\
\hline
Ratées & 27 & 42 & 12 & 11 & 92\\
Ratées & 22 & 48 & 16 & 12 & 98\\
\hline
Total & 55 & 73 & 59 & 51 & 238\\
Total & 45 & 82 & 61 & 53 & 241\\
\hline
\end{tabular}
\end{center}
\item Proportion de photos réussies
\[
\frac{146}{238} = 0.61 = 61\%
\frac{143}{241} = 0.59 = 59\%
\]
\item
\begin{itemize}
\item De juin à juillet
\[
\frac{42 - 27}{27} = \frac{15}{27} = 0.56 = 55\%
\frac{48 - 22}{22} = \frac{26}{22} = 1.18 = 118\%
\]
\item De juillet à août
\[
\frac{12 - 42}{42} = \frac{-30}{42} = -0.71 = -71\%
\frac{16 - 48}{48} = \frac{-32}{48} = -0.67 = -66\%
\]
\item De août à septembre
\[
\frac{11 - 12}{12} = \frac{-1}{12} = -0.08 = -8\%
\frac{12 - 16}{16} = \frac{-4}{16} = -0.25 = -25\%
\]
\end{itemize}
\end{enumerate}
\end{solution}
\printsolutionstype{exercise}
\end{document}

View File

@ -17,17 +17,17 @@
Détailler les calculs suivants et donner le résultat sous la forme d'une fraction irréductible.
\begin{multicols}{3}
\begin{enumerate}[label={\Alph*=}]
\item $\dfrac{4}{8} + \dfrac{7}{8}$
\item $\dfrac{4}{2} + 6$
\item $\dfrac{4}{2} + \dfrac{5}{2}$
\item $\dfrac{6}{8} + 1$
\item $\dfrac{8}{7} + \dfrac{5}{49}$
\item $\dfrac{4}{9} + \dfrac{7}{9}$
\item $\dfrac{9}{10} + \dfrac{1}{10}$
\item $\dfrac{1}{8} + \dfrac{3}{7}$
\item $\dfrac{6}{2} \times 3$
\item $\dfrac{2}{10} \times \dfrac{9}{10}$
\item $\dfrac{7}{2} \times 7$
\item $\dfrac{1}{7} \times \dfrac{9}{7}$
\item $\dfrac{- 4}{4} \times \dfrac{- 9}{36}$
\item $\dfrac{\dfrac{6}{7}}{\dfrac{10}{6}}$
\item $\dfrac{- 2}{7} \times \dfrac{- 8}{42}$
\item $\dfrac{\dfrac{8}{4}}{\dfrac{5}{4}}$
\end{enumerate}
\end{multicols}
\end{exercise}
@ -35,15 +35,15 @@
\begin{solution}
\begin{enumerate}[label={\Alph*=}]
\item $\dfrac{4}{8} + \dfrac{7}{8}=\dfrac{4 + 7}{8}=\dfrac{11}{8} = \dfrac{11}{8}$
\item $\dfrac{4}{2} + 6=\dfrac{4}{2} + \dfrac{6}{1}=\dfrac{4}{2} + \dfrac{6 \times 2}{1 \times 2}=\dfrac{4}{2} + \dfrac{12}{2}=\dfrac{4 + 12}{2}=\dfrac{16}{2} = 8$
\item $\dfrac{8}{7} + \dfrac{5}{49}=\dfrac{8 \times 7}{7 \times 7} + \dfrac{5}{49}=\dfrac{56}{49} + \dfrac{5}{49}=\dfrac{56 + 5}{49}=\dfrac{61}{49} = \dfrac{61}{49}$
\item $\dfrac{4}{9} + \dfrac{7}{9}=\dfrac{4 + 7}{9}=\dfrac{11}{9} = \dfrac{11}{9}$
\item $\dfrac{4}{2} + \dfrac{5}{2}=\dfrac{4 + 5}{2}=\dfrac{9}{2} = \dfrac{9}{2}$
\item $\dfrac{6}{8} + 1=\dfrac{6}{8} + \dfrac{1}{1}=\dfrac{6}{8} + \dfrac{1 \times 8}{1 \times 8}=\dfrac{6}{8} + \dfrac{8}{8}=\dfrac{6 + 8}{8}=\dfrac{14}{8} = \dfrac{7}{4}$
\item $\dfrac{9}{10} + \dfrac{1}{10}=\dfrac{9 + 1}{10}=\dfrac{10}{10} = 1$
\item $\dfrac{1}{8} + \dfrac{3}{7}=\dfrac{1 \times 7}{8 \times 7} + \dfrac{3 \times 8}{7 \times 8}=\dfrac{7}{56} + \dfrac{24}{56}=\dfrac{7 + 24}{56}=\dfrac{31}{56} = \dfrac{31}{56}$
\item $\dfrac{6}{2} \times 3=\dfrac{6 \times 3}{2}=\dfrac{18}{2} = 9$
\item $\dfrac{2}{10} \times \dfrac{9}{10}=\dfrac{2 \times 9}{10 \times 10}=\dfrac{18}{100} = \dfrac{9}{50}$
\item $\dfrac{- 4}{4} \times \dfrac{- 9}{36}=\dfrac{- 4(- 9)}{4 \times 36}=\dfrac{36}{144} = \dfrac{1}{4}$
\item $\dfrac{\dfrac{6}{7}}{\dfrac{10}{6}}=\dfrac{6}{7} \times \dfrac{6}{10}=\dfrac{6 \times 6}{7 \times 10}=\dfrac{36}{70} = \dfrac{18}{35}$
\item $\dfrac{7}{2} \times 7=\dfrac{7 \times 7}{2}=\dfrac{49}{2} = \dfrac{49}{2}$
\item $\dfrac{1}{7} \times \dfrac{9}{7}=\dfrac{1 \times 9}{7 \times 7}=\dfrac{9}{49} = \dfrac{9}{49}$
\item $\dfrac{- 2}{7} \times \dfrac{- 8}{42}=\dfrac{- 2(- 8)}{7 \times 42}=\dfrac{16}{294} = \dfrac{8}{147}$
\item $\dfrac{\dfrac{8}{4}}{\dfrac{5}{4}}=\dfrac{8}{4} \times \dfrac{4}{5}=\dfrac{8 \times 4}{4 \times 5}=\dfrac{32}{20} = \dfrac{8}{5}$
\end{enumerate}
\end{solution}
@ -52,10 +52,10 @@
Le radar a pris des photos pendant l'été:
\begin{itemize}
\item en juin, il y a eu 48 photos prises dont 27 ratées.
\item en juillet, il y a eu 39 photos réussies et 45 ratées.
\item en août, il y a eu 59 photos dont une proportion de 0.27 de photos ratées.
\item en septembre, il y a eu 5 photos ratées, ce qui correspondait à 10.87\% des photos prises.
\item en juin, il y a eu 47 photos prises dont 23 ratées.
\item en juillet, il y a eu 46 photos réussies et 45 ratées.
\item en août, il y a eu 65 photos dont une proportion de 0.31 de photos ratées.
\item en septembre, il y a eu 14 photos ratées, ce qui correspondait à 23.73\% des photos prises.
\end{itemize}
\begin{enumerate}
@ -87,37 +87,35 @@
\hline
& Juin & Juillet & Août & Septembre & Total\\
\hline
Réussies & 21 & 39 & 43 & 41 & 144\\
Réussies & 24 & 46 & 45 & 45 & 160\\
\hline
Ratées & 27 & 45 & 16 & 5 & 93\\
Ratées & 23 & 45 & 20 & 14 & 102\\
\hline
Total & 48 & 84 & 59 & 46 & 237\\
Total & 47 & 91 & 65 & 59 & 262\\
\hline
\end{tabular}
\end{center}
\item Proportion de photos réussies
\[
\frac{144}{237} = 0.61 = 60\%
\frac{160}{262} = 0.61 = 61\%
\]
\item
\begin{itemize}
\item De juin à juillet
\[
\frac{45 - 27}{27} = \frac{18}{27} = 0.67 = 66\%
\frac{45 - 23}{23} = \frac{22}{23} = 0.96 = 95\%
\]
\item De juillet à août
\[
\frac{16 - 45}{45} = \frac{-29}{45} = -0.64 = -64\%
\frac{20 - 45}{45} = \frac{-25}{45} = -0.56 = -55\%
\]
\item De août à septembre
\[
\frac{5 - 16}{16} = \frac{-11}{16} = -0.69 = -68\%
\frac{14 - 20}{20} = \frac{-6}{20} = -0.3 = -30\%
\]
\end{itemize}
\end{enumerate}
\end{solution}
\printsolutionstype{exercise}
\end{document}

View File

@ -17,17 +17,17 @@
Détailler les calculs suivants et donner le résultat sous la forme d'une fraction irréductible.
\begin{multicols}{3}
\begin{enumerate}[label={\Alph*=}]
\item $\dfrac{10}{8} + \dfrac{2}{8}$
\item $\dfrac{5}{10} + 3$
\item $\dfrac{4}{10} + \dfrac{5}{10}$
\item $\dfrac{7}{3} + 7$
\item $\dfrac{1}{9} + \dfrac{1}{90}$
\item $\dfrac{7}{9} + \dfrac{9}{4}$
\item $\dfrac{5}{2} + \dfrac{10}{2}$
\item $\dfrac{3}{6} + \dfrac{5}{6}$
\item $\dfrac{7}{6} \times 9$
\item $\dfrac{1}{9} \times \dfrac{4}{9}$
\item $\dfrac{7}{10} \times 7$
\item $\dfrac{5}{3} \times \dfrac{1}{3}$
\item $\dfrac{7}{3} \times \dfrac{- 2}{21}$
\item $\dfrac{\dfrac{4}{7}}{\dfrac{5}{9}}$
\item $\dfrac{4}{8} \times \dfrac{- 6}{64}$
\item $\dfrac{\dfrac{4}{6}}{\dfrac{7}{6}}$
\end{enumerate}
\end{multicols}
\end{exercise}
@ -35,15 +35,15 @@
\begin{solution}
\begin{enumerate}[label={\Alph*=}]
\item $\dfrac{10}{8} + \dfrac{2}{8}=\dfrac{10 + 2}{8}=\dfrac{12}{8} = \dfrac{3}{2}$
\item $\dfrac{5}{10} + 3=\dfrac{5}{10} + \dfrac{3}{1}=\dfrac{5}{10} + \dfrac{3 \times 10}{1 \times 10}=\dfrac{5}{10} + \dfrac{30}{10}=\dfrac{5 + 30}{10}=\dfrac{35}{10} = \dfrac{7}{2}$
\item $\dfrac{1}{9} + \dfrac{1}{90}=\dfrac{1 \times 10}{9 \times 10} + \dfrac{1}{90}=\dfrac{10}{90} + \dfrac{1}{90}=\dfrac{10 + 1}{90}=\dfrac{11}{90} = \dfrac{11}{90}$
\item $\dfrac{7}{9} + \dfrac{9}{4}=\dfrac{7 \times 4}{9 \times 4} + \dfrac{9 \times 9}{4 \times 9}=\dfrac{28}{36} + \dfrac{81}{36}=\dfrac{28 + 81}{36}=\dfrac{109}{36} = \dfrac{109}{36}$
\item $\dfrac{4}{10} + \dfrac{5}{10}=\dfrac{4 + 5}{10}=\dfrac{9}{10} = \dfrac{9}{10}$
\item $\dfrac{7}{3} + 7=\dfrac{7}{3} + \dfrac{7}{1}=\dfrac{7}{3} + \dfrac{7 \times 3}{1 \times 3}=\dfrac{7}{3} + \dfrac{21}{3}=\dfrac{7 + 21}{3}=\dfrac{28}{3} = \dfrac{28}{3}$
\item $\dfrac{5}{2} + \dfrac{10}{2}=\dfrac{5 + 10}{2}=\dfrac{15}{2} = \dfrac{15}{2}$
\item $\dfrac{3}{6} + \dfrac{5}{6}=\dfrac{3 + 5}{6}=\dfrac{8}{6} = \dfrac{4}{3}$
\item $\dfrac{7}{6} \times 9=\dfrac{7 \times 9}{6}=\dfrac{63}{6} = \dfrac{21}{2}$
\item $\dfrac{1}{9} \times \dfrac{4}{9}=\dfrac{1 \times 4}{9 \times 9}=\dfrac{4}{81} = \dfrac{4}{81}$
\item $\dfrac{7}{3} \times \dfrac{- 2}{21}=\dfrac{7(- 2)}{3 \times 21}=\dfrac{- 14}{63} = \dfrac{- 2}{9}$
\item $\dfrac{\dfrac{4}{7}}{\dfrac{5}{9}}=\dfrac{4}{7} \times \dfrac{9}{5}=\dfrac{4 \times 9}{7 \times 5}=\dfrac{36}{35} = \dfrac{36}{35}$
\item $\dfrac{7}{10} \times 7=\dfrac{7 \times 7}{10}=\dfrac{49}{10} = \dfrac{49}{10}$
\item $\dfrac{5}{3} \times \dfrac{1}{3}=\dfrac{5 \times 1}{3 \times 3}=\dfrac{5}{9} = \dfrac{5}{9}$
\item $\dfrac{4}{8} \times \dfrac{- 6}{64}=\dfrac{4(- 6)}{8 \times 64}=\dfrac{- 24}{512} = \dfrac{- 3}{64}$
\item $\dfrac{\dfrac{4}{6}}{\dfrac{7}{6}}=\dfrac{4}{6} \times \dfrac{6}{7}=\dfrac{4 \times 6}{6 \times 7}=\dfrac{24}{42} = \dfrac{4}{7}$
\end{enumerate}
\end{solution}
@ -52,10 +52,10 @@
Le radar a pris des photos pendant l'été:
\begin{itemize}
\item en juin, il y a eu 51 photos prises dont 24 ratées.
\item en juillet, il y a eu 40 photos réussies et 49 ratées.
\item en août, il y a eu 61 photos dont une proportion de 0.28 de photos ratées.
\item en septembre, il y a eu 10 photos ratées, ce qui correspondait à 18.18\% des photos prises.
\item en juin, il y a eu 46 photos prises dont 20 ratées.
\item en juillet, il y a eu 46 photos réussies et 41 ratées.
\item en août, il y a eu 62 photos dont une proportion de 0.24 de photos ratées.
\item en septembre, il y a eu 6 photos ratées, ce qui correspondait à 12.5\% des photos prises.
\end{itemize}
\begin{enumerate}
@ -87,37 +87,35 @@
\hline
& Juin & Juillet & Août & Septembre & Total\\
\hline
Réussies & 27 & 40 & 44 & 45 & 156\\
Réussies & 26 & 46 & 47 & 42 & 161\\
\hline
Ratées & 24 & 49 & 17 & 10 & 100\\
Ratées & 20 & 41 & 15 & 6 & 82\\
\hline
Total & 51 & 89 & 61 & 55 & 256\\
Total & 46 & 87 & 62 & 48 & 243\\
\hline
\end{tabular}
\end{center}
\item Proportion de photos réussies
\[
\frac{156}{256} = 0.61 = 60\%
\frac{161}{243} = 0.66 = 66\%
\]
\item
\begin{itemize}
\item De juin à juillet
\[
\frac{49 - 24}{24} = \frac{25}{24} = 1.04 = 104\%
\frac{41 - 20}{20} = \frac{21}{20} = 1.05 = 105\%
\]
\item De juillet à août
\[
\frac{17 - 49}{49} = \frac{-32}{49} = -0.65 = -65\%
\frac{15 - 41}{41} = \frac{-26}{41} = -0.63 = -63\%
\]
\item De août à septembre
\[
\frac{10 - 17}{17} = \frac{-7}{17} = -0.41 = -41\%
\frac{6 - 15}{15} = \frac{-9}{15} = -0.6 = -60\%
\]
\end{itemize}
\end{enumerate}
\end{solution}
\printsolutionstype{exercise}
\end{document}

View File

@ -17,17 +17,17 @@
Détailler les calculs suivants et donner le résultat sous la forme d'une fraction irréductible.
\begin{multicols}{3}
\begin{enumerate}[label={\Alph*=}]
\item $\dfrac{6}{2} + \dfrac{7}{2}$
\item $\dfrac{4}{7} + 2$
\item $\dfrac{9}{5} + \dfrac{6}{5}$
\item $\dfrac{7}{10} + 1$
\item $\dfrac{7}{2} + \dfrac{1}{16}$
\item $\dfrac{8}{7} + \dfrac{4}{7}$
\item $\dfrac{3}{6} + \dfrac{4}{24}$
\item $\dfrac{1}{4} + \dfrac{5}{10}$
\item $\dfrac{5}{4} \times 10$
\item $\dfrac{7}{8} \times \dfrac{4}{8}$
\item $\dfrac{8}{3} \times 9$
\item $\dfrac{6}{5} \times \dfrac{10}{5}$
\item $\dfrac{3}{4} \times \dfrac{- 2}{12}$
\item $\dfrac{\dfrac{4}{10}}{\dfrac{10}{4}}$
\item $\dfrac{- 3}{6} \times \dfrac{- 6}{24}$
\item $\dfrac{\dfrac{1}{9}}{\dfrac{5}{7}}$
\end{enumerate}
\end{multicols}
\end{exercise}
@ -35,15 +35,15 @@
\begin{solution}
\begin{enumerate}[label={\Alph*=}]
\item $\dfrac{6}{2} + \dfrac{7}{2}=\dfrac{6 + 7}{2}=\dfrac{13}{2} = \dfrac{13}{2}$
\item $\dfrac{4}{7} + 2=\dfrac{4}{7} + \dfrac{2}{1}=\dfrac{4}{7} + \dfrac{2 \times 7}{1 \times 7}=\dfrac{4}{7} + \dfrac{14}{7}=\dfrac{4 + 14}{7}=\dfrac{18}{7} = \dfrac{18}{7}$
\item $\dfrac{7}{2} + \dfrac{1}{16}=\dfrac{7 \times 8}{2 \times 8} + \dfrac{1}{16}=\dfrac{56}{16} + \dfrac{1}{16}=\dfrac{56 + 1}{16}=\dfrac{57}{16} = \dfrac{57}{16}$
\item $\dfrac{8}{7} + \dfrac{4}{7}=\dfrac{8 + 4}{7}=\dfrac{12}{7} = \dfrac{12}{7}$
\item $\dfrac{9}{5} + \dfrac{6}{5}=\dfrac{9 + 6}{5}=\dfrac{15}{5} = 3$
\item $\dfrac{7}{10} + 1=\dfrac{7}{10} + \dfrac{1}{1}=\dfrac{7}{10} + \dfrac{1 \times 10}{1 \times 10}=\dfrac{7}{10} + \dfrac{10}{10}=\dfrac{7 + 10}{10}=\dfrac{17}{10} = \dfrac{17}{10}$
\item $\dfrac{3}{6} + \dfrac{4}{24}=\dfrac{3 \times 4}{6 \times 4} + \dfrac{4}{24}=\dfrac{12}{24} + \dfrac{4}{24}=\dfrac{12 + 4}{24}=\dfrac{16}{24} = \dfrac{2}{3}$
\item $\dfrac{1}{4} + \dfrac{5}{10}=\dfrac{1 \times 5}{4 \times 5} + \dfrac{5 \times 2}{10 \times 2}=\dfrac{5}{20} + \dfrac{10}{20}=\dfrac{5 + 10}{20}=\dfrac{15}{20} = \dfrac{3}{4}$
\item $\dfrac{5}{4} \times 10=\dfrac{5 \times 10}{4}=\dfrac{50}{4} = \dfrac{25}{2}$
\item $\dfrac{7}{8} \times \dfrac{4}{8}=\dfrac{7 \times 4}{8 \times 8}=\dfrac{28}{64} = \dfrac{7}{16}$
\item $\dfrac{3}{4} \times \dfrac{- 2}{12}=\dfrac{3(- 2)}{4 \times 12}=\dfrac{- 6}{48} = \dfrac{- 1}{8}$
\item $\dfrac{\dfrac{4}{10}}{\dfrac{10}{4}}=\dfrac{4}{10} \times \dfrac{4}{10}=\dfrac{4 \times 4}{10 \times 10}=\dfrac{16}{100} = \dfrac{4}{25}$
\item $\dfrac{8}{3} \times 9=\dfrac{8 \times 9}{3}=\dfrac{72}{3} = 24$
\item $\dfrac{6}{5} \times \dfrac{10}{5}=\dfrac{6 \times 10}{5 \times 5}=\dfrac{60}{25} = \dfrac{12}{5}$
\item $\dfrac{- 3}{6} \times \dfrac{- 6}{24}=\dfrac{- 3(- 6)}{6 \times 24}=\dfrac{18}{144} = \dfrac{1}{8}$
\item $\dfrac{\dfrac{1}{9}}{\dfrac{5}{7}}=\dfrac{1}{9} \times \dfrac{7}{5}=\dfrac{1 \times 7}{9 \times 5}=\dfrac{7}{45} = \dfrac{7}{45}$
\end{enumerate}
\end{solution}
@ -52,10 +52,10 @@
Le radar a pris des photos pendant l'été:
\begin{itemize}
\item en juin, il y a eu 47 photos prises dont 26 ratées.
\item en juillet, il y a eu 31 photos réussies et 37 ratées.
\item en juin, il y a eu 52 photos prises dont 29 ratées.
\item en juillet, il y a eu 40 photos réussies et 37 ratées.
\item en août, il y a eu 60 photos dont une proportion de 0.32 de photos ratées.
\item en septembre, il y a eu 7 photos ratées, ce qui correspondait à 14.89\% des photos prises.
\item en septembre, il y a eu 9 photos ratées, ce qui correspondait à 15.79\% des photos prises.
\end{itemize}
\begin{enumerate}
@ -87,23 +87,23 @@
\hline
& Juin & Juillet & Août & Septembre & Total\\
\hline
Réussies & 21 & 31 & 41 & 40 & 133\\
Réussies & 23 & 40 & 41 & 48 & 152\\
\hline
Ratées & 26 & 37 & 19 & 7 & 89\\
Ratées & 29 & 37 & 19 & 9 & 94\\
\hline
Total & 47 & 68 & 60 & 47 & 222\\
Total & 52 & 77 & 60 & 57 & 246\\
\hline
\end{tabular}
\end{center}
\item Proportion de photos réussies
\[
\frac{133}{222} = 0.6 = 59\%
\frac{152}{246} = 0.62 = 61\%
\]
\item
\begin{itemize}
\item De juin à juillet
\[
\frac{37 - 26}{26} = \frac{11}{26} = 0.42 = 42\%
\frac{37 - 29}{29} = \frac{8}{29} = 0.28 = 27\%
\]
\item De juillet à août
\[
@ -111,13 +111,11 @@
\]
\item De août à septembre
\[
\frac{7 - 19}{19} = \frac{-12}{19} = -0.63 = -63\%
\frac{9 - 19}{19} = \frac{-10}{19} = -0.53 = -52\%
\]
\end{itemize}
\end{enumerate}
\end{solution}
\printsolutionstype{exercise}
\end{document}

View File

@ -17,17 +17,17 @@
Détailler les calculs suivants et donner le résultat sous la forme d'une fraction irréductible.
\begin{multicols}{3}
\begin{enumerate}[label={\Alph*=}]
\item $\dfrac{1}{4} + \dfrac{3}{4}$
\item $\dfrac{6}{7} + 3$
\item $\dfrac{7}{10} + \dfrac{4}{10}$
\item $\dfrac{3}{4} + 8$
\item $\dfrac{8}{10} + \dfrac{8}{70}$
\item $\dfrac{7}{8} + \dfrac{7}{9}$
\item $\dfrac{1}{2} + \dfrac{10}{6}$
\item $\dfrac{2}{3} + \dfrac{8}{2}$
\item $\dfrac{9}{5} \times 2$
\item $\dfrac{6}{2} \times \dfrac{10}{2}$
\item $\dfrac{9}{10} \times 4$
\item $\dfrac{9}{6} \times \dfrac{7}{6}$
\item $\dfrac{- 8}{4} \times \dfrac{7}{16}$
\item $\dfrac{\dfrac{9}{7}}{\dfrac{4}{3}}$
\item $\dfrac{- 9}{4} \times \dfrac{8}{12}$
\item $\dfrac{\dfrac{5}{2}}{\dfrac{7}{9}}$
\end{enumerate}
\end{multicols}
\end{exercise}
@ -35,15 +35,15 @@
\begin{solution}
\begin{enumerate}[label={\Alph*=}]
\item $\dfrac{1}{4} + \dfrac{3}{4}=\dfrac{1 + 3}{4}=\dfrac{4}{4} = 1$
\item $\dfrac{6}{7} + 3=\dfrac{6}{7} + \dfrac{3}{1}=\dfrac{6}{7} + \dfrac{3 \times 7}{1 \times 7}=\dfrac{6}{7} + \dfrac{21}{7}=\dfrac{6 + 21}{7}=\dfrac{27}{7} = \dfrac{27}{7}$
\item $\dfrac{8}{10} + \dfrac{8}{70}=\dfrac{8 \times 7}{10 \times 7} + \dfrac{8}{70}=\dfrac{56}{70} + \dfrac{8}{70}=\dfrac{56 + 8}{70}=\dfrac{64}{70} = \dfrac{32}{35}$
\item $\dfrac{7}{8} + \dfrac{7}{9}=\dfrac{7 \times 9}{8 \times 9} + \dfrac{7 \times 8}{9 \times 8}=\dfrac{63}{72} + \dfrac{56}{72}=\dfrac{63 + 56}{72}=\dfrac{119}{72} = \dfrac{119}{72}$
\item $\dfrac{7}{10} + \dfrac{4}{10}=\dfrac{7 + 4}{10}=\dfrac{11}{10} = \dfrac{11}{10}$
\item $\dfrac{3}{4} + 8=\dfrac{3}{4} + \dfrac{8}{1}=\dfrac{3}{4} + \dfrac{8 \times 4}{1 \times 4}=\dfrac{3}{4} + \dfrac{32}{4}=\dfrac{3 + 32}{4}=\dfrac{35}{4} = \dfrac{35}{4}$
\item $\dfrac{1}{2} + \dfrac{10}{6}=\dfrac{1 \times 3}{2 \times 3} + \dfrac{10}{6}=\dfrac{3}{6} + \dfrac{10}{6}=\dfrac{3 + 10}{6}=\dfrac{13}{6} = \dfrac{13}{6}$
\item $\dfrac{2}{3} + \dfrac{8}{2}=\dfrac{2 \times 2}{3 \times 2} + \dfrac{8 \times 3}{2 \times 3}=\dfrac{4}{6} + \dfrac{24}{6}=\dfrac{4 + 24}{6}=\dfrac{28}{6} = \dfrac{14}{3}$
\item $\dfrac{9}{5} \times 2=\dfrac{9 \times 2}{5}=\dfrac{18}{5} = \dfrac{18}{5}$
\item $\dfrac{6}{2} \times \dfrac{10}{2}=\dfrac{6 \times 10}{2 \times 2}=\dfrac{60}{4} = 15$
\item $\dfrac{- 8}{4} \times \dfrac{7}{16}=\dfrac{- 8 \times 7}{4 \times 16}=\dfrac{- 56}{64} = \dfrac{- 7}{8}$
\item $\dfrac{\dfrac{9}{7}}{\dfrac{4}{3}}=\dfrac{9}{7} \times \dfrac{3}{4}=\dfrac{9 \times 3}{7 \times 4}=\dfrac{27}{28} = \dfrac{27}{28}$
\item $\dfrac{9}{10} \times 4=\dfrac{9 \times 4}{10}=\dfrac{36}{10} = \dfrac{18}{5}$
\item $\dfrac{9}{6} \times \dfrac{7}{6}=\dfrac{9 \times 7}{6 \times 6}=\dfrac{63}{36} = \dfrac{7}{4}$
\item $\dfrac{- 9}{4} \times \dfrac{8}{12}=\dfrac{- 9 \times 8}{4 \times 12}=\dfrac{- 72}{48} = \dfrac{- 3}{2}$
\item $\dfrac{\dfrac{5}{2}}{\dfrac{7}{9}}=\dfrac{5}{2} \times \dfrac{9}{7}=\dfrac{5 \times 9}{2 \times 7}=\dfrac{45}{14} = \dfrac{45}{14}$
\end{enumerate}
\end{solution}
@ -52,10 +52,10 @@
Le radar a pris des photos pendant l'été:
\begin{itemize}
\item en juin, il y a eu 48 photos prises dont 27 ratées.
\item en juillet, il y a eu 31 photos réussies et 32 ratées.
\item en août, il y a eu 61 photos dont une proportion de 0.3 de photos ratées.
\item en septembre, il y a eu 9 photos ratées, ce qui correspondait à 18.37\% des photos prises.
\item en juin, il y a eu 47 photos prises dont 27 ratées.
\item en juillet, il y a eu 47 photos réussies et 31 ratées.
\item en août, il y a eu 62 photos dont une proportion de 0.26 de photos ratées.
\item en septembre, il y a eu 11 photos ratées, ce qui correspondait à 20.0\% des photos prises.
\end{itemize}
\begin{enumerate}
@ -87,37 +87,35 @@
\hline
& Juin & Juillet & Août & Septembre & Total\\
\hline
Réussies & 21 & 31 & 43 & 40 & 135\\
Réussies & 20 & 47 & 46 & 44 & 157\\
\hline
Ratées & 27 & 32 & 18 & 9 & 86\\
Ratées & 27 & 31 & 16 & 11 & 85\\
\hline
Total & 48 & 63 & 61 & 49 & 221\\
Total & 47 & 78 & 62 & 55 & 242\\
\hline
\end{tabular}
\end{center}
\item Proportion de photos réussies
\[
\frac{135}{221} = 0.61 = 61\%
\frac{157}{242} = 0.65 = 64\%
\]
\item
\begin{itemize}
\item De juin à juillet
\[
\frac{32 - 27}{27} = \frac{5}{27} = 0.19 = 18\%
\frac{31 - 27}{27} = \frac{4}{27} = 0.15 = 14\%
\]
\item De juillet à août
\[
\frac{18 - 32}{32} = \frac{-14}{32} = -0.44 = -43\%
\frac{16 - 31}{31} = \frac{-15}{31} = -0.48 = -48\%
\]
\item De août à septembre
\[
\frac{9 - 18}{18} = \frac{-9}{18} = -0.5 = -50\%
\frac{11 - 16}{16} = \frac{-5}{16} = -0.31 = -31\%
\]
\end{itemize}
\end{enumerate}
\end{solution}
\printsolutionstype{exercise}
\end{document}

View File

@ -17,17 +17,17 @@
Détailler les calculs suivants et donner le résultat sous la forme d'une fraction irréductible.
\begin{multicols}{3}
\begin{enumerate}[label={\Alph*=}]
\item $\dfrac{2}{6} + \dfrac{8}{6}$
\item $\dfrac{6}{3} + 5$
\item $\dfrac{2}{4} + \dfrac{3}{4}$
\item $\dfrac{5}{4} + 10$
\item $\dfrac{1}{9} + \dfrac{6}{36}$
\item $\dfrac{6}{10} + \dfrac{8}{6}$
\item $\dfrac{2}{7} + \dfrac{10}{42}$
\item $\dfrac{7}{8} + \dfrac{6}{10}$
\item $\dfrac{1}{4} \times 4$
\item $\dfrac{3}{10} \times \dfrac{3}{10}$
\item $\dfrac{5}{2} \times 8$
\item $\dfrac{2}{10} \times \dfrac{5}{10}$
\item $\dfrac{- 8}{6} \times \dfrac{2}{24}$
\item $\dfrac{\dfrac{4}{10}}{\dfrac{2}{10}}$
\item $\dfrac{- 1}{7} \times \dfrac{- 2}{70}$
\item $\dfrac{\dfrac{1}{7}}{\dfrac{7}{3}}$
\end{enumerate}
\end{multicols}
\end{exercise}
@ -35,15 +35,15 @@
\begin{solution}
\begin{enumerate}[label={\Alph*=}]
\item $\dfrac{2}{6} + \dfrac{8}{6}=\dfrac{2 + 8}{6}=\dfrac{10}{6} = \dfrac{5}{3}$
\item $\dfrac{6}{3} + 5=\dfrac{6}{3} + \dfrac{5}{1}=\dfrac{6}{3} + \dfrac{5 \times 3}{1 \times 3}=\dfrac{6}{3} + \dfrac{15}{3}=\dfrac{6 + 15}{3}=\dfrac{21}{3} = 7$
\item $\dfrac{1}{9} + \dfrac{6}{36}=\dfrac{1 \times 4}{9 \times 4} + \dfrac{6}{36}=\dfrac{4}{36} + \dfrac{6}{36}=\dfrac{4 + 6}{36}=\dfrac{10}{36} = \dfrac{5}{18}$
\item $\dfrac{6}{10} + \dfrac{8}{6}=\dfrac{6 \times 3}{10 \times 3} + \dfrac{8 \times 5}{6 \times 5}=\dfrac{18}{30} + \dfrac{40}{30}=\dfrac{18 + 40}{30}=\dfrac{58}{30} = \dfrac{29}{15}$
\item $\dfrac{2}{4} + \dfrac{3}{4}=\dfrac{2 + 3}{4}=\dfrac{5}{4} = \dfrac{5}{4}$
\item $\dfrac{5}{4} + 10=\dfrac{5}{4} + \dfrac{10}{1}=\dfrac{5}{4} + \dfrac{10 \times 4}{1 \times 4}=\dfrac{5}{4} + \dfrac{40}{4}=\dfrac{5 + 40}{4}=\dfrac{45}{4} = \dfrac{45}{4}$
\item $\dfrac{2}{7} + \dfrac{10}{42}=\dfrac{2 \times 6}{7 \times 6} + \dfrac{10}{42}=\dfrac{12}{42} + \dfrac{10}{42}=\dfrac{12 + 10}{42}=\dfrac{22}{42} = \dfrac{11}{21}$
\item $\dfrac{7}{8} + \dfrac{6}{10}=\dfrac{7 \times 5}{8 \times 5} + \dfrac{6 \times 4}{10 \times 4}=\dfrac{35}{40} + \dfrac{24}{40}=\dfrac{35 + 24}{40}=\dfrac{59}{40} = \dfrac{59}{40}$
\item $\dfrac{1}{4} \times 4=\dfrac{1 \times 4}{4}=\dfrac{4}{4} = 1$
\item $\dfrac{3}{10} \times \dfrac{3}{10}=\dfrac{3 \times 3}{10 \times 10}=\dfrac{9}{100} = \dfrac{9}{100}$
\item $\dfrac{- 8}{6} \times \dfrac{2}{24}=\dfrac{- 8 \times 2}{6 \times 24}=\dfrac{- 16}{144} = \dfrac{- 1}{9}$
\item $\dfrac{\dfrac{4}{10}}{\dfrac{2}{10}}=\dfrac{4}{10} \times \dfrac{10}{2}=\dfrac{4 \times 10}{10 \times 2}=\dfrac{40}{20} = 2$
\item $\dfrac{5}{2} \times 8=\dfrac{5 \times 8}{2}=\dfrac{40}{2} = 20$
\item $\dfrac{2}{10} \times \dfrac{5}{10}=\dfrac{2 \times 5}{10 \times 10}=\dfrac{10}{100} = \dfrac{1}{10}$
\item $\dfrac{- 1}{7} \times \dfrac{- 2}{70}=\dfrac{- 1(- 2)}{7 \times 70}=\dfrac{2}{490} = \dfrac{1}{245}$
\item $\dfrac{\dfrac{1}{7}}{\dfrac{7}{3}}=\dfrac{1}{7} \times \dfrac{3}{7}=\dfrac{1 \times 3}{7 \times 7}=\dfrac{3}{49} = \dfrac{3}{49}$
\end{enumerate}
\end{solution}
@ -52,10 +52,10 @@
Le radar a pris des photos pendant l'été:
\begin{itemize}
\item en juin, il y a eu 46 photos prises dont 22 ratées.
\item en juillet, il y a eu 49 photos réussies et 39 ratées.
\item en août, il y a eu 54 photos dont une proportion de 0.24 de photos ratées.
\item en septembre, il y a eu 8 photos ratées, ce qui correspondait à 14.81\% des photos prises.
\item en juin, il y a eu 46 photos prises dont 20 ratées.
\item en juillet, il y a eu 33 photos réussies et 41 ratées.
\item en août, il y a eu 53 photos dont une proportion de 0.21 de photos ratées.
\item en septembre, il y a eu 8 photos ratées, ce qui correspondait à 16.33\% des photos prises.
\end{itemize}
\begin{enumerate}
@ -87,37 +87,35 @@
\hline
& Juin & Juillet & Août & Septembre & Total\\
\hline
Réussies & 24 & 49 & 41 & 46 & 160\\
Réussies & 26 & 33 & 42 & 41 & 142\\
\hline
Ratées & 22 & 39 & 13 & 8 & 82\\
Ratées & 20 & 41 & 11 & 8 & 80\\
\hline
Total & 46 & 88 & 54 & 54 & 242\\
Total & 46 & 74 & 53 & 49 & 222\\
\hline
\end{tabular}
\end{center}
\item Proportion de photos réussies
\[
\frac{160}{242} = 0.66 = 66\%
\frac{142}{222} = 0.64 = 63\%
\]
\item
\begin{itemize}
\item De juin à juillet
\[
\frac{39 - 22}{22} = \frac{17}{22} = 0.77 = 77\%
\frac{41 - 20}{20} = \frac{21}{20} = 1.05 = 105\%
\]
\item De juillet à août
\[
\frac{13 - 39}{39} = \frac{-26}{39} = -0.67 = -66\%
\frac{11 - 41}{41} = \frac{-30}{41} = -0.73 = -73\%
\]
\item De août à septembre
\[
\frac{8 - 13}{13} = \frac{-5}{13} = -0.38 = -38\%
\frac{8 - 11}{11} = \frac{-3}{11} = -0.27 = -27\%
\]
\end{itemize}
\end{enumerate}
\end{solution}
\printsolutionstype{exercise}
\end{document}

View File

@ -17,17 +17,17 @@
Détailler les calculs suivants et donner le résultat sous la forme d'une fraction irréductible.
\begin{multicols}{3}
\begin{enumerate}[label={\Alph*=}]
\item $\dfrac{6}{3} + \dfrac{1}{3}$
\item $\dfrac{1}{8} + 10$
\item $\dfrac{8}{10} + \dfrac{9}{10}$
\item $\dfrac{6}{9} + 4$
\item $\dfrac{1}{9} + \dfrac{3}{18}$
\item $\dfrac{9}{7} + \dfrac{7}{3}$
\item $\dfrac{9}{2} + \dfrac{7}{10}$
\item $\dfrac{10}{5} + \dfrac{5}{6}$
\item $\dfrac{7}{4} \times 2$
\item $\dfrac{7}{3} \times \dfrac{10}{3}$
\item $\dfrac{6}{5} \times 3$
\item $\dfrac{8}{5} \times \dfrac{10}{5}$
\item $\dfrac{- 10}{2} \times \dfrac{- 9}{4}$
\item $\dfrac{\dfrac{3}{6}}{\dfrac{6}{8}}$
\item $\dfrac{- 4}{7} \times \dfrac{6}{14}$
\item $\dfrac{\dfrac{7}{10}}{\dfrac{6}{3}}$
\end{enumerate}
\end{multicols}
\end{exercise}
@ -35,15 +35,15 @@
\begin{solution}
\begin{enumerate}[label={\Alph*=}]
\item $\dfrac{6}{3} + \dfrac{1}{3}=\dfrac{6 + 1}{3}=\dfrac{7}{3} = \dfrac{7}{3}$
\item $\dfrac{1}{8} + 10=\dfrac{1}{8} + \dfrac{10}{1}=\dfrac{1}{8} + \dfrac{10 \times 8}{1 \times 8}=\dfrac{1}{8} + \dfrac{80}{8}=\dfrac{1 + 80}{8}=\dfrac{81}{8} = \dfrac{81}{8}$
\item $\dfrac{1}{9} + \dfrac{3}{18}=\dfrac{1 \times 2}{9 \times 2} + \dfrac{3}{18}=\dfrac{2}{18} + \dfrac{3}{18}=\dfrac{2 + 3}{18}=\dfrac{5}{18} = \dfrac{5}{18}$
\item $\dfrac{9}{7} + \dfrac{7}{3}=\dfrac{9 \times 3}{7 \times 3} + \dfrac{7 \times 7}{3 \times 7}=\dfrac{27}{21} + \dfrac{49}{21}=\dfrac{27 + 49}{21}=\dfrac{76}{21} = \dfrac{76}{21}$
\item $\dfrac{8}{10} + \dfrac{9}{10}=\dfrac{8 + 9}{10}=\dfrac{17}{10} = \dfrac{17}{10}$
\item $\dfrac{6}{9} + 4=\dfrac{6}{9} + \dfrac{4}{1}=\dfrac{6}{9} + \dfrac{4 \times 9}{1 \times 9}=\dfrac{6}{9} + \dfrac{36}{9}=\dfrac{6 + 36}{9}=\dfrac{42}{9} = \dfrac{14}{3}$
\item $\dfrac{9}{2} + \dfrac{7}{10}=\dfrac{9 \times 5}{2 \times 5} + \dfrac{7}{10}=\dfrac{45}{10} + \dfrac{7}{10}=\dfrac{45 + 7}{10}=\dfrac{52}{10} = \dfrac{26}{5}$
\item $\dfrac{10}{5} + \dfrac{5}{6}=\dfrac{10 \times 6}{5 \times 6} + \dfrac{5 \times 5}{6 \times 5}=\dfrac{60}{30} + \dfrac{25}{30}=\dfrac{60 + 25}{30}=\dfrac{85}{30} = \dfrac{17}{6}$
\item $\dfrac{7}{4} \times 2=\dfrac{7 \times 2}{4}=\dfrac{14}{4} = \dfrac{7}{2}$
\item $\dfrac{7}{3} \times \dfrac{10}{3}=\dfrac{7 \times 10}{3 \times 3}=\dfrac{70}{9} = \dfrac{70}{9}$
\item $\dfrac{- 10}{2} \times \dfrac{- 9}{4}=\dfrac{- 10(- 9)}{2 \times 4}=\dfrac{90}{8} = \dfrac{45}{4}$
\item $\dfrac{\dfrac{3}{6}}{\dfrac{6}{8}}=\dfrac{3}{6} \times \dfrac{8}{6}=\dfrac{3 \times 8}{6 \times 6}=\dfrac{24}{36} = \dfrac{2}{3}$
\item $\dfrac{6}{5} \times 3=\dfrac{6 \times 3}{5}=\dfrac{18}{5} = \dfrac{18}{5}$
\item $\dfrac{8}{5} \times \dfrac{10}{5}=\dfrac{8 \times 10}{5 \times 5}=\dfrac{80}{25} = \dfrac{16}{5}$
\item $\dfrac{- 4}{7} \times \dfrac{6}{14}=\dfrac{- 4 \times 6}{7 \times 14}=\dfrac{- 24}{98} = \dfrac{- 12}{49}$
\item $\dfrac{\dfrac{7}{10}}{\dfrac{6}{3}}=\dfrac{7}{10} \times \dfrac{3}{6}=\dfrac{7 \times 3}{10 \times 6}=\dfrac{21}{60} = \dfrac{7}{20}$
\end{enumerate}
\end{solution}
@ -52,10 +52,10 @@
Le radar a pris des photos pendant l'été:
\begin{itemize}
\item en juin, il y a eu 52 photos prises dont 26 ratées.
\item en juillet, il y a eu 42 photos réussies et 41 ratées.
\item en août, il y a eu 65 photos dont une proportion de 0.25 de photos ratées.
\item en septembre, il y a eu 12 photos ratées, ce qui correspondait à 19.35\% des photos prises.
\item en juin, il y a eu 44 photos prises dont 22 ratées.
\item en juillet, il y a eu 41 photos réussies et 40 ratées.
\item en août, il y a eu 57 photos dont une proportion de 0.26 de photos ratées.
\item en septembre, il y a eu 14 photos ratées, ce qui correspondait à 24.14\% des photos prises.
\end{itemize}
\begin{enumerate}
@ -87,37 +87,35 @@
\hline
& Juin & Juillet & Août & Septembre & Total\\
\hline
Réussies & 26 & 42 & 49 & 50 & 167\\
Réussies & 22 & 41 & 42 & 44 & 149\\
\hline
Ratées & 26 & 41 & 16 & 12 & 95\\
Ratées & 22 & 40 & 15 & 14 & 91\\
\hline
Total & 52 & 83 & 65 & 62 & 262\\
Total & 44 & 81 & 57 & 58 & 240\\
\hline
\end{tabular}
\end{center}
\item Proportion de photos réussies
\[
\frac{167}{262} = 0.64 = 63\%
\frac{149}{240} = 0.62 = 62\%
\]
\item
\begin{itemize}
\item De juin à juillet
\[
\frac{41 - 26}{26} = \frac{15}{26} = 0.58 = 57\%
\frac{40 - 22}{22} = \frac{18}{22} = 0.82 = 81\%
\]
\item De juillet à août
\[
\frac{16 - 41}{41} = \frac{-25}{41} = -0.61 = -60\%
\frac{15 - 40}{40} = \frac{-25}{40} = -0.62 = -62\%
\]
\item De août à septembre
\[
\frac{12 - 16}{16} = \frac{-4}{16} = -0.25 = -25\%
\frac{14 - 15}{15} = \frac{-1}{15} = -0.07 = -6\%
\]
\end{itemize}
\end{enumerate}
\end{solution}
\printsolutionstype{exercise}
\end{document}

View File

@ -17,17 +17,17 @@
Détailler les calculs suivants et donner le résultat sous la forme d'une fraction irréductible.
\begin{multicols}{3}
\begin{enumerate}[label={\Alph*=}]
\item $\dfrac{5}{6} + \dfrac{9}{6}$
\item $\dfrac{2}{8} + 8$
\item $\dfrac{1}{5} + \dfrac{1}{5}$
\item $\dfrac{9}{4} + 5$
\item $\dfrac{2}{10} + \dfrac{2}{80}$
\item $\dfrac{2}{7} + \dfrac{8}{5}$
\item $\dfrac{4}{8} + \dfrac{3}{64}$
\item $\dfrac{9}{10} + \dfrac{2}{6}$
\item $\dfrac{3}{4} \times 1$
\item $\dfrac{7}{3} \times \dfrac{2}{3}$
\item $\dfrac{8}{3} \times 6$
\item $\dfrac{6}{3} \times \dfrac{2}{3}$
\item $\dfrac{- 2}{5} \times \dfrac{- 7}{20}$
\item $\dfrac{\dfrac{3}{6}}{\dfrac{5}{8}}$
\item $\dfrac{10}{7} \times \dfrac{- 6}{21}$
\item $\dfrac{\dfrac{6}{2}}{\dfrac{3}{2}}$
\end{enumerate}
\end{multicols}
\end{exercise}
@ -35,15 +35,15 @@
\begin{solution}
\begin{enumerate}[label={\Alph*=}]
\item $\dfrac{5}{6} + \dfrac{9}{6}=\dfrac{5 + 9}{6}=\dfrac{14}{6} = \dfrac{7}{3}$
\item $\dfrac{2}{8} + 8=\dfrac{2}{8} + \dfrac{8}{1}=\dfrac{2}{8} + \dfrac{8 \times 8}{1 \times 8}=\dfrac{2}{8} + \dfrac{64}{8}=\dfrac{2 + 64}{8}=\dfrac{66}{8} = \dfrac{33}{4}$
\item $\dfrac{2}{10} + \dfrac{2}{80}=\dfrac{2 \times 8}{10 \times 8} + \dfrac{2}{80}=\dfrac{16}{80} + \dfrac{2}{80}=\dfrac{16 + 2}{80}=\dfrac{18}{80} = \dfrac{9}{40}$
\item $\dfrac{2}{7} + \dfrac{8}{5}=\dfrac{2 \times 5}{7 \times 5} + \dfrac{8 \times 7}{5 \times 7}=\dfrac{10}{35} + \dfrac{56}{35}=\dfrac{10 + 56}{35}=\dfrac{66}{35} = \dfrac{66}{35}$
\item $\dfrac{1}{5} + \dfrac{1}{5}=\dfrac{1 + 1}{5}=\dfrac{2}{5} = \dfrac{2}{5}$
\item $\dfrac{9}{4} + 5=\dfrac{9}{4} + \dfrac{5}{1}=\dfrac{9}{4} + \dfrac{5 \times 4}{1 \times 4}=\dfrac{9}{4} + \dfrac{20}{4}=\dfrac{9 + 20}{4}=\dfrac{29}{4} = \dfrac{29}{4}$
\item $\dfrac{4}{8} + \dfrac{3}{64}=\dfrac{4 \times 8}{8 \times 8} + \dfrac{3}{64}=\dfrac{32}{64} + \dfrac{3}{64}=\dfrac{32 + 3}{64}=\dfrac{35}{64} = \dfrac{35}{64}$
\item $\dfrac{9}{10} + \dfrac{2}{6}=\dfrac{9 \times 3}{10 \times 3} + \dfrac{2 \times 5}{6 \times 5}=\dfrac{27}{30} + \dfrac{10}{30}=\dfrac{27 + 10}{30}=\dfrac{37}{30} = \dfrac{37}{30}$
\item $\dfrac{3}{4} \times 1=\dfrac{3}{4} = \dfrac{3}{4}$
\item $\dfrac{7}{3} \times \dfrac{2}{3}=\dfrac{7 \times 2}{3 \times 3}=\dfrac{14}{9} = \dfrac{14}{9}$
\item $\dfrac{- 2}{5} \times \dfrac{- 7}{20}=\dfrac{- 2(- 7)}{5 \times 20}=\dfrac{14}{100} = \dfrac{7}{50}$
\item $\dfrac{\dfrac{3}{6}}{\dfrac{5}{8}}=\dfrac{3}{6} \times \dfrac{8}{5}=\dfrac{3 \times 8}{6 \times 5}=\dfrac{24}{30} = \dfrac{4}{5}$
\item $\dfrac{8}{3} \times 6=\dfrac{8 \times 6}{3}=\dfrac{48}{3} = 16$
\item $\dfrac{6}{3} \times \dfrac{2}{3}=\dfrac{6 \times 2}{3 \times 3}=\dfrac{12}{9} = \dfrac{4}{3}$
\item $\dfrac{10}{7} \times \dfrac{- 6}{21}=\dfrac{10(- 6)}{7 \times 21}=\dfrac{- 60}{147} = \dfrac{- 20}{49}$
\item $\dfrac{\dfrac{6}{2}}{\dfrac{3}{2}}=\dfrac{6}{2} \times \dfrac{2}{3}=\dfrac{6 \times 2}{2 \times 3}=\dfrac{12}{6} = 2$
\end{enumerate}
\end{solution}
@ -52,10 +52,10 @@
Le radar a pris des photos pendant l'été:
\begin{itemize}
\item en juin, il y a eu 43 photos prises dont 23 ratées.
\item en juillet, il y a eu 35 photos réussies et 37 ratées.
\item en août, il y a eu 64 photos dont une proportion de 0.31 de photos ratées.
\item en septembre, il y a eu 12 photos ratées, ce qui correspondait à 21.82\% des photos prises.
\item en juin, il y a eu 59 photos prises dont 29 ratées.
\item en juillet, il y a eu 37 photos réussies et 47 ratées.
\item en août, il y a eu 55 photos dont une proportion de 0.27 de photos ratées.
\item en septembre, il y a eu 10 photos ratées, ce qui correspondait à 18.52\% des photos prises.
\end{itemize}
\begin{enumerate}
@ -87,37 +87,35 @@
\hline
& Juin & Juillet & Août & Septembre & Total\\
\hline
Réussies & 20 & 35 & 44 & 43 & 142\\
Réussies & 30 & 37 & 40 & 44 & 151\\
\hline
Ratées & 23 & 37 & 20 & 12 & 92\\
Ratées & 29 & 47 & 15 & 10 & 101\\
\hline
Total & 43 & 72 & 64 & 55 & 234\\
Total & 59 & 84 & 55 & 54 & 252\\
\hline
\end{tabular}
\end{center}
\item Proportion de photos réussies
\[
\frac{142}{234} = 0.61 = 60\%
\frac{151}{252} = 0.6 = 59\%
\]
\item
\begin{itemize}
\item De juin à juillet
\[
\frac{37 - 23}{23} = \frac{14}{23} = 0.61 = 60\%
\frac{47 - 29}{29} = \frac{18}{29} = 0.62 = 62\%
\]
\item De juillet à août
\[
\frac{20 - 37}{37} = \frac{-17}{37} = -0.46 = -45\%
\frac{15 - 47}{47} = \frac{-32}{47} = -0.68 = -68\%
\]
\item De août à septembre
\[
\frac{12 - 20}{20} = \frac{-8}{20} = -0.4 = -40\%
\frac{10 - 15}{15} = \frac{-5}{15} = -0.33 = -33\%
\]
\end{itemize}
\end{enumerate}
\end{solution}
\printsolutionstype{exercise}
\end{document}

View File

@ -17,17 +17,17 @@
Détailler les calculs suivants et donner le résultat sous la forme d'une fraction irréductible.
\begin{multicols}{3}
\begin{enumerate}[label={\Alph*=}]
\item $\dfrac{3}{10} + \dfrac{5}{10}$
\item $\dfrac{1}{10} + 1$
\item $\dfrac{8}{6} + \dfrac{1}{6}$
\item $\dfrac{2}{5} + 2$
\item $\dfrac{2}{7} + \dfrac{9}{49}$
\item $\dfrac{10}{3} + \dfrac{5}{8}$
\item $\dfrac{8}{3} + \dfrac{7}{6}$
\item $\dfrac{5}{9} + \dfrac{3}{7}$
\item $\dfrac{2}{4} \times 5$
\item $\dfrac{7}{8} \times 2$
\item $\dfrac{3}{7} \times \dfrac{4}{7}$
\item $\dfrac{10}{4} \times \dfrac{- 10}{28}$
\item $\dfrac{\dfrac{6}{4}}{\dfrac{9}{3}}$
\item $\dfrac{- 3}{10} \times \dfrac{8}{20}$
\item $\dfrac{\dfrac{9}{6}}{\dfrac{2}{8}}$
\end{enumerate}
\end{multicols}
\end{exercise}
@ -35,15 +35,15 @@
\begin{solution}
\begin{enumerate}[label={\Alph*=}]
\item $\dfrac{3}{10} + \dfrac{5}{10}=\dfrac{3 + 5}{10}=\dfrac{8}{10} = \dfrac{4}{5}$
\item $\dfrac{1}{10} + 1=\dfrac{1}{10} + \dfrac{1}{1}=\dfrac{1}{10} + \dfrac{1 \times 10}{1 \times 10}=\dfrac{1}{10} + \dfrac{10}{10}=\dfrac{1 + 10}{10}=\dfrac{11}{10} = \dfrac{11}{10}$
\item $\dfrac{2}{7} + \dfrac{9}{49}=\dfrac{2 \times 7}{7 \times 7} + \dfrac{9}{49}=\dfrac{14}{49} + \dfrac{9}{49}=\dfrac{14 + 9}{49}=\dfrac{23}{49} = \dfrac{23}{49}$
\item $\dfrac{10}{3} + \dfrac{5}{8}=\dfrac{10 \times 8}{3 \times 8} + \dfrac{5 \times 3}{8 \times 3}=\dfrac{80}{24} + \dfrac{15}{24}=\dfrac{80 + 15}{24}=\dfrac{95}{24} = \dfrac{95}{24}$
\item $\dfrac{8}{6} + \dfrac{1}{6}=\dfrac{8 + 1}{6}=\dfrac{9}{6} = \dfrac{3}{2}$
\item $\dfrac{2}{5} + 2=\dfrac{2}{5} + \dfrac{2}{1}=\dfrac{2}{5} + \dfrac{2 \times 5}{1 \times 5}=\dfrac{2}{5} + \dfrac{10}{5}=\dfrac{2 + 10}{5}=\dfrac{12}{5} = \dfrac{12}{5}$
\item $\dfrac{8}{3} + \dfrac{7}{6}=\dfrac{8 \times 2}{3 \times 2} + \dfrac{7}{6}=\dfrac{16}{6} + \dfrac{7}{6}=\dfrac{16 + 7}{6}=\dfrac{23}{6} = \dfrac{23}{6}$
\item $\dfrac{5}{9} + \dfrac{3}{7}=\dfrac{5 \times 7}{9 \times 7} + \dfrac{3 \times 9}{7 \times 9}=\dfrac{35}{63} + \dfrac{27}{63}=\dfrac{35 + 27}{63}=\dfrac{62}{63} = \dfrac{62}{63}$
\item $\dfrac{2}{4} \times 5=\dfrac{2 \times 5}{4}=\dfrac{10}{4} = \dfrac{5}{2}$
\item $\dfrac{7}{8} \times 2=\dfrac{7 \times 2}{8}=\dfrac{14}{8} = \dfrac{7}{4}$
\item $\dfrac{3}{7} \times \dfrac{4}{7}=\dfrac{3 \times 4}{7 \times 7}=\dfrac{12}{49} = \dfrac{12}{49}$
\item $\dfrac{10}{4} \times \dfrac{- 10}{28}=\dfrac{10(- 10)}{4 \times 28}=\dfrac{- 100}{112} = \dfrac{- 25}{28}$
\item $\dfrac{\dfrac{6}{4}}{\dfrac{9}{3}}=\dfrac{6}{4} \times \dfrac{3}{9}=\dfrac{6 \times 3}{4 \times 9}=\dfrac{18}{36} = \dfrac{1}{2}$
\item $\dfrac{- 3}{10} \times \dfrac{8}{20}=\dfrac{- 3 \times 8}{10 \times 20}=\dfrac{- 24}{200} = \dfrac{- 3}{25}$
\item $\dfrac{\dfrac{9}{6}}{\dfrac{2}{8}}=\dfrac{9}{6} \times \dfrac{8}{2}=\dfrac{9 \times 8}{6 \times 2}=\dfrac{72}{12} = 6$
\end{enumerate}
\end{solution}
@ -52,10 +52,10 @@
Le radar a pris des photos pendant l'été:
\begin{itemize}
\item en juin, il y a eu 54 photos prises dont 26 ratées.
\item en juillet, il y a eu 44 photos réussies et 32 ratées.
\item en août, il y a eu 61 photos dont une proportion de 0.26 de photos ratées.
\item en septembre, il y a eu 7 photos ratées, ce qui correspondait à 14.0\% des photos prises.
\item en juin, il y a eu 52 photos prises dont 28 ratées.
\item en juillet, il y a eu 41 photos réussies et 48 ratées.
\item en août, il y a eu 60 photos dont une proportion de 0.33 de photos ratées.
\item en septembre, il y a eu 9 photos ratées, ce qui correspondait à 17.65\% des photos prises.
\end{itemize}
\begin{enumerate}
@ -87,37 +87,35 @@
\hline
& Juin & Juillet & Août & Septembre & Total\\
\hline
Réussies & 28 & 44 & 45 & 43 & 160\\
Réussies & 24 & 41 & 40 & 42 & 147\\
\hline
Ratées & 26 & 32 & 16 & 7 & 81\\
Ratées & 28 & 48 & 20 & 9 & 105\\
\hline
Total & 54 & 76 & 61 & 50 & 241\\
Total & 52 & 89 & 60 & 51 & 252\\
\hline
\end{tabular}
\end{center}
\item Proportion de photos réussies
\[
\frac{160}{241} = 0.66 = 66\%
\frac{147}{252} = 0.58 = 58\%
\]
\item
\begin{itemize}
\item De juin à juillet
\[
\frac{32 - 26}{26} = \frac{6}{26} = 0.23 = 23\%
\frac{48 - 28}{28} = \frac{20}{28} = 0.71 = 71\%
\]
\item De juillet à août
\[
\frac{16 - 32}{32} = \frac{-16}{32} = -0.5 = -50\%
\frac{20 - 48}{48} = \frac{-28}{48} = -0.58 = -58\%
\]
\item De août à septembre
\[
\frac{7 - 16}{16} = \frac{-9}{16} = -0.56 = -56\%
\frac{9 - 20}{20} = \frac{-11}{20} = -0.55 = -55\%
\]
\end{itemize}
\end{enumerate}
\end{solution}
\printsolutionstype{exercise}
\end{document}

View File

@ -17,17 +17,17 @@
Détailler les calculs suivants et donner le résultat sous la forme d'une fraction irréductible.
\begin{multicols}{3}
\begin{enumerate}[label={\Alph*=}]
\item $\dfrac{9}{8} + \dfrac{7}{8}$
\item $\dfrac{9}{4} + 9$
\item $\dfrac{5}{6} + \dfrac{10}{6}$
\item $\dfrac{3}{2} + 8$
\item $\dfrac{3}{7} + \dfrac{8}{14}$
\item $\dfrac{5}{10} + \dfrac{2}{10}$
\item $\dfrac{9}{3} + \dfrac{10}{12}$
\item $\dfrac{4}{5} + \dfrac{6}{4}$
\item $\dfrac{5}{8} \times 1$
\item $\dfrac{1}{4} \times \dfrac{5}{4}$
\item $\dfrac{5}{7} \times 1$
\item $\dfrac{6}{8} \times \dfrac{9}{8}$
\item $\dfrac{1}{8} \times \dfrac{- 9}{80}$
\item $\dfrac{\dfrac{8}{10}}{\dfrac{7}{2}}$
\item $\dfrac{4}{3} \times \dfrac{- 1}{6}$
\item $\dfrac{\dfrac{2}{10}}{\dfrac{5}{2}}$
\end{enumerate}
\end{multicols}
\end{exercise}
@ -35,15 +35,15 @@
\begin{solution}
\begin{enumerate}[label={\Alph*=}]
\item $\dfrac{9}{8} + \dfrac{7}{8}=\dfrac{9 + 7}{8}=\dfrac{16}{8} = 2$
\item $\dfrac{9}{4} + 9=\dfrac{9}{4} + \dfrac{9}{1}=\dfrac{9}{4} + \dfrac{9 \times 4}{1 \times 4}=\dfrac{9}{4} + \dfrac{36}{4}=\dfrac{9 + 36}{4}=\dfrac{45}{4} = \dfrac{45}{4}$
\item $\dfrac{3}{7} + \dfrac{8}{14}=\dfrac{3 \times 2}{7 \times 2} + \dfrac{8}{14}=\dfrac{6}{14} + \dfrac{8}{14}=\dfrac{6 + 8}{14}=\dfrac{14}{14} = 1$
\item $\dfrac{5}{10} + \dfrac{2}{10}=\dfrac{5 + 2}{10}=\dfrac{7}{10} = \dfrac{7}{10}$
\item $\dfrac{5}{6} + \dfrac{10}{6}=\dfrac{5 + 10}{6}=\dfrac{15}{6} = \dfrac{5}{2}$
\item $\dfrac{3}{2} + 8=\dfrac{3}{2} + \dfrac{8}{1}=\dfrac{3}{2} + \dfrac{8 \times 2}{1 \times 2}=\dfrac{3}{2} + \dfrac{16}{2}=\dfrac{3 + 16}{2}=\dfrac{19}{2} = \dfrac{19}{2}$
\item $\dfrac{9}{3} + \dfrac{10}{12}=\dfrac{9 \times 4}{3 \times 4} + \dfrac{10}{12}=\dfrac{36}{12} + \dfrac{10}{12}=\dfrac{36 + 10}{12}=\dfrac{46}{12} = \dfrac{23}{6}$
\item $\dfrac{4}{5} + \dfrac{6}{4}=\dfrac{4 \times 4}{5 \times 4} + \dfrac{6 \times 5}{4 \times 5}=\dfrac{16}{20} + \dfrac{30}{20}=\dfrac{16 + 30}{20}=\dfrac{46}{20} = \dfrac{23}{10}$
\item $\dfrac{5}{8} \times 1=\dfrac{5}{8} = \dfrac{5}{8}$
\item $\dfrac{1}{4} \times \dfrac{5}{4}=\dfrac{1 \times 5}{4 \times 4}=\dfrac{5}{16} = \dfrac{5}{16}$
\item $\dfrac{1}{8} \times \dfrac{- 9}{80}=\dfrac{1(- 9)}{8 \times 80}=\dfrac{- 9}{640} = \dfrac{- 9}{640}$
\item $\dfrac{\dfrac{8}{10}}{\dfrac{7}{2}}=\dfrac{8}{10} \times \dfrac{2}{7}=\dfrac{8 \times 2}{10 \times 7}=\dfrac{16}{70} = \dfrac{8}{35}$
\item $\dfrac{5}{7} \times 1=\dfrac{5}{7} = \dfrac{5}{7}$
\item $\dfrac{6}{8} \times \dfrac{9}{8}=\dfrac{6 \times 9}{8 \times 8}=\dfrac{54}{64} = \dfrac{27}{32}$
\item $\dfrac{4}{3} \times \dfrac{- 1}{6}=\dfrac{4(- 1)}{3 \times 6}=\dfrac{- 4}{18} = \dfrac{- 2}{9}$
\item $\dfrac{\dfrac{2}{10}}{\dfrac{5}{2}}=\dfrac{2}{10} \times \dfrac{2}{5}=\dfrac{2 \times 2}{10 \times 5}=\dfrac{4}{50} = \dfrac{2}{25}$
\end{enumerate}
\end{solution}
@ -52,10 +52,10 @@
Le radar a pris des photos pendant l'été:
\begin{itemize}
\item en juin, il y a eu 56 photos prises dont 29 ratées.
\item en juillet, il y a eu 50 photos réussies et 37 ratées.
\item en août, il y a eu 60 photos dont une proportion de 0.17 de photos ratées.
\item en septembre, il y a eu 11 photos ratées, ce qui correspondait à 21.15\% des photos prises.
\item en juin, il y a eu 51 photos prises dont 22 ratées.
\item en juillet, il y a eu 38 photos réussies et 45 ratées.
\item en août, il y a eu 67 photos dont une proportion de 0.28 de photos ratées.
\item en septembre, il y a eu 13 photos ratées, ce qui correspondait à 22.03\% des photos prises.
\end{itemize}
\begin{enumerate}
@ -87,37 +87,35 @@
\hline
& Juin & Juillet & Août & Septembre & Total\\
\hline
Réussies & 27 & 50 & 50 & 41 & 168\\
Réussies & 29 & 38 & 48 & 46 & 161\\
\hline
Ratées & 29 & 37 & 10 & 11 & 87\\
Ratées & 22 & 45 & 19 & 13 & 99\\
\hline
Total & 56 & 87 & 60 & 52 & 255\\
Total & 51 & 83 & 67 & 59 & 260\\
\hline
\end{tabular}
\end{center}
\item Proportion de photos réussies
\[
\frac{168}{255} = 0.66 = 65\%
\frac{161}{260} = 0.62 = 61\%
\]
\item
\begin{itemize}
\item De juin à juillet
\[
\frac{37 - 29}{29} = \frac{8}{29} = 0.28 = 27\%
\frac{45 - 22}{22} = \frac{23}{22} = 1.05 = 104\%
\]
\item De juillet à août
\[
\frac{10 - 37}{37} = \frac{-27}{37} = -0.73 = -72\%
\frac{19 - 45}{45} = \frac{-26}{45} = -0.58 = -57\%
\]
\item De août à septembre
\[
\frac{11 - 10}{10} = \frac{1}{10} = 0.1 = 10\%
\frac{13 - 19}{19} = \frac{-6}{19} = -0.32 = -31\%
\]
\end{itemize}
\end{enumerate}
\end{solution}
\printsolutionstype{exercise}
\end{document}

View File

@ -17,17 +17,17 @@
Détailler les calculs suivants et donner le résultat sous la forme d'une fraction irréductible.
\begin{multicols}{3}
\begin{enumerate}[label={\Alph*=}]
\item $\dfrac{9}{7} + \dfrac{3}{7}$
\item $\dfrac{5}{7} + 6$
\item $\dfrac{10}{2} + \dfrac{4}{2}$
\item $\dfrac{10}{2} + 6$
\item $\dfrac{1}{7} + \dfrac{10}{14}$
\item $\dfrac{6}{8} + \dfrac{1}{7}$
\item $\dfrac{8}{9} + \dfrac{6}{45}$
\item $\dfrac{5}{3} + \dfrac{7}{6}$
\item $\dfrac{1}{5} \times 1$
\item $\dfrac{5}{8} \times \dfrac{7}{8}$
\item $\dfrac{7}{6} \times 5$
\item $\dfrac{7}{4} \times \dfrac{8}{4}$
\item $\dfrac{- 7}{2} \times \dfrac{- 1}{14}$
\item $\dfrac{\dfrac{7}{6}}{\dfrac{10}{9}}$
\item $\dfrac{3}{4} \times \dfrac{2}{20}$
\item $\dfrac{\dfrac{6}{5}}{\dfrac{10}{3}}$
\end{enumerate}
\end{multicols}
\end{exercise}
@ -35,15 +35,15 @@
\begin{solution}
\begin{enumerate}[label={\Alph*=}]
\item $\dfrac{9}{7} + \dfrac{3}{7}=\dfrac{9 + 3}{7}=\dfrac{12}{7} = \dfrac{12}{7}$
\item $\dfrac{5}{7} + 6=\dfrac{5}{7} + \dfrac{6}{1}=\dfrac{5}{7} + \dfrac{6 \times 7}{1 \times 7}=\dfrac{5}{7} + \dfrac{42}{7}=\dfrac{5 + 42}{7}=\dfrac{47}{7} = \dfrac{47}{7}$
\item $\dfrac{1}{7} + \dfrac{10}{14}=\dfrac{1 \times 2}{7 \times 2} + \dfrac{10}{14}=\dfrac{2}{14} + \dfrac{10}{14}=\dfrac{2 + 10}{14}=\dfrac{12}{14} = \dfrac{6}{7}$
\item $\dfrac{6}{8} + \dfrac{1}{7}=\dfrac{6 \times 7}{8 \times 7} + \dfrac{1 \times 8}{7 \times 8}=\dfrac{42}{56} + \dfrac{8}{56}=\dfrac{42 + 8}{56}=\dfrac{50}{56} = \dfrac{25}{28}$
\item $\dfrac{10}{2} + \dfrac{4}{2}=\dfrac{10 + 4}{2}=\dfrac{14}{2} = 7$
\item $\dfrac{10}{2} + 6=\dfrac{10}{2} + \dfrac{6}{1}=\dfrac{10}{2} + \dfrac{6 \times 2}{1 \times 2}=\dfrac{10}{2} + \dfrac{12}{2}=\dfrac{10 + 12}{2}=\dfrac{22}{2} = 11$
\item $\dfrac{8}{9} + \dfrac{6}{45}=\dfrac{8 \times 5}{9 \times 5} + \dfrac{6}{45}=\dfrac{40}{45} + \dfrac{6}{45}=\dfrac{40 + 6}{45}=\dfrac{46}{45} = \dfrac{46}{45}$
\item $\dfrac{5}{3} + \dfrac{7}{6}=\dfrac{5 \times 2}{3 \times 2} + \dfrac{7}{6}=\dfrac{10}{6} + \dfrac{7}{6}=\dfrac{10 + 7}{6}=\dfrac{17}{6} = \dfrac{17}{6}$
\item $\dfrac{1}{5} \times 1=\dfrac{1}{5} = \dfrac{1}{5}$
\item $\dfrac{5}{8} \times \dfrac{7}{8}=\dfrac{5 \times 7}{8 \times 8}=\dfrac{35}{64} = \dfrac{35}{64}$
\item $\dfrac{- 7}{2} \times \dfrac{- 1}{14}=\dfrac{- 7(- 1)}{2 \times 14}=\dfrac{7}{28} = \dfrac{1}{4}$
\item $\dfrac{\dfrac{7}{6}}{\dfrac{10}{9}}=\dfrac{7}{6} \times \dfrac{9}{10}=\dfrac{7 \times 9}{6 \times 10}=\dfrac{63}{60} = \dfrac{21}{20}$
\item $\dfrac{7}{6} \times 5=\dfrac{7 \times 5}{6}=\dfrac{35}{6} = \dfrac{35}{6}$
\item $\dfrac{7}{4} \times \dfrac{8}{4}=\dfrac{7 \times 8}{4 \times 4}=\dfrac{56}{16} = \dfrac{7}{2}$
\item $\dfrac{3}{4} \times \dfrac{2}{20}=\dfrac{3 \times 2}{4 \times 20}=\dfrac{6}{80} = \dfrac{3}{40}$
\item $\dfrac{\dfrac{6}{5}}{\dfrac{10}{3}}=\dfrac{6}{5} \times \dfrac{3}{10}=\dfrac{6 \times 3}{5 \times 10}=\dfrac{18}{50} = \dfrac{9}{25}$
\end{enumerate}
\end{solution}
@ -52,10 +52,10 @@
Le radar a pris des photos pendant l'été:
\begin{itemize}
\item en juin, il y a eu 55 photos prises dont 26 ratées.
\item en juillet, il y a eu 39 photos réussies et 33 ratées.
\item en août, il y a eu 66 photos dont une proportion de 0.3 de photos ratées.
\item en septembre, il y a eu 11 photos ratées, ce qui correspondait à 19.64\% des photos prises.
\item en juin, il y a eu 44 photos prises dont 20 ratées.
\item en juillet, il y a eu 40 photos réussies et 37 ratées.
\item en août, il y a eu 61 photos dont une proportion de 0.33 de photos ratées.
\item en septembre, il y a eu 6 photos ratées, ce qui correspondait à 13.04\% des photos prises.
\end{itemize}
\begin{enumerate}
@ -87,37 +87,35 @@
\hline
& Juin & Juillet & Août & Septembre & Total\\
\hline
Réussies & 29 & 39 & 46 & 45 & 159\\
Réussies & 24 & 40 & 41 & 40 & 145\\
\hline
Ratées & 26 & 33 & 20 & 11 & 90\\
Ratées & 20 & 37 & 20 & 6 & 83\\
\hline
Total & 55 & 72 & 66 & 56 & 249\\
Total & 44 & 77 & 61 & 46 & 228\\
\hline
\end{tabular}
\end{center}
\item Proportion de photos réussies
\[
\frac{159}{249} = 0.64 = 63\%
\frac{145}{228} = 0.64 = 63\%
\]
\item
\begin{itemize}
\item De juin à juillet
\[
\frac{33 - 26}{26} = \frac{7}{26} = 0.27 = 26\%
\frac{37 - 20}{20} = \frac{17}{20} = 0.85 = 85\%
\]
\item De juillet à août
\[
\frac{20 - 33}{33} = \frac{-13}{33} = -0.39 = -39\%
\frac{20 - 37}{37} = \frac{-17}{37} = -0.46 = -45\%
\]
\item De août à septembre
\[
\frac{11 - 20}{20} = \frac{-9}{20} = -0.45 = -45\%
\frac{6 - 20}{20} = \frac{-14}{20} = -0.7 = -70\%
\]
\end{itemize}
\end{enumerate}
\end{solution}
\printsolutionstype{exercise}
\end{document}

View File

@ -17,17 +17,17 @@
Détailler les calculs suivants et donner le résultat sous la forme d'une fraction irréductible.
\begin{multicols}{3}
\begin{enumerate}[label={\Alph*=}]
\item $\dfrac{2}{9} + \dfrac{7}{9}$
\item $\dfrac{2}{8} + 5$
\item $\dfrac{2}{7} + \dfrac{9}{7}$
\item $\dfrac{2}{9} + 8$
\item $\dfrac{2}{5} + \dfrac{1}{30}$
\item $\dfrac{3}{8} + \dfrac{5}{3}$
\item $\dfrac{1}{8} + \dfrac{3}{8}$
\item $\dfrac{3}{2} + \dfrac{9}{7}$
\item $\dfrac{7}{5} \times 2$
\item $\dfrac{10}{9} \times \dfrac{3}{9}$
\item $\dfrac{1}{10} \times 2$
\item $\dfrac{7}{3} \times \dfrac{10}{3}$
\item $\dfrac{9}{7} \times \dfrac{3}{49}$
\item $\dfrac{\dfrac{3}{7}}{\dfrac{2}{9}}$
\item $\dfrac{9}{3} \times \dfrac{2}{6}$
\item $\dfrac{\dfrac{2}{6}}{\dfrac{8}{7}}$
\end{enumerate}
\end{multicols}
\end{exercise}
@ -35,15 +35,15 @@
\begin{solution}
\begin{enumerate}[label={\Alph*=}]
\item $\dfrac{2}{9} + \dfrac{7}{9}=\dfrac{2 + 7}{9}=\dfrac{9}{9} = 1$
\item $\dfrac{2}{8} + 5=\dfrac{2}{8} + \dfrac{5}{1}=\dfrac{2}{8} + \dfrac{5 \times 8}{1 \times 8}=\dfrac{2}{8} + \dfrac{40}{8}=\dfrac{2 + 40}{8}=\dfrac{42}{8} = \dfrac{21}{4}$
\item $\dfrac{2}{5} + \dfrac{1}{30}=\dfrac{2 \times 6}{5 \times 6} + \dfrac{1}{30}=\dfrac{12}{30} + \dfrac{1}{30}=\dfrac{12 + 1}{30}=\dfrac{13}{30} = \dfrac{13}{30}$
\item $\dfrac{3}{8} + \dfrac{5}{3}=\dfrac{3 \times 3}{8 \times 3} + \dfrac{5 \times 8}{3 \times 8}=\dfrac{9}{24} + \dfrac{40}{24}=\dfrac{9 + 40}{24}=\dfrac{49}{24} = \dfrac{49}{24}$
\item $\dfrac{2}{7} + \dfrac{9}{7}=\dfrac{2 + 9}{7}=\dfrac{11}{7} = \dfrac{11}{7}$
\item $\dfrac{2}{9} + 8=\dfrac{2}{9} + \dfrac{8}{1}=\dfrac{2}{9} + \dfrac{8 \times 9}{1 \times 9}=\dfrac{2}{9} + \dfrac{72}{9}=\dfrac{2 + 72}{9}=\dfrac{74}{9} = \dfrac{74}{9}$
\item $\dfrac{1}{8} + \dfrac{3}{8}=\dfrac{1 + 3}{8}=\dfrac{4}{8} = \dfrac{1}{2}$
\item $\dfrac{3}{2} + \dfrac{9}{7}=\dfrac{3 \times 7}{2 \times 7} + \dfrac{9 \times 2}{7 \times 2}=\dfrac{21}{14} + \dfrac{18}{14}=\dfrac{21 + 18}{14}=\dfrac{39}{14} = \dfrac{39}{14}$
\item $\dfrac{7}{5} \times 2=\dfrac{7 \times 2}{5}=\dfrac{14}{5} = \dfrac{14}{5}$
\item $\dfrac{10}{9} \times \dfrac{3}{9}=\dfrac{10 \times 3}{9 \times 9}=\dfrac{30}{81} = \dfrac{10}{27}$
\item $\dfrac{9}{7} \times \dfrac{3}{49}=\dfrac{9 \times 3}{7 \times 49}=\dfrac{27}{343} = \dfrac{27}{343}$
\item $\dfrac{\dfrac{3}{7}}{\dfrac{2}{9}}=\dfrac{3}{7} \times \dfrac{9}{2}=\dfrac{3 \times 9}{7 \times 2}=\dfrac{27}{14} = \dfrac{27}{14}$
\item $\dfrac{1}{10} \times 2=\dfrac{1 \times 2}{10}=\dfrac{2}{10} = \dfrac{1}{5}$
\item $\dfrac{7}{3} \times \dfrac{10}{3}=\dfrac{7 \times 10}{3 \times 3}=\dfrac{70}{9} = \dfrac{70}{9}$
\item $\dfrac{9}{3} \times \dfrac{2}{6}=\dfrac{9 \times 2}{3 \times 6}=\dfrac{18}{18} = 1$
\item $\dfrac{\dfrac{2}{6}}{\dfrac{8}{7}}=\dfrac{2}{6} \times \dfrac{7}{8}=\dfrac{2 \times 7}{6 \times 8}=\dfrac{14}{48} = \dfrac{7}{24}$
\end{enumerate}
\end{solution}
@ -52,10 +52,10 @@
Le radar a pris des photos pendant l'été:
\begin{itemize}
\item en juin, il y a eu 45 photos prises dont 21 ratées.
\item en juillet, il y a eu 36 photos réussies et 41 ratées.
\item en août, il y a eu 54 photos dont une proportion de 0.26 de photos ratées.
\item en septembre, il y a eu 5 photos ratées, ce qui correspondait à 9.43\% des photos prises.
\item en juin, il y a eu 44 photos prises dont 24 ratées.
\item en juillet, il y a eu 32 photos réussies et 36 ratées.
\item en août, il y a eu 59 photos dont une proportion de 0.25 de photos ratées.
\item en septembre, il y a eu 12 photos ratées, ce qui correspondait à 21.82\% des photos prises.
\end{itemize}
\begin{enumerate}
@ -87,37 +87,35 @@
\hline
& Juin & Juillet & Août & Septembre & Total\\
\hline
Réussies & 24 & 36 & 40 & 48 & 148\\
Réussies & 20 & 32 & 44 & 43 & 139\\
\hline
Ratées & 21 & 41 & 14 & 5 & 81\\
Ratées & 24 & 36 & 15 & 12 & 87\\
\hline
Total & 45 & 77 & 54 & 53 & 229\\
Total & 44 & 68 & 59 & 55 & 226\\
\hline
\end{tabular}
\end{center}
\item Proportion de photos réussies
\[
\frac{148}{229} = 0.65 = 64\%
\frac{139}{226} = 0.62 = 61\%
\]
\item
\begin{itemize}
\item De juin à juillet
\[
\frac{41 - 21}{21} = \frac{20}{21} = 0.95 = 95\%
\frac{36 - 24}{24} = \frac{12}{24} = 0.5 = 50\%
\]
\item De juillet à août
\[
\frac{14 - 41}{41} = \frac{-27}{41} = -0.66 = -65\%
\frac{15 - 36}{36} = \frac{-21}{36} = -0.58 = -58\%
\]
\item De août à septembre
\[
\frac{5 - 14}{14} = \frac{-9}{14} = -0.64 = -64\%
\frac{12 - 15}{15} = \frac{-3}{15} = -0.2 = -20\%
\]
\end{itemize}
\end{enumerate}
\end{solution}
\printsolutionstype{exercise}
\end{document}

View File

@ -17,17 +17,17 @@
Détailler les calculs suivants et donner le résultat sous la forme d'une fraction irréductible.
\begin{multicols}{3}
\begin{enumerate}[label={\Alph*=}]
\item $\dfrac{8}{3} + \dfrac{4}{3}$
\item $\dfrac{8}{7} + 7$
\item $\dfrac{2}{8} + \dfrac{10}{8}$
\item $\dfrac{3}{4} + 4$
\item $\dfrac{7}{8} + \dfrac{6}{72}$
\item $\dfrac{8}{3} + \dfrac{10}{7}$
\item $\dfrac{6}{10} + \dfrac{8}{60}$
\item $\dfrac{6}{8} + \dfrac{9}{7}$
\item $\dfrac{2}{7} \times 9$
\item $\dfrac{4}{9} \times \dfrac{7}{9}$
\item $\dfrac{9}{10} \times 7$
\item $\dfrac{2}{3} \times \dfrac{8}{3}$
\item $\dfrac{- 7}{8} \times \dfrac{7}{56}$
\item $\dfrac{\dfrac{8}{10}}{\dfrac{1}{8}}$
\item $\dfrac{1}{9} \times \dfrac{- 7}{81}$
\item $\dfrac{\dfrac{7}{9}}{\dfrac{6}{9}}$
\end{enumerate}
\end{multicols}
\end{exercise}
@ -35,15 +35,15 @@
\begin{solution}
\begin{enumerate}[label={\Alph*=}]
\item $\dfrac{8}{3} + \dfrac{4}{3}=\dfrac{8 + 4}{3}=\dfrac{12}{3} = 4$
\item $\dfrac{8}{7} + 7=\dfrac{8}{7} + \dfrac{7}{1}=\dfrac{8}{7} + \dfrac{7 \times 7}{1 \times 7}=\dfrac{8}{7} + \dfrac{49}{7}=\dfrac{8 + 49}{7}=\dfrac{57}{7} = \dfrac{57}{7}$
\item $\dfrac{7}{8} + \dfrac{6}{72}=\dfrac{7 \times 9}{8 \times 9} + \dfrac{6}{72}=\dfrac{63}{72} + \dfrac{6}{72}=\dfrac{63 + 6}{72}=\dfrac{69}{72} = \dfrac{23}{24}$
\item $\dfrac{8}{3} + \dfrac{10}{7}=\dfrac{8 \times 7}{3 \times 7} + \dfrac{10 \times 3}{7 \times 3}=\dfrac{56}{21} + \dfrac{30}{21}=\dfrac{56 + 30}{21}=\dfrac{86}{21} = \dfrac{86}{21}$
\item $\dfrac{2}{8} + \dfrac{10}{8}=\dfrac{2 + 10}{8}=\dfrac{12}{8} = \dfrac{3}{2}$
\item $\dfrac{3}{4} + 4=\dfrac{3}{4} + \dfrac{4}{1}=\dfrac{3}{4} + \dfrac{4 \times 4}{1 \times 4}=\dfrac{3}{4} + \dfrac{16}{4}=\dfrac{3 + 16}{4}=\dfrac{19}{4} = \dfrac{19}{4}$
\item $\dfrac{6}{10} + \dfrac{8}{60}=\dfrac{6 \times 6}{10 \times 6} + \dfrac{8}{60}=\dfrac{36}{60} + \dfrac{8}{60}=\dfrac{36 + 8}{60}=\dfrac{44}{60} = \dfrac{11}{15}$
\item $\dfrac{6}{8} + \dfrac{9}{7}=\dfrac{6 \times 7}{8 \times 7} + \dfrac{9 \times 8}{7 \times 8}=\dfrac{42}{56} + \dfrac{72}{56}=\dfrac{42 + 72}{56}=\dfrac{114}{56} = \dfrac{57}{28}$
\item $\dfrac{2}{7} \times 9=\dfrac{2 \times 9}{7}=\dfrac{18}{7} = \dfrac{18}{7}$
\item $\dfrac{4}{9} \times \dfrac{7}{9}=\dfrac{4 \times 7}{9 \times 9}=\dfrac{28}{81} = \dfrac{28}{81}$
\item $\dfrac{- 7}{8} \times \dfrac{7}{56}=\dfrac{- 7 \times 7}{8 \times 56}=\dfrac{- 49}{448} = \dfrac{- 7}{64}$
\item $\dfrac{\dfrac{8}{10}}{\dfrac{1}{8}}=\dfrac{8}{10} \times \dfrac{8}{1}=\dfrac{8 \times 8}{10 \times 1}=\dfrac{64}{10} = \dfrac{32}{5}$
\item $\dfrac{9}{10} \times 7=\dfrac{9 \times 7}{10}=\dfrac{63}{10} = \dfrac{63}{10}$
\item $\dfrac{2}{3} \times \dfrac{8}{3}=\dfrac{2 \times 8}{3 \times 3}=\dfrac{16}{9} = \dfrac{16}{9}$
\item $\dfrac{1}{9} \times \dfrac{- 7}{81}=\dfrac{1(- 7)}{9 \times 81}=\dfrac{- 7}{729} = \dfrac{- 7}{729}$
\item $\dfrac{\dfrac{7}{9}}{\dfrac{6}{9}}=\dfrac{7}{9} \times \dfrac{9}{6}=\dfrac{7 \times 9}{9 \times 6}=\dfrac{63}{54} = \dfrac{7}{6}$
\end{enumerate}
\end{solution}
@ -52,10 +52,10 @@
Le radar a pris des photos pendant l'été:
\begin{itemize}
\item en juin, il y a eu 50 photos prises dont 23 ratées.
\item en juillet, il y a eu 31 photos réussies et 48 ratées.
\item en août, il y a eu 60 photos dont une proportion de 0.32 de photos ratées.
\item en septembre, il y a eu 11 photos ratées, ce qui correspondait à 20.75\% des photos prises.
\item en juin, il y a eu 41 photos prises dont 21 ratées.
\item en juillet, il y a eu 49 photos réussies et 42 ratées.
\item en août, il y a eu 52 photos dont une proportion de 0.19 de photos ratées.
\item en septembre, il y a eu 11 photos ratées, ce qui correspondait à 18.64\% des photos prises.
\end{itemize}
\begin{enumerate}
@ -87,37 +87,35 @@
\hline
& Juin & Juillet & Août & Septembre & Total\\
\hline
Réussies & 27 & 31 & 41 & 42 & 141\\
Réussies & 20 & 49 & 42 & 48 & 159\\
\hline
Ratées & 23 & 48 & 19 & 11 & 101\\
Ratées & 21 & 42 & 10 & 11 & 84\\
\hline
Total & 50 & 79 & 60 & 53 & 242\\
Total & 41 & 91 & 52 & 59 & 243\\
\hline
\end{tabular}
\end{center}
\item Proportion de photos réussies
\[
\frac{141}{242} = 0.58 = 58\%
\frac{159}{243} = 0.65 = 65\%
\]
\item
\begin{itemize}
\item De juin à juillet
\[
\frac{48 - 23}{23} = \frac{25}{23} = 1.09 = 108\%
\frac{42 - 21}{21} = \frac{21}{21} = 1.0 = 100\%
\]
\item De juillet à août
\[
\frac{19 - 48}{48} = \frac{-29}{48} = -0.6 = -60\%
\frac{10 - 42}{42} = \frac{-32}{42} = -0.76 = -76\%
\]
\item De août à septembre
\[
\frac{11 - 19}{19} = \frac{-8}{19} = -0.42 = -42\%
\frac{11 - 10}{10} = \frac{1}{10} = 0.1 = 10\%
\]
\end{itemize}
\end{enumerate}
\end{solution}
\printsolutionstype{exercise}
\end{document}

View File

@ -17,17 +17,17 @@
Détailler les calculs suivants et donner le résultat sous la forme d'une fraction irréductible.
\begin{multicols}{3}
\begin{enumerate}[label={\Alph*=}]
\item $\dfrac{6}{7} + \dfrac{8}{7}$
\item $\dfrac{6}{8} + 10$
\item $\dfrac{1}{9} + \dfrac{4}{9}$
\item $\dfrac{2}{5} + 1$
\item $\dfrac{1}{2} + \dfrac{3}{8}$
\item $\dfrac{6}{2} + \dfrac{5}{10}$
\item $\dfrac{10}{3} + \dfrac{2}{15}$
\item $\dfrac{5}{2} + \dfrac{10}{2}$
\item $\dfrac{6}{10} \times 1$
\item $\dfrac{3}{5} \times \dfrac{3}{5}$
\item $\dfrac{1}{7} \times 6$
\item $\dfrac{6}{9} \times \dfrac{10}{9}$
\item $\dfrac{- 1}{9} \times \dfrac{3}{63}$
\item $\dfrac{\dfrac{7}{3}}{\dfrac{4}{3}}$
\item $\dfrac{4}{5} \times \dfrac{- 10}{15}$
\item $\dfrac{\dfrac{10}{7}}{\dfrac{3}{8}}$
\end{enumerate}
\end{multicols}
\end{exercise}
@ -35,15 +35,15 @@
\begin{solution}
\begin{enumerate}[label={\Alph*=}]
\item $\dfrac{6}{7} + \dfrac{8}{7}=\dfrac{6 + 8}{7}=\dfrac{14}{7} = 2$
\item $\dfrac{6}{8} + 10=\dfrac{6}{8} + \dfrac{10}{1}=\dfrac{6}{8} + \dfrac{10 \times 8}{1 \times 8}=\dfrac{6}{8} + \dfrac{80}{8}=\dfrac{6 + 80}{8}=\dfrac{86}{8} = \dfrac{43}{4}$
\item $\dfrac{1}{2} + \dfrac{3}{8}=\dfrac{1 \times 4}{2 \times 4} + \dfrac{3}{8}=\dfrac{4}{8} + \dfrac{3}{8}=\dfrac{4 + 3}{8}=\dfrac{7}{8} = \dfrac{7}{8}$
\item $\dfrac{6}{2} + \dfrac{5}{10}=\dfrac{6 \times 5}{2 \times 5} + \dfrac{5}{10}=\dfrac{30}{10} + \dfrac{5}{10}=\dfrac{30 + 5}{10}=\dfrac{35}{10} = \dfrac{7}{2}$
\item $\dfrac{1}{9} + \dfrac{4}{9}=\dfrac{1 + 4}{9}=\dfrac{5}{9} = \dfrac{5}{9}$
\item $\dfrac{2}{5} + 1=\dfrac{2}{5} + \dfrac{1}{1}=\dfrac{2}{5} + \dfrac{1 \times 5}{1 \times 5}=\dfrac{2}{5} + \dfrac{5}{5}=\dfrac{2 + 5}{5}=\dfrac{7}{5} = \dfrac{7}{5}$
\item $\dfrac{10}{3} + \dfrac{2}{15}=\dfrac{10 \times 5}{3 \times 5} + \dfrac{2}{15}=\dfrac{50}{15} + \dfrac{2}{15}=\dfrac{50 + 2}{15}=\dfrac{52}{15} = \dfrac{52}{15}$
\item $\dfrac{5}{2} + \dfrac{10}{2}=\dfrac{5 + 10}{2}=\dfrac{15}{2} = \dfrac{15}{2}$
\item $\dfrac{6}{10} \times 1=\dfrac{6}{10} = \dfrac{3}{5}$
\item $\dfrac{3}{5} \times \dfrac{3}{5}=\dfrac{3 \times 3}{5 \times 5}=\dfrac{9}{25} = \dfrac{9}{25}$
\item $\dfrac{- 1}{9} \times \dfrac{3}{63}=\dfrac{- 1 \times 3}{9 \times 63}=\dfrac{- 3}{567} = \dfrac{- 1}{189}$
\item $\dfrac{\dfrac{7}{3}}{\dfrac{4}{3}}=\dfrac{7}{3} \times \dfrac{3}{4}=\dfrac{7 \times 3}{3 \times 4}=\dfrac{21}{12} = \dfrac{7}{4}$
\item $\dfrac{1}{7} \times 6=\dfrac{1 \times 6}{7}=\dfrac{6}{7} = \dfrac{6}{7}$
\item $\dfrac{6}{9} \times \dfrac{10}{9}=\dfrac{6 \times 10}{9 \times 9}=\dfrac{60}{81} = \dfrac{20}{27}$
\item $\dfrac{4}{5} \times \dfrac{- 10}{15}=\dfrac{4(- 10)}{5 \times 15}=\dfrac{- 40}{75} = \dfrac{- 8}{15}$
\item $\dfrac{\dfrac{10}{7}}{\dfrac{3}{8}}=\dfrac{10}{7} \times \dfrac{8}{3}=\dfrac{10 \times 8}{7 \times 3}=\dfrac{80}{21} = \dfrac{80}{21}$
\end{enumerate}
\end{solution}
@ -52,10 +52,10 @@
Le radar a pris des photos pendant l'été:
\begin{itemize}
\item en juin, il y a eu 52 photos prises dont 26 ratées.
\item en juillet, il y a eu 48 photos réussies et 35 ratées.
\item en août, il y a eu 69 photos dont une proportion de 0.28 de photos ratées.
\item en septembre, il y a eu 11 photos ratées, ce qui correspondait à 19.3\% des photos prises.
\item en juin, il y a eu 52 photos prises dont 28 ratées.
\item en juillet, il y a eu 49 photos réussies et 41 ratées.
\item en août, il y a eu 60 photos dont une proportion de 0.22 de photos ratées.
\item en septembre, il y a eu 5 photos ratées, ce qui correspondait à 9.43\% des photos prises.
\end{itemize}
\begin{enumerate}
@ -87,37 +87,35 @@
\hline
& Juin & Juillet & Août & Septembre & Total\\
\hline
Réussies & 26 & 48 & 50 & 46 & 170\\
Réussies & 24 & 49 & 47 & 48 & 168\\
\hline
Ratées & 26 & 35 & 19 & 11 & 91\\
Ratées & 28 & 41 & 13 & 5 & 87\\
\hline
Total & 52 & 83 & 69 & 57 & 261\\
Total & 52 & 90 & 60 & 53 & 255\\
\hline
\end{tabular}
\end{center}
\item Proportion de photos réussies
\[
\frac{170}{261} = 0.65 = 65\%
\frac{168}{255} = 0.66 = 65\%
\]
\item
\begin{itemize}
\item De juin à juillet
\[
\frac{35 - 26}{26} = \frac{9}{26} = 0.35 = 34\%
\frac{41 - 28}{28} = \frac{13}{28} = 0.46 = 46\%
\]
\item De juillet à août
\[
\frac{19 - 35}{35} = \frac{-16}{35} = -0.46 = -45\%
\frac{13 - 41}{41} = \frac{-28}{41} = -0.68 = -68\%
\]
\item De août à septembre
\[
\frac{11 - 19}{19} = \frac{-8}{19} = -0.42 = -42\%
\frac{5 - 13}{13} = \frac{-8}{13} = -0.62 = -61\%
\]
\end{itemize}
\end{enumerate}
\end{solution}
\printsolutionstype{exercise}
\end{document}

View File

@ -17,17 +17,17 @@
Détailler les calculs suivants et donner le résultat sous la forme d'une fraction irréductible.
\begin{multicols}{3}
\begin{enumerate}[label={\Alph*=}]
\item $\dfrac{7}{9} + \dfrac{1}{9}$
\item $\dfrac{9}{3} + 1$
\item $\dfrac{7}{9} + \dfrac{10}{9}$
\item $\dfrac{5}{10} + 3$
\item $\dfrac{1}{2} + \dfrac{9}{6}$
\item $\dfrac{6}{9} + \dfrac{3}{2}$
\item $\dfrac{8}{7} + \dfrac{4}{7}$
\item $\dfrac{10}{4} + \dfrac{5}{2}$
\item $\dfrac{4}{8} \times 3$
\item $\dfrac{5}{4} \times \dfrac{1}{4}$
\item $\dfrac{4}{3} \times 1$
\item $\dfrac{2}{10} \times \dfrac{8}{10}$
\item $\dfrac{- 2}{3} \times \dfrac{- 2}{30}$
\item $\dfrac{\dfrac{5}{8}}{\dfrac{7}{3}}$
\item $\dfrac{- 5}{9} \times \dfrac{7}{45}$
\item $\dfrac{\dfrac{9}{6}}{\dfrac{3}{7}}$
\end{enumerate}
\end{multicols}
\end{exercise}
@ -35,15 +35,15 @@
\begin{solution}
\begin{enumerate}[label={\Alph*=}]
\item $\dfrac{7}{9} + \dfrac{1}{9}=\dfrac{7 + 1}{9}=\dfrac{8}{9} = \dfrac{8}{9}$
\item $\dfrac{9}{3} + 1=\dfrac{9}{3} + \dfrac{1}{1}=\dfrac{9}{3} + \dfrac{1 \times 3}{1 \times 3}=\dfrac{9}{3} + \dfrac{3}{3}=\dfrac{9 + 3}{3}=\dfrac{12}{3} = 4$
\item $\dfrac{1}{2} + \dfrac{9}{6}=\dfrac{1 \times 3}{2 \times 3} + \dfrac{9}{6}=\dfrac{3}{6} + \dfrac{9}{6}=\dfrac{3 + 9}{6}=\dfrac{12}{6} = 2$
\item $\dfrac{6}{9} + \dfrac{3}{2}=\dfrac{6 \times 2}{9 \times 2} + \dfrac{3 \times 9}{2 \times 9}=\dfrac{12}{18} + \dfrac{27}{18}=\dfrac{12 + 27}{18}=\dfrac{39}{18} = \dfrac{13}{6}$
\item $\dfrac{7}{9} + \dfrac{10}{9}=\dfrac{7 + 10}{9}=\dfrac{17}{9} = \dfrac{17}{9}$
\item $\dfrac{5}{10} + 3=\dfrac{5}{10} + \dfrac{3}{1}=\dfrac{5}{10} + \dfrac{3 \times 10}{1 \times 10}=\dfrac{5}{10} + \dfrac{30}{10}=\dfrac{5 + 30}{10}=\dfrac{35}{10} = \dfrac{7}{2}$
\item $\dfrac{8}{7} + \dfrac{4}{7}=\dfrac{8 + 4}{7}=\dfrac{12}{7} = \dfrac{12}{7}$
\item $\dfrac{10}{4} + \dfrac{5}{2}=\dfrac{10}{4} + \dfrac{5 \times 2}{2 \times 2}=\dfrac{10}{4} + \dfrac{10}{4}=\dfrac{10 + 10}{4}=\dfrac{20}{4} = 5$
\item $\dfrac{4}{8} \times 3=\dfrac{4 \times 3}{8}=\dfrac{12}{8} = \dfrac{3}{2}$
\item $\dfrac{5}{4} \times \dfrac{1}{4}=\dfrac{5 \times 1}{4 \times 4}=\dfrac{5}{16} = \dfrac{5}{16}$
\item $\dfrac{- 2}{3} \times \dfrac{- 2}{30}=\dfrac{- 2(- 2)}{3 \times 30}=\dfrac{4}{90} = \dfrac{2}{45}$
\item $\dfrac{\dfrac{5}{8}}{\dfrac{7}{3}}=\dfrac{5}{8} \times \dfrac{3}{7}=\dfrac{5 \times 3}{8 \times 7}=\dfrac{15}{56} = \dfrac{15}{56}$
\item $\dfrac{4}{3} \times 1=\dfrac{4}{3} = \dfrac{4}{3}$
\item $\dfrac{2}{10} \times \dfrac{8}{10}=\dfrac{2 \times 8}{10 \times 10}=\dfrac{16}{100} = \dfrac{4}{25}$
\item $\dfrac{- 5}{9} \times \dfrac{7}{45}=\dfrac{- 5 \times 7}{9 \times 45}=\dfrac{- 35}{405} = \dfrac{- 7}{81}$
\item $\dfrac{\dfrac{9}{6}}{\dfrac{3}{7}}=\dfrac{9}{6} \times \dfrac{7}{3}=\dfrac{9 \times 7}{6 \times 3}=\dfrac{63}{18} = \dfrac{7}{2}$
\end{enumerate}
\end{solution}
@ -52,10 +52,10 @@
Le radar a pris des photos pendant l'été:
\begin{itemize}
\item en juin, il y a eu 49 photos prises dont 22 ratées.
\item en juillet, il y a eu 44 photos réussies et 43 ratées.
\item en août, il y a eu 65 photos dont une proportion de 0.28 de photos ratées.
\item en septembre, il y a eu 14 photos ratées, ce qui correspondait à 25.45\% des photos prises.
\item en juin, il y a eu 50 photos prises dont 28 ratées.
\item en juillet, il y a eu 48 photos réussies et 44 ratées.
\item en août, il y a eu 58 photos dont une proportion de 0.17 de photos ratées.
\item en septembre, il y a eu 6 photos ratées, ce qui correspondait à 10.71\% des photos prises.
\end{itemize}
\begin{enumerate}
@ -87,37 +87,35 @@
\hline
& Juin & Juillet & Août & Septembre & Total\\
\hline
Réussies & 27 & 44 & 47 & 41 & 159\\
Réussies & 22 & 48 & 48 & 50 & 168\\
\hline
Ratées & 22 & 43 & 18 & 14 & 97\\
Ratées & 28 & 44 & 10 & 6 & 88\\
\hline
Total & 49 & 87 & 65 & 55 & 256\\
Total & 50 & 92 & 58 & 56 & 256\\
\hline
\end{tabular}
\end{center}
\item Proportion de photos réussies
\[
\frac{159}{256} = 0.62 = 62\%
\frac{168}{256} = 0.66 = 65\%
\]
\item
\begin{itemize}
\item De juin à juillet
\[
\frac{43 - 22}{22} = \frac{21}{22} = 0.95 = 95\%
\frac{44 - 28}{28} = \frac{16}{28} = 0.57 = 57\%
\]
\item De juillet à août
\[
\frac{18 - 43}{43} = \frac{-25}{43} = -0.58 = -58\%
\frac{10 - 44}{44} = \frac{-34}{44} = -0.77 = -77\%
\]
\item De août à septembre
\[
\frac{14 - 18}{18} = \frac{-4}{18} = -0.22 = -22\%
\frac{6 - 10}{10} = \frac{-4}{10} = -0.4 = -40\%
\]
\end{itemize}
\end{enumerate}
\end{solution}
\printsolutionstype{exercise}
\end{document}

View File

@ -17,17 +17,17 @@
Détailler les calculs suivants et donner le résultat sous la forme d'une fraction irréductible.
\begin{multicols}{3}
\begin{enumerate}[label={\Alph*=}]
\item $\dfrac{6}{9} + \dfrac{1}{9}$
\item $\dfrac{3}{2} + 4$
\item $\dfrac{7}{6} + \dfrac{5}{6}$
\item $\dfrac{10}{3} + 8$
\item $\dfrac{10}{8} + \dfrac{5}{80}$
\item $\dfrac{8}{5} + \dfrac{3}{10}$
\item $\dfrac{7}{4} + \dfrac{2}{28}$
\item $\dfrac{6}{8} + \dfrac{5}{3}$
\item $\dfrac{7}{2} \times 7$
\item $\dfrac{5}{9} \times \dfrac{6}{9}$
\item $\dfrac{8}{5} \times 10$
\item $\dfrac{5}{7} \times \dfrac{4}{7}$
\item $\dfrac{8}{3} \times \dfrac{- 9}{9}$
\item $\dfrac{\dfrac{2}{4}}{\dfrac{1}{6}}$
\item $\dfrac{- 1}{9} \times \dfrac{10}{72}$
\item $\dfrac{\dfrac{7}{6}}{\dfrac{8}{2}}$
\end{enumerate}
\end{multicols}
\end{exercise}
@ -35,15 +35,15 @@
\begin{solution}
\begin{enumerate}[label={\Alph*=}]
\item $\dfrac{6}{9} + \dfrac{1}{9}=\dfrac{6 + 1}{9}=\dfrac{7}{9} = \dfrac{7}{9}$
\item $\dfrac{3}{2} + 4=\dfrac{3}{2} + \dfrac{4}{1}=\dfrac{3}{2} + \dfrac{4 \times 2}{1 \times 2}=\dfrac{3}{2} + \dfrac{8}{2}=\dfrac{3 + 8}{2}=\dfrac{11}{2} = \dfrac{11}{2}$
\item $\dfrac{10}{8} + \dfrac{5}{80}=\dfrac{10 \times 10}{8 \times 10} + \dfrac{5}{80}=\dfrac{100}{80} + \dfrac{5}{80}=\dfrac{100 + 5}{80}=\dfrac{105}{80} = \dfrac{21}{16}$
\item $\dfrac{8}{5} + \dfrac{3}{10}=\dfrac{8 \times 2}{5 \times 2} + \dfrac{3}{10}=\dfrac{16}{10} + \dfrac{3}{10}=\dfrac{16 + 3}{10}=\dfrac{19}{10} = \dfrac{19}{10}$
\item $\dfrac{7}{6} + \dfrac{5}{6}=\dfrac{7 + 5}{6}=\dfrac{12}{6} = 2$
\item $\dfrac{10}{3} + 8=\dfrac{10}{3} + \dfrac{8}{1}=\dfrac{10}{3} + \dfrac{8 \times 3}{1 \times 3}=\dfrac{10}{3} + \dfrac{24}{3}=\dfrac{10 + 24}{3}=\dfrac{34}{3} = \dfrac{34}{3}$
\item $\dfrac{7}{4} + \dfrac{2}{28}=\dfrac{7 \times 7}{4 \times 7} + \dfrac{2}{28}=\dfrac{49}{28} + \dfrac{2}{28}=\dfrac{49 + 2}{28}=\dfrac{51}{28} = \dfrac{51}{28}$
\item $\dfrac{6}{8} + \dfrac{5}{3}=\dfrac{6 \times 3}{8 \times 3} + \dfrac{5 \times 8}{3 \times 8}=\dfrac{18}{24} + \dfrac{40}{24}=\dfrac{18 + 40}{24}=\dfrac{58}{24} = \dfrac{29}{12}$
\item $\dfrac{7}{2} \times 7=\dfrac{7 \times 7}{2}=\dfrac{49}{2} = \dfrac{49}{2}$
\item $\dfrac{5}{9} \times \dfrac{6}{9}=\dfrac{5 \times 6}{9 \times 9}=\dfrac{30}{81} = \dfrac{10}{27}$
\item $\dfrac{8}{3} \times \dfrac{- 9}{9}=\dfrac{8(- 9)}{3 \times 9}=\dfrac{- 72}{27} = \dfrac{- 8}{3}$
\item $\dfrac{\dfrac{2}{4}}{\dfrac{1}{6}}=\dfrac{2}{4} \times \dfrac{6}{1}=\dfrac{2 \times 6}{4 \times 1}=\dfrac{12}{4} = 3$
\item $\dfrac{8}{5} \times 10=\dfrac{8 \times 10}{5}=\dfrac{80}{5} = 16$
\item $\dfrac{5}{7} \times \dfrac{4}{7}=\dfrac{5 \times 4}{7 \times 7}=\dfrac{20}{49} = \dfrac{20}{49}$
\item $\dfrac{- 1}{9} \times \dfrac{10}{72}=\dfrac{- 1 \times 10}{9 \times 72}=\dfrac{- 10}{648} = \dfrac{- 5}{324}$
\item $\dfrac{\dfrac{7}{6}}{\dfrac{8}{2}}=\dfrac{7}{6} \times \dfrac{2}{8}=\dfrac{7 \times 2}{6 \times 8}=\dfrac{14}{48} = \dfrac{7}{24}$
\end{enumerate}
\end{solution}
@ -52,10 +52,10 @@
Le radar a pris des photos pendant l'été:
\begin{itemize}
\item en juin, il y a eu 59 photos prises dont 30 ratées.
\item en juillet, il y a eu 49 photos réussies et 31 ratées.
\item en août, il y a eu 59 photos dont une proportion de 0.24 de photos ratées.
\item en septembre, il y a eu 13 photos ratées, ce qui correspondait à 20.63\% des photos prises.
\item en juin, il y a eu 54 photos prises dont 27 ratées.
\item en juillet, il y a eu 43 photos réussies et 34 ratées.
\item en août, il y a eu 64 photos dont une proportion de 0.31 de photos ratées.
\item en septembre, il y a eu 5 photos ratées, ce qui correspondait à 11.11\% des photos prises.
\end{itemize}
\begin{enumerate}
@ -87,37 +87,35 @@
\hline
& Juin & Juillet & Août & Septembre & Total\\
\hline
Réussies & 29 & 49 & 45 & 50 & 173\\
Réussies & 27 & 43 & 44 & 40 & 154\\
\hline
Ratées & 30 & 31 & 14 & 13 & 88\\
Ratées & 27 & 34 & 20 & 5 & 86\\
\hline
Total & 59 & 80 & 59 & 63 & 261\\
Total & 54 & 77 & 64 & 45 & 240\\
\hline
\end{tabular}
\end{center}
\item Proportion de photos réussies
\[
\frac{173}{261} = 0.66 = 66\%
\frac{154}{240} = 0.64 = 64\%
\]
\item
\begin{itemize}
\item De juin à juillet
\[
\frac{31 - 30}{30} = \frac{1}{30} = 0.03 = 3\%
\frac{34 - 27}{27} = \frac{7}{27} = 0.26 = 25\%
\]
\item De juillet à août
\[
\frac{14 - 31}{31} = \frac{-17}{31} = -0.55 = -54\%
\frac{20 - 34}{34} = \frac{-14}{34} = -0.41 = -41\%
\]
\item De août à septembre
\[
\frac{13 - 14}{14} = \frac{-1}{14} = -0.07 = -7\%
\frac{5 - 20}{20} = \frac{-15}{20} = -0.75 = -75\%
\]
\end{itemize}
\end{enumerate}
\end{solution}
\printsolutionstype{exercise}
\end{document}

View File

@ -17,17 +17,17 @@
Détailler les calculs suivants et donner le résultat sous la forme d'une fraction irréductible.
\begin{multicols}{3}
\begin{enumerate}[label={\Alph*=}]
\item $\dfrac{9}{7} + \dfrac{2}{7}$
\item $\dfrac{3}{9} + 5$
\item $\dfrac{2}{8} + \dfrac{1}{8}$
\item $\dfrac{3}{5} + 10$
\item $\dfrac{5}{2} + \dfrac{4}{12}$
\item $\dfrac{8}{6} + \dfrac{9}{7}$
\item $\dfrac{2}{4} + \dfrac{8}{24}$
\item $\dfrac{2}{9} + \dfrac{10}{7}$
\item $\dfrac{6}{10} \times 3$
\item $\dfrac{4}{6} \times \dfrac{9}{6}$
\item $\dfrac{4}{6} \times 4$
\item $\dfrac{2}{3} \times \dfrac{10}{3}$
\item $\dfrac{3}{10} \times \dfrac{- 9}{90}$
\item $\dfrac{\dfrac{9}{5}}{\dfrac{10}{3}}$
\item $\dfrac{- 9}{7} \times \dfrac{- 2}{14}$
\item $\dfrac{\dfrac{4}{6}}{\dfrac{3}{6}}$
\end{enumerate}
\end{multicols}
\end{exercise}
@ -35,15 +35,15 @@
\begin{solution}
\begin{enumerate}[label={\Alph*=}]
\item $\dfrac{9}{7} + \dfrac{2}{7}=\dfrac{9 + 2}{7}=\dfrac{11}{7} = \dfrac{11}{7}$
\item $\dfrac{3}{9} + 5=\dfrac{3}{9} + \dfrac{5}{1}=\dfrac{3}{9} + \dfrac{5 \times 9}{1 \times 9}=\dfrac{3}{9} + \dfrac{45}{9}=\dfrac{3 + 45}{9}=\dfrac{48}{9} = \dfrac{16}{3}$
\item $\dfrac{5}{2} + \dfrac{4}{12}=\dfrac{5 \times 6}{2 \times 6} + \dfrac{4}{12}=\dfrac{30}{12} + \dfrac{4}{12}=\dfrac{30 + 4}{12}=\dfrac{34}{12} = \dfrac{17}{6}$
\item $\dfrac{8}{6} + \dfrac{9}{7}=\dfrac{8 \times 7}{6 \times 7} + \dfrac{9 \times 6}{7 \times 6}=\dfrac{56}{42} + \dfrac{54}{42}=\dfrac{56 + 54}{42}=\dfrac{110}{42} = \dfrac{55}{21}$
\item $\dfrac{2}{8} + \dfrac{1}{8}=\dfrac{2 + 1}{8}=\dfrac{3}{8} = \dfrac{3}{8}$
\item $\dfrac{3}{5} + 10=\dfrac{3}{5} + \dfrac{10}{1}=\dfrac{3}{5} + \dfrac{10 \times 5}{1 \times 5}=\dfrac{3}{5} + \dfrac{50}{5}=\dfrac{3 + 50}{5}=\dfrac{53}{5} = \dfrac{53}{5}$
\item $\dfrac{2}{4} + \dfrac{8}{24}=\dfrac{2 \times 6}{4 \times 6} + \dfrac{8}{24}=\dfrac{12}{24} + \dfrac{8}{24}=\dfrac{12 + 8}{24}=\dfrac{20}{24} = \dfrac{5}{6}$
\item $\dfrac{2}{9} + \dfrac{10}{7}=\dfrac{2 \times 7}{9 \times 7} + \dfrac{10 \times 9}{7 \times 9}=\dfrac{14}{63} + \dfrac{90}{63}=\dfrac{14 + 90}{63}=\dfrac{104}{63} = \dfrac{104}{63}$
\item $\dfrac{6}{10} \times 3=\dfrac{6 \times 3}{10}=\dfrac{18}{10} = \dfrac{9}{5}$
\item $\dfrac{4}{6} \times \dfrac{9}{6}=\dfrac{4 \times 9}{6 \times 6}=\dfrac{36}{36} = 1$
\item $\dfrac{3}{10} \times \dfrac{- 9}{90}=\dfrac{3(- 9)}{10 \times 90}=\dfrac{- 27}{900} = \dfrac{- 3}{100}$
\item $\dfrac{\dfrac{9}{5}}{\dfrac{10}{3}}=\dfrac{9}{5} \times \dfrac{3}{10}=\dfrac{9 \times 3}{5 \times 10}=\dfrac{27}{50} = \dfrac{27}{50}$
\item $\dfrac{4}{6} \times 4=\dfrac{4 \times 4}{6}=\dfrac{16}{6} = \dfrac{8}{3}$
\item $\dfrac{2}{3} \times \dfrac{10}{3}=\dfrac{2 \times 10}{3 \times 3}=\dfrac{20}{9} = \dfrac{20}{9}$
\item $\dfrac{- 9}{7} \times \dfrac{- 2}{14}=\dfrac{- 9(- 2)}{7 \times 14}=\dfrac{18}{98} = \dfrac{9}{49}$
\item $\dfrac{\dfrac{4}{6}}{\dfrac{3}{6}}=\dfrac{4}{6} \times \dfrac{6}{3}=\dfrac{4 \times 6}{6 \times 3}=\dfrac{24}{18} = \dfrac{4}{3}$
\end{enumerate}
\end{solution}
@ -52,10 +52,10 @@
Le radar a pris des photos pendant l'été:
\begin{itemize}
\item en juin, il y a eu 54 photos prises dont 30 ratées.
\item en juillet, il y a eu 44 photos réussies et 48 ratées.
\item en août, il y a eu 53 photos dont une proportion de 0.23 de photos ratées.
\item en septembre, il y a eu 10 photos ratées, ce qui correspondait à 17.86\% des photos prises.
\item en juin, il y a eu 52 photos prises dont 26 ratées.
\item en juillet, il y a eu 47 photos réussies et 42 ratées.
\item en août, il y a eu 61 photos dont une proportion de 0.31 de photos ratées.
\item en septembre, il y a eu 11 photos ratées, ce qui correspondait à 21.57\% des photos prises.
\end{itemize}
\begin{enumerate}
@ -87,37 +87,35 @@
\hline
& Juin & Juillet & Août & Septembre & Total\\
\hline
Réussies & 24 & 44 & 41 & 46 & 155\\
Réussies & 26 & 47 & 42 & 40 & 155\\
\hline
Ratées & 30 & 48 & 12 & 10 & 100\\
Ratées & 26 & 42 & 19 & 11 & 98\\
\hline
Total & 54 & 92 & 53 & 56 & 255\\
Total & 52 & 89 & 61 & 51 & 253\\
\hline
\end{tabular}
\end{center}
\item Proportion de photos réussies
\[
\frac{155}{255} = 0.61 = 60\%
\frac{155}{253} = 0.61 = 61\%
\]
\item
\begin{itemize}
\item De juin à juillet
\[
\frac{48 - 30}{30} = \frac{18}{30} = 0.6 = 60\%
\frac{42 - 26}{26} = \frac{16}{26} = 0.62 = 61\%
\]
\item De juillet à août
\[
\frac{12 - 48}{48} = \frac{-36}{48} = -0.75 = -75\%
\frac{19 - 42}{42} = \frac{-23}{42} = -0.55 = -54\%
\]
\item De août à septembre
\[
\frac{10 - 12}{12} = \frac{-2}{12} = -0.17 = -16\%
\frac{11 - 19}{19} = \frac{-8}{19} = -0.42 = -42\%
\]
\end{itemize}
\end{enumerate}
\end{solution}
\printsolutionstype{exercise}
\end{document}

View File

@ -17,17 +17,17 @@
Détailler les calculs suivants et donner le résultat sous la forme d'une fraction irréductible.
\begin{multicols}{3}
\begin{enumerate}[label={\Alph*=}]
\item $\dfrac{9}{5} + \dfrac{8}{5}$
\item $\dfrac{2}{9} + 3$
\item $\dfrac{2}{3} + \dfrac{2}{3}$
\item $\dfrac{5}{8} + 9$
\item $\dfrac{4}{5} + \dfrac{10}{25}$
\item $\dfrac{6}{3} + \dfrac{10}{7}$
\item $\dfrac{3}{2} + \dfrac{9}{2}$
\item $\dfrac{5}{7} + \dfrac{4}{9}$
\item $\dfrac{3}{8} \times 2$
\item $\dfrac{1}{2} \times \dfrac{9}{2}$
\item $\dfrac{4}{10} \times 1$
\item $\dfrac{9}{4} \times \dfrac{8}{4}$
\item $\dfrac{- 2}{7} \times \dfrac{- 8}{70}$
\item $\dfrac{\dfrac{1}{4}}{\dfrac{9}{7}}$
\item $\dfrac{- 10}{6} \times \dfrac{- 5}{18}$
\item $\dfrac{\dfrac{9}{2}}{\dfrac{4}{7}}$
\end{enumerate}
\end{multicols}
\end{exercise}
@ -35,15 +35,15 @@
\begin{solution}
\begin{enumerate}[label={\Alph*=}]
\item $\dfrac{9}{5} + \dfrac{8}{5}=\dfrac{9 + 8}{5}=\dfrac{17}{5} = \dfrac{17}{5}$
\item $\dfrac{2}{9} + 3=\dfrac{2}{9} + \dfrac{3}{1}=\dfrac{2}{9} + \dfrac{3 \times 9}{1 \times 9}=\dfrac{2}{9} + \dfrac{27}{9}=\dfrac{2 + 27}{9}=\dfrac{29}{9} = \dfrac{29}{9}$
\item $\dfrac{4}{5} + \dfrac{10}{25}=\dfrac{4 \times 5}{5 \times 5} + \dfrac{10}{25}=\dfrac{20}{25} + \dfrac{10}{25}=\dfrac{20 + 10}{25}=\dfrac{30}{25} = \dfrac{6}{5}$
\item $\dfrac{6}{3} + \dfrac{10}{7}=\dfrac{6 \times 7}{3 \times 7} + \dfrac{10 \times 3}{7 \times 3}=\dfrac{42}{21} + \dfrac{30}{21}=\dfrac{42 + 30}{21}=\dfrac{72}{21} = \dfrac{24}{7}$
\item $\dfrac{2}{3} + \dfrac{2}{3}=\dfrac{2 + 2}{3}=\dfrac{4}{3} = \dfrac{4}{3}$
\item $\dfrac{5}{8} + 9=\dfrac{5}{8} + \dfrac{9}{1}=\dfrac{5}{8} + \dfrac{9 \times 8}{1 \times 8}=\dfrac{5}{8} + \dfrac{72}{8}=\dfrac{5 + 72}{8}=\dfrac{77}{8} = \dfrac{77}{8}$
\item $\dfrac{3}{2} + \dfrac{9}{2}=\dfrac{3 + 9}{2}=\dfrac{12}{2} = 6$
\item $\dfrac{5}{7} + \dfrac{4}{9}=\dfrac{5 \times 9}{7 \times 9} + \dfrac{4 \times 7}{9 \times 7}=\dfrac{45}{63} + \dfrac{28}{63}=\dfrac{45 + 28}{63}=\dfrac{73}{63} = \dfrac{73}{63}$
\item $\dfrac{3}{8} \times 2=\dfrac{3 \times 2}{8}=\dfrac{6}{8} = \dfrac{3}{4}$
\item $\dfrac{1}{2} \times \dfrac{9}{2}=\dfrac{1 \times 9}{2 \times 2}=\dfrac{9}{4} = \dfrac{9}{4}$
\item $\dfrac{- 2}{7} \times \dfrac{- 8}{70}=\dfrac{- 2(- 8)}{7 \times 70}=\dfrac{16}{490} = \dfrac{8}{245}$
\item $\dfrac{\dfrac{1}{4}}{\dfrac{9}{7}}=\dfrac{1}{4} \times \dfrac{7}{9}=\dfrac{1 \times 7}{4 \times 9}=\dfrac{7}{36} = \dfrac{7}{36}$
\item $\dfrac{4}{10} \times 1=\dfrac{4}{10} = \dfrac{2}{5}$
\item $\dfrac{9}{4} \times \dfrac{8}{4}=\dfrac{9 \times 8}{4 \times 4}=\dfrac{72}{16} = \dfrac{9}{2}$
\item $\dfrac{- 10}{6} \times \dfrac{- 5}{18}=\dfrac{- 10(- 5)}{6 \times 18}=\dfrac{50}{108} = \dfrac{25}{54}$
\item $\dfrac{\dfrac{9}{2}}{\dfrac{4}{7}}=\dfrac{9}{2} \times \dfrac{7}{4}=\dfrac{9 \times 7}{2 \times 4}=\dfrac{63}{8} = \dfrac{63}{8}$
\end{enumerate}
\end{solution}
@ -52,10 +52,10 @@
Le radar a pris des photos pendant l'été:
\begin{itemize}
\item en juin, il y a eu 54 photos prises dont 25 ratées.
\item en juillet, il y a eu 39 photos réussies et 37 ratées.
\item en août, il y a eu 58 photos dont une proportion de 0.26 de photos ratées.
\item en septembre, il y a eu 9 photos ratées, ce qui correspondait à 15.52\% des photos prises.
\item en juin, il y a eu 43 photos prises dont 20 ratées.
\item en juillet, il y a eu 32 photos réussies et 34 ratées.
\item en août, il y a eu 60 photos dont une proportion de 0.17 de photos ratées.
\item en septembre, il y a eu 10 photos ratées, ce qui correspondait à 20.0\% des photos prises.
\end{itemize}
\begin{enumerate}
@ -87,37 +87,35 @@
\hline
& Juin & Juillet & Août & Septembre & Total\\
\hline
Réussies & 29 & 39 & 43 & 49 & 160\\
Réussies & 23 & 32 & 50 & 40 & 145\\
\hline
Ratées & 25 & 37 & 15 & 9 & 86\\
Ratées & 20 & 34 & 10 & 10 & 74\\
\hline
Total & 54 & 76 & 58 & 58 & 246\\
Total & 43 & 66 & 60 & 50 & 219\\
\hline
\end{tabular}
\end{center}
\item Proportion de photos réussies
\[
\frac{160}{246} = 0.65 = 65\%
\frac{145}{219} = 0.66 = 66\%
\]
\item
\begin{itemize}
\item De juin à juillet
\[
\frac{37 - 25}{25} = \frac{12}{25} = 0.48 = 48\%
\frac{34 - 20}{20} = \frac{14}{20} = 0.7 = 70\%
\]
\item De juillet à août
\[
\frac{15 - 37}{37} = \frac{-22}{37} = -0.59 = -59\%
\frac{10 - 34}{34} = \frac{-24}{34} = -0.71 = -70\%
\]
\item De août à septembre
\[
\frac{9 - 15}{15} = \frac{-6}{15} = -0.4 = -40\%
\frac{10 - 10}{10} = \frac{0}{10} = 0.0 = 0\%
\]
\end{itemize}
\end{enumerate}
\end{solution}
\printsolutionstype{exercise}
\end{document}

View File

@ -17,17 +17,17 @@
Détailler les calculs suivants et donner le résultat sous la forme d'une fraction irréductible.
\begin{multicols}{3}
\begin{enumerate}[label={\Alph*=}]
\item $\dfrac{2}{9} + \dfrac{2}{9}$
\item $\dfrac{9}{6} + 9$
\item $\dfrac{10}{2} + \dfrac{4}{2}$
\item $\dfrac{7}{2} + 7$
\item $\dfrac{9}{4} + \dfrac{8}{12}$
\item $\dfrac{1}{7} + \dfrac{6}{8}$
\item $\dfrac{9}{10} + \dfrac{7}{10}$
\item $\dfrac{5}{10} + \dfrac{7}{10}$
\item $\dfrac{6}{10} \times 4$
\item $\dfrac{3}{4} \times \dfrac{2}{4}$
\item $\dfrac{4}{9} \times 7$
\item $\dfrac{4}{7} \times \dfrac{6}{7}$
\item $\dfrac{- 6}{10} \times \dfrac{6}{30}$
\item $\dfrac{\dfrac{7}{3}}{\dfrac{4}{3}}$
\item $\dfrac{- 2}{7} \times \dfrac{2}{42}$
\item $\dfrac{\dfrac{9}{10}}{\dfrac{6}{4}}$
\end{enumerate}
\end{multicols}
\end{exercise}
@ -35,15 +35,15 @@
\begin{solution}
\begin{enumerate}[label={\Alph*=}]
\item $\dfrac{2}{9} + \dfrac{2}{9}=\dfrac{2 + 2}{9}=\dfrac{4}{9} = \dfrac{4}{9}$
\item $\dfrac{9}{6} + 9=\dfrac{9}{6} + \dfrac{9}{1}=\dfrac{9}{6} + \dfrac{9 \times 6}{1 \times 6}=\dfrac{9}{6} + \dfrac{54}{6}=\dfrac{9 + 54}{6}=\dfrac{63}{6} = \dfrac{21}{2}$
\item $\dfrac{9}{4} + \dfrac{8}{12}=\dfrac{9 \times 3}{4 \times 3} + \dfrac{8}{12}=\dfrac{27}{12} + \dfrac{8}{12}=\dfrac{27 + 8}{12}=\dfrac{35}{12} = \dfrac{35}{12}$
\item $\dfrac{1}{7} + \dfrac{6}{8}=\dfrac{1 \times 8}{7 \times 8} + \dfrac{6 \times 7}{8 \times 7}=\dfrac{8}{56} + \dfrac{42}{56}=\dfrac{8 + 42}{56}=\dfrac{50}{56} = \dfrac{25}{28}$
\item $\dfrac{10}{2} + \dfrac{4}{2}=\dfrac{10 + 4}{2}=\dfrac{14}{2} = 7$
\item $\dfrac{7}{2} + 7=\dfrac{7}{2} + \dfrac{7}{1}=\dfrac{7}{2} + \dfrac{7 \times 2}{1 \times 2}=\dfrac{7}{2} + \dfrac{14}{2}=\dfrac{7 + 14}{2}=\dfrac{21}{2} = \dfrac{21}{2}$
\item $\dfrac{9}{10} + \dfrac{7}{10}=\dfrac{9 + 7}{10}=\dfrac{16}{10} = \dfrac{8}{5}$
\item $\dfrac{5}{10} + \dfrac{7}{10}=\dfrac{5 + 7}{10}=\dfrac{12}{10} = \dfrac{6}{5}$
\item $\dfrac{6}{10} \times 4=\dfrac{6 \times 4}{10}=\dfrac{24}{10} = \dfrac{12}{5}$
\item $\dfrac{3}{4} \times \dfrac{2}{4}=\dfrac{3 \times 2}{4 \times 4}=\dfrac{6}{16} = \dfrac{3}{8}$
\item $\dfrac{- 6}{10} \times \dfrac{6}{30}=\dfrac{- 6 \times 6}{10 \times 30}=\dfrac{- 36}{300} = \dfrac{- 3}{25}$
\item $\dfrac{\dfrac{7}{3}}{\dfrac{4}{3}}=\dfrac{7}{3} \times \dfrac{3}{4}=\dfrac{7 \times 3}{3 \times 4}=\dfrac{21}{12} = \dfrac{7}{4}$
\item $\dfrac{4}{9} \times 7=\dfrac{4 \times 7}{9}=\dfrac{28}{9} = \dfrac{28}{9}$
\item $\dfrac{4}{7} \times \dfrac{6}{7}=\dfrac{4 \times 6}{7 \times 7}=\dfrac{24}{49} = \dfrac{24}{49}$
\item $\dfrac{- 2}{7} \times \dfrac{2}{42}=\dfrac{- 2 \times 2}{7 \times 42}=\dfrac{- 4}{294} = \dfrac{- 2}{147}$
\item $\dfrac{\dfrac{9}{10}}{\dfrac{6}{4}}=\dfrac{9}{10} \times \dfrac{4}{6}=\dfrac{9 \times 4}{10 \times 6}=\dfrac{36}{60} = \dfrac{3}{5}$
\end{enumerate}
\end{solution}
@ -52,10 +52,10 @@
Le radar a pris des photos pendant l'été:
\begin{itemize}
\item en juin, il y a eu 52 photos prises dont 25 ratées.
\item en juillet, il y a eu 49 photos réussies et 36 ratées.
\item en août, il y a eu 60 photos dont une proportion de 0.22 de photos ratées.
\item en septembre, il y a eu 5 photos ratées, ce qui correspondait à 9.62\% des photos prises.
\item en juin, il y a eu 52 photos prises dont 26 ratées.
\item en juillet, il y a eu 30 photos réussies et 40 ratées.
\item en août, il y a eu 61 photos dont une proportion de 0.21 de photos ratées.
\item en septembre, il y a eu 9 photos ratées, ce qui correspondait à 17.65\% des photos prises.
\end{itemize}
\begin{enumerate}
@ -87,37 +87,35 @@
\hline
& Juin & Juillet & Août & Septembre & Total\\
\hline
Réussies & 27 & 49 & 47 & 47 & 170\\
Réussies & 26 & 30 & 48 & 42 & 146\\
\hline
Ratées & 25 & 36 & 13 & 5 & 79\\
Ratées & 26 & 40 & 13 & 9 & 88\\
\hline
Total & 52 & 85 & 60 & 52 & 249\\
Total & 52 & 70 & 61 & 51 & 234\\
\hline
\end{tabular}
\end{center}
\item Proportion de photos réussies
\[
\frac{170}{249} = 0.68 = 68\%
\frac{146}{234} = 0.62 = 62\%
\]
\item
\begin{itemize}
\item De juin à juillet
\[
\frac{36 - 25}{25} = \frac{11}{25} = 0.44 = 44\%
\frac{40 - 26}{26} = \frac{14}{26} = 0.54 = 53\%
\]
\item De juillet à août
\[
\frac{13 - 36}{36} = \frac{-23}{36} = -0.64 = -63\%
\frac{13 - 40}{40} = \frac{-27}{40} = -0.68 = -67\%
\]
\item De août à septembre
\[
\frac{5 - 13}{13} = \frac{-8}{13} = -0.62 = -61\%
\frac{9 - 13}{13} = \frac{-4}{13} = -0.31 = -30\%
\]
\end{itemize}
\end{enumerate}
\end{solution}
\printsolutionstype{exercise}
\end{document}

View File

@ -17,17 +17,17 @@
Détailler les calculs suivants et donner le résultat sous la forme d'une fraction irréductible.
\begin{multicols}{3}
\begin{enumerate}[label={\Alph*=}]
\item $\dfrac{1}{10} + \dfrac{2}{10}$
\item $\dfrac{4}{6} + 5$
\item $\dfrac{7}{5} + \dfrac{6}{5}$
\item $\dfrac{7}{6} + 4$
\item $\dfrac{10}{3} + \dfrac{5}{27}$
\item $\dfrac{1}{10} + \dfrac{2}{9}$
\item $\dfrac{9}{4} + \dfrac{6}{36}$
\item $\dfrac{2}{8} + \dfrac{7}{8}$
\item $\dfrac{5}{9} \times 9$
\item $\dfrac{8}{7} \times \dfrac{9}{7}$
\item $\dfrac{1}{4} \times 8$
\item $\dfrac{3}{7} \times \dfrac{10}{7}$
\item $\dfrac{- 6}{7} \times \dfrac{10}{28}$
\item $\dfrac{\dfrac{1}{7}}{\dfrac{1}{9}}$
\item $\dfrac{- 8}{3} \times \dfrac{- 3}{12}$
\item $\dfrac{\dfrac{1}{5}}{\dfrac{4}{5}}$
\end{enumerate}
\end{multicols}
\end{exercise}
@ -35,15 +35,15 @@
\begin{solution}
\begin{enumerate}[label={\Alph*=}]
\item $\dfrac{1}{10} + \dfrac{2}{10}=\dfrac{1 + 2}{10}=\dfrac{3}{10} = \dfrac{3}{10}$
\item $\dfrac{4}{6} + 5=\dfrac{4}{6} + \dfrac{5}{1}=\dfrac{4}{6} + \dfrac{5 \times 6}{1 \times 6}=\dfrac{4}{6} + \dfrac{30}{6}=\dfrac{4 + 30}{6}=\dfrac{34}{6} = \dfrac{17}{3}$
\item $\dfrac{10}{3} + \dfrac{5}{27}=\dfrac{10 \times 9}{3 \times 9} + \dfrac{5}{27}=\dfrac{90}{27} + \dfrac{5}{27}=\dfrac{90 + 5}{27}=\dfrac{95}{27} = \dfrac{95}{27}$
\item $\dfrac{1}{10} + \dfrac{2}{9}=\dfrac{1 \times 9}{10 \times 9} + \dfrac{2 \times 10}{9 \times 10}=\dfrac{9}{90} + \dfrac{20}{90}=\dfrac{9 + 20}{90}=\dfrac{29}{90} = \dfrac{29}{90}$
\item $\dfrac{7}{5} + \dfrac{6}{5}=\dfrac{7 + 6}{5}=\dfrac{13}{5} = \dfrac{13}{5}$
\item $\dfrac{7}{6} + 4=\dfrac{7}{6} + \dfrac{4}{1}=\dfrac{7}{6} + \dfrac{4 \times 6}{1 \times 6}=\dfrac{7}{6} + \dfrac{24}{6}=\dfrac{7 + 24}{6}=\dfrac{31}{6} = \dfrac{31}{6}$
\item $\dfrac{9}{4} + \dfrac{6}{36}=\dfrac{9 \times 9}{4 \times 9} + \dfrac{6}{36}=\dfrac{81}{36} + \dfrac{6}{36}=\dfrac{81 + 6}{36}=\dfrac{87}{36} = \dfrac{29}{12}$
\item $\dfrac{2}{8} + \dfrac{7}{8}=\dfrac{2 + 7}{8}=\dfrac{9}{8} = \dfrac{9}{8}$
\item $\dfrac{5}{9} \times 9=\dfrac{5 \times 9}{9}=\dfrac{45}{9} = 5$
\item $\dfrac{8}{7} \times \dfrac{9}{7}=\dfrac{8 \times 9}{7 \times 7}=\dfrac{72}{49} = \dfrac{72}{49}$
\item $\dfrac{- 6}{7} \times \dfrac{10}{28}=\dfrac{- 6 \times 10}{7 \times 28}=\dfrac{- 60}{196} = \dfrac{- 15}{49}$
\item $\dfrac{\dfrac{1}{7}}{\dfrac{1}{9}}=\dfrac{1}{7} \times \dfrac{9}{1}=\dfrac{1 \times 9}{7 \times 1}=\dfrac{9}{7} = \dfrac{9}{7}$
\item $\dfrac{1}{4} \times 8=\dfrac{1 \times 8}{4}=\dfrac{8}{4} = 2$
\item $\dfrac{3}{7} \times \dfrac{10}{7}=\dfrac{3 \times 10}{7 \times 7}=\dfrac{30}{49} = \dfrac{30}{49}$
\item $\dfrac{- 8}{3} \times \dfrac{- 3}{12}=\dfrac{- 8(- 3)}{3 \times 12}=\dfrac{24}{36} = \dfrac{2}{3}$
\item $\dfrac{\dfrac{1}{5}}{\dfrac{4}{5}}=\dfrac{1}{5} \times \dfrac{5}{4}=\dfrac{1 \times 5}{5 \times 4}=\dfrac{5}{20} = \dfrac{1}{4}$
\end{enumerate}
\end{solution}
@ -52,10 +52,10 @@
Le radar a pris des photos pendant l'été:
\begin{itemize}
\item en juin, il y a eu 55 photos prises dont 27 ratées.
\item en juillet, il y a eu 35 photos réussies et 45 ratées.
\item en août, il y a eu 66 photos dont une proportion de 0.29 de photos ratées.
\item en septembre, il y a eu 10 photos ratées, ce qui correspondait à 19.61\% des photos prises.
\item en juin, il y a eu 47 photos prises dont 25 ratées.
\item en juillet, il y a eu 45 photos réussies et 31 ratées.
\item en août, il y a eu 65 photos dont une proportion de 0.31 de photos ratées.
\item en septembre, il y a eu 11 photos ratées, ce qui correspondait à 18.97\% des photos prises.
\end{itemize}
\begin{enumerate}
@ -87,37 +87,35 @@
\hline
& Juin & Juillet & Août & Septembre & Total\\
\hline
Réussies & 28 & 35 & 47 & 41 & 151\\
Réussies & 22 & 45 & 45 & 47 & 159\\
\hline
Ratées & 27 & 45 & 19 & 10 & 101\\
Ratées & 25 & 31 & 20 & 11 & 87\\
\hline
Total & 55 & 80 & 66 & 51 & 252\\
Total & 47 & 76 & 65 & 58 & 246\\
\hline
\end{tabular}
\end{center}
\item Proportion de photos réussies
\[
\frac{151}{252} = 0.6 = 59\%
\frac{159}{246} = 0.65 = 64\%
\]
\item
\begin{itemize}
\item De juin à juillet
\[
\frac{45 - 27}{27} = \frac{18}{27} = 0.67 = 66\%
\frac{31 - 25}{25} = \frac{6}{25} = 0.24 = 24\%
\]
\item De juillet à août
\[
\frac{19 - 45}{45} = \frac{-26}{45} = -0.58 = -57\%
\frac{20 - 31}{31} = \frac{-11}{31} = -0.35 = -35\%
\]
\item De août à septembre
\[
\frac{10 - 19}{19} = \frac{-9}{19} = -0.47 = -47\%
\frac{11 - 20}{20} = \frac{-9}{20} = -0.45 = -45\%
\]
\end{itemize}
\end{enumerate}
\end{solution}
\printsolutionstype{exercise}
\end{document}

View File

@ -17,17 +17,17 @@
Détailler les calculs suivants et donner le résultat sous la forme d'une fraction irréductible.
\begin{multicols}{3}
\begin{enumerate}[label={\Alph*=}]
\item $\dfrac{10}{4} + \dfrac{10}{4}$
\item $\dfrac{4}{5} + 7$
\item $\dfrac{6}{4} + \dfrac{8}{4}$
\item $\dfrac{10}{3} + 3$
\item $\dfrac{8}{6} + \dfrac{2}{30}$
\item $\dfrac{7}{2} + \dfrac{3}{2}$
\item $\dfrac{6}{4} + \dfrac{5}{40}$
\item $\dfrac{4}{8} + \dfrac{7}{9}$
\item $\dfrac{8}{2} \times 3$
\item $\dfrac{1}{9} \times \dfrac{5}{9}$
\item $\dfrac{3}{2} \times 3$
\item $\dfrac{10}{9} \times \dfrac{1}{9}$
\item $\dfrac{3}{9} \times \dfrac{1}{18}$
\item $\dfrac{\dfrac{6}{7}}{\dfrac{4}{3}}$
\item $\dfrac{3}{10} \times \dfrac{- 8}{100}$
\item $\dfrac{\dfrac{2}{6}}{\dfrac{10}{7}}$
\end{enumerate}
\end{multicols}
\end{exercise}
@ -35,15 +35,15 @@
\begin{solution}
\begin{enumerate}[label={\Alph*=}]
\item $\dfrac{10}{4} + \dfrac{10}{4}=\dfrac{10 + 10}{4}=\dfrac{20}{4} = 5$
\item $\dfrac{4}{5} + 7=\dfrac{4}{5} + \dfrac{7}{1}=\dfrac{4}{5} + \dfrac{7 \times 5}{1 \times 5}=\dfrac{4}{5} + \dfrac{35}{5}=\dfrac{4 + 35}{5}=\dfrac{39}{5} = \dfrac{39}{5}$
\item $\dfrac{8}{6} + \dfrac{2}{30}=\dfrac{8 \times 5}{6 \times 5} + \dfrac{2}{30}=\dfrac{40}{30} + \dfrac{2}{30}=\dfrac{40 + 2}{30}=\dfrac{42}{30} = \dfrac{7}{5}$
\item $\dfrac{7}{2} + \dfrac{3}{2}=\dfrac{7 + 3}{2}=\dfrac{10}{2} = 5$
\item $\dfrac{6}{4} + \dfrac{8}{4}=\dfrac{6 + 8}{4}=\dfrac{14}{4} = \dfrac{7}{2}$
\item $\dfrac{10}{3} + 3=\dfrac{10}{3} + \dfrac{3}{1}=\dfrac{10}{3} + \dfrac{3 \times 3}{1 \times 3}=\dfrac{10}{3} + \dfrac{9}{3}=\dfrac{10 + 9}{3}=\dfrac{19}{3} = \dfrac{19}{3}$
\item $\dfrac{6}{4} + \dfrac{5}{40}=\dfrac{6 \times 10}{4 \times 10} + \dfrac{5}{40}=\dfrac{60}{40} + \dfrac{5}{40}=\dfrac{60 + 5}{40}=\dfrac{65}{40} = \dfrac{13}{8}$
\item $\dfrac{4}{8} + \dfrac{7}{9}=\dfrac{4 \times 9}{8 \times 9} + \dfrac{7 \times 8}{9 \times 8}=\dfrac{36}{72} + \dfrac{56}{72}=\dfrac{36 + 56}{72}=\dfrac{92}{72} = \dfrac{23}{18}$
\item $\dfrac{8}{2} \times 3=\dfrac{8 \times 3}{2}=\dfrac{24}{2} = 12$
\item $\dfrac{1}{9} \times \dfrac{5}{9}=\dfrac{1 \times 5}{9 \times 9}=\dfrac{5}{81} = \dfrac{5}{81}$
\item $\dfrac{3}{9} \times \dfrac{1}{18}=\dfrac{3 \times 1}{9 \times 18}=\dfrac{3}{162} = \dfrac{1}{54}$
\item $\dfrac{\dfrac{6}{7}}{\dfrac{4}{3}}=\dfrac{6}{7} \times \dfrac{3}{4}=\dfrac{6 \times 3}{7 \times 4}=\dfrac{18}{28} = \dfrac{9}{14}$
\item $\dfrac{3}{2} \times 3=\dfrac{3 \times 3}{2}=\dfrac{9}{2} = \dfrac{9}{2}$
\item $\dfrac{10}{9} \times \dfrac{1}{9}=\dfrac{10 \times 1}{9 \times 9}=\dfrac{10}{81} = \dfrac{10}{81}$
\item $\dfrac{3}{10} \times \dfrac{- 8}{100}=\dfrac{3(- 8)}{10 \times 100}=\dfrac{- 24}{1000} = \dfrac{- 3}{125}$
\item $\dfrac{\dfrac{2}{6}}{\dfrac{10}{7}}=\dfrac{2}{6} \times \dfrac{7}{10}=\dfrac{2 \times 7}{6 \times 10}=\dfrac{14}{60} = \dfrac{7}{30}$
\end{enumerate}
\end{solution}
@ -52,10 +52,10 @@
Le radar a pris des photos pendant l'été:
\begin{itemize}
\item en juin, il y a eu 42 photos prises dont 20 ratées.
\item en juillet, il y a eu 30 photos réussies et 36 ratées.
\item en août, il y a eu 57 photos dont une proportion de 0.21 de photos ratées.
\item en septembre, il y a eu 7 photos ratées, ce qui correspondait à 14.58\% des photos prises.
\item en juin, il y a eu 50 photos prises dont 29 ratées.
\item en juillet, il y a eu 43 photos réussies et 39 ratées.
\item en août, il y a eu 52 photos dont une proportion de 0.23 de photos ratées.
\item en septembre, il y a eu 7 photos ratées, ce qui correspondait à 14.89\% des photos prises.
\end{itemize}
\begin{enumerate}
@ -87,27 +87,27 @@
\hline
& Juin & Juillet & Août & Septembre & Total\\
\hline
Réussies & 22 & 30 & 45 & 41 & 138\\
Réussies & 21 & 43 & 40 & 40 & 144\\
\hline
Ratées & 20 & 36 & 12 & 7 & 75\\
Ratées & 29 & 39 & 12 & 7 & 87\\
\hline
Total & 42 & 66 & 57 & 48 & 213\\
Total & 50 & 82 & 52 & 47 & 231\\
\hline
\end{tabular}
\end{center}
\item Proportion de photos réussies
\[
\frac{138}{213} = 0.65 = 64\%
\frac{144}{231} = 0.62 = 62\%
\]
\item
\begin{itemize}
\item De juin à juillet
\[
\frac{36 - 20}{20} = \frac{16}{20} = 0.8 = 80\%
\frac{39 - 29}{29} = \frac{10}{29} = 0.34 = 34\%
\]
\item De juillet à août
\[
\frac{12 - 36}{36} = \frac{-24}{36} = -0.67 = -66\%
\frac{12 - 39}{39} = \frac{-27}{39} = -0.69 = -69\%
\]
\item De août à septembre
\[
@ -118,6 +118,4 @@
\end{solution}
\printsolutionstype{exercise}
\end{document}

View File

@ -17,17 +17,17 @@
Détailler les calculs suivants et donner le résultat sous la forme d'une fraction irréductible.
\begin{multicols}{3}
\begin{enumerate}[label={\Alph*=}]
\item $\dfrac{9}{7} + \dfrac{4}{7}$
\item $\dfrac{1}{3} + 7$
\item $\dfrac{7}{10} + \dfrac{1}{10}$
\item $\dfrac{1}{5} + 2$
\item $\dfrac{9}{2} + \dfrac{4}{10}$
\item $\dfrac{1}{6} + \dfrac{9}{10}$
\item $\dfrac{9}{10} + \dfrac{2}{50}$
\item $\dfrac{2}{6} + \dfrac{9}{2}$
\item $\dfrac{1}{7} \times 10$
\item $\dfrac{10}{2} \times \dfrac{8}{2}$
\item $\dfrac{7}{10} \times 2$
\item $\dfrac{8}{3} \times \dfrac{5}{3}$
\item $\dfrac{- 10}{2} \times \dfrac{- 4}{18}$
\item $\dfrac{\dfrac{6}{3}}{\dfrac{4}{10}}$
\item $\dfrac{8}{9} \times \dfrac{7}{36}$
\item $\dfrac{\dfrac{1}{6}}{\dfrac{9}{4}}$
\end{enumerate}
\end{multicols}
\end{exercise}
@ -35,15 +35,15 @@
\begin{solution}
\begin{enumerate}[label={\Alph*=}]
\item $\dfrac{9}{7} + \dfrac{4}{7}=\dfrac{9 + 4}{7}=\dfrac{13}{7} = \dfrac{13}{7}$
\item $\dfrac{1}{3} + 7=\dfrac{1}{3} + \dfrac{7}{1}=\dfrac{1}{3} + \dfrac{7 \times 3}{1 \times 3}=\dfrac{1}{3} + \dfrac{21}{3}=\dfrac{1 + 21}{3}=\dfrac{22}{3} = \dfrac{22}{3}$
\item $\dfrac{9}{2} + \dfrac{4}{10}=\dfrac{9 \times 5}{2 \times 5} + \dfrac{4}{10}=\dfrac{45}{10} + \dfrac{4}{10}=\dfrac{45 + 4}{10}=\dfrac{49}{10} = \dfrac{49}{10}$
\item $\dfrac{1}{6} + \dfrac{9}{10}=\dfrac{1 \times 5}{6 \times 5} + \dfrac{9 \times 3}{10 \times 3}=\dfrac{5}{30} + \dfrac{27}{30}=\dfrac{5 + 27}{30}=\dfrac{32}{30} = \dfrac{16}{15}$
\item $\dfrac{7}{10} + \dfrac{1}{10}=\dfrac{7 + 1}{10}=\dfrac{8}{10} = \dfrac{4}{5}$
\item $\dfrac{1}{5} + 2=\dfrac{1}{5} + \dfrac{2}{1}=\dfrac{1}{5} + \dfrac{2 \times 5}{1 \times 5}=\dfrac{1}{5} + \dfrac{10}{5}=\dfrac{1 + 10}{5}=\dfrac{11}{5} = \dfrac{11}{5}$
\item $\dfrac{9}{10} + \dfrac{2}{50}=\dfrac{9 \times 5}{10 \times 5} + \dfrac{2}{50}=\dfrac{45}{50} + \dfrac{2}{50}=\dfrac{45 + 2}{50}=\dfrac{47}{50} = \dfrac{47}{50}$
\item $\dfrac{2}{6} + \dfrac{9}{2}=\dfrac{2}{6} + \dfrac{9 \times 3}{2 \times 3}=\dfrac{2}{6} + \dfrac{27}{6}=\dfrac{2 + 27}{6}=\dfrac{29}{6} = \dfrac{29}{6}$
\item $\dfrac{1}{7} \times 10=\dfrac{1 \times 10}{7}=\dfrac{10}{7} = \dfrac{10}{7}$
\item $\dfrac{10}{2} \times \dfrac{8}{2}=\dfrac{10 \times 8}{2 \times 2}=\dfrac{80}{4} = 20$
\item $\dfrac{- 10}{2} \times \dfrac{- 4}{18}=\dfrac{- 10(- 4)}{2 \times 18}=\dfrac{40}{36} = \dfrac{10}{9}$
\item $\dfrac{\dfrac{6}{3}}{\dfrac{4}{10}}=\dfrac{6}{3} \times \dfrac{10}{4}=\dfrac{6 \times 10}{3 \times 4}=\dfrac{60}{12} = 5$
\item $\dfrac{7}{10} \times 2=\dfrac{7 \times 2}{10}=\dfrac{14}{10} = \dfrac{7}{5}$
\item $\dfrac{8}{3} \times \dfrac{5}{3}=\dfrac{8 \times 5}{3 \times 3}=\dfrac{40}{9} = \dfrac{40}{9}$
\item $\dfrac{8}{9} \times \dfrac{7}{36}=\dfrac{8 \times 7}{9 \times 36}=\dfrac{56}{324} = \dfrac{14}{81}$
\item $\dfrac{\dfrac{1}{6}}{\dfrac{9}{4}}=\dfrac{1}{6} \times \dfrac{4}{9}=\dfrac{1 \times 4}{6 \times 9}=\dfrac{4}{54} = \dfrac{2}{27}$
\end{enumerate}
\end{solution}
@ -52,10 +52,10 @@
Le radar a pris des photos pendant l'été:
\begin{itemize}
\item en juin, il y a eu 42 photos prises dont 22 ratées.
\item en juillet, il y a eu 44 photos réussies et 32 ratées.
\item en août, il y a eu 54 photos dont une proportion de 0.26 de photos ratées.
\item en septembre, il y a eu 10 photos ratées, ce qui correspondait à 18.87\% des photos prises.
\item en juin, il y a eu 51 photos prises dont 22 ratées.
\item en juillet, il y a eu 48 photos réussies et 43 ratées.
\item en août, il y a eu 62 photos dont une proportion de 0.21 de photos ratées.
\item en septembre, il y a eu 14 photos ratées, ce qui correspondait à 24.56\% des photos prises.
\end{itemize}
\begin{enumerate}
@ -87,37 +87,35 @@
\hline
& Juin & Juillet & Août & Septembre & Total\\
\hline
Réussies & 20 & 44 & 40 & 43 & 147\\
Réussies & 29 & 48 & 49 & 43 & 169\\
\hline
Ratées & 22 & 32 & 14 & 10 & 78\\
Ratées & 22 & 43 & 13 & 14 & 92\\
\hline
Total & 42 & 76 & 54 & 53 & 225\\
Total & 51 & 91 & 62 & 57 & 261\\
\hline
\end{tabular}
\end{center}
\item Proportion de photos réussies
\[
\frac{147}{225} = 0.65 = 65\%
\frac{169}{261} = 0.65 = 64\%
\]
\item
\begin{itemize}
\item De juin à juillet
\[
\frac{32 - 22}{22} = \frac{10}{22} = 0.45 = 45\%
\frac{43 - 22}{22} = \frac{21}{22} = 0.95 = 95\%
\]
\item De juillet à août
\[
\frac{14 - 32}{32} = \frac{-18}{32} = -0.56 = -56\%
\frac{13 - 43}{43} = \frac{-30}{43} = -0.7 = -69\%
\]
\item De août à septembre
\[
\frac{10 - 14}{14} = \frac{-4}{14} = -0.29 = -28\%
\frac{14 - 13}{13} = \frac{1}{13} = 0.08 = 7\%
\]
\end{itemize}
\end{enumerate}
\end{solution}
\printsolutionstype{exercise}
\end{document}

View File

@ -17,17 +17,17 @@
Détailler les calculs suivants et donner le résultat sous la forme d'une fraction irréductible.
\begin{multicols}{3}
\begin{enumerate}[label={\Alph*=}]
\item $\dfrac{1}{3} + \dfrac{9}{3}$
\item $\dfrac{5}{2} + 2$
\item $\dfrac{4}{7} + \dfrac{10}{7}$
\item $\dfrac{5}{6} + 6$
\item $\dfrac{10}{2} + \dfrac{6}{2}$
\item $\dfrac{5}{6} + \dfrac{4}{9}$
\item $\dfrac{8}{6} + \dfrac{1}{6}$
\item $\dfrac{10}{7} + \dfrac{10}{3}$
\item $\dfrac{6}{8} \times 5$
\item $\dfrac{5}{2} \times \dfrac{5}{2}$
\item $\dfrac{2}{4} \times 7$
\item $\dfrac{4}{10} \times \dfrac{2}{10}$
\item $\dfrac{6}{2} \times \dfrac{- 4}{10}$
\item $\dfrac{\dfrac{7}{9}}{\dfrac{7}{2}}$
\item $\dfrac{7}{2} \times \dfrac{- 6}{20}$
\item $\dfrac{\dfrac{5}{10}}{\dfrac{6}{9}}$
\end{enumerate}
\end{multicols}
\end{exercise}
@ -35,15 +35,15 @@
\begin{solution}
\begin{enumerate}[label={\Alph*=}]
\item $\dfrac{1}{3} + \dfrac{9}{3}=\dfrac{1 + 9}{3}=\dfrac{10}{3} = \dfrac{10}{3}$
\item $\dfrac{5}{2} + 2=\dfrac{5}{2} + \dfrac{2}{1}=\dfrac{5}{2} + \dfrac{2 \times 2}{1 \times 2}=\dfrac{5}{2} + \dfrac{4}{2}=\dfrac{5 + 4}{2}=\dfrac{9}{2} = \dfrac{9}{2}$
\item $\dfrac{10}{2} + \dfrac{6}{2}=\dfrac{10 + 6}{2}=\dfrac{16}{2} = 8$
\item $\dfrac{5}{6} + \dfrac{4}{9}=\dfrac{5 \times 3}{6 \times 3} + \dfrac{4 \times 2}{9 \times 2}=\dfrac{15}{18} + \dfrac{8}{18}=\dfrac{15 + 8}{18}=\dfrac{23}{18} = \dfrac{23}{18}$
\item $\dfrac{4}{7} + \dfrac{10}{7}=\dfrac{4 + 10}{7}=\dfrac{14}{7} = 2$
\item $\dfrac{5}{6} + 6=\dfrac{5}{6} + \dfrac{6}{1}=\dfrac{5}{6} + \dfrac{6 \times 6}{1 \times 6}=\dfrac{5}{6} + \dfrac{36}{6}=\dfrac{5 + 36}{6}=\dfrac{41}{6} = \dfrac{41}{6}$
\item $\dfrac{8}{6} + \dfrac{1}{6}=\dfrac{8 + 1}{6}=\dfrac{9}{6} = \dfrac{3}{2}$
\item $\dfrac{10}{7} + \dfrac{10}{3}=\dfrac{10 \times 3}{7 \times 3} + \dfrac{10 \times 7}{3 \times 7}=\dfrac{30}{21} + \dfrac{70}{21}=\dfrac{30 + 70}{21}=\dfrac{100}{21} = \dfrac{100}{21}$
\item $\dfrac{6}{8} \times 5=\dfrac{6 \times 5}{8}=\dfrac{30}{8} = \dfrac{15}{4}$
\item $\dfrac{5}{2} \times \dfrac{5}{2}=\dfrac{5 \times 5}{2 \times 2}=\dfrac{25}{4} = \dfrac{25}{4}$
\item $\dfrac{6}{2} \times \dfrac{- 4}{10}=\dfrac{6(- 4)}{2 \times 10}=\dfrac{- 24}{20} = \dfrac{- 6}{5}$
\item $\dfrac{\dfrac{7}{9}}{\dfrac{7}{2}}=\dfrac{7}{9} \times \dfrac{2}{7}=\dfrac{7 \times 2}{9 \times 7}=\dfrac{14}{63} = \dfrac{2}{9}$
\item $\dfrac{2}{4} \times 7=\dfrac{2 \times 7}{4}=\dfrac{14}{4} = \dfrac{7}{2}$
\item $\dfrac{4}{10} \times \dfrac{2}{10}=\dfrac{4 \times 2}{10 \times 10}=\dfrac{8}{100} = \dfrac{2}{25}$
\item $\dfrac{7}{2} \times \dfrac{- 6}{20}=\dfrac{7(- 6)}{2 \times 20}=\dfrac{- 42}{40} = \dfrac{- 21}{20}$
\item $\dfrac{\dfrac{5}{10}}{\dfrac{6}{9}}=\dfrac{5}{10} \times \dfrac{9}{6}=\dfrac{5 \times 9}{10 \times 6}=\dfrac{45}{60} = \dfrac{3}{4}$
\end{enumerate}
\end{solution}
@ -52,10 +52,10 @@
Le radar a pris des photos pendant l'été:
\begin{itemize}
\item en juin, il y a eu 51 photos prises dont 25 ratées.
\item en juillet, il y a eu 37 photos réussies et 32 ratées.
\item en août, il y a eu 65 photos dont une proportion de 0.26 de photos ratées.
\item en septembre, il y a eu 8 photos ratées, ce qui correspondait à 14.55\% des photos prises.
\item en juin, il y a eu 47 photos prises dont 22 ratées.
\item en juillet, il y a eu 43 photos réussies et 30 ratées.
\item en août, il y a eu 66 photos dont une proportion de 0.3 de photos ratées.
\item en septembre, il y a eu 8 photos ratées, ce qui correspondait à 16.67\% des photos prises.
\end{itemize}
\begin{enumerate}
@ -87,37 +87,35 @@
\hline
& Juin & Juillet & Août & Septembre & Total\\
\hline
Réussies & 26 & 37 & 48 & 47 & 158\\
Réussies & 25 & 43 & 46 & 40 & 154\\
\hline
Ratées & 25 & 32 & 17 & 8 & 82\\
Ratées & 22 & 30 & 20 & 8 & 80\\
\hline
Total & 51 & 69 & 65 & 55 & 240\\
Total & 47 & 73 & 66 & 48 & 234\\
\hline
\end{tabular}
\end{center}
\item Proportion de photos réussies
\[
\frac{158}{240} = 0.66 = 65\%
\frac{154}{234} = 0.66 = 65\%
\]
\item
\begin{itemize}
\item De juin à juillet
\[
\frac{32 - 25}{25} = \frac{7}{25} = 0.28 = 28\%
\frac{30 - 22}{22} = \frac{8}{22} = 0.36 = 36\%
\]
\item De juillet à août
\[
\frac{17 - 32}{32} = \frac{-15}{32} = -0.47 = -46\%
\frac{20 - 30}{30} = \frac{-10}{30} = -0.33 = -33\%
\]
\item De août à septembre
\[
\frac{8 - 17}{17} = \frac{-9}{17} = -0.53 = -52\%
\frac{8 - 20}{20} = \frac{-12}{20} = -0.6 = -60\%
\]
\end{itemize}
\end{enumerate}
\end{solution}
\printsolutionstype{exercise}
\end{document}

View File

@ -17,17 +17,17 @@
Détailler les calculs suivants et donner le résultat sous la forme d'une fraction irréductible.
\begin{multicols}{3}
\begin{enumerate}[label={\Alph*=}]
\item $\dfrac{4}{6} + \dfrac{1}{6}$
\item $\dfrac{6}{7} + 6$
\item $\dfrac{5}{6} + \dfrac{4}{6}$
\item $\dfrac{3}{6} + 7$
\item $\dfrac{5}{4} + \dfrac{6}{24}$
\item $\dfrac{3}{6} + \dfrac{6}{8}$
\item $\dfrac{6}{7} + \dfrac{1}{63}$
\item $\dfrac{6}{3} + \dfrac{1}{3}$
\item $\dfrac{3}{7} \times 7$
\item $\dfrac{8}{4} \times \dfrac{5}{4}$
\item $\dfrac{9}{4} \times 1$
\item $\dfrac{3}{4} \times \dfrac{2}{4}$
\item $\dfrac{4}{10} \times \dfrac{- 5}{50}$
\item $\dfrac{\dfrac{10}{9}}{\dfrac{5}{7}}$
\item $\dfrac{7}{3} \times \dfrac{- 10}{21}$
\item $\dfrac{\dfrac{2}{9}}{\dfrac{2}{4}}$
\end{enumerate}
\end{multicols}
\end{exercise}
@ -35,15 +35,15 @@
\begin{solution}
\begin{enumerate}[label={\Alph*=}]
\item $\dfrac{4}{6} + \dfrac{1}{6}=\dfrac{4 + 1}{6}=\dfrac{5}{6} = \dfrac{5}{6}$
\item $\dfrac{6}{7} + 6=\dfrac{6}{7} + \dfrac{6}{1}=\dfrac{6}{7} + \dfrac{6 \times 7}{1 \times 7}=\dfrac{6}{7} + \dfrac{42}{7}=\dfrac{6 + 42}{7}=\dfrac{48}{7} = \dfrac{48}{7}$
\item $\dfrac{5}{4} + \dfrac{6}{24}=\dfrac{5 \times 6}{4 \times 6} + \dfrac{6}{24}=\dfrac{30}{24} + \dfrac{6}{24}=\dfrac{30 + 6}{24}=\dfrac{36}{24} = \dfrac{3}{2}$
\item $\dfrac{3}{6} + \dfrac{6}{8}=\dfrac{3 \times 4}{6 \times 4} + \dfrac{6 \times 3}{8 \times 3}=\dfrac{12}{24} + \dfrac{18}{24}=\dfrac{12 + 18}{24}=\dfrac{30}{24} = \dfrac{5}{4}$
\item $\dfrac{5}{6} + \dfrac{4}{6}=\dfrac{5 + 4}{6}=\dfrac{9}{6} = \dfrac{3}{2}$
\item $\dfrac{3}{6} + 7=\dfrac{3}{6} + \dfrac{7}{1}=\dfrac{3}{6} + \dfrac{7 \times 6}{1 \times 6}=\dfrac{3}{6} + \dfrac{42}{6}=\dfrac{3 + 42}{6}=\dfrac{45}{6} = \dfrac{15}{2}$
\item $\dfrac{6}{7} + \dfrac{1}{63}=\dfrac{6 \times 9}{7 \times 9} + \dfrac{1}{63}=\dfrac{54}{63} + \dfrac{1}{63}=\dfrac{54 + 1}{63}=\dfrac{55}{63} = \dfrac{55}{63}$
\item $\dfrac{6}{3} + \dfrac{1}{3}=\dfrac{6 + 1}{3}=\dfrac{7}{3} = \dfrac{7}{3}$
\item $\dfrac{3}{7} \times 7=\dfrac{3 \times 7}{7}=\dfrac{21}{7} = 3$
\item $\dfrac{8}{4} \times \dfrac{5}{4}=\dfrac{8 \times 5}{4 \times 4}=\dfrac{40}{16} = \dfrac{5}{2}$
\item $\dfrac{4}{10} \times \dfrac{- 5}{50}=\dfrac{4(- 5)}{10 \times 50}=\dfrac{- 20}{500} = \dfrac{- 1}{25}$
\item $\dfrac{\dfrac{10}{9}}{\dfrac{5}{7}}=\dfrac{10}{9} \times \dfrac{7}{5}=\dfrac{10 \times 7}{9 \times 5}=\dfrac{70}{45} = \dfrac{14}{9}$
\item $\dfrac{9}{4} \times 1=\dfrac{9}{4} = \dfrac{9}{4}$
\item $\dfrac{3}{4} \times \dfrac{2}{4}=\dfrac{3 \times 2}{4 \times 4}=\dfrac{6}{16} = \dfrac{3}{8}$
\item $\dfrac{7}{3} \times \dfrac{- 10}{21}=\dfrac{7(- 10)}{3 \times 21}=\dfrac{- 70}{63} = \dfrac{- 10}{9}$
\item $\dfrac{\dfrac{2}{9}}{\dfrac{2}{4}}=\dfrac{2}{9} \times \dfrac{4}{2}=\dfrac{2 \times 4}{9 \times 2}=\dfrac{8}{18} = \dfrac{4}{9}$
\end{enumerate}
\end{solution}
@ -52,10 +52,10 @@
Le radar a pris des photos pendant l'été:
\begin{itemize}
\item en juin, il y a eu 50 photos prises dont 20 ratées.
\item en juillet, il y a eu 47 photos réussies et 44 ratées.
\item en août, il y a eu 53 photos dont une proportion de 0.25 de photos ratées.
\item en septembre, il y a eu 9 photos ratées, ce qui correspondait à 16.98\% des photos prises.
\item en juin, il y a eu 51 photos prises dont 22 ratées.
\item en juillet, il y a eu 48 photos réussies et 49 ratées.
\item en août, il y a eu 62 photos dont une proportion de 0.29 de photos ratées.
\item en septembre, il y a eu 15 photos ratées, ce qui correspondait à 23.08\% des photos prises.
\end{itemize}
\begin{enumerate}
@ -87,37 +87,35 @@
\hline
& Juin & Juillet & Août & Septembre & Total\\
\hline
Réussies & 30 & 47 & 40 & 44 & 161\\
Réussies & 29 & 48 & 44 & 50 & 171\\
\hline
Ratées & 20 & 44 & 13 & 9 & 86\\
Ratées & 22 & 49 & 18 & 15 & 104\\
\hline
Total & 50 & 91 & 53 & 53 & 247\\
Total & 51 & 97 & 62 & 65 & 275\\
\hline
\end{tabular}
\end{center}
\item Proportion de photos réussies
\[
\frac{161}{247} = 0.65 = 65\%
\frac{171}{275} = 0.62 = 62\%
\]
\item
\begin{itemize}
\item De juin à juillet
\[
\frac{44 - 20}{20} = \frac{24}{20} = 1.2 = 120\%
\frac{49 - 22}{22} = \frac{27}{22} = 1.23 = 122\%
\]
\item De juillet à août
\[
\frac{13 - 44}{44} = \frac{-31}{44} = -0.7 = -70\%
\frac{18 - 49}{49} = \frac{-31}{49} = -0.63 = -63\%
\]
\item De août à septembre
\[
\frac{9 - 13}{13} = \frac{-4}{13} = -0.31 = -30\%
\frac{15 - 18}{18} = \frac{-3}{18} = -0.17 = -16\%
\]
\end{itemize}
\end{enumerate}
\end{solution}
\printsolutionstype{exercise}
\end{document}

View File

@ -17,17 +17,17 @@
Détailler les calculs suivants et donner le résultat sous la forme d'une fraction irréductible.
\begin{multicols}{3}
\begin{enumerate}[label={\Alph*=}]
\item $\dfrac{3}{7} + \dfrac{3}{7}$
\item $\dfrac{8}{3} + 9$
\item $\dfrac{6}{5} + \dfrac{3}{5}$
\item $\dfrac{7}{2} + 5$
\item $\dfrac{1}{4} + \dfrac{3}{28}$
\item $\dfrac{4}{7} + \dfrac{6}{2}$
\item $\dfrac{5}{4} + \dfrac{2}{32}$
\item $\dfrac{10}{2} + \dfrac{10}{9}$
\item $\dfrac{9}{4} \times 8$
\item $\dfrac{5}{7} \times \dfrac{3}{7}$
\item $\dfrac{10}{3} \times 4$
\item $\dfrac{3}{2} \times \dfrac{7}{2}$
\item $\dfrac{- 10}{8} \times \dfrac{- 2}{40}$
\item $\dfrac{\dfrac{4}{9}}{\dfrac{6}{9}}$
\item $\dfrac{- 10}{3} \times \dfrac{8}{12}$
\item $\dfrac{\dfrac{9}{3}}{\dfrac{1}{4}}$
\end{enumerate}
\end{multicols}
\end{exercise}
@ -35,15 +35,15 @@
\begin{solution}
\begin{enumerate}[label={\Alph*=}]
\item $\dfrac{3}{7} + \dfrac{3}{7}=\dfrac{3 + 3}{7}=\dfrac{6}{7} = \dfrac{6}{7}$
\item $\dfrac{8}{3} + 9=\dfrac{8}{3} + \dfrac{9}{1}=\dfrac{8}{3} + \dfrac{9 \times 3}{1 \times 3}=\dfrac{8}{3} + \dfrac{27}{3}=\dfrac{8 + 27}{3}=\dfrac{35}{3} = \dfrac{35}{3}$
\item $\dfrac{1}{4} + \dfrac{3}{28}=\dfrac{1 \times 7}{4 \times 7} + \dfrac{3}{28}=\dfrac{7}{28} + \dfrac{3}{28}=\dfrac{7 + 3}{28}=\dfrac{10}{28} = \dfrac{5}{14}$
\item $\dfrac{4}{7} + \dfrac{6}{2}=\dfrac{4 \times 2}{7 \times 2} + \dfrac{6 \times 7}{2 \times 7}=\dfrac{8}{14} + \dfrac{42}{14}=\dfrac{8 + 42}{14}=\dfrac{50}{14} = \dfrac{25}{7}$
\item $\dfrac{6}{5} + \dfrac{3}{5}=\dfrac{6 + 3}{5}=\dfrac{9}{5} = \dfrac{9}{5}$
\item $\dfrac{7}{2} + 5=\dfrac{7}{2} + \dfrac{5}{1}=\dfrac{7}{2} + \dfrac{5 \times 2}{1 \times 2}=\dfrac{7}{2} + \dfrac{10}{2}=\dfrac{7 + 10}{2}=\dfrac{17}{2} = \dfrac{17}{2}$
\item $\dfrac{5}{4} + \dfrac{2}{32}=\dfrac{5 \times 8}{4 \times 8} + \dfrac{2}{32}=\dfrac{40}{32} + \dfrac{2}{32}=\dfrac{40 + 2}{32}=\dfrac{42}{32} = \dfrac{21}{16}$
\item $\dfrac{10}{2} + \dfrac{10}{9}=\dfrac{10 \times 9}{2 \times 9} + \dfrac{10 \times 2}{9 \times 2}=\dfrac{90}{18} + \dfrac{20}{18}=\dfrac{90 + 20}{18}=\dfrac{110}{18} = \dfrac{55}{9}$
\item $\dfrac{9}{4} \times 8=\dfrac{9 \times 8}{4}=\dfrac{72}{4} = 18$
\item $\dfrac{5}{7} \times \dfrac{3}{7}=\dfrac{5 \times 3}{7 \times 7}=\dfrac{15}{49} = \dfrac{15}{49}$
\item $\dfrac{- 10}{8} \times \dfrac{- 2}{40}=\dfrac{- 10(- 2)}{8 \times 40}=\dfrac{20}{320} = \dfrac{1}{16}$
\item $\dfrac{\dfrac{4}{9}}{\dfrac{6}{9}}=\dfrac{4}{9} \times \dfrac{9}{6}=\dfrac{4 \times 9}{9 \times 6}=\dfrac{36}{54} = \dfrac{2}{3}$
\item $\dfrac{10}{3} \times 4=\dfrac{10 \times 4}{3}=\dfrac{40}{3} = \dfrac{40}{3}$
\item $\dfrac{3}{2} \times \dfrac{7}{2}=\dfrac{3 \times 7}{2 \times 2}=\dfrac{21}{4} = \dfrac{21}{4}$
\item $\dfrac{- 10}{3} \times \dfrac{8}{12}=\dfrac{- 10 \times 8}{3 \times 12}=\dfrac{- 80}{36} = \dfrac{- 20}{9}$
\item $\dfrac{\dfrac{9}{3}}{\dfrac{1}{4}}=\dfrac{9}{3} \times \dfrac{4}{1}=\dfrac{9 \times 4}{3 \times 1}=\dfrac{36}{3} = 12$
\end{enumerate}
\end{solution}
@ -52,10 +52,10 @@
Le radar a pris des photos pendant l'été:
\begin{itemize}
\item en juin, il y a eu 45 photos prises dont 21 ratées.
\item en juillet, il y a eu 39 photos réussies et 30 ratées.
\item en août, il y a eu 54 photos dont une proportion de 0.19 de photos ratées.
\item en septembre, il y a eu 7 photos ratées, ce qui correspondait à 14.0\% des photos prises.
\item en juin, il y a eu 47 photos prises dont 23 ratées.
\item en juillet, il y a eu 34 photos réussies et 47 ratées.
\item en août, il y a eu 67 photos dont une proportion de 0.3 de photos ratées.
\item en septembre, il y a eu 12 photos ratées, ce qui correspondait à 22.22\% des photos prises.
\end{itemize}
\begin{enumerate}
@ -87,37 +87,35 @@
\hline
& Juin & Juillet & Août & Septembre & Total\\
\hline
Réussies & 24 & 39 & 44 & 43 & 150\\
Réussies & 24 & 34 & 47 & 42 & 147\\
\hline
Ratées & 21 & 30 & 10 & 7 & 68\\
Ratées & 23 & 47 & 20 & 12 & 102\\
\hline
Total & 45 & 69 & 54 & 50 & 218\\
Total & 47 & 81 & 67 & 54 & 249\\
\hline
\end{tabular}
\end{center}
\item Proportion de photos réussies
\[
\frac{150}{218} = 0.69 = 68\%
\frac{147}{249} = 0.59 = 59\%
\]
\item
\begin{itemize}
\item De juin à juillet
\[
\frac{30 - 21}{21} = \frac{9}{21} = 0.43 = 42\%
\frac{47 - 23}{23} = \frac{24}{23} = 1.04 = 104\%
\]
\item De juillet à août
\[
\frac{10 - 30}{30} = \frac{-20}{30} = -0.67 = -66\%
\frac{20 - 47}{47} = \frac{-27}{47} = -0.57 = -57\%
\]
\item De août à septembre
\[
\frac{7 - 10}{10} = \frac{-3}{10} = -0.3 = -30\%
\frac{12 - 20}{20} = \frac{-8}{20} = -0.4 = -40\%
\]
\end{itemize}
\end{enumerate}
\end{solution}
\printsolutionstype{exercise}
\end{document}

View File

@ -17,17 +17,17 @@
Détailler les calculs suivants et donner le résultat sous la forme d'une fraction irréductible.
\begin{multicols}{3}
\begin{enumerate}[label={\Alph*=}]
\item $\dfrac{5}{4} + \dfrac{7}{4}$
\item $\dfrac{2}{4} + 5$
\item $\dfrac{3}{4} + \dfrac{7}{4}$
\item $\dfrac{4}{3} + 7$
\item $\dfrac{2}{5} + \dfrac{9}{30}$
\item $\dfrac{1}{7} + \dfrac{4}{6}$
\item $\dfrac{5}{4} + \dfrac{3}{20}$
\item $\dfrac{6}{9} + \dfrac{7}{5}$
\item $\dfrac{8}{9} \times 3$
\item $\dfrac{2}{5} \times \dfrac{7}{5}$
\item $\dfrac{4}{10} \times 5$
\item $\dfrac{1}{9} \times \dfrac{4}{9}$
\item $\dfrac{4}{7} \times \dfrac{- 2}{70}$
\item $\dfrac{\dfrac{5}{4}}{\dfrac{1}{3}}$
\item $\dfrac{- 5}{5} \times \dfrac{7}{45}$
\item $\dfrac{\dfrac{1}{2}}{\dfrac{10}{9}}$
\end{enumerate}
\end{multicols}
\end{exercise}
@ -35,15 +35,15 @@
\begin{solution}
\begin{enumerate}[label={\Alph*=}]
\item $\dfrac{5}{4} + \dfrac{7}{4}=\dfrac{5 + 7}{4}=\dfrac{12}{4} = 3$
\item $\dfrac{2}{4} + 5=\dfrac{2}{4} + \dfrac{5}{1}=\dfrac{2}{4} + \dfrac{5 \times 4}{1 \times 4}=\dfrac{2}{4} + \dfrac{20}{4}=\dfrac{2 + 20}{4}=\dfrac{22}{4} = \dfrac{11}{2}$
\item $\dfrac{2}{5} + \dfrac{9}{30}=\dfrac{2 \times 6}{5 \times 6} + \dfrac{9}{30}=\dfrac{12}{30} + \dfrac{9}{30}=\dfrac{12 + 9}{30}=\dfrac{21}{30} = \dfrac{7}{10}$
\item $\dfrac{1}{7} + \dfrac{4}{6}=\dfrac{1 \times 6}{7 \times 6} + \dfrac{4 \times 7}{6 \times 7}=\dfrac{6}{42} + \dfrac{28}{42}=\dfrac{6 + 28}{42}=\dfrac{34}{42} = \dfrac{17}{21}$
\item $\dfrac{3}{4} + \dfrac{7}{4}=\dfrac{3 + 7}{4}=\dfrac{10}{4} = \dfrac{5}{2}$
\item $\dfrac{4}{3} + 7=\dfrac{4}{3} + \dfrac{7}{1}=\dfrac{4}{3} + \dfrac{7 \times 3}{1 \times 3}=\dfrac{4}{3} + \dfrac{21}{3}=\dfrac{4 + 21}{3}=\dfrac{25}{3} = \dfrac{25}{3}$
\item $\dfrac{5}{4} + \dfrac{3}{20}=\dfrac{5 \times 5}{4 \times 5} + \dfrac{3}{20}=\dfrac{25}{20} + \dfrac{3}{20}=\dfrac{25 + 3}{20}=\dfrac{28}{20} = \dfrac{7}{5}$
\item $\dfrac{6}{9} + \dfrac{7}{5}=\dfrac{6 \times 5}{9 \times 5} + \dfrac{7 \times 9}{5 \times 9}=\dfrac{30}{45} + \dfrac{63}{45}=\dfrac{30 + 63}{45}=\dfrac{93}{45} = \dfrac{31}{15}$
\item $\dfrac{8}{9} \times 3=\dfrac{8 \times 3}{9}=\dfrac{24}{9} = \dfrac{8}{3}$
\item $\dfrac{2}{5} \times \dfrac{7}{5}=\dfrac{2 \times 7}{5 \times 5}=\dfrac{14}{25} = \dfrac{14}{25}$
\item $\dfrac{4}{7} \times \dfrac{- 2}{70}=\dfrac{4(- 2)}{7 \times 70}=\dfrac{- 8}{490} = \dfrac{- 4}{245}$
\item $\dfrac{\dfrac{5}{4}}{\dfrac{1}{3}}=\dfrac{5}{4} \times \dfrac{3}{1}=\dfrac{5 \times 3}{4 \times 1}=\dfrac{15}{4} = \dfrac{15}{4}$
\item $\dfrac{4}{10} \times 5=\dfrac{4 \times 5}{10}=\dfrac{20}{10} = 2$
\item $\dfrac{1}{9} \times \dfrac{4}{9}=\dfrac{1 \times 4}{9 \times 9}=\dfrac{4}{81} = \dfrac{4}{81}$
\item $\dfrac{- 5}{5} \times \dfrac{7}{45}=\dfrac{- 5 \times 7}{5 \times 45}=\dfrac{- 35}{225} = \dfrac{- 7}{45}$
\item $\dfrac{\dfrac{1}{2}}{\dfrac{10}{9}}=\dfrac{1}{2} \times \dfrac{9}{10}=\dfrac{1 \times 9}{2 \times 10}=\dfrac{9}{20} = \dfrac{9}{20}$
\end{enumerate}
\end{solution}
@ -52,10 +52,10 @@
Le radar a pris des photos pendant l'été:
\begin{itemize}
\item en juin, il y a eu 51 photos prises dont 24 ratées.
\item en juillet, il y a eu 42 photos réussies et 39 ratées.
\item en août, il y a eu 55 photos dont une proportion de 0.25 de photos ratées.
\item en septembre, il y a eu 14 photos ratées, ce qui correspondait à 25.0\% des photos prises.
\item en juin, il y a eu 52 photos prises dont 24 ratées.
\item en juillet, il y a eu 47 photos réussies et 31 ratées.
\item en août, il y a eu 63 photos dont une proportion de 0.21 de photos ratées.
\item en septembre, il y a eu 14 photos ratées, ce qui correspondait à 23.73\% des photos prises.
\end{itemize}
\begin{enumerate}
@ -87,37 +87,35 @@
\hline
& Juin & Juillet & Août & Septembre & Total\\
\hline
Réussies & 27 & 42 & 41 & 42 & 152\\
Réussies & 28 & 47 & 50 & 45 & 170\\
\hline
Ratées & 24 & 39 & 14 & 14 & 91\\
Ratées & 24 & 31 & 13 & 14 & 82\\
\hline
Total & 51 & 81 & 55 & 56 & 243\\
Total & 52 & 78 & 63 & 59 & 252\\
\hline
\end{tabular}
\end{center}
\item Proportion de photos réussies
\[
\frac{152}{243} = 0.63 = 62\%
\frac{170}{252} = 0.67 = 67\%
\]
\item
\begin{itemize}
\item De juin à juillet
\[
\frac{39 - 24}{24} = \frac{15}{24} = 0.62 = 62\%
\frac{31 - 24}{24} = \frac{7}{24} = 0.29 = 29\%
\]
\item De juillet à août
\[
\frac{14 - 39}{39} = \frac{-25}{39} = -0.64 = -64\%
\frac{13 - 31}{31} = \frac{-18}{31} = -0.58 = -58\%
\]
\item De août à septembre
\[
\frac{14 - 14}{14} = \frac{0}{14} = 0.0 = 0\%
\frac{14 - 13}{13} = \frac{1}{13} = 0.08 = 7\%
\]
\end{itemize}
\end{enumerate}
\end{solution}
\printsolutionstype{exercise}
\end{document}

View File

@ -17,17 +17,17 @@
Détailler les calculs suivants et donner le résultat sous la forme d'une fraction irréductible.
\begin{multicols}{3}
\begin{enumerate}[label={\Alph*=}]
\item $\dfrac{7}{2} + \dfrac{8}{2}$
\item $\dfrac{8}{6} + 5$
\item $\dfrac{9}{5} + \dfrac{9}{5}$
\item $\dfrac{4}{5} + 7$
\item $\dfrac{10}{6} + \dfrac{9}{48}$
\item $\dfrac{9}{7} + \dfrac{6}{2}$
\item $\dfrac{2}{9} + \dfrac{2}{63}$
\item $\dfrac{2}{6} + \dfrac{2}{4}$
\item $\dfrac{2}{6} \times 6$
\item $\dfrac{1}{7} \times \dfrac{3}{7}$
\item $\dfrac{2}{4} \times 5$
\item $\dfrac{6}{8} \times \dfrac{5}{8}$
\item $\dfrac{- 7}{7} \times \dfrac{2}{35}$
\item $\dfrac{\dfrac{10}{8}}{\dfrac{6}{2}}$
\item $\dfrac{- 9}{7} \times \dfrac{- 1}{42}$
\item $\dfrac{\dfrac{6}{9}}{\dfrac{7}{6}}$
\end{enumerate}
\end{multicols}
\end{exercise}
@ -35,15 +35,15 @@
\begin{solution}
\begin{enumerate}[label={\Alph*=}]
\item $\dfrac{7}{2} + \dfrac{8}{2}=\dfrac{7 + 8}{2}=\dfrac{15}{2} = \dfrac{15}{2}$
\item $\dfrac{8}{6} + 5=\dfrac{8}{6} + \dfrac{5}{1}=\dfrac{8}{6} + \dfrac{5 \times 6}{1 \times 6}=\dfrac{8}{6} + \dfrac{30}{6}=\dfrac{8 + 30}{6}=\dfrac{38}{6} = \dfrac{19}{3}$
\item $\dfrac{10}{6} + \dfrac{9}{48}=\dfrac{10 \times 8}{6 \times 8} + \dfrac{9}{48}=\dfrac{80}{48} + \dfrac{9}{48}=\dfrac{80 + 9}{48}=\dfrac{89}{48} = \dfrac{89}{48}$
\item $\dfrac{9}{7} + \dfrac{6}{2}=\dfrac{9 \times 2}{7 \times 2} + \dfrac{6 \times 7}{2 \times 7}=\dfrac{18}{14} + \dfrac{42}{14}=\dfrac{18 + 42}{14}=\dfrac{60}{14} = \dfrac{30}{7}$
\item $\dfrac{9}{5} + \dfrac{9}{5}=\dfrac{9 + 9}{5}=\dfrac{18}{5} = \dfrac{18}{5}$
\item $\dfrac{4}{5} + 7=\dfrac{4}{5} + \dfrac{7}{1}=\dfrac{4}{5} + \dfrac{7 \times 5}{1 \times 5}=\dfrac{4}{5} + \dfrac{35}{5}=\dfrac{4 + 35}{5}=\dfrac{39}{5} = \dfrac{39}{5}$
\item $\dfrac{2}{9} + \dfrac{2}{63}=\dfrac{2 \times 7}{9 \times 7} + \dfrac{2}{63}=\dfrac{14}{63} + \dfrac{2}{63}=\dfrac{14 + 2}{63}=\dfrac{16}{63} = \dfrac{16}{63}$
\item $\dfrac{2}{6} + \dfrac{2}{4}=\dfrac{2 \times 2}{6 \times 2} + \dfrac{2 \times 3}{4 \times 3}=\dfrac{4}{12} + \dfrac{6}{12}=\dfrac{4 + 6}{12}=\dfrac{10}{12} = \dfrac{5}{6}$
\item $\dfrac{2}{6} \times 6=\dfrac{2 \times 6}{6}=\dfrac{12}{6} = 2$
\item $\dfrac{1}{7} \times \dfrac{3}{7}=\dfrac{1 \times 3}{7 \times 7}=\dfrac{3}{49} = \dfrac{3}{49}$
\item $\dfrac{- 7}{7} \times \dfrac{2}{35}=\dfrac{- 7 \times 2}{7 \times 35}=\dfrac{- 14}{245} = \dfrac{- 2}{35}$
\item $\dfrac{\dfrac{10}{8}}{\dfrac{6}{2}}=\dfrac{10}{8} \times \dfrac{2}{6}=\dfrac{10 \times 2}{8 \times 6}=\dfrac{20}{48} = \dfrac{5}{12}$
\item $\dfrac{2}{4} \times 5=\dfrac{2 \times 5}{4}=\dfrac{10}{4} = \dfrac{5}{2}$
\item $\dfrac{6}{8} \times \dfrac{5}{8}=\dfrac{6 \times 5}{8 \times 8}=\dfrac{30}{64} = \dfrac{15}{32}$
\item $\dfrac{- 9}{7} \times \dfrac{- 1}{42}=\dfrac{- 9(- 1)}{7 \times 42}=\dfrac{9}{294} = \dfrac{3}{98}$
\item $\dfrac{\dfrac{6}{9}}{\dfrac{7}{6}}=\dfrac{6}{9} \times \dfrac{6}{7}=\dfrac{6 \times 6}{9 \times 7}=\dfrac{36}{63} = \dfrac{4}{7}$
\end{enumerate}
\end{solution}
@ -52,10 +52,10 @@
Le radar a pris des photos pendant l'été:
\begin{itemize}
\item en juin, il y a eu 50 photos prises dont 26 ratées.
\item en juillet, il y a eu 43 photos réussies et 32 ratées.
\item en août, il y a eu 61 photos dont une proportion de 0.2 de photos ratées.
\item en septembre, il y a eu 5 photos ratées, ce qui correspondait à 9.26\% des photos prises.
\item en juin, il y a eu 46 photos prises dont 25 ratées.
\item en juillet, il y a eu 35 photos réussies et 44 ratées.
\item en août, il y a eu 52 photos dont une proportion de 0.21 de photos ratées.
\item en septembre, il y a eu 9 photos ratées, ce qui correspondait à 17.31\% des photos prises.
\end{itemize}
\begin{enumerate}
@ -87,37 +87,35 @@
\hline
& Juin & Juillet & Août & Septembre & Total\\
\hline
Réussies & 24 & 43 & 49 & 49 & 165\\
Réussies & 21 & 35 & 41 & 43 & 140\\
\hline
Ratées & 26 & 32 & 12 & 5 & 75\\
Ratées & 25 & 44 & 11 & 9 & 89\\
\hline
Total & 50 & 75 & 61 & 54 & 240\\
Total & 46 & 79 & 52 & 52 & 229\\
\hline
\end{tabular}
\end{center}
\item Proportion de photos réussies
\[
\frac{165}{240} = 0.69 = 68\%
\frac{140}{229} = 0.61 = 61\%
\]
\item
\begin{itemize}
\item De juin à juillet
\[
\frac{32 - 26}{26} = \frac{6}{26} = 0.23 = 23\%
\frac{44 - 25}{25} = \frac{19}{25} = 0.76 = 76\%
\]
\item De juillet à août
\[
\frac{12 - 32}{32} = \frac{-20}{32} = -0.62 = -62\%
\frac{11 - 44}{44} = \frac{-33}{44} = -0.75 = -75\%
\]
\item De août à septembre
\[
\frac{5 - 12}{12} = \frac{-7}{12} = -0.58 = -58\%
\frac{9 - 11}{11} = \frac{-2}{11} = -0.18 = -18\%
\]
\end{itemize}
\end{enumerate}
\end{solution}
\printsolutionstype{exercise}
\end{document}

View File

@ -17,17 +17,17 @@
Détailler les calculs suivants et donner le résultat sous la forme d'une fraction irréductible.
\begin{multicols}{3}
\begin{enumerate}[label={\Alph*=}]
\item $\dfrac{10}{5} + \dfrac{2}{5}$
\item $\dfrac{8}{9} + 6$
\item $\dfrac{10}{2} + \dfrac{3}{2}$
\item $\dfrac{6}{9} + 7$
\item $\dfrac{3}{4} + \dfrac{1}{24}$
\item $\dfrac{2}{4} + \dfrac{1}{2}$
\item $\dfrac{3}{9} + \dfrac{8}{45}$
\item $\dfrac{2}{3} + \dfrac{10}{4}$
\item $\dfrac{2}{4} \times 1$
\item $\dfrac{1}{5} \times \dfrac{1}{5}$
\item $\dfrac{5}{6} \times 9$
\item $\dfrac{6}{10} \times \dfrac{5}{10}$
\item $\dfrac{- 2}{10} \times \dfrac{- 5}{20}$
\item $\dfrac{\dfrac{4}{7}}{\dfrac{7}{4}}$
\item $\dfrac{- 6}{9} \times \dfrac{- 4}{90}$
\item $\dfrac{\dfrac{6}{7}}{\dfrac{1}{8}}$
\end{enumerate}
\end{multicols}
\end{exercise}
@ -35,15 +35,15 @@
\begin{solution}
\begin{enumerate}[label={\Alph*=}]
\item $\dfrac{10}{5} + \dfrac{2}{5}=\dfrac{10 + 2}{5}=\dfrac{12}{5} = \dfrac{12}{5}$
\item $\dfrac{8}{9} + 6=\dfrac{8}{9} + \dfrac{6}{1}=\dfrac{8}{9} + \dfrac{6 \times 9}{1 \times 9}=\dfrac{8}{9} + \dfrac{54}{9}=\dfrac{8 + 54}{9}=\dfrac{62}{9} = \dfrac{62}{9}$
\item $\dfrac{3}{4} + \dfrac{1}{24}=\dfrac{3 \times 6}{4 \times 6} + \dfrac{1}{24}=\dfrac{18}{24} + \dfrac{1}{24}=\dfrac{18 + 1}{24}=\dfrac{19}{24} = \dfrac{19}{24}$
\item $\dfrac{2}{4} + \dfrac{1}{2}=\dfrac{2}{4} + \dfrac{1 \times 2}{2 \times 2}=\dfrac{2}{4} + \dfrac{2}{4}=\dfrac{2 + 2}{4}=\dfrac{4}{4} = 1$
\item $\dfrac{10}{2} + \dfrac{3}{2}=\dfrac{10 + 3}{2}=\dfrac{13}{2} = \dfrac{13}{2}$
\item $\dfrac{6}{9} + 7=\dfrac{6}{9} + \dfrac{7}{1}=\dfrac{6}{9} + \dfrac{7 \times 9}{1 \times 9}=\dfrac{6}{9} + \dfrac{63}{9}=\dfrac{6 + 63}{9}=\dfrac{69}{9} = \dfrac{23}{3}$
\item $\dfrac{3}{9} + \dfrac{8}{45}=\dfrac{3 \times 5}{9 \times 5} + \dfrac{8}{45}=\dfrac{15}{45} + \dfrac{8}{45}=\dfrac{15 + 8}{45}=\dfrac{23}{45} = \dfrac{23}{45}$
\item $\dfrac{2}{3} + \dfrac{10}{4}=\dfrac{2 \times 4}{3 \times 4} + \dfrac{10 \times 3}{4 \times 3}=\dfrac{8}{12} + \dfrac{30}{12}=\dfrac{8 + 30}{12}=\dfrac{38}{12} = \dfrac{19}{6}$
\item $\dfrac{2}{4} \times 1=\dfrac{2}{4} = \dfrac{1}{2}$
\item $\dfrac{1}{5} \times \dfrac{1}{5}=\dfrac{1 \times 1}{5 \times 5}=\dfrac{1}{25} = \dfrac{1}{25}$
\item $\dfrac{- 2}{10} \times \dfrac{- 5}{20}=\dfrac{- 2(- 5)}{10 \times 20}=\dfrac{10}{200} = \dfrac{1}{20}$
\item $\dfrac{\dfrac{4}{7}}{\dfrac{7}{4}}=\dfrac{4}{7} \times \dfrac{4}{7}=\dfrac{4 \times 4}{7 \times 7}=\dfrac{16}{49} = \dfrac{16}{49}$
\item $\dfrac{5}{6} \times 9=\dfrac{5 \times 9}{6}=\dfrac{45}{6} = \dfrac{15}{2}$
\item $\dfrac{6}{10} \times \dfrac{5}{10}=\dfrac{6 \times 5}{10 \times 10}=\dfrac{30}{100} = \dfrac{3}{10}$
\item $\dfrac{- 6}{9} \times \dfrac{- 4}{90}=\dfrac{- 6(- 4)}{9 \times 90}=\dfrac{24}{810} = \dfrac{4}{135}$
\item $\dfrac{\dfrac{6}{7}}{\dfrac{1}{8}}=\dfrac{6}{7} \times \dfrac{8}{1}=\dfrac{6 \times 8}{7 \times 1}=\dfrac{48}{7} = \dfrac{48}{7}$
\end{enumerate}
\end{solution}
@ -52,10 +52,10 @@
Le radar a pris des photos pendant l'été:
\begin{itemize}
\item en juin, il y a eu 52 photos prises dont 28 ratées.
\item en juillet, il y a eu 39 photos réussies et 36 ratées.
\item en août, il y a eu 51 photos dont une proportion de 0.2 de photos ratées.
\item en septembre, il y a eu 5 photos ratées, ce qui correspondait à 10.42\% des photos prises.
\item en juin, il y a eu 50 photos prises dont 27 ratées.
\item en juillet, il y a eu 46 photos réussies et 35 ratées.
\item en août, il y a eu 65 photos dont une proportion de 0.29 de photos ratées.
\item en septembre, il y a eu 5 photos ratées, ce qui correspondait à 9.43\% des photos prises.
\end{itemize}
\begin{enumerate}
@ -87,37 +87,35 @@
\hline
& Juin & Juillet & Août & Septembre & Total\\
\hline
Réussies & 24 & 39 & 41 & 43 & 147\\
Réussies & 23 & 46 & 46 & 48 & 163\\
\hline
Ratées & 28 & 36 & 10 & 5 & 79\\
Ratées & 27 & 35 & 19 & 5 & 86\\
\hline
Total & 52 & 75 & 51 & 48 & 226\\
Total & 50 & 81 & 65 & 53 & 249\\
\hline
\end{tabular}
\end{center}
\item Proportion de photos réussies
\[
\frac{147}{226} = 0.65 = 65\%
\frac{163}{249} = 0.65 = 65\%
\]
\item
\begin{itemize}
\item De juin à juillet
\[
\frac{36 - 28}{28} = \frac{8}{28} = 0.29 = 28\%
\frac{35 - 27}{27} = \frac{8}{27} = 0.3 = 29\%
\]
\item De juillet à août
\[
\frac{10 - 36}{36} = \frac{-26}{36} = -0.72 = -72\%
\frac{19 - 35}{35} = \frac{-16}{35} = -0.46 = -45\%
\]
\item De août à septembre
\[
\frac{5 - 10}{10} = \frac{-5}{10} = -0.5 = -50\%
\frac{5 - 19}{19} = \frac{-14}{19} = -0.74 = -73\%
\]
\end{itemize}
\end{enumerate}
\end{solution}
\printsolutionstype{exercise}
\end{document}

View File

@ -17,17 +17,17 @@
Détailler les calculs suivants et donner le résultat sous la forme d'une fraction irréductible.
\begin{multicols}{3}
\begin{enumerate}[label={\Alph*=}]
\item $\dfrac{6}{8} + \dfrac{7}{8}$
\item $\dfrac{9}{5} + 8$
\item $\dfrac{3}{8} + \dfrac{7}{8}$
\item $\dfrac{1}{2} + 2$
\item $\dfrac{2}{5} + \dfrac{10}{35}$
\item $\dfrac{6}{3} + \dfrac{8}{6}$
\item $\dfrac{4}{9} + \dfrac{7}{81}$
\item $\dfrac{6}{8} + \dfrac{9}{6}$
\item $\dfrac{7}{4} \times 8$
\item $\dfrac{4}{8} \times \dfrac{5}{8}$
\item $\dfrac{9}{4} \times 5$
\item $\dfrac{8}{2} \times \dfrac{10}{2}$
\item $\dfrac{- 6}{2} \times \dfrac{- 2}{10}$
\item $\dfrac{\dfrac{3}{10}}{\dfrac{8}{10}}$
\item $\dfrac{- 4}{7} \times \dfrac{9}{49}$
\item $\dfrac{\dfrac{4}{7}}{\dfrac{5}{7}}$
\end{enumerate}
\end{multicols}
\end{exercise}
@ -35,15 +35,15 @@
\begin{solution}
\begin{enumerate}[label={\Alph*=}]
\item $\dfrac{6}{8} + \dfrac{7}{8}=\dfrac{6 + 7}{8}=\dfrac{13}{8} = \dfrac{13}{8}$
\item $\dfrac{9}{5} + 8=\dfrac{9}{5} + \dfrac{8}{1}=\dfrac{9}{5} + \dfrac{8 \times 5}{1 \times 5}=\dfrac{9}{5} + \dfrac{40}{5}=\dfrac{9 + 40}{5}=\dfrac{49}{5} = \dfrac{49}{5}$
\item $\dfrac{2}{5} + \dfrac{10}{35}=\dfrac{2 \times 7}{5 \times 7} + \dfrac{10}{35}=\dfrac{14}{35} + \dfrac{10}{35}=\dfrac{14 + 10}{35}=\dfrac{24}{35} = \dfrac{24}{35}$
\item $\dfrac{6}{3} + \dfrac{8}{6}=\dfrac{6 \times 2}{3 \times 2} + \dfrac{8}{6}=\dfrac{12}{6} + \dfrac{8}{6}=\dfrac{12 + 8}{6}=\dfrac{20}{6} = \dfrac{10}{3}$
\item $\dfrac{3}{8} + \dfrac{7}{8}=\dfrac{3 + 7}{8}=\dfrac{10}{8} = \dfrac{5}{4}$
\item $\dfrac{1}{2} + 2=\dfrac{1}{2} + \dfrac{2}{1}=\dfrac{1}{2} + \dfrac{2 \times 2}{1 \times 2}=\dfrac{1}{2} + \dfrac{4}{2}=\dfrac{1 + 4}{2}=\dfrac{5}{2} = \dfrac{5}{2}$
\item $\dfrac{4}{9} + \dfrac{7}{81}=\dfrac{4 \times 9}{9 \times 9} + \dfrac{7}{81}=\dfrac{36}{81} + \dfrac{7}{81}=\dfrac{36 + 7}{81}=\dfrac{43}{81} = \dfrac{43}{81}$
\item $\dfrac{6}{8} + \dfrac{9}{6}=\dfrac{6 \times 3}{8 \times 3} + \dfrac{9 \times 4}{6 \times 4}=\dfrac{18}{24} + \dfrac{36}{24}=\dfrac{18 + 36}{24}=\dfrac{54}{24} = \dfrac{9}{4}$
\item $\dfrac{7}{4} \times 8=\dfrac{7 \times 8}{4}=\dfrac{56}{4} = 14$
\item $\dfrac{4}{8} \times \dfrac{5}{8}=\dfrac{4 \times 5}{8 \times 8}=\dfrac{20}{64} = \dfrac{5}{16}$
\item $\dfrac{- 6}{2} \times \dfrac{- 2}{10}=\dfrac{- 6(- 2)}{2 \times 10}=\dfrac{12}{20} = \dfrac{3}{5}$
\item $\dfrac{\dfrac{3}{10}}{\dfrac{8}{10}}=\dfrac{3}{10} \times \dfrac{10}{8}=\dfrac{3 \times 10}{10 \times 8}=\dfrac{30}{80} = \dfrac{3}{8}$
\item $\dfrac{9}{4} \times 5=\dfrac{9 \times 5}{4}=\dfrac{45}{4} = \dfrac{45}{4}$
\item $\dfrac{8}{2} \times \dfrac{10}{2}=\dfrac{8 \times 10}{2 \times 2}=\dfrac{80}{4} = 20$
\item $\dfrac{- 4}{7} \times \dfrac{9}{49}=\dfrac{- 4 \times 9}{7 \times 49}=\dfrac{- 36}{343} = \dfrac{- 36}{343}$
\item $\dfrac{\dfrac{4}{7}}{\dfrac{5}{7}}=\dfrac{4}{7} \times \dfrac{7}{5}=\dfrac{4 \times 7}{7 \times 5}=\dfrac{28}{35} = \dfrac{4}{5}$
\end{enumerate}
\end{solution}
@ -52,10 +52,10 @@
Le radar a pris des photos pendant l'été:
\begin{itemize}
\item en juin, il y a eu 45 photos prises dont 20 ratées.
\item en juillet, il y a eu 39 photos réussies et 50 ratées.
\item en août, il y a eu 60 photos dont une proportion de 0.23 de photos ratées.
\item en septembre, il y a eu 15 photos ratées, ce qui correspondait à 26.32\% des photos prises.
\item en juin, il y a eu 49 photos prises dont 24 ratées.
\item en juillet, il y a eu 35 photos réussies et 34 ratées.
\item en août, il y a eu 58 photos dont une proportion de 0.29 de photos ratées.
\item en septembre, il y a eu 6 photos ratées, ce qui correspondait à 11.76\% des photos prises.
\end{itemize}
\begin{enumerate}
@ -87,37 +87,35 @@
\hline
& Juin & Juillet & Août & Septembre & Total\\
\hline
Réussies & 25 & 39 & 46 & 42 & 152\\
Réussies & 25 & 35 & 41 & 45 & 146\\
\hline
Ratées & 20 & 50 & 14 & 15 & 99\\
Ratées & 24 & 34 & 17 & 6 & 81\\
\hline
Total & 45 & 89 & 60 & 57 & 251\\
Total & 49 & 69 & 58 & 51 & 227\\
\hline
\end{tabular}
\end{center}
\item Proportion de photos réussies
\[
\frac{152}{251} = 0.61 = 60\%
\frac{146}{227} = 0.64 = 64\%
\]
\item
\begin{itemize}
\item De juin à juillet
\[
\frac{50 - 20}{20} = \frac{30}{20} = 1.5 = 150\%
\frac{34 - 24}{24} = \frac{10}{24} = 0.42 = 41\%
\]
\item De juillet à août
\[
\frac{14 - 50}{50} = \frac{-36}{50} = -0.72 = -72\%
\frac{17 - 34}{34} = \frac{-17}{34} = -0.5 = -50\%
\]
\item De août à septembre
\[
\frac{15 - 14}{14} = \frac{1}{14} = 0.07 = 7\%
\frac{6 - 17}{17} = \frac{-11}{17} = -0.65 = -64\%
\]
\end{itemize}
\end{enumerate}
\end{solution}
\printsolutionstype{exercise}
\end{document}

View File

@ -17,17 +17,17 @@
Détailler les calculs suivants et donner le résultat sous la forme d'une fraction irréductible.
\begin{multicols}{3}
\begin{enumerate}[label={\Alph*=}]
\item $\dfrac{1}{2} + \dfrac{8}{2}$
\item $\dfrac{3}{8} + 5$
\item $\dfrac{9}{5} + \dfrac{3}{5}$
\item $\dfrac{1}{5} + 6$
\item $\dfrac{9}{7} + \dfrac{8}{63}$
\item $\dfrac{2}{10} + \dfrac{7}{10}$
\item $\dfrac{6}{3} + \dfrac{1}{27}$
\item $\dfrac{5}{7} + \dfrac{2}{6}$
\item $\dfrac{8}{2} \times 10$
\item $\dfrac{9}{2} \times \dfrac{8}{2}$
\item $\dfrac{8}{6} \times 1$
\item $\dfrac{7}{9} \times \dfrac{4}{9}$
\item $\dfrac{- 9}{7} \times \dfrac{8}{14}$
\item $\dfrac{\dfrac{1}{7}}{\dfrac{9}{4}}$
\item $\dfrac{9}{5} \times \dfrac{- 9}{10}$
\item $\dfrac{\dfrac{1}{5}}{\dfrac{2}{10}}$
\end{enumerate}
\end{multicols}
\end{exercise}
@ -35,15 +35,15 @@
\begin{solution}
\begin{enumerate}[label={\Alph*=}]
\item $\dfrac{1}{2} + \dfrac{8}{2}=\dfrac{1 + 8}{2}=\dfrac{9}{2} = \dfrac{9}{2}$
\item $\dfrac{3}{8} + 5=\dfrac{3}{8} + \dfrac{5}{1}=\dfrac{3}{8} + \dfrac{5 \times 8}{1 \times 8}=\dfrac{3}{8} + \dfrac{40}{8}=\dfrac{3 + 40}{8}=\dfrac{43}{8} = \dfrac{43}{8}$
\item $\dfrac{9}{7} + \dfrac{8}{63}=\dfrac{9 \times 9}{7 \times 9} + \dfrac{8}{63}=\dfrac{81}{63} + \dfrac{8}{63}=\dfrac{81 + 8}{63}=\dfrac{89}{63} = \dfrac{89}{63}$
\item $\dfrac{2}{10} + \dfrac{7}{10}=\dfrac{2 + 7}{10}=\dfrac{9}{10} = \dfrac{9}{10}$
\item $\dfrac{9}{5} + \dfrac{3}{5}=\dfrac{9 + 3}{5}=\dfrac{12}{5} = \dfrac{12}{5}$
\item $\dfrac{1}{5} + 6=\dfrac{1}{5} + \dfrac{6}{1}=\dfrac{1}{5} + \dfrac{6 \times 5}{1 \times 5}=\dfrac{1}{5} + \dfrac{30}{5}=\dfrac{1 + 30}{5}=\dfrac{31}{5} = \dfrac{31}{5}$
\item $\dfrac{6}{3} + \dfrac{1}{27}=\dfrac{6 \times 9}{3 \times 9} + \dfrac{1}{27}=\dfrac{54}{27} + \dfrac{1}{27}=\dfrac{54 + 1}{27}=\dfrac{55}{27} = \dfrac{55}{27}$
\item $\dfrac{5}{7} + \dfrac{2}{6}=\dfrac{5 \times 6}{7 \times 6} + \dfrac{2 \times 7}{6 \times 7}=\dfrac{30}{42} + \dfrac{14}{42}=\dfrac{30 + 14}{42}=\dfrac{44}{42} = \dfrac{22}{21}$
\item $\dfrac{8}{2} \times 10=\dfrac{8 \times 10}{2}=\dfrac{80}{2} = 40$
\item $\dfrac{9}{2} \times \dfrac{8}{2}=\dfrac{9 \times 8}{2 \times 2}=\dfrac{72}{4} = 18$
\item $\dfrac{- 9}{7} \times \dfrac{8}{14}=\dfrac{- 9 \times 8}{7 \times 14}=\dfrac{- 72}{98} = \dfrac{- 36}{49}$
\item $\dfrac{\dfrac{1}{7}}{\dfrac{9}{4}}=\dfrac{1}{7} \times \dfrac{4}{9}=\dfrac{1 \times 4}{7 \times 9}=\dfrac{4}{63} = \dfrac{4}{63}$
\item $\dfrac{8}{6} \times 1=\dfrac{8}{6} = \dfrac{4}{3}$
\item $\dfrac{7}{9} \times \dfrac{4}{9}=\dfrac{7 \times 4}{9 \times 9}=\dfrac{28}{81} = \dfrac{28}{81}$
\item $\dfrac{9}{5} \times \dfrac{- 9}{10}=\dfrac{9(- 9)}{5 \times 10}=\dfrac{- 81}{50} = \dfrac{- 81}{50}$
\item $\dfrac{\dfrac{1}{5}}{\dfrac{2}{10}}=\dfrac{1}{5} \times \dfrac{10}{2}=\dfrac{1 \times 10}{5 \times 2}=\dfrac{10}{10} = 1$
\end{enumerate}
\end{solution}
@ -52,10 +52,10 @@
Le radar a pris des photos pendant l'été:
\begin{itemize}
\item en juin, il y a eu 46 photos prises dont 25 ratées.
\item en juillet, il y a eu 45 photos réussies et 33 ratées.
\item en août, il y a eu 66 photos dont une proportion de 0.24 de photos ratées.
\item en septembre, il y a eu 13 photos ratées, ce qui correspondait à 23.21\% des photos prises.
\item en juin, il y a eu 47 photos prises dont 22 ratées.
\item en juillet, il y a eu 32 photos réussies et 30 ratées.
\item en août, il y a eu 69 photos dont une proportion de 0.28 de photos ratées.
\item en septembre, il y a eu 13 photos ratées, ce qui correspondait à 24.07\% des photos prises.
\end{itemize}
\begin{enumerate}
@ -87,37 +87,35 @@
\hline
& Juin & Juillet & Août & Septembre & Total\\
\hline
Réussies & 21 & 45 & 50 & 43 & 159\\
Réussies & 25 & 32 & 50 & 41 & 148\\
\hline
Ratées & 25 & 33 & 16 & 13 & 87\\
Ratées & 22 & 30 & 19 & 13 & 84\\
\hline
Total & 46 & 78 & 66 & 56 & 246\\
Total & 47 & 62 & 69 & 54 & 232\\
\hline
\end{tabular}
\end{center}
\item Proportion de photos réussies
\[
\frac{159}{246} = 0.65 = 64\%
\frac{148}{232} = 0.64 = 63\%
\]
\item
\begin{itemize}
\item De juin à juillet
\[
\frac{33 - 25}{25} = \frac{8}{25} = 0.32 = 32\%
\frac{30 - 22}{22} = \frac{8}{22} = 0.36 = 36\%
\]
\item De juillet à août
\[
\frac{16 - 33}{33} = \frac{-17}{33} = -0.52 = -51\%
\frac{19 - 30}{30} = \frac{-11}{30} = -0.37 = -36\%
\]
\item De août à septembre
\[
\frac{13 - 16}{16} = \frac{-3}{16} = -0.19 = -18\%
\frac{13 - 19}{19} = \frac{-6}{19} = -0.32 = -31\%
\]
\end{itemize}
\end{enumerate}
\end{solution}
\printsolutionstype{exercise}
\end{document}

View File

@ -17,17 +17,17 @@
Détailler les calculs suivants et donner le résultat sous la forme d'une fraction irréductible.
\begin{multicols}{3}
\begin{enumerate}[label={\Alph*=}]
\item $\dfrac{3}{6} + \dfrac{7}{6}$
\item $\dfrac{2}{7} + 3$
\item $\dfrac{8}{4} + \dfrac{5}{4}$
\item $\dfrac{8}{5} + 5$
\item $\dfrac{6}{2} + \dfrac{4}{14}$
\item $\dfrac{4}{8} + \dfrac{2}{3}$
\item $\dfrac{9}{7} + \dfrac{8}{14}$
\item $\dfrac{9}{6} + \dfrac{5}{6}$
\item $\dfrac{4}{8} \times 8$
\item $\dfrac{4}{2} \times \dfrac{7}{2}$
\item $\dfrac{10}{9} \times 4$
\item $\dfrac{9}{4} \times \dfrac{5}{4}$
\item $\dfrac{10}{8} \times \dfrac{- 2}{64}$
\item $\dfrac{\dfrac{4}{6}}{\dfrac{7}{9}}$
\item $\dfrac{- 8}{9} \times \dfrac{- 6}{27}$
\item $\dfrac{\dfrac{9}{8}}{\dfrac{3}{9}}$
\end{enumerate}
\end{multicols}
\end{exercise}
@ -35,15 +35,15 @@
\begin{solution}
\begin{enumerate}[label={\Alph*=}]
\item $\dfrac{3}{6} + \dfrac{7}{6}=\dfrac{3 + 7}{6}=\dfrac{10}{6} = \dfrac{5}{3}$
\item $\dfrac{2}{7} + 3=\dfrac{2}{7} + \dfrac{3}{1}=\dfrac{2}{7} + \dfrac{3 \times 7}{1 \times 7}=\dfrac{2}{7} + \dfrac{21}{7}=\dfrac{2 + 21}{7}=\dfrac{23}{7} = \dfrac{23}{7}$
\item $\dfrac{6}{2} + \dfrac{4}{14}=\dfrac{6 \times 7}{2 \times 7} + \dfrac{4}{14}=\dfrac{42}{14} + \dfrac{4}{14}=\dfrac{42 + 4}{14}=\dfrac{46}{14} = \dfrac{23}{7}$
\item $\dfrac{4}{8} + \dfrac{2}{3}=\dfrac{4 \times 3}{8 \times 3} + \dfrac{2 \times 8}{3 \times 8}=\dfrac{12}{24} + \dfrac{16}{24}=\dfrac{12 + 16}{24}=\dfrac{28}{24} = \dfrac{7}{6}$
\item $\dfrac{8}{4} + \dfrac{5}{4}=\dfrac{8 + 5}{4}=\dfrac{13}{4} = \dfrac{13}{4}$
\item $\dfrac{8}{5} + 5=\dfrac{8}{5} + \dfrac{5}{1}=\dfrac{8}{5} + \dfrac{5 \times 5}{1 \times 5}=\dfrac{8}{5} + \dfrac{25}{5}=\dfrac{8 + 25}{5}=\dfrac{33}{5} = \dfrac{33}{5}$
\item $\dfrac{9}{7} + \dfrac{8}{14}=\dfrac{9 \times 2}{7 \times 2} + \dfrac{8}{14}=\dfrac{18}{14} + \dfrac{8}{14}=\dfrac{18 + 8}{14}=\dfrac{26}{14} = \dfrac{13}{7}$
\item $\dfrac{9}{6} + \dfrac{5}{6}=\dfrac{9 + 5}{6}=\dfrac{14}{6} = \dfrac{7}{3}$
\item $\dfrac{4}{8} \times 8=\dfrac{4 \times 8}{8}=\dfrac{32}{8} = 4$
\item $\dfrac{4}{2} \times \dfrac{7}{2}=\dfrac{4 \times 7}{2 \times 2}=\dfrac{28}{4} = 7$
\item $\dfrac{10}{8} \times \dfrac{- 2}{64}=\dfrac{10(- 2)}{8 \times 64}=\dfrac{- 20}{512} = \dfrac{- 5}{128}$
\item $\dfrac{\dfrac{4}{6}}{\dfrac{7}{9}}=\dfrac{4}{6} \times \dfrac{9}{7}=\dfrac{4 \times 9}{6 \times 7}=\dfrac{36}{42} = \dfrac{6}{7}$
\item $\dfrac{10}{9} \times 4=\dfrac{10 \times 4}{9}=\dfrac{40}{9} = \dfrac{40}{9}$
\item $\dfrac{9}{4} \times \dfrac{5}{4}=\dfrac{9 \times 5}{4 \times 4}=\dfrac{45}{16} = \dfrac{45}{16}$
\item $\dfrac{- 8}{9} \times \dfrac{- 6}{27}=\dfrac{- 8(- 6)}{9 \times 27}=\dfrac{48}{243} = \dfrac{16}{81}$
\item $\dfrac{\dfrac{9}{8}}{\dfrac{3}{9}}=\dfrac{9}{8} \times \dfrac{9}{3}=\dfrac{9 \times 9}{8 \times 3}=\dfrac{81}{24} = \dfrac{27}{8}$
\end{enumerate}
\end{solution}
@ -52,10 +52,10 @@
Le radar a pris des photos pendant l'été:
\begin{itemize}
\item en juin, il y a eu 58 photos prises dont 30 ratées.
\item en juillet, il y a eu 34 photos réussies et 33 ratées.
\item en août, il y a eu 57 photos dont une proportion de 0.19 de photos ratées.
\item en septembre, il y a eu 10 photos ratées, ce qui correspondait à 18.52\% des photos prises.
\item en juin, il y a eu 45 photos prises dont 20 ratées.
\item en juillet, il y a eu 30 photos réussies et 30 ratées.
\item en août, il y a eu 64 photos dont une proportion de 0.27 de photos ratées.
\item en septembre, il y a eu 15 photos ratées, ce qui correspondait à 26.79\% des photos prises.
\end{itemize}
\begin{enumerate}
@ -87,37 +87,35 @@
\hline
& Juin & Juillet & Août & Septembre & Total\\
\hline
Réussies & 28 & 34 & 46 & 44 & 152\\
Réussies & 25 & 30 & 47 & 41 & 143\\
\hline
Ratées & 30 & 33 & 11 & 10 & 84\\
Ratées & 20 & 30 & 17 & 15 & 82\\
\hline
Total & 58 & 67 & 57 & 54 & 236\\
Total & 45 & 60 & 64 & 56 & 225\\
\hline
\end{tabular}
\end{center}
\item Proportion de photos réussies
\[
\frac{152}{236} = 0.64 = 64\%
\frac{143}{225} = 0.64 = 63\%
\]
\item
\begin{itemize}
\item De juin à juillet
\[
\frac{33 - 30}{30} = \frac{3}{30} = 0.1 = 10\%
\frac{30 - 20}{20} = \frac{10}{20} = 0.5 = 50\%
\]
\item De juillet à août
\[
\frac{11 - 33}{33} = \frac{-22}{33} = -0.67 = -66\%
\frac{17 - 30}{30} = \frac{-13}{30} = -0.43 = -43\%
\]
\item De août à septembre
\[
\frac{10 - 11}{11} = \frac{-1}{11} = -0.09 = -9\%
\frac{15 - 17}{17} = \frac{-2}{17} = -0.12 = -11\%
\]
\end{itemize}
\end{enumerate}
\end{solution}
\printsolutionstype{exercise}
\end{document}

View File

@ -17,17 +17,17 @@
Détailler les calculs suivants et donner le résultat sous la forme d'une fraction irréductible.
\begin{multicols}{3}
\begin{enumerate}[label={\Alph*=}]
\item $\dfrac{7}{8} + \dfrac{5}{8}$
\item $\dfrac{1}{5} + 6$
\item $\dfrac{10}{5} + \dfrac{4}{5}$
\item $\dfrac{1}{3} + 6$
\item $\dfrac{3}{7} + \dfrac{4}{35}$
\item $\dfrac{1}{3} + \dfrac{8}{10}$
\item $\dfrac{1}{2} + \dfrac{4}{4}$
\item $\dfrac{8}{3} + \dfrac{6}{9}$
\item $\dfrac{2}{9} \times 6$
\item $\dfrac{7}{5} \times \dfrac{7}{5}$
\item $\dfrac{10}{6} \times 2$
\item $\dfrac{3}{2} \times \dfrac{7}{2}$
\item $\dfrac{- 7}{4} \times \dfrac{- 4}{12}$
\item $\dfrac{\dfrac{8}{7}}{\dfrac{3}{7}}$
\item $\dfrac{- 2}{6} \times \dfrac{10}{36}$
\item $\dfrac{\dfrac{2}{9}}{\dfrac{5}{10}}$
\end{enumerate}
\end{multicols}
\end{exercise}
@ -35,15 +35,15 @@
\begin{solution}
\begin{enumerate}[label={\Alph*=}]
\item $\dfrac{7}{8} + \dfrac{5}{8}=\dfrac{7 + 5}{8}=\dfrac{12}{8} = \dfrac{3}{2}$
\item $\dfrac{1}{5} + 6=\dfrac{1}{5} + \dfrac{6}{1}=\dfrac{1}{5} + \dfrac{6 \times 5}{1 \times 5}=\dfrac{1}{5} + \dfrac{30}{5}=\dfrac{1 + 30}{5}=\dfrac{31}{5} = \dfrac{31}{5}$
\item $\dfrac{3}{7} + \dfrac{4}{35}=\dfrac{3 \times 5}{7 \times 5} + \dfrac{4}{35}=\dfrac{15}{35} + \dfrac{4}{35}=\dfrac{15 + 4}{35}=\dfrac{19}{35} = \dfrac{19}{35}$
\item $\dfrac{1}{3} + \dfrac{8}{10}=\dfrac{1 \times 10}{3 \times 10} + \dfrac{8 \times 3}{10 \times 3}=\dfrac{10}{30} + \dfrac{24}{30}=\dfrac{10 + 24}{30}=\dfrac{34}{30} = \dfrac{17}{15}$
\item $\dfrac{10}{5} + \dfrac{4}{5}=\dfrac{10 + 4}{5}=\dfrac{14}{5} = \dfrac{14}{5}$
\item $\dfrac{1}{3} + 6=\dfrac{1}{3} + \dfrac{6}{1}=\dfrac{1}{3} + \dfrac{6 \times 3}{1 \times 3}=\dfrac{1}{3} + \dfrac{18}{3}=\dfrac{1 + 18}{3}=\dfrac{19}{3} = \dfrac{19}{3}$
\item $\dfrac{1}{2} + \dfrac{4}{4}=\dfrac{1 \times 2}{2 \times 2} + \dfrac{4}{4}=\dfrac{2}{4} + \dfrac{4}{4}=\dfrac{2 + 4}{4}=\dfrac{6}{4} = \dfrac{3}{2}$
\item $\dfrac{8}{3} + \dfrac{6}{9}=\dfrac{8 \times 3}{3 \times 3} + \dfrac{6}{9}=\dfrac{24}{9} + \dfrac{6}{9}=\dfrac{24 + 6}{9}=\dfrac{30}{9} = \dfrac{10}{3}$
\item $\dfrac{2}{9} \times 6=\dfrac{2 \times 6}{9}=\dfrac{12}{9} = \dfrac{4}{3}$
\item $\dfrac{7}{5} \times \dfrac{7}{5}=\dfrac{7 \times 7}{5 \times 5}=\dfrac{49}{25} = \dfrac{49}{25}$
\item $\dfrac{- 7}{4} \times \dfrac{- 4}{12}=\dfrac{- 7(- 4)}{4 \times 12}=\dfrac{28}{48} = \dfrac{7}{12}$
\item $\dfrac{\dfrac{8}{7}}{\dfrac{3}{7}}=\dfrac{8}{7} \times \dfrac{7}{3}=\dfrac{8 \times 7}{7 \times 3}=\dfrac{56}{21} = \dfrac{8}{3}$
\item $\dfrac{10}{6} \times 2=\dfrac{10 \times 2}{6}=\dfrac{20}{6} = \dfrac{10}{3}$
\item $\dfrac{3}{2} \times \dfrac{7}{2}=\dfrac{3 \times 7}{2 \times 2}=\dfrac{21}{4} = \dfrac{21}{4}$
\item $\dfrac{- 2}{6} \times \dfrac{10}{36}=\dfrac{- 2 \times 10}{6 \times 36}=\dfrac{- 20}{216} = \dfrac{- 5}{54}$
\item $\dfrac{\dfrac{2}{9}}{\dfrac{5}{10}}=\dfrac{2}{9} \times \dfrac{10}{5}=\dfrac{2 \times 10}{9 \times 5}=\dfrac{20}{45} = \dfrac{4}{9}$
\end{enumerate}
\end{solution}
@ -52,10 +52,10 @@
Le radar a pris des photos pendant l'été:
\begin{itemize}
\item en juin, il y a eu 46 photos prises dont 21 ratées.
\item en juillet, il y a eu 48 photos réussies et 45 ratées.
\item en août, il y a eu 60 photos dont une proportion de 0.23 de photos ratées.
\item en septembre, il y a eu 10 photos ratées, ce qui correspondait à 16.67\% des photos prises.
\item en juin, il y a eu 48 photos prises dont 22 ratées.
\item en juillet, il y a eu 40 photos réussies et 31 ratées.
\item en août, il y a eu 59 photos dont une proportion de 0.19 de photos ratées.
\item en septembre, il y a eu 6 photos ratées, ce qui correspondait à 11.11\% des photos prises.
\end{itemize}
\begin{enumerate}
@ -87,37 +87,35 @@
\hline
& Juin & Juillet & Août & Septembre & Total\\
\hline
Réussies & 25 & 48 & 46 & 50 & 169\\
Réussies & 26 & 40 & 48 & 48 & 162\\
\hline
Ratées & 21 & 45 & 14 & 10 & 90\\
Ratées & 22 & 31 & 11 & 6 & 70\\
\hline
Total & 46 & 93 & 60 & 60 & 259\\
Total & 48 & 71 & 59 & 54 & 232\\
\hline
\end{tabular}
\end{center}
\item Proportion de photos réussies
\[
\frac{169}{259} = 0.65 = 65\%
\frac{162}{232} = 0.7 = 69\%
\]
\item
\begin{itemize}
\item De juin à juillet
\[
\frac{45 - 21}{21} = \frac{24}{21} = 1.14 = 114\%
\frac{31 - 22}{22} = \frac{9}{22} = 0.41 = 40\%
\]
\item De juillet à août
\[
\frac{14 - 45}{45} = \frac{-31}{45} = -0.69 = -68\%
\frac{11 - 31}{31} = \frac{-20}{31} = -0.65 = -64\%
\]
\item De août à septembre
\[
\frac{10 - 14}{14} = \frac{-4}{14} = -0.29 = -28\%
\frac{6 - 11}{11} = \frac{-5}{11} = -0.45 = -45\%
\]
\end{itemize}
\end{enumerate}
\end{solution}
\printsolutionstype{exercise}
\end{document}

View File

@ -17,17 +17,17 @@
Détailler les calculs suivants et donner le résultat sous la forme d'une fraction irréductible.
\begin{multicols}{3}
\begin{enumerate}[label={\Alph*=}]
\item $\dfrac{5}{4} + \dfrac{6}{4}$
\item $\dfrac{7}{4} + 6$
\item $\dfrac{1}{2} + \dfrac{3}{2}$
\item $\dfrac{4}{6} + 4$
\item $\dfrac{8}{4} + \dfrac{9}{20}$
\item $\dfrac{3}{10} + \dfrac{3}{7}$
\item $\dfrac{6}{5} + \dfrac{7}{10}$
\item $\dfrac{6}{2} + \dfrac{4}{10}$
\item $\dfrac{1}{4} \times 2$
\item $\dfrac{3}{9} \times \dfrac{10}{9}$
\item $\dfrac{6}{2} \times 7$
\item $\dfrac{5}{8} \times \dfrac{4}{8}$
\item $\dfrac{6}{2} \times \dfrac{4}{10}$
\item $\dfrac{\dfrac{8}{9}}{\dfrac{9}{5}}$
\item $\dfrac{- 6}{5} \times \dfrac{4}{15}$
\item $\dfrac{\dfrac{10}{8}}{\dfrac{3}{6}}$
\end{enumerate}
\end{multicols}
\end{exercise}
@ -35,15 +35,15 @@
\begin{solution}
\begin{enumerate}[label={\Alph*=}]
\item $\dfrac{5}{4} + \dfrac{6}{4}=\dfrac{5 + 6}{4}=\dfrac{11}{4} = \dfrac{11}{4}$
\item $\dfrac{7}{4} + 6=\dfrac{7}{4} + \dfrac{6}{1}=\dfrac{7}{4} + \dfrac{6 \times 4}{1 \times 4}=\dfrac{7}{4} + \dfrac{24}{4}=\dfrac{7 + 24}{4}=\dfrac{31}{4} = \dfrac{31}{4}$
\item $\dfrac{8}{4} + \dfrac{9}{20}=\dfrac{8 \times 5}{4 \times 5} + \dfrac{9}{20}=\dfrac{40}{20} + \dfrac{9}{20}=\dfrac{40 + 9}{20}=\dfrac{49}{20} = \dfrac{49}{20}$
\item $\dfrac{3}{10} + \dfrac{3}{7}=\dfrac{3 \times 7}{10 \times 7} + \dfrac{3 \times 10}{7 \times 10}=\dfrac{21}{70} + \dfrac{30}{70}=\dfrac{21 + 30}{70}=\dfrac{51}{70} = \dfrac{51}{70}$
\item $\dfrac{1}{2} + \dfrac{3}{2}=\dfrac{1 + 3}{2}=\dfrac{4}{2} = 2$
\item $\dfrac{4}{6} + 4=\dfrac{4}{6} + \dfrac{4}{1}=\dfrac{4}{6} + \dfrac{4 \times 6}{1 \times 6}=\dfrac{4}{6} + \dfrac{24}{6}=\dfrac{4 + 24}{6}=\dfrac{28}{6} = \dfrac{14}{3}$
\item $\dfrac{6}{5} + \dfrac{7}{10}=\dfrac{6 \times 2}{5 \times 2} + \dfrac{7}{10}=\dfrac{12}{10} + \dfrac{7}{10}=\dfrac{12 + 7}{10}=\dfrac{19}{10} = \dfrac{19}{10}$
\item $\dfrac{6}{2} + \dfrac{4}{10}=\dfrac{6 \times 5}{2 \times 5} + \dfrac{4}{10}=\dfrac{30}{10} + \dfrac{4}{10}=\dfrac{30 + 4}{10}=\dfrac{34}{10} = \dfrac{17}{5}$
\item $\dfrac{1}{4} \times 2=\dfrac{1 \times 2}{4}=\dfrac{2}{4} = \dfrac{1}{2}$
\item $\dfrac{3}{9} \times \dfrac{10}{9}=\dfrac{3 \times 10}{9 \times 9}=\dfrac{30}{81} = \dfrac{10}{27}$
\item $\dfrac{6}{2} \times \dfrac{4}{10}=\dfrac{6 \times 4}{2 \times 10}=\dfrac{24}{20} = \dfrac{6}{5}$
\item $\dfrac{\dfrac{8}{9}}{\dfrac{9}{5}}=\dfrac{8}{9} \times \dfrac{5}{9}=\dfrac{8 \times 5}{9 \times 9}=\dfrac{40}{81} = \dfrac{40}{81}$
\item $\dfrac{6}{2} \times 7=\dfrac{6 \times 7}{2}=\dfrac{42}{2} = 21$
\item $\dfrac{5}{8} \times \dfrac{4}{8}=\dfrac{5 \times 4}{8 \times 8}=\dfrac{20}{64} = \dfrac{5}{16}$
\item $\dfrac{- 6}{5} \times \dfrac{4}{15}=\dfrac{- 6 \times 4}{5 \times 15}=\dfrac{- 24}{75} = \dfrac{- 8}{25}$
\item $\dfrac{\dfrac{10}{8}}{\dfrac{3}{6}}=\dfrac{10}{8} \times \dfrac{6}{3}=\dfrac{10 \times 6}{8 \times 3}=\dfrac{60}{24} = \dfrac{5}{2}$
\end{enumerate}
\end{solution}
@ -52,10 +52,10 @@
Le radar a pris des photos pendant l'été:
\begin{itemize}
\item en juin, il y a eu 55 photos prises dont 27 ratées.
\item en juillet, il y a eu 31 photos réussies et 42 ratées.
\item en août, il y a eu 59 photos dont une proportion de 0.2 de photos ratées.
\item en septembre, il y a eu 11 photos ratées, ce qui correspondait à 21.57\% des photos prises.
\item en juin, il y a eu 45 photos prises dont 22 ratées.
\item en juillet, il y a eu 34 photos réussies et 48 ratées.
\item en août, il y a eu 61 photos dont une proportion de 0.26 de photos ratées.
\item en septembre, il y a eu 12 photos ratées, ce qui correspondait à 22.64\% des photos prises.
\end{itemize}
\begin{enumerate}
@ -87,37 +87,35 @@
\hline
& Juin & Juillet & Août & Septembre & Total\\
\hline
Réussies & 28 & 31 & 47 & 40 & 146\\
Réussies & 23 & 34 & 45 & 41 & 143\\
\hline
Ratées & 27 & 42 & 12 & 11 & 92\\
Ratées & 22 & 48 & 16 & 12 & 98\\
\hline
Total & 55 & 73 & 59 & 51 & 238\\
Total & 45 & 82 & 61 & 53 & 241\\
\hline
\end{tabular}
\end{center}
\item Proportion de photos réussies
\[
\frac{146}{238} = 0.61 = 61\%
\frac{143}{241} = 0.59 = 59\%
\]
\item
\begin{itemize}
\item De juin à juillet
\[
\frac{42 - 27}{27} = \frac{15}{27} = 0.56 = 55\%
\frac{48 - 22}{22} = \frac{26}{22} = 1.18 = 118\%
\]
\item De juillet à août
\[
\frac{12 - 42}{42} = \frac{-30}{42} = -0.71 = -71\%
\frac{16 - 48}{48} = \frac{-32}{48} = -0.67 = -66\%
\]
\item De août à septembre
\[
\frac{11 - 12}{12} = \frac{-1}{12} = -0.08 = -8\%
\frac{12 - 16}{16} = \frac{-4}{16} = -0.25 = -25\%
\]
\end{itemize}
\end{enumerate}
\end{solution}
\printsolutionstype{exercise}
\end{document}

View File

@ -17,17 +17,17 @@
Détailler les calculs suivants et donner le résultat sous la forme d'une fraction irréductible.
\begin{multicols}{3}
\begin{enumerate}[label={\Alph*=}]
\item $\dfrac{4}{8} + \dfrac{7}{8}$
\item $\dfrac{4}{2} + 6$
\item $\dfrac{4}{2} + \dfrac{5}{2}$
\item $\dfrac{6}{8} + 1$
\item $\dfrac{8}{7} + \dfrac{5}{49}$
\item $\dfrac{4}{9} + \dfrac{7}{9}$
\item $\dfrac{9}{10} + \dfrac{1}{10}$
\item $\dfrac{1}{8} + \dfrac{3}{7}$
\item $\dfrac{6}{2} \times 3$
\item $\dfrac{2}{10} \times \dfrac{9}{10}$
\item $\dfrac{7}{2} \times 7$
\item $\dfrac{1}{7} \times \dfrac{9}{7}$
\item $\dfrac{- 4}{4} \times \dfrac{- 9}{36}$
\item $\dfrac{\dfrac{6}{7}}{\dfrac{10}{6}}$
\item $\dfrac{- 2}{7} \times \dfrac{- 8}{42}$
\item $\dfrac{\dfrac{8}{4}}{\dfrac{5}{4}}$
\end{enumerate}
\end{multicols}
\end{exercise}
@ -35,15 +35,15 @@
\begin{solution}
\begin{enumerate}[label={\Alph*=}]
\item $\dfrac{4}{8} + \dfrac{7}{8}=\dfrac{4 + 7}{8}=\dfrac{11}{8} = \dfrac{11}{8}$
\item $\dfrac{4}{2} + 6=\dfrac{4}{2} + \dfrac{6}{1}=\dfrac{4}{2} + \dfrac{6 \times 2}{1 \times 2}=\dfrac{4}{2} + \dfrac{12}{2}=\dfrac{4 + 12}{2}=\dfrac{16}{2} = 8$
\item $\dfrac{8}{7} + \dfrac{5}{49}=\dfrac{8 \times 7}{7 \times 7} + \dfrac{5}{49}=\dfrac{56}{49} + \dfrac{5}{49}=\dfrac{56 + 5}{49}=\dfrac{61}{49} = \dfrac{61}{49}$
\item $\dfrac{4}{9} + \dfrac{7}{9}=\dfrac{4 + 7}{9}=\dfrac{11}{9} = \dfrac{11}{9}$
\item $\dfrac{4}{2} + \dfrac{5}{2}=\dfrac{4 + 5}{2}=\dfrac{9}{2} = \dfrac{9}{2}$
\item $\dfrac{6}{8} + 1=\dfrac{6}{8} + \dfrac{1}{1}=\dfrac{6}{8} + \dfrac{1 \times 8}{1 \times 8}=\dfrac{6}{8} + \dfrac{8}{8}=\dfrac{6 + 8}{8}=\dfrac{14}{8} = \dfrac{7}{4}$
\item $\dfrac{9}{10} + \dfrac{1}{10}=\dfrac{9 + 1}{10}=\dfrac{10}{10} = 1$
\item $\dfrac{1}{8} + \dfrac{3}{7}=\dfrac{1 \times 7}{8 \times 7} + \dfrac{3 \times 8}{7 \times 8}=\dfrac{7}{56} + \dfrac{24}{56}=\dfrac{7 + 24}{56}=\dfrac{31}{56} = \dfrac{31}{56}$
\item $\dfrac{6}{2} \times 3=\dfrac{6 \times 3}{2}=\dfrac{18}{2} = 9$
\item $\dfrac{2}{10} \times \dfrac{9}{10}=\dfrac{2 \times 9}{10 \times 10}=\dfrac{18}{100} = \dfrac{9}{50}$
\item $\dfrac{- 4}{4} \times \dfrac{- 9}{36}=\dfrac{- 4(- 9)}{4 \times 36}=\dfrac{36}{144} = \dfrac{1}{4}$
\item $\dfrac{\dfrac{6}{7}}{\dfrac{10}{6}}=\dfrac{6}{7} \times \dfrac{6}{10}=\dfrac{6 \times 6}{7 \times 10}=\dfrac{36}{70} = \dfrac{18}{35}$
\item $\dfrac{7}{2} \times 7=\dfrac{7 \times 7}{2}=\dfrac{49}{2} = \dfrac{49}{2}$
\item $\dfrac{1}{7} \times \dfrac{9}{7}=\dfrac{1 \times 9}{7 \times 7}=\dfrac{9}{49} = \dfrac{9}{49}$
\item $\dfrac{- 2}{7} \times \dfrac{- 8}{42}=\dfrac{- 2(- 8)}{7 \times 42}=\dfrac{16}{294} = \dfrac{8}{147}$
\item $\dfrac{\dfrac{8}{4}}{\dfrac{5}{4}}=\dfrac{8}{4} \times \dfrac{4}{5}=\dfrac{8 \times 4}{4 \times 5}=\dfrac{32}{20} = \dfrac{8}{5}$
\end{enumerate}
\end{solution}
@ -52,10 +52,10 @@
Le radar a pris des photos pendant l'été:
\begin{itemize}
\item en juin, il y a eu 48 photos prises dont 27 ratées.
\item en juillet, il y a eu 39 photos réussies et 45 ratées.
\item en août, il y a eu 59 photos dont une proportion de 0.27 de photos ratées.
\item en septembre, il y a eu 5 photos ratées, ce qui correspondait à 10.87\% des photos prises.
\item en juin, il y a eu 47 photos prises dont 23 ratées.
\item en juillet, il y a eu 46 photos réussies et 45 ratées.
\item en août, il y a eu 65 photos dont une proportion de 0.31 de photos ratées.
\item en septembre, il y a eu 14 photos ratées, ce qui correspondait à 23.73\% des photos prises.
\end{itemize}
\begin{enumerate}
@ -87,37 +87,35 @@
\hline
& Juin & Juillet & Août & Septembre & Total\\
\hline
Réussies & 21 & 39 & 43 & 41 & 144\\
Réussies & 24 & 46 & 45 & 45 & 160\\
\hline
Ratées & 27 & 45 & 16 & 5 & 93\\
Ratées & 23 & 45 & 20 & 14 & 102\\
\hline
Total & 48 & 84 & 59 & 46 & 237\\
Total & 47 & 91 & 65 & 59 & 262\\
\hline
\end{tabular}
\end{center}
\item Proportion de photos réussies
\[
\frac{144}{237} = 0.61 = 60\%
\frac{160}{262} = 0.61 = 61\%
\]
\item
\begin{itemize}
\item De juin à juillet
\[
\frac{45 - 27}{27} = \frac{18}{27} = 0.67 = 66\%
\frac{45 - 23}{23} = \frac{22}{23} = 0.96 = 95\%
\]
\item De juillet à août
\[
\frac{16 - 45}{45} = \frac{-29}{45} = -0.64 = -64\%
\frac{20 - 45}{45} = \frac{-25}{45} = -0.56 = -55\%
\]
\item De août à septembre
\[
\frac{5 - 16}{16} = \frac{-11}{16} = -0.69 = -68\%
\frac{14 - 20}{20} = \frac{-6}{20} = -0.3 = -30\%
\]
\end{itemize}
\end{enumerate}
\end{solution}
\printsolutionstype{exercise}
\end{document}

View File

@ -17,17 +17,17 @@
Détailler les calculs suivants et donner le résultat sous la forme d'une fraction irréductible.
\begin{multicols}{3}
\begin{enumerate}[label={\Alph*=}]
\item $\dfrac{10}{8} + \dfrac{2}{8}$
\item $\dfrac{5}{10} + 3$
\item $\dfrac{4}{10} + \dfrac{5}{10}$
\item $\dfrac{7}{3} + 7$
\item $\dfrac{1}{9} + \dfrac{1}{90}$
\item $\dfrac{7}{9} + \dfrac{9}{4}$
\item $\dfrac{5}{2} + \dfrac{10}{2}$
\item $\dfrac{3}{6} + \dfrac{5}{6}$
\item $\dfrac{7}{6} \times 9$
\item $\dfrac{1}{9} \times \dfrac{4}{9}$
\item $\dfrac{7}{10} \times 7$
\item $\dfrac{5}{3} \times \dfrac{1}{3}$
\item $\dfrac{7}{3} \times \dfrac{- 2}{21}$
\item $\dfrac{\dfrac{4}{7}}{\dfrac{5}{9}}$
\item $\dfrac{4}{8} \times \dfrac{- 6}{64}$
\item $\dfrac{\dfrac{4}{6}}{\dfrac{7}{6}}$
\end{enumerate}
\end{multicols}
\end{exercise}
@ -35,15 +35,15 @@
\begin{solution}
\begin{enumerate}[label={\Alph*=}]
\item $\dfrac{10}{8} + \dfrac{2}{8}=\dfrac{10 + 2}{8}=\dfrac{12}{8} = \dfrac{3}{2}$
\item $\dfrac{5}{10} + 3=\dfrac{5}{10} + \dfrac{3}{1}=\dfrac{5}{10} + \dfrac{3 \times 10}{1 \times 10}=\dfrac{5}{10} + \dfrac{30}{10}=\dfrac{5 + 30}{10}=\dfrac{35}{10} = \dfrac{7}{2}$
\item $\dfrac{1}{9} + \dfrac{1}{90}=\dfrac{1 \times 10}{9 \times 10} + \dfrac{1}{90}=\dfrac{10}{90} + \dfrac{1}{90}=\dfrac{10 + 1}{90}=\dfrac{11}{90} = \dfrac{11}{90}$
\item $\dfrac{7}{9} + \dfrac{9}{4}=\dfrac{7 \times 4}{9 \times 4} + \dfrac{9 \times 9}{4 \times 9}=\dfrac{28}{36} + \dfrac{81}{36}=\dfrac{28 + 81}{36}=\dfrac{109}{36} = \dfrac{109}{36}$
\item $\dfrac{4}{10} + \dfrac{5}{10}=\dfrac{4 + 5}{10}=\dfrac{9}{10} = \dfrac{9}{10}$
\item $\dfrac{7}{3} + 7=\dfrac{7}{3} + \dfrac{7}{1}=\dfrac{7}{3} + \dfrac{7 \times 3}{1 \times 3}=\dfrac{7}{3} + \dfrac{21}{3}=\dfrac{7 + 21}{3}=\dfrac{28}{3} = \dfrac{28}{3}$
\item $\dfrac{5}{2} + \dfrac{10}{2}=\dfrac{5 + 10}{2}=\dfrac{15}{2} = \dfrac{15}{2}$
\item $\dfrac{3}{6} + \dfrac{5}{6}=\dfrac{3 + 5}{6}=\dfrac{8}{6} = \dfrac{4}{3}$
\item $\dfrac{7}{6} \times 9=\dfrac{7 \times 9}{6}=\dfrac{63}{6} = \dfrac{21}{2}$
\item $\dfrac{1}{9} \times \dfrac{4}{9}=\dfrac{1 \times 4}{9 \times 9}=\dfrac{4}{81} = \dfrac{4}{81}$
\item $\dfrac{7}{3} \times \dfrac{- 2}{21}=\dfrac{7(- 2)}{3 \times 21}=\dfrac{- 14}{63} = \dfrac{- 2}{9}$
\item $\dfrac{\dfrac{4}{7}}{\dfrac{5}{9}}=\dfrac{4}{7} \times \dfrac{9}{5}=\dfrac{4 \times 9}{7 \times 5}=\dfrac{36}{35} = \dfrac{36}{35}$
\item $\dfrac{7}{10} \times 7=\dfrac{7 \times 7}{10}=\dfrac{49}{10} = \dfrac{49}{10}$
\item $\dfrac{5}{3} \times \dfrac{1}{3}=\dfrac{5 \times 1}{3 \times 3}=\dfrac{5}{9} = \dfrac{5}{9}$
\item $\dfrac{4}{8} \times \dfrac{- 6}{64}=\dfrac{4(- 6)}{8 \times 64}=\dfrac{- 24}{512} = \dfrac{- 3}{64}$
\item $\dfrac{\dfrac{4}{6}}{\dfrac{7}{6}}=\dfrac{4}{6} \times \dfrac{6}{7}=\dfrac{4 \times 6}{6 \times 7}=\dfrac{24}{42} = \dfrac{4}{7}$
\end{enumerate}
\end{solution}
@ -52,10 +52,10 @@
Le radar a pris des photos pendant l'été:
\begin{itemize}
\item en juin, il y a eu 51 photos prises dont 24 ratées.
\item en juillet, il y a eu 40 photos réussies et 49 ratées.
\item en août, il y a eu 61 photos dont une proportion de 0.28 de photos ratées.
\item en septembre, il y a eu 10 photos ratées, ce qui correspondait à 18.18\% des photos prises.
\item en juin, il y a eu 46 photos prises dont 20 ratées.
\item en juillet, il y a eu 46 photos réussies et 41 ratées.
\item en août, il y a eu 62 photos dont une proportion de 0.24 de photos ratées.
\item en septembre, il y a eu 6 photos ratées, ce qui correspondait à 12.5\% des photos prises.
\end{itemize}
\begin{enumerate}
@ -87,37 +87,35 @@
\hline
& Juin & Juillet & Août & Septembre & Total\\
\hline
Réussies & 27 & 40 & 44 & 45 & 156\\
Réussies & 26 & 46 & 47 & 42 & 161\\
\hline
Ratées & 24 & 49 & 17 & 10 & 100\\
Ratées & 20 & 41 & 15 & 6 & 82\\
\hline
Total & 51 & 89 & 61 & 55 & 256\\
Total & 46 & 87 & 62 & 48 & 243\\
\hline
\end{tabular}
\end{center}
\item Proportion de photos réussies
\[
\frac{156}{256} = 0.61 = 60\%
\frac{161}{243} = 0.66 = 66\%
\]
\item
\begin{itemize}
\item De juin à juillet
\[
\frac{49 - 24}{24} = \frac{25}{24} = 1.04 = 104\%
\frac{41 - 20}{20} = \frac{21}{20} = 1.05 = 105\%
\]
\item De juillet à août
\[
\frac{17 - 49}{49} = \frac{-32}{49} = -0.65 = -65\%
\frac{15 - 41}{41} = \frac{-26}{41} = -0.63 = -63\%
\]
\item De août à septembre
\[
\frac{10 - 17}{17} = \frac{-7}{17} = -0.41 = -41\%
\frac{6 - 15}{15} = \frac{-9}{15} = -0.6 = -60\%
\]
\end{itemize}
\end{enumerate}
\end{solution}
\printsolutionstype{exercise}
\end{document}

View File

@ -17,17 +17,17 @@
Détailler les calculs suivants et donner le résultat sous la forme d'une fraction irréductible.
\begin{multicols}{3}
\begin{enumerate}[label={\Alph*=}]
\item $\dfrac{6}{2} + \dfrac{7}{2}$
\item $\dfrac{4}{7} + 2$
\item $\dfrac{9}{5} + \dfrac{6}{5}$
\item $\dfrac{7}{10} + 1$
\item $\dfrac{7}{2} + \dfrac{1}{16}$
\item $\dfrac{8}{7} + \dfrac{4}{7}$
\item $\dfrac{3}{6} + \dfrac{4}{24}$
\item $\dfrac{1}{4} + \dfrac{5}{10}$
\item $\dfrac{5}{4} \times 10$
\item $\dfrac{7}{8} \times \dfrac{4}{8}$
\item $\dfrac{8}{3} \times 9$
\item $\dfrac{6}{5} \times \dfrac{10}{5}$
\item $\dfrac{3}{4} \times \dfrac{- 2}{12}$
\item $\dfrac{\dfrac{4}{10}}{\dfrac{10}{4}}$
\item $\dfrac{- 3}{6} \times \dfrac{- 6}{24}$
\item $\dfrac{\dfrac{1}{9}}{\dfrac{5}{7}}$
\end{enumerate}
\end{multicols}
\end{exercise}
@ -35,15 +35,15 @@
\begin{solution}
\begin{enumerate}[label={\Alph*=}]
\item $\dfrac{6}{2} + \dfrac{7}{2}=\dfrac{6 + 7}{2}=\dfrac{13}{2} = \dfrac{13}{2}$
\item $\dfrac{4}{7} + 2=\dfrac{4}{7} + \dfrac{2}{1}=\dfrac{4}{7} + \dfrac{2 \times 7}{1 \times 7}=\dfrac{4}{7} + \dfrac{14}{7}=\dfrac{4 + 14}{7}=\dfrac{18}{7} = \dfrac{18}{7}$
\item $\dfrac{7}{2} + \dfrac{1}{16}=\dfrac{7 \times 8}{2 \times 8} + \dfrac{1}{16}=\dfrac{56}{16} + \dfrac{1}{16}=\dfrac{56 + 1}{16}=\dfrac{57}{16} = \dfrac{57}{16}$
\item $\dfrac{8}{7} + \dfrac{4}{7}=\dfrac{8 + 4}{7}=\dfrac{12}{7} = \dfrac{12}{7}$
\item $\dfrac{9}{5} + \dfrac{6}{5}=\dfrac{9 + 6}{5}=\dfrac{15}{5} = 3$
\item $\dfrac{7}{10} + 1=\dfrac{7}{10} + \dfrac{1}{1}=\dfrac{7}{10} + \dfrac{1 \times 10}{1 \times 10}=\dfrac{7}{10} + \dfrac{10}{10}=\dfrac{7 + 10}{10}=\dfrac{17}{10} = \dfrac{17}{10}$
\item $\dfrac{3}{6} + \dfrac{4}{24}=\dfrac{3 \times 4}{6 \times 4} + \dfrac{4}{24}=\dfrac{12}{24} + \dfrac{4}{24}=\dfrac{12 + 4}{24}=\dfrac{16}{24} = \dfrac{2}{3}$
\item $\dfrac{1}{4} + \dfrac{5}{10}=\dfrac{1 \times 5}{4 \times 5} + \dfrac{5 \times 2}{10 \times 2}=\dfrac{5}{20} + \dfrac{10}{20}=\dfrac{5 + 10}{20}=\dfrac{15}{20} = \dfrac{3}{4}$
\item $\dfrac{5}{4} \times 10=\dfrac{5 \times 10}{4}=\dfrac{50}{4} = \dfrac{25}{2}$
\item $\dfrac{7}{8} \times \dfrac{4}{8}=\dfrac{7 \times 4}{8 \times 8}=\dfrac{28}{64} = \dfrac{7}{16}$
\item $\dfrac{3}{4} \times \dfrac{- 2}{12}=\dfrac{3(- 2)}{4 \times 12}=\dfrac{- 6}{48} = \dfrac{- 1}{8}$
\item $\dfrac{\dfrac{4}{10}}{\dfrac{10}{4}}=\dfrac{4}{10} \times \dfrac{4}{10}=\dfrac{4 \times 4}{10 \times 10}=\dfrac{16}{100} = \dfrac{4}{25}$
\item $\dfrac{8}{3} \times 9=\dfrac{8 \times 9}{3}=\dfrac{72}{3} = 24$
\item $\dfrac{6}{5} \times \dfrac{10}{5}=\dfrac{6 \times 10}{5 \times 5}=\dfrac{60}{25} = \dfrac{12}{5}$
\item $\dfrac{- 3}{6} \times \dfrac{- 6}{24}=\dfrac{- 3(- 6)}{6 \times 24}=\dfrac{18}{144} = \dfrac{1}{8}$
\item $\dfrac{\dfrac{1}{9}}{\dfrac{5}{7}}=\dfrac{1}{9} \times \dfrac{7}{5}=\dfrac{1 \times 7}{9 \times 5}=\dfrac{7}{45} = \dfrac{7}{45}$
\end{enumerate}
\end{solution}
@ -52,10 +52,10 @@
Le radar a pris des photos pendant l'été:
\begin{itemize}
\item en juin, il y a eu 47 photos prises dont 26 ratées.
\item en juillet, il y a eu 31 photos réussies et 37 ratées.
\item en juin, il y a eu 52 photos prises dont 29 ratées.
\item en juillet, il y a eu 40 photos réussies et 37 ratées.
\item en août, il y a eu 60 photos dont une proportion de 0.32 de photos ratées.
\item en septembre, il y a eu 7 photos ratées, ce qui correspondait à 14.89\% des photos prises.
\item en septembre, il y a eu 9 photos ratées, ce qui correspondait à 15.79\% des photos prises.
\end{itemize}
\begin{enumerate}
@ -87,23 +87,23 @@
\hline
& Juin & Juillet & Août & Septembre & Total\\
\hline
Réussies & 21 & 31 & 41 & 40 & 133\\
Réussies & 23 & 40 & 41 & 48 & 152\\
\hline
Ratées & 26 & 37 & 19 & 7 & 89\\
Ratées & 29 & 37 & 19 & 9 & 94\\
\hline
Total & 47 & 68 & 60 & 47 & 222\\
Total & 52 & 77 & 60 & 57 & 246\\
\hline
\end{tabular}
\end{center}
\item Proportion de photos réussies
\[
\frac{133}{222} = 0.6 = 59\%
\frac{152}{246} = 0.62 = 61\%
\]
\item
\begin{itemize}
\item De juin à juillet
\[
\frac{37 - 26}{26} = \frac{11}{26} = 0.42 = 42\%
\frac{37 - 29}{29} = \frac{8}{29} = 0.28 = 27\%
\]
\item De juillet à août
\[
@ -111,13 +111,11 @@
\]
\item De août à septembre
\[
\frac{7 - 19}{19} = \frac{-12}{19} = -0.63 = -63\%
\frac{9 - 19}{19} = \frac{-10}{19} = -0.53 = -52\%
\]
\end{itemize}
\end{enumerate}
\end{solution}
\printsolutionstype{exercise}
\end{document}

View File

@ -17,17 +17,17 @@
Détailler les calculs suivants et donner le résultat sous la forme d'une fraction irréductible.
\begin{multicols}{3}
\begin{enumerate}[label={\Alph*=}]
\item $\dfrac{1}{4} + \dfrac{3}{4}$
\item $\dfrac{6}{7} + 3$
\item $\dfrac{7}{10} + \dfrac{4}{10}$
\item $\dfrac{3}{4} + 8$
\item $\dfrac{8}{10} + \dfrac{8}{70}$
\item $\dfrac{7}{8} + \dfrac{7}{9}$
\item $\dfrac{1}{2} + \dfrac{10}{6}$
\item $\dfrac{2}{3} + \dfrac{8}{2}$
\item $\dfrac{9}{5} \times 2$
\item $\dfrac{6}{2} \times \dfrac{10}{2}$
\item $\dfrac{9}{10} \times 4$
\item $\dfrac{9}{6} \times \dfrac{7}{6}$
\item $\dfrac{- 8}{4} \times \dfrac{7}{16}$
\item $\dfrac{\dfrac{9}{7}}{\dfrac{4}{3}}$
\item $\dfrac{- 9}{4} \times \dfrac{8}{12}$
\item $\dfrac{\dfrac{5}{2}}{\dfrac{7}{9}}$
\end{enumerate}
\end{multicols}
\end{exercise}
@ -35,15 +35,15 @@
\begin{solution}
\begin{enumerate}[label={\Alph*=}]
\item $\dfrac{1}{4} + \dfrac{3}{4}=\dfrac{1 + 3}{4}=\dfrac{4}{4} = 1$
\item $\dfrac{6}{7} + 3=\dfrac{6}{7} + \dfrac{3}{1}=\dfrac{6}{7} + \dfrac{3 \times 7}{1 \times 7}=\dfrac{6}{7} + \dfrac{21}{7}=\dfrac{6 + 21}{7}=\dfrac{27}{7} = \dfrac{27}{7}$
\item $\dfrac{8}{10} + \dfrac{8}{70}=\dfrac{8 \times 7}{10 \times 7} + \dfrac{8}{70}=\dfrac{56}{70} + \dfrac{8}{70}=\dfrac{56 + 8}{70}=\dfrac{64}{70} = \dfrac{32}{35}$
\item $\dfrac{7}{8} + \dfrac{7}{9}=\dfrac{7 \times 9}{8 \times 9} + \dfrac{7 \times 8}{9 \times 8}=\dfrac{63}{72} + \dfrac{56}{72}=\dfrac{63 + 56}{72}=\dfrac{119}{72} = \dfrac{119}{72}$
\item $\dfrac{7}{10} + \dfrac{4}{10}=\dfrac{7 + 4}{10}=\dfrac{11}{10} = \dfrac{11}{10}$
\item $\dfrac{3}{4} + 8=\dfrac{3}{4} + \dfrac{8}{1}=\dfrac{3}{4} + \dfrac{8 \times 4}{1 \times 4}=\dfrac{3}{4} + \dfrac{32}{4}=\dfrac{3 + 32}{4}=\dfrac{35}{4} = \dfrac{35}{4}$
\item $\dfrac{1}{2} + \dfrac{10}{6}=\dfrac{1 \times 3}{2 \times 3} + \dfrac{10}{6}=\dfrac{3}{6} + \dfrac{10}{6}=\dfrac{3 + 10}{6}=\dfrac{13}{6} = \dfrac{13}{6}$
\item $\dfrac{2}{3} + \dfrac{8}{2}=\dfrac{2 \times 2}{3 \times 2} + \dfrac{8 \times 3}{2 \times 3}=\dfrac{4}{6} + \dfrac{24}{6}=\dfrac{4 + 24}{6}=\dfrac{28}{6} = \dfrac{14}{3}$
\item $\dfrac{9}{5} \times 2=\dfrac{9 \times 2}{5}=\dfrac{18}{5} = \dfrac{18}{5}$
\item $\dfrac{6}{2} \times \dfrac{10}{2}=\dfrac{6 \times 10}{2 \times 2}=\dfrac{60}{4} = 15$
\item $\dfrac{- 8}{4} \times \dfrac{7}{16}=\dfrac{- 8 \times 7}{4 \times 16}=\dfrac{- 56}{64} = \dfrac{- 7}{8}$
\item $\dfrac{\dfrac{9}{7}}{\dfrac{4}{3}}=\dfrac{9}{7} \times \dfrac{3}{4}=\dfrac{9 \times 3}{7 \times 4}=\dfrac{27}{28} = \dfrac{27}{28}$
\item $\dfrac{9}{10} \times 4=\dfrac{9 \times 4}{10}=\dfrac{36}{10} = \dfrac{18}{5}$
\item $\dfrac{9}{6} \times \dfrac{7}{6}=\dfrac{9 \times 7}{6 \times 6}=\dfrac{63}{36} = \dfrac{7}{4}$
\item $\dfrac{- 9}{4} \times \dfrac{8}{12}=\dfrac{- 9 \times 8}{4 \times 12}=\dfrac{- 72}{48} = \dfrac{- 3}{2}$
\item $\dfrac{\dfrac{5}{2}}{\dfrac{7}{9}}=\dfrac{5}{2} \times \dfrac{9}{7}=\dfrac{5 \times 9}{2 \times 7}=\dfrac{45}{14} = \dfrac{45}{14}$
\end{enumerate}
\end{solution}
@ -52,10 +52,10 @@
Le radar a pris des photos pendant l'été:
\begin{itemize}
\item en juin, il y a eu 48 photos prises dont 27 ratées.
\item en juillet, il y a eu 31 photos réussies et 32 ratées.
\item en août, il y a eu 61 photos dont une proportion de 0.3 de photos ratées.
\item en septembre, il y a eu 9 photos ratées, ce qui correspondait à 18.37\% des photos prises.
\item en juin, il y a eu 47 photos prises dont 27 ratées.
\item en juillet, il y a eu 47 photos réussies et 31 ratées.
\item en août, il y a eu 62 photos dont une proportion de 0.26 de photos ratées.
\item en septembre, il y a eu 11 photos ratées, ce qui correspondait à 20.0\% des photos prises.
\end{itemize}
\begin{enumerate}
@ -87,37 +87,35 @@
\hline
& Juin & Juillet & Août & Septembre & Total\\
\hline
Réussies & 21 & 31 & 43 & 40 & 135\\
Réussies & 20 & 47 & 46 & 44 & 157\\
\hline
Ratées & 27 & 32 & 18 & 9 & 86\\
Ratées & 27 & 31 & 16 & 11 & 85\\
\hline
Total & 48 & 63 & 61 & 49 & 221\\
Total & 47 & 78 & 62 & 55 & 242\\
\hline
\end{tabular}
\end{center}
\item Proportion de photos réussies
\[
\frac{135}{221} = 0.61 = 61\%
\frac{157}{242} = 0.65 = 64\%
\]
\item
\begin{itemize}
\item De juin à juillet
\[
\frac{32 - 27}{27} = \frac{5}{27} = 0.19 = 18\%
\frac{31 - 27}{27} = \frac{4}{27} = 0.15 = 14\%
\]
\item De juillet à août
\[
\frac{18 - 32}{32} = \frac{-14}{32} = -0.44 = -43\%
\frac{16 - 31}{31} = \frac{-15}{31} = -0.48 = -48\%
\]
\item De août à septembre
\[
\frac{9 - 18}{18} = \frac{-9}{18} = -0.5 = -50\%
\frac{11 - 16}{16} = \frac{-5}{16} = -0.31 = -31\%
\]
\end{itemize}
\end{enumerate}
\end{solution}
\printsolutionstype{exercise}
\end{document}

View File

@ -17,17 +17,17 @@
Détailler les calculs suivants et donner le résultat sous la forme d'une fraction irréductible.
\begin{multicols}{3}
\begin{enumerate}[label={\Alph*=}]
\item $\dfrac{2}{6} + \dfrac{8}{6}$
\item $\dfrac{6}{3} + 5$
\item $\dfrac{2}{4} + \dfrac{3}{4}$
\item $\dfrac{5}{4} + 10$
\item $\dfrac{1}{9} + \dfrac{6}{36}$
\item $\dfrac{6}{10} + \dfrac{8}{6}$
\item $\dfrac{2}{7} + \dfrac{10}{42}$
\item $\dfrac{7}{8} + \dfrac{6}{10}$
\item $\dfrac{1}{4} \times 4$
\item $\dfrac{3}{10} \times \dfrac{3}{10}$
\item $\dfrac{5}{2} \times 8$
\item $\dfrac{2}{10} \times \dfrac{5}{10}$
\item $\dfrac{- 8}{6} \times \dfrac{2}{24}$
\item $\dfrac{\dfrac{4}{10}}{\dfrac{2}{10}}$
\item $\dfrac{- 1}{7} \times \dfrac{- 2}{70}$
\item $\dfrac{\dfrac{1}{7}}{\dfrac{7}{3}}$
\end{enumerate}
\end{multicols}
\end{exercise}
@ -35,15 +35,15 @@
\begin{solution}
\begin{enumerate}[label={\Alph*=}]
\item $\dfrac{2}{6} + \dfrac{8}{6}=\dfrac{2 + 8}{6}=\dfrac{10}{6} = \dfrac{5}{3}$
\item $\dfrac{6}{3} + 5=\dfrac{6}{3} + \dfrac{5}{1}=\dfrac{6}{3} + \dfrac{5 \times 3}{1 \times 3}=\dfrac{6}{3} + \dfrac{15}{3}=\dfrac{6 + 15}{3}=\dfrac{21}{3} = 7$
\item $\dfrac{1}{9} + \dfrac{6}{36}=\dfrac{1 \times 4}{9 \times 4} + \dfrac{6}{36}=\dfrac{4}{36} + \dfrac{6}{36}=\dfrac{4 + 6}{36}=\dfrac{10}{36} = \dfrac{5}{18}$
\item $\dfrac{6}{10} + \dfrac{8}{6}=\dfrac{6 \times 3}{10 \times 3} + \dfrac{8 \times 5}{6 \times 5}=\dfrac{18}{30} + \dfrac{40}{30}=\dfrac{18 + 40}{30}=\dfrac{58}{30} = \dfrac{29}{15}$
\item $\dfrac{2}{4} + \dfrac{3}{4}=\dfrac{2 + 3}{4}=\dfrac{5}{4} = \dfrac{5}{4}$
\item $\dfrac{5}{4} + 10=\dfrac{5}{4} + \dfrac{10}{1}=\dfrac{5}{4} + \dfrac{10 \times 4}{1 \times 4}=\dfrac{5}{4} + \dfrac{40}{4}=\dfrac{5 + 40}{4}=\dfrac{45}{4} = \dfrac{45}{4}$
\item $\dfrac{2}{7} + \dfrac{10}{42}=\dfrac{2 \times 6}{7 \times 6} + \dfrac{10}{42}=\dfrac{12}{42} + \dfrac{10}{42}=\dfrac{12 + 10}{42}=\dfrac{22}{42} = \dfrac{11}{21}$
\item $\dfrac{7}{8} + \dfrac{6}{10}=\dfrac{7 \times 5}{8 \times 5} + \dfrac{6 \times 4}{10 \times 4}=\dfrac{35}{40} + \dfrac{24}{40}=\dfrac{35 + 24}{40}=\dfrac{59}{40} = \dfrac{59}{40}$
\item $\dfrac{1}{4} \times 4=\dfrac{1 \times 4}{4}=\dfrac{4}{4} = 1$
\item $\dfrac{3}{10} \times \dfrac{3}{10}=\dfrac{3 \times 3}{10 \times 10}=\dfrac{9}{100} = \dfrac{9}{100}$
\item $\dfrac{- 8}{6} \times \dfrac{2}{24}=\dfrac{- 8 \times 2}{6 \times 24}=\dfrac{- 16}{144} = \dfrac{- 1}{9}$
\item $\dfrac{\dfrac{4}{10}}{\dfrac{2}{10}}=\dfrac{4}{10} \times \dfrac{10}{2}=\dfrac{4 \times 10}{10 \times 2}=\dfrac{40}{20} = 2$
\item $\dfrac{5}{2} \times 8=\dfrac{5 \times 8}{2}=\dfrac{40}{2} = 20$
\item $\dfrac{2}{10} \times \dfrac{5}{10}=\dfrac{2 \times 5}{10 \times 10}=\dfrac{10}{100} = \dfrac{1}{10}$
\item $\dfrac{- 1}{7} \times \dfrac{- 2}{70}=\dfrac{- 1(- 2)}{7 \times 70}=\dfrac{2}{490} = \dfrac{1}{245}$
\item $\dfrac{\dfrac{1}{7}}{\dfrac{7}{3}}=\dfrac{1}{7} \times \dfrac{3}{7}=\dfrac{1 \times 3}{7 \times 7}=\dfrac{3}{49} = \dfrac{3}{49}$
\end{enumerate}
\end{solution}
@ -52,10 +52,10 @@
Le radar a pris des photos pendant l'été:
\begin{itemize}
\item en juin, il y a eu 46 photos prises dont 22 ratées.
\item en juillet, il y a eu 49 photos réussies et 39 ratées.
\item en août, il y a eu 54 photos dont une proportion de 0.24 de photos ratées.
\item en septembre, il y a eu 8 photos ratées, ce qui correspondait à 14.81\% des photos prises.
\item en juin, il y a eu 46 photos prises dont 20 ratées.
\item en juillet, il y a eu 33 photos réussies et 41 ratées.
\item en août, il y a eu 53 photos dont une proportion de 0.21 de photos ratées.
\item en septembre, il y a eu 8 photos ratées, ce qui correspondait à 16.33\% des photos prises.
\end{itemize}
\begin{enumerate}
@ -87,37 +87,35 @@
\hline
& Juin & Juillet & Août & Septembre & Total\\
\hline
Réussies & 24 & 49 & 41 & 46 & 160\\
Réussies & 26 & 33 & 42 & 41 & 142\\
\hline
Ratées & 22 & 39 & 13 & 8 & 82\\
Ratées & 20 & 41 & 11 & 8 & 80\\
\hline
Total & 46 & 88 & 54 & 54 & 242\\
Total & 46 & 74 & 53 & 49 & 222\\
\hline
\end{tabular}
\end{center}
\item Proportion de photos réussies
\[
\frac{160}{242} = 0.66 = 66\%
\frac{142}{222} = 0.64 = 63\%
\]
\item
\begin{itemize}
\item De juin à juillet
\[
\frac{39 - 22}{22} = \frac{17}{22} = 0.77 = 77\%
\frac{41 - 20}{20} = \frac{21}{20} = 1.05 = 105\%
\]
\item De juillet à août
\[
\frac{13 - 39}{39} = \frac{-26}{39} = -0.67 = -66\%
\frac{11 - 41}{41} = \frac{-30}{41} = -0.73 = -73\%
\]
\item De août à septembre
\[
\frac{8 - 13}{13} = \frac{-5}{13} = -0.38 = -38\%
\frac{8 - 11}{11} = \frac{-3}{11} = -0.27 = -27\%
\]
\end{itemize}
\end{enumerate}
\end{solution}
\printsolutionstype{exercise}
\end{document}

View File

@ -17,17 +17,17 @@
Détailler les calculs suivants et donner le résultat sous la forme d'une fraction irréductible.
\begin{multicols}{3}
\begin{enumerate}[label={\Alph*=}]
\item $\dfrac{6}{3} + \dfrac{1}{3}$
\item $\dfrac{1}{8} + 10$
\item $\dfrac{8}{10} + \dfrac{9}{10}$
\item $\dfrac{6}{9} + 4$
\item $\dfrac{1}{9} + \dfrac{3}{18}$
\item $\dfrac{9}{7} + \dfrac{7}{3}$
\item $\dfrac{9}{2} + \dfrac{7}{10}$
\item $\dfrac{10}{5} + \dfrac{5}{6}$
\item $\dfrac{7}{4} \times 2$
\item $\dfrac{7}{3} \times \dfrac{10}{3}$
\item $\dfrac{6}{5} \times 3$
\item $\dfrac{8}{5} \times \dfrac{10}{5}$
\item $\dfrac{- 10}{2} \times \dfrac{- 9}{4}$
\item $\dfrac{\dfrac{3}{6}}{\dfrac{6}{8}}$
\item $\dfrac{- 4}{7} \times \dfrac{6}{14}$
\item $\dfrac{\dfrac{7}{10}}{\dfrac{6}{3}}$
\end{enumerate}
\end{multicols}
\end{exercise}
@ -35,15 +35,15 @@
\begin{solution}
\begin{enumerate}[label={\Alph*=}]
\item $\dfrac{6}{3} + \dfrac{1}{3}=\dfrac{6 + 1}{3}=\dfrac{7}{3} = \dfrac{7}{3}$
\item $\dfrac{1}{8} + 10=\dfrac{1}{8} + \dfrac{10}{1}=\dfrac{1}{8} + \dfrac{10 \times 8}{1 \times 8}=\dfrac{1}{8} + \dfrac{80}{8}=\dfrac{1 + 80}{8}=\dfrac{81}{8} = \dfrac{81}{8}$
\item $\dfrac{1}{9} + \dfrac{3}{18}=\dfrac{1 \times 2}{9 \times 2} + \dfrac{3}{18}=\dfrac{2}{18} + \dfrac{3}{18}=\dfrac{2 + 3}{18}=\dfrac{5}{18} = \dfrac{5}{18}$
\item $\dfrac{9}{7} + \dfrac{7}{3}=\dfrac{9 \times 3}{7 \times 3} + \dfrac{7 \times 7}{3 \times 7}=\dfrac{27}{21} + \dfrac{49}{21}=\dfrac{27 + 49}{21}=\dfrac{76}{21} = \dfrac{76}{21}$
\item $\dfrac{8}{10} + \dfrac{9}{10}=\dfrac{8 + 9}{10}=\dfrac{17}{10} = \dfrac{17}{10}$
\item $\dfrac{6}{9} + 4=\dfrac{6}{9} + \dfrac{4}{1}=\dfrac{6}{9} + \dfrac{4 \times 9}{1 \times 9}=\dfrac{6}{9} + \dfrac{36}{9}=\dfrac{6 + 36}{9}=\dfrac{42}{9} = \dfrac{14}{3}$
\item $\dfrac{9}{2} + \dfrac{7}{10}=\dfrac{9 \times 5}{2 \times 5} + \dfrac{7}{10}=\dfrac{45}{10} + \dfrac{7}{10}=\dfrac{45 + 7}{10}=\dfrac{52}{10} = \dfrac{26}{5}$
\item $\dfrac{10}{5} + \dfrac{5}{6}=\dfrac{10 \times 6}{5 \times 6} + \dfrac{5 \times 5}{6 \times 5}=\dfrac{60}{30} + \dfrac{25}{30}=\dfrac{60 + 25}{30}=\dfrac{85}{30} = \dfrac{17}{6}$
\item $\dfrac{7}{4} \times 2=\dfrac{7 \times 2}{4}=\dfrac{14}{4} = \dfrac{7}{2}$
\item $\dfrac{7}{3} \times \dfrac{10}{3}=\dfrac{7 \times 10}{3 \times 3}=\dfrac{70}{9} = \dfrac{70}{9}$
\item $\dfrac{- 10}{2} \times \dfrac{- 9}{4}=\dfrac{- 10(- 9)}{2 \times 4}=\dfrac{90}{8} = \dfrac{45}{4}$
\item $\dfrac{\dfrac{3}{6}}{\dfrac{6}{8}}=\dfrac{3}{6} \times \dfrac{8}{6}=\dfrac{3 \times 8}{6 \times 6}=\dfrac{24}{36} = \dfrac{2}{3}$
\item $\dfrac{6}{5} \times 3=\dfrac{6 \times 3}{5}=\dfrac{18}{5} = \dfrac{18}{5}$
\item $\dfrac{8}{5} \times \dfrac{10}{5}=\dfrac{8 \times 10}{5 \times 5}=\dfrac{80}{25} = \dfrac{16}{5}$
\item $\dfrac{- 4}{7} \times \dfrac{6}{14}=\dfrac{- 4 \times 6}{7 \times 14}=\dfrac{- 24}{98} = \dfrac{- 12}{49}$
\item $\dfrac{\dfrac{7}{10}}{\dfrac{6}{3}}=\dfrac{7}{10} \times \dfrac{3}{6}=\dfrac{7 \times 3}{10 \times 6}=\dfrac{21}{60} = \dfrac{7}{20}$
\end{enumerate}
\end{solution}
@ -52,10 +52,10 @@
Le radar a pris des photos pendant l'été:
\begin{itemize}
\item en juin, il y a eu 52 photos prises dont 26 ratées.
\item en juillet, il y a eu 42 photos réussies et 41 ratées.
\item en août, il y a eu 65 photos dont une proportion de 0.25 de photos ratées.
\item en septembre, il y a eu 12 photos ratées, ce qui correspondait à 19.35\% des photos prises.
\item en juin, il y a eu 44 photos prises dont 22 ratées.
\item en juillet, il y a eu 41 photos réussies et 40 ratées.
\item en août, il y a eu 57 photos dont une proportion de 0.26 de photos ratées.
\item en septembre, il y a eu 14 photos ratées, ce qui correspondait à 24.14\% des photos prises.
\end{itemize}
\begin{enumerate}
@ -87,37 +87,35 @@
\hline
& Juin & Juillet & Août & Septembre & Total\\
\hline
Réussies & 26 & 42 & 49 & 50 & 167\\
Réussies & 22 & 41 & 42 & 44 & 149\\
\hline
Ratées & 26 & 41 & 16 & 12 & 95\\
Ratées & 22 & 40 & 15 & 14 & 91\\
\hline
Total & 52 & 83 & 65 & 62 & 262\\
Total & 44 & 81 & 57 & 58 & 240\\
\hline
\end{tabular}
\end{center}
\item Proportion de photos réussies
\[
\frac{167}{262} = 0.64 = 63\%
\frac{149}{240} = 0.62 = 62\%
\]
\item
\begin{itemize}
\item De juin à juillet
\[
\frac{41 - 26}{26} = \frac{15}{26} = 0.58 = 57\%
\frac{40 - 22}{22} = \frac{18}{22} = 0.82 = 81\%
\]
\item De juillet à août
\[
\frac{16 - 41}{41} = \frac{-25}{41} = -0.61 = -60\%
\frac{15 - 40}{40} = \frac{-25}{40} = -0.62 = -62\%
\]
\item De août à septembre
\[
\frac{12 - 16}{16} = \frac{-4}{16} = -0.25 = -25\%
\frac{14 - 15}{15} = \frac{-1}{15} = -0.07 = -6\%
\]
\end{itemize}
\end{enumerate}
\end{solution}
\printsolutionstype{exercise}
\end{document}

View File

@ -17,17 +17,17 @@
Détailler les calculs suivants et donner le résultat sous la forme d'une fraction irréductible.
\begin{multicols}{3}
\begin{enumerate}[label={\Alph*=}]
\item $\dfrac{5}{6} + \dfrac{9}{6}$
\item $\dfrac{2}{8} + 8$
\item $\dfrac{1}{5} + \dfrac{1}{5}$
\item $\dfrac{9}{4} + 5$
\item $\dfrac{2}{10} + \dfrac{2}{80}$
\item $\dfrac{2}{7} + \dfrac{8}{5}$
\item $\dfrac{4}{8} + \dfrac{3}{64}$
\item $\dfrac{9}{10} + \dfrac{2}{6}$
\item $\dfrac{3}{4} \times 1$
\item $\dfrac{7}{3} \times \dfrac{2}{3}$
\item $\dfrac{8}{3} \times 6$
\item $\dfrac{6}{3} \times \dfrac{2}{3}$
\item $\dfrac{- 2}{5} \times \dfrac{- 7}{20}$
\item $\dfrac{\dfrac{3}{6}}{\dfrac{5}{8}}$
\item $\dfrac{10}{7} \times \dfrac{- 6}{21}$
\item $\dfrac{\dfrac{6}{2}}{\dfrac{3}{2}}$
\end{enumerate}
\end{multicols}
\end{exercise}
@ -35,15 +35,15 @@
\begin{solution}
\begin{enumerate}[label={\Alph*=}]
\item $\dfrac{5}{6} + \dfrac{9}{6}=\dfrac{5 + 9}{6}=\dfrac{14}{6} = \dfrac{7}{3}$
\item $\dfrac{2}{8} + 8=\dfrac{2}{8} + \dfrac{8}{1}=\dfrac{2}{8} + \dfrac{8 \times 8}{1 \times 8}=\dfrac{2}{8} + \dfrac{64}{8}=\dfrac{2 + 64}{8}=\dfrac{66}{8} = \dfrac{33}{4}$
\item $\dfrac{2}{10} + \dfrac{2}{80}=\dfrac{2 \times 8}{10 \times 8} + \dfrac{2}{80}=\dfrac{16}{80} + \dfrac{2}{80}=\dfrac{16 + 2}{80}=\dfrac{18}{80} = \dfrac{9}{40}$
\item $\dfrac{2}{7} + \dfrac{8}{5}=\dfrac{2 \times 5}{7 \times 5} + \dfrac{8 \times 7}{5 \times 7}=\dfrac{10}{35} + \dfrac{56}{35}=\dfrac{10 + 56}{35}=\dfrac{66}{35} = \dfrac{66}{35}$
\item $\dfrac{1}{5} + \dfrac{1}{5}=\dfrac{1 + 1}{5}=\dfrac{2}{5} = \dfrac{2}{5}$
\item $\dfrac{9}{4} + 5=\dfrac{9}{4} + \dfrac{5}{1}=\dfrac{9}{4} + \dfrac{5 \times 4}{1 \times 4}=\dfrac{9}{4} + \dfrac{20}{4}=\dfrac{9 + 20}{4}=\dfrac{29}{4} = \dfrac{29}{4}$
\item $\dfrac{4}{8} + \dfrac{3}{64}=\dfrac{4 \times 8}{8 \times 8} + \dfrac{3}{64}=\dfrac{32}{64} + \dfrac{3}{64}=\dfrac{32 + 3}{64}=\dfrac{35}{64} = \dfrac{35}{64}$
\item $\dfrac{9}{10} + \dfrac{2}{6}=\dfrac{9 \times 3}{10 \times 3} + \dfrac{2 \times 5}{6 \times 5}=\dfrac{27}{30} + \dfrac{10}{30}=\dfrac{27 + 10}{30}=\dfrac{37}{30} = \dfrac{37}{30}$
\item $\dfrac{3}{4} \times 1=\dfrac{3}{4} = \dfrac{3}{4}$
\item $\dfrac{7}{3} \times \dfrac{2}{3}=\dfrac{7 \times 2}{3 \times 3}=\dfrac{14}{9} = \dfrac{14}{9}$
\item $\dfrac{- 2}{5} \times \dfrac{- 7}{20}=\dfrac{- 2(- 7)}{5 \times 20}=\dfrac{14}{100} = \dfrac{7}{50}$
\item $\dfrac{\dfrac{3}{6}}{\dfrac{5}{8}}=\dfrac{3}{6} \times \dfrac{8}{5}=\dfrac{3 \times 8}{6 \times 5}=\dfrac{24}{30} = \dfrac{4}{5}$
\item $\dfrac{8}{3} \times 6=\dfrac{8 \times 6}{3}=\dfrac{48}{3} = 16$
\item $\dfrac{6}{3} \times \dfrac{2}{3}=\dfrac{6 \times 2}{3 \times 3}=\dfrac{12}{9} = \dfrac{4}{3}$
\item $\dfrac{10}{7} \times \dfrac{- 6}{21}=\dfrac{10(- 6)}{7 \times 21}=\dfrac{- 60}{147} = \dfrac{- 20}{49}$
\item $\dfrac{\dfrac{6}{2}}{\dfrac{3}{2}}=\dfrac{6}{2} \times \dfrac{2}{3}=\dfrac{6 \times 2}{2 \times 3}=\dfrac{12}{6} = 2$
\end{enumerate}
\end{solution}
@ -52,10 +52,10 @@
Le radar a pris des photos pendant l'été:
\begin{itemize}
\item en juin, il y a eu 43 photos prises dont 23 ratées.
\item en juillet, il y a eu 35 photos réussies et 37 ratées.
\item en août, il y a eu 64 photos dont une proportion de 0.31 de photos ratées.
\item en septembre, il y a eu 12 photos ratées, ce qui correspondait à 21.82\% des photos prises.
\item en juin, il y a eu 59 photos prises dont 29 ratées.
\item en juillet, il y a eu 37 photos réussies et 47 ratées.
\item en août, il y a eu 55 photos dont une proportion de 0.27 de photos ratées.
\item en septembre, il y a eu 10 photos ratées, ce qui correspondait à 18.52\% des photos prises.
\end{itemize}
\begin{enumerate}
@ -87,37 +87,35 @@
\hline
& Juin & Juillet & Août & Septembre & Total\\
\hline
Réussies & 20 & 35 & 44 & 43 & 142\\
Réussies & 30 & 37 & 40 & 44 & 151\\
\hline
Ratées & 23 & 37 & 20 & 12 & 92\\
Ratées & 29 & 47 & 15 & 10 & 101\\
\hline
Total & 43 & 72 & 64 & 55 & 234\\
Total & 59 & 84 & 55 & 54 & 252\\
\hline
\end{tabular}
\end{center}
\item Proportion de photos réussies
\[
\frac{142}{234} = 0.61 = 60\%
\frac{151}{252} = 0.6 = 59\%
\]
\item
\begin{itemize}
\item De juin à juillet
\[
\frac{37 - 23}{23} = \frac{14}{23} = 0.61 = 60\%
\frac{47 - 29}{29} = \frac{18}{29} = 0.62 = 62\%
\]
\item De juillet à août
\[
\frac{20 - 37}{37} = \frac{-17}{37} = -0.46 = -45\%
\frac{15 - 47}{47} = \frac{-32}{47} = -0.68 = -68\%
\]
\item De août à septembre
\[
\frac{12 - 20}{20} = \frac{-8}{20} = -0.4 = -40\%
\frac{10 - 15}{15} = \frac{-5}{15} = -0.33 = -33\%
\]
\end{itemize}
\end{enumerate}
\end{solution}
\printsolutionstype{exercise}
\end{document}

View File

@ -17,17 +17,17 @@
Détailler les calculs suivants et donner le résultat sous la forme d'une fraction irréductible.
\begin{multicols}{3}
\begin{enumerate}[label={\Alph*=}]
\item $\dfrac{3}{10} + \dfrac{5}{10}$
\item $\dfrac{1}{10} + 1$
\item $\dfrac{8}{6} + \dfrac{1}{6}$
\item $\dfrac{2}{5} + 2$
\item $\dfrac{2}{7} + \dfrac{9}{49}$
\item $\dfrac{10}{3} + \dfrac{5}{8}$
\item $\dfrac{8}{3} + \dfrac{7}{6}$
\item $\dfrac{5}{9} + \dfrac{3}{7}$
\item $\dfrac{2}{4} \times 5$
\item $\dfrac{7}{8} \times 2$
\item $\dfrac{3}{7} \times \dfrac{4}{7}$
\item $\dfrac{10}{4} \times \dfrac{- 10}{28}$
\item $\dfrac{\dfrac{6}{4}}{\dfrac{9}{3}}$
\item $\dfrac{- 3}{10} \times \dfrac{8}{20}$
\item $\dfrac{\dfrac{9}{6}}{\dfrac{2}{8}}$
\end{enumerate}
\end{multicols}
\end{exercise}
@ -35,15 +35,15 @@
\begin{solution}
\begin{enumerate}[label={\Alph*=}]
\item $\dfrac{3}{10} + \dfrac{5}{10}=\dfrac{3 + 5}{10}=\dfrac{8}{10} = \dfrac{4}{5}$
\item $\dfrac{1}{10} + 1=\dfrac{1}{10} + \dfrac{1}{1}=\dfrac{1}{10} + \dfrac{1 \times 10}{1 \times 10}=\dfrac{1}{10} + \dfrac{10}{10}=\dfrac{1 + 10}{10}=\dfrac{11}{10} = \dfrac{11}{10}$
\item $\dfrac{2}{7} + \dfrac{9}{49}=\dfrac{2 \times 7}{7 \times 7} + \dfrac{9}{49}=\dfrac{14}{49} + \dfrac{9}{49}=\dfrac{14 + 9}{49}=\dfrac{23}{49} = \dfrac{23}{49}$
\item $\dfrac{10}{3} + \dfrac{5}{8}=\dfrac{10 \times 8}{3 \times 8} + \dfrac{5 \times 3}{8 \times 3}=\dfrac{80}{24} + \dfrac{15}{24}=\dfrac{80 + 15}{24}=\dfrac{95}{24} = \dfrac{95}{24}$
\item $\dfrac{8}{6} + \dfrac{1}{6}=\dfrac{8 + 1}{6}=\dfrac{9}{6} = \dfrac{3}{2}$
\item $\dfrac{2}{5} + 2=\dfrac{2}{5} + \dfrac{2}{1}=\dfrac{2}{5} + \dfrac{2 \times 5}{1 \times 5}=\dfrac{2}{5} + \dfrac{10}{5}=\dfrac{2 + 10}{5}=\dfrac{12}{5} = \dfrac{12}{5}$
\item $\dfrac{8}{3} + \dfrac{7}{6}=\dfrac{8 \times 2}{3 \times 2} + \dfrac{7}{6}=\dfrac{16}{6} + \dfrac{7}{6}=\dfrac{16 + 7}{6}=\dfrac{23}{6} = \dfrac{23}{6}$
\item $\dfrac{5}{9} + \dfrac{3}{7}=\dfrac{5 \times 7}{9 \times 7} + \dfrac{3 \times 9}{7 \times 9}=\dfrac{35}{63} + \dfrac{27}{63}=\dfrac{35 + 27}{63}=\dfrac{62}{63} = \dfrac{62}{63}$
\item $\dfrac{2}{4} \times 5=\dfrac{2 \times 5}{4}=\dfrac{10}{4} = \dfrac{5}{2}$
\item $\dfrac{7}{8} \times 2=\dfrac{7 \times 2}{8}=\dfrac{14}{8} = \dfrac{7}{4}$
\item $\dfrac{3}{7} \times \dfrac{4}{7}=\dfrac{3 \times 4}{7 \times 7}=\dfrac{12}{49} = \dfrac{12}{49}$
\item $\dfrac{10}{4} \times \dfrac{- 10}{28}=\dfrac{10(- 10)}{4 \times 28}=\dfrac{- 100}{112} = \dfrac{- 25}{28}$
\item $\dfrac{\dfrac{6}{4}}{\dfrac{9}{3}}=\dfrac{6}{4} \times \dfrac{3}{9}=\dfrac{6 \times 3}{4 \times 9}=\dfrac{18}{36} = \dfrac{1}{2}$
\item $\dfrac{- 3}{10} \times \dfrac{8}{20}=\dfrac{- 3 \times 8}{10 \times 20}=\dfrac{- 24}{200} = \dfrac{- 3}{25}$
\item $\dfrac{\dfrac{9}{6}}{\dfrac{2}{8}}=\dfrac{9}{6} \times \dfrac{8}{2}=\dfrac{9 \times 8}{6 \times 2}=\dfrac{72}{12} = 6$
\end{enumerate}
\end{solution}
@ -52,10 +52,10 @@
Le radar a pris des photos pendant l'été:
\begin{itemize}
\item en juin, il y a eu 54 photos prises dont 26 ratées.
\item en juillet, il y a eu 44 photos réussies et 32 ratées.
\item en août, il y a eu 61 photos dont une proportion de 0.26 de photos ratées.
\item en septembre, il y a eu 7 photos ratées, ce qui correspondait à 14.0\% des photos prises.
\item en juin, il y a eu 52 photos prises dont 28 ratées.
\item en juillet, il y a eu 41 photos réussies et 48 ratées.
\item en août, il y a eu 60 photos dont une proportion de 0.33 de photos ratées.
\item en septembre, il y a eu 9 photos ratées, ce qui correspondait à 17.65\% des photos prises.
\end{itemize}
\begin{enumerate}
@ -87,37 +87,35 @@
\hline
& Juin & Juillet & Août & Septembre & Total\\
\hline
Réussies & 28 & 44 & 45 & 43 & 160\\
Réussies & 24 & 41 & 40 & 42 & 147\\
\hline
Ratées & 26 & 32 & 16 & 7 & 81\\
Ratées & 28 & 48 & 20 & 9 & 105\\
\hline
Total & 54 & 76 & 61 & 50 & 241\\
Total & 52 & 89 & 60 & 51 & 252\\
\hline
\end{tabular}
\end{center}
\item Proportion de photos réussies
\[
\frac{160}{241} = 0.66 = 66\%
\frac{147}{252} = 0.58 = 58\%
\]
\item
\begin{itemize}
\item De juin à juillet
\[
\frac{32 - 26}{26} = \frac{6}{26} = 0.23 = 23\%
\frac{48 - 28}{28} = \frac{20}{28} = 0.71 = 71\%
\]
\item De juillet à août
\[
\frac{16 - 32}{32} = \frac{-16}{32} = -0.5 = -50\%
\frac{20 - 48}{48} = \frac{-28}{48} = -0.58 = -58\%
\]
\item De août à septembre
\[
\frac{7 - 16}{16} = \frac{-9}{16} = -0.56 = -56\%
\frac{9 - 20}{20} = \frac{-11}{20} = -0.55 = -55\%
\]
\end{itemize}
\end{enumerate}
\end{solution}
\printsolutionstype{exercise}
\end{document}

View File

@ -17,17 +17,17 @@
Détailler les calculs suivants et donner le résultat sous la forme d'une fraction irréductible.
\begin{multicols}{3}
\begin{enumerate}[label={\Alph*=}]
\item $\dfrac{9}{8} + \dfrac{7}{8}$
\item $\dfrac{9}{4} + 9$
\item $\dfrac{5}{6} + \dfrac{10}{6}$
\item $\dfrac{3}{2} + 8$
\item $\dfrac{3}{7} + \dfrac{8}{14}$
\item $\dfrac{5}{10} + \dfrac{2}{10}$
\item $\dfrac{9}{3} + \dfrac{10}{12}$
\item $\dfrac{4}{5} + \dfrac{6}{4}$
\item $\dfrac{5}{8} \times 1$
\item $\dfrac{1}{4} \times \dfrac{5}{4}$
\item $\dfrac{5}{7} \times 1$
\item $\dfrac{6}{8} \times \dfrac{9}{8}$
\item $\dfrac{1}{8} \times \dfrac{- 9}{80}$
\item $\dfrac{\dfrac{8}{10}}{\dfrac{7}{2}}$
\item $\dfrac{4}{3} \times \dfrac{- 1}{6}$
\item $\dfrac{\dfrac{2}{10}}{\dfrac{5}{2}}$
\end{enumerate}
\end{multicols}
\end{exercise}
@ -35,15 +35,15 @@
\begin{solution}
\begin{enumerate}[label={\Alph*=}]
\item $\dfrac{9}{8} + \dfrac{7}{8}=\dfrac{9 + 7}{8}=\dfrac{16}{8} = 2$
\item $\dfrac{9}{4} + 9=\dfrac{9}{4} + \dfrac{9}{1}=\dfrac{9}{4} + \dfrac{9 \times 4}{1 \times 4}=\dfrac{9}{4} + \dfrac{36}{4}=\dfrac{9 + 36}{4}=\dfrac{45}{4} = \dfrac{45}{4}$
\item $\dfrac{3}{7} + \dfrac{8}{14}=\dfrac{3 \times 2}{7 \times 2} + \dfrac{8}{14}=\dfrac{6}{14} + \dfrac{8}{14}=\dfrac{6 + 8}{14}=\dfrac{14}{14} = 1$
\item $\dfrac{5}{10} + \dfrac{2}{10}=\dfrac{5 + 2}{10}=\dfrac{7}{10} = \dfrac{7}{10}$
\item $\dfrac{5}{6} + \dfrac{10}{6}=\dfrac{5 + 10}{6}=\dfrac{15}{6} = \dfrac{5}{2}$
\item $\dfrac{3}{2} + 8=\dfrac{3}{2} + \dfrac{8}{1}=\dfrac{3}{2} + \dfrac{8 \times 2}{1 \times 2}=\dfrac{3}{2} + \dfrac{16}{2}=\dfrac{3 + 16}{2}=\dfrac{19}{2} = \dfrac{19}{2}$
\item $\dfrac{9}{3} + \dfrac{10}{12}=\dfrac{9 \times 4}{3 \times 4} + \dfrac{10}{12}=\dfrac{36}{12} + \dfrac{10}{12}=\dfrac{36 + 10}{12}=\dfrac{46}{12} = \dfrac{23}{6}$
\item $\dfrac{4}{5} + \dfrac{6}{4}=\dfrac{4 \times 4}{5 \times 4} + \dfrac{6 \times 5}{4 \times 5}=\dfrac{16}{20} + \dfrac{30}{20}=\dfrac{16 + 30}{20}=\dfrac{46}{20} = \dfrac{23}{10}$
\item $\dfrac{5}{8} \times 1=\dfrac{5}{8} = \dfrac{5}{8}$
\item $\dfrac{1}{4} \times \dfrac{5}{4}=\dfrac{1 \times 5}{4 \times 4}=\dfrac{5}{16} = \dfrac{5}{16}$
\item $\dfrac{1}{8} \times \dfrac{- 9}{80}=\dfrac{1(- 9)}{8 \times 80}=\dfrac{- 9}{640} = \dfrac{- 9}{640}$
\item $\dfrac{\dfrac{8}{10}}{\dfrac{7}{2}}=\dfrac{8}{10} \times \dfrac{2}{7}=\dfrac{8 \times 2}{10 \times 7}=\dfrac{16}{70} = \dfrac{8}{35}$
\item $\dfrac{5}{7} \times 1=\dfrac{5}{7} = \dfrac{5}{7}$
\item $\dfrac{6}{8} \times \dfrac{9}{8}=\dfrac{6 \times 9}{8 \times 8}=\dfrac{54}{64} = \dfrac{27}{32}$
\item $\dfrac{4}{3} \times \dfrac{- 1}{6}=\dfrac{4(- 1)}{3 \times 6}=\dfrac{- 4}{18} = \dfrac{- 2}{9}$
\item $\dfrac{\dfrac{2}{10}}{\dfrac{5}{2}}=\dfrac{2}{10} \times \dfrac{2}{5}=\dfrac{2 \times 2}{10 \times 5}=\dfrac{4}{50} = \dfrac{2}{25}$
\end{enumerate}
\end{solution}
@ -52,10 +52,10 @@
Le radar a pris des photos pendant l'été:
\begin{itemize}
\item en juin, il y a eu 56 photos prises dont 29 ratées.
\item en juillet, il y a eu 50 photos réussies et 37 ratées.
\item en août, il y a eu 60 photos dont une proportion de 0.17 de photos ratées.
\item en septembre, il y a eu 11 photos ratées, ce qui correspondait à 21.15\% des photos prises.
\item en juin, il y a eu 51 photos prises dont 22 ratées.
\item en juillet, il y a eu 38 photos réussies et 45 ratées.
\item en août, il y a eu 67 photos dont une proportion de 0.28 de photos ratées.
\item en septembre, il y a eu 13 photos ratées, ce qui correspondait à 22.03\% des photos prises.
\end{itemize}
\begin{enumerate}
@ -87,37 +87,35 @@
\hline
& Juin & Juillet & Août & Septembre & Total\\
\hline
Réussies & 27 & 50 & 50 & 41 & 168\\
Réussies & 29 & 38 & 48 & 46 & 161\\
\hline
Ratées & 29 & 37 & 10 & 11 & 87\\
Ratées & 22 & 45 & 19 & 13 & 99\\
\hline
Total & 56 & 87 & 60 & 52 & 255\\
Total & 51 & 83 & 67 & 59 & 260\\
\hline
\end{tabular}
\end{center}
\item Proportion de photos réussies
\[
\frac{168}{255} = 0.66 = 65\%
\frac{161}{260} = 0.62 = 61\%
\]
\item
\begin{itemize}
\item De juin à juillet
\[
\frac{37 - 29}{29} = \frac{8}{29} = 0.28 = 27\%
\frac{45 - 22}{22} = \frac{23}{22} = 1.05 = 104\%
\]
\item De juillet à août
\[
\frac{10 - 37}{37} = \frac{-27}{37} = -0.73 = -72\%
\frac{19 - 45}{45} = \frac{-26}{45} = -0.58 = -57\%
\]
\item De août à septembre
\[
\frac{11 - 10}{10} = \frac{1}{10} = 0.1 = 10\%
\frac{13 - 19}{19} = \frac{-6}{19} = -0.32 = -31\%
\]
\end{itemize}
\end{enumerate}
\end{solution}
\printsolutionstype{exercise}
\end{document}

View File

@ -17,17 +17,17 @@
Détailler les calculs suivants et donner le résultat sous la forme d'une fraction irréductible.
\begin{multicols}{3}
\begin{enumerate}[label={\Alph*=}]
\item $\dfrac{9}{7} + \dfrac{3}{7}$
\item $\dfrac{5}{7} + 6$
\item $\dfrac{10}{2} + \dfrac{4}{2}$
\item $\dfrac{10}{2} + 6$
\item $\dfrac{1}{7} + \dfrac{10}{14}$
\item $\dfrac{6}{8} + \dfrac{1}{7}$
\item $\dfrac{8}{9} + \dfrac{6}{45}$
\item $\dfrac{5}{3} + \dfrac{7}{6}$
\item $\dfrac{1}{5} \times 1$
\item $\dfrac{5}{8} \times \dfrac{7}{8}$
\item $\dfrac{7}{6} \times 5$
\item $\dfrac{7}{4} \times \dfrac{8}{4}$
\item $\dfrac{- 7}{2} \times \dfrac{- 1}{14}$
\item $\dfrac{\dfrac{7}{6}}{\dfrac{10}{9}}$
\item $\dfrac{3}{4} \times \dfrac{2}{20}$
\item $\dfrac{\dfrac{6}{5}}{\dfrac{10}{3}}$
\end{enumerate}
\end{multicols}
\end{exercise}
@ -35,15 +35,15 @@
\begin{solution}
\begin{enumerate}[label={\Alph*=}]
\item $\dfrac{9}{7} + \dfrac{3}{7}=\dfrac{9 + 3}{7}=\dfrac{12}{7} = \dfrac{12}{7}$
\item $\dfrac{5}{7} + 6=\dfrac{5}{7} + \dfrac{6}{1}=\dfrac{5}{7} + \dfrac{6 \times 7}{1 \times 7}=\dfrac{5}{7} + \dfrac{42}{7}=\dfrac{5 + 42}{7}=\dfrac{47}{7} = \dfrac{47}{7}$
\item $\dfrac{1}{7} + \dfrac{10}{14}=\dfrac{1 \times 2}{7 \times 2} + \dfrac{10}{14}=\dfrac{2}{14} + \dfrac{10}{14}=\dfrac{2 + 10}{14}=\dfrac{12}{14} = \dfrac{6}{7}$
\item $\dfrac{6}{8} + \dfrac{1}{7}=\dfrac{6 \times 7}{8 \times 7} + \dfrac{1 \times 8}{7 \times 8}=\dfrac{42}{56} + \dfrac{8}{56}=\dfrac{42 + 8}{56}=\dfrac{50}{56} = \dfrac{25}{28}$
\item $\dfrac{10}{2} + \dfrac{4}{2}=\dfrac{10 + 4}{2}=\dfrac{14}{2} = 7$
\item $\dfrac{10}{2} + 6=\dfrac{10}{2} + \dfrac{6}{1}=\dfrac{10}{2} + \dfrac{6 \times 2}{1 \times 2}=\dfrac{10}{2} + \dfrac{12}{2}=\dfrac{10 + 12}{2}=\dfrac{22}{2} = 11$
\item $\dfrac{8}{9} + \dfrac{6}{45}=\dfrac{8 \times 5}{9 \times 5} + \dfrac{6}{45}=\dfrac{40}{45} + \dfrac{6}{45}=\dfrac{40 + 6}{45}=\dfrac{46}{45} = \dfrac{46}{45}$
\item $\dfrac{5}{3} + \dfrac{7}{6}=\dfrac{5 \times 2}{3 \times 2} + \dfrac{7}{6}=\dfrac{10}{6} + \dfrac{7}{6}=\dfrac{10 + 7}{6}=\dfrac{17}{6} = \dfrac{17}{6}$
\item $\dfrac{1}{5} \times 1=\dfrac{1}{5} = \dfrac{1}{5}$
\item $\dfrac{5}{8} \times \dfrac{7}{8}=\dfrac{5 \times 7}{8 \times 8}=\dfrac{35}{64} = \dfrac{35}{64}$
\item $\dfrac{- 7}{2} \times \dfrac{- 1}{14}=\dfrac{- 7(- 1)}{2 \times 14}=\dfrac{7}{28} = \dfrac{1}{4}$
\item $\dfrac{\dfrac{7}{6}}{\dfrac{10}{9}}=\dfrac{7}{6} \times \dfrac{9}{10}=\dfrac{7 \times 9}{6 \times 10}=\dfrac{63}{60} = \dfrac{21}{20}$
\item $\dfrac{7}{6} \times 5=\dfrac{7 \times 5}{6}=\dfrac{35}{6} = \dfrac{35}{6}$
\item $\dfrac{7}{4} \times \dfrac{8}{4}=\dfrac{7 \times 8}{4 \times 4}=\dfrac{56}{16} = \dfrac{7}{2}$
\item $\dfrac{3}{4} \times \dfrac{2}{20}=\dfrac{3 \times 2}{4 \times 20}=\dfrac{6}{80} = \dfrac{3}{40}$
\item $\dfrac{\dfrac{6}{5}}{\dfrac{10}{3}}=\dfrac{6}{5} \times \dfrac{3}{10}=\dfrac{6 \times 3}{5 \times 10}=\dfrac{18}{50} = \dfrac{9}{25}$
\end{enumerate}
\end{solution}
@ -52,10 +52,10 @@
Le radar a pris des photos pendant l'été:
\begin{itemize}
\item en juin, il y a eu 55 photos prises dont 26 ratées.
\item en juillet, il y a eu 39 photos réussies et 33 ratées.
\item en août, il y a eu 66 photos dont une proportion de 0.3 de photos ratées.
\item en septembre, il y a eu 11 photos ratées, ce qui correspondait à 19.64\% des photos prises.
\item en juin, il y a eu 44 photos prises dont 20 ratées.
\item en juillet, il y a eu 40 photos réussies et 37 ratées.
\item en août, il y a eu 61 photos dont une proportion de 0.33 de photos ratées.
\item en septembre, il y a eu 6 photos ratées, ce qui correspondait à 13.04\% des photos prises.
\end{itemize}
\begin{enumerate}
@ -87,37 +87,35 @@
\hline
& Juin & Juillet & Août & Septembre & Total\\
\hline
Réussies & 29 & 39 & 46 & 45 & 159\\
Réussies & 24 & 40 & 41 & 40 & 145\\
\hline
Ratées & 26 & 33 & 20 & 11 & 90\\
Ratées & 20 & 37 & 20 & 6 & 83\\
\hline
Total & 55 & 72 & 66 & 56 & 249\\
Total & 44 & 77 & 61 & 46 & 228\\
\hline
\end{tabular}
\end{center}
\item Proportion de photos réussies
\[
\frac{159}{249} = 0.64 = 63\%
\frac{145}{228} = 0.64 = 63\%
\]
\item
\begin{itemize}
\item De juin à juillet
\[
\frac{33 - 26}{26} = \frac{7}{26} = 0.27 = 26\%
\frac{37 - 20}{20} = \frac{17}{20} = 0.85 = 85\%
\]
\item De juillet à août
\[
\frac{20 - 33}{33} = \frac{-13}{33} = -0.39 = -39\%
\frac{20 - 37}{37} = \frac{-17}{37} = -0.46 = -45\%
\]
\item De août à septembre
\[
\frac{11 - 20}{20} = \frac{-9}{20} = -0.45 = -45\%
\frac{6 - 20}{20} = \frac{-14}{20} = -0.7 = -70\%
\]
\end{itemize}
\end{enumerate}
\end{solution}
\printsolutionstype{exercise}
\end{document}

View File

@ -17,17 +17,17 @@
Détailler les calculs suivants et donner le résultat sous la forme d'une fraction irréductible.
\begin{multicols}{3}
\begin{enumerate}[label={\Alph*=}]
\item $\dfrac{2}{9} + \dfrac{7}{9}$
\item $\dfrac{2}{8} + 5$
\item $\dfrac{2}{7} + \dfrac{9}{7}$
\item $\dfrac{2}{9} + 8$
\item $\dfrac{2}{5} + \dfrac{1}{30}$
\item $\dfrac{3}{8} + \dfrac{5}{3}$
\item $\dfrac{1}{8} + \dfrac{3}{8}$
\item $\dfrac{3}{2} + \dfrac{9}{7}$
\item $\dfrac{7}{5} \times 2$
\item $\dfrac{10}{9} \times \dfrac{3}{9}$
\item $\dfrac{1}{10} \times 2$
\item $\dfrac{7}{3} \times \dfrac{10}{3}$
\item $\dfrac{9}{7} \times \dfrac{3}{49}$
\item $\dfrac{\dfrac{3}{7}}{\dfrac{2}{9}}$
\item $\dfrac{9}{3} \times \dfrac{2}{6}$
\item $\dfrac{\dfrac{2}{6}}{\dfrac{8}{7}}$
\end{enumerate}
\end{multicols}
\end{exercise}
@ -35,15 +35,15 @@
\begin{solution}
\begin{enumerate}[label={\Alph*=}]
\item $\dfrac{2}{9} + \dfrac{7}{9}=\dfrac{2 + 7}{9}=\dfrac{9}{9} = 1$
\item $\dfrac{2}{8} + 5=\dfrac{2}{8} + \dfrac{5}{1}=\dfrac{2}{8} + \dfrac{5 \times 8}{1 \times 8}=\dfrac{2}{8} + \dfrac{40}{8}=\dfrac{2 + 40}{8}=\dfrac{42}{8} = \dfrac{21}{4}$
\item $\dfrac{2}{5} + \dfrac{1}{30}=\dfrac{2 \times 6}{5 \times 6} + \dfrac{1}{30}=\dfrac{12}{30} + \dfrac{1}{30}=\dfrac{12 + 1}{30}=\dfrac{13}{30} = \dfrac{13}{30}$
\item $\dfrac{3}{8} + \dfrac{5}{3}=\dfrac{3 \times 3}{8 \times 3} + \dfrac{5 \times 8}{3 \times 8}=\dfrac{9}{24} + \dfrac{40}{24}=\dfrac{9 + 40}{24}=\dfrac{49}{24} = \dfrac{49}{24}$
\item $\dfrac{2}{7} + \dfrac{9}{7}=\dfrac{2 + 9}{7}=\dfrac{11}{7} = \dfrac{11}{7}$
\item $\dfrac{2}{9} + 8=\dfrac{2}{9} + \dfrac{8}{1}=\dfrac{2}{9} + \dfrac{8 \times 9}{1 \times 9}=\dfrac{2}{9} + \dfrac{72}{9}=\dfrac{2 + 72}{9}=\dfrac{74}{9} = \dfrac{74}{9}$
\item $\dfrac{1}{8} + \dfrac{3}{8}=\dfrac{1 + 3}{8}=\dfrac{4}{8} = \dfrac{1}{2}$
\item $\dfrac{3}{2} + \dfrac{9}{7}=\dfrac{3 \times 7}{2 \times 7} + \dfrac{9 \times 2}{7 \times 2}=\dfrac{21}{14} + \dfrac{18}{14}=\dfrac{21 + 18}{14}=\dfrac{39}{14} = \dfrac{39}{14}$
\item $\dfrac{7}{5} \times 2=\dfrac{7 \times 2}{5}=\dfrac{14}{5} = \dfrac{14}{5}$
\item $\dfrac{10}{9} \times \dfrac{3}{9}=\dfrac{10 \times 3}{9 \times 9}=\dfrac{30}{81} = \dfrac{10}{27}$
\item $\dfrac{9}{7} \times \dfrac{3}{49}=\dfrac{9 \times 3}{7 \times 49}=\dfrac{27}{343} = \dfrac{27}{343}$
\item $\dfrac{\dfrac{3}{7}}{\dfrac{2}{9}}=\dfrac{3}{7} \times \dfrac{9}{2}=\dfrac{3 \times 9}{7 \times 2}=\dfrac{27}{14} = \dfrac{27}{14}$
\item $\dfrac{1}{10} \times 2=\dfrac{1 \times 2}{10}=\dfrac{2}{10} = \dfrac{1}{5}$
\item $\dfrac{7}{3} \times \dfrac{10}{3}=\dfrac{7 \times 10}{3 \times 3}=\dfrac{70}{9} = \dfrac{70}{9}$
\item $\dfrac{9}{3} \times \dfrac{2}{6}=\dfrac{9 \times 2}{3 \times 6}=\dfrac{18}{18} = 1$
\item $\dfrac{\dfrac{2}{6}}{\dfrac{8}{7}}=\dfrac{2}{6} \times \dfrac{7}{8}=\dfrac{2 \times 7}{6 \times 8}=\dfrac{14}{48} = \dfrac{7}{24}$
\end{enumerate}
\end{solution}
@ -52,10 +52,10 @@
Le radar a pris des photos pendant l'été:
\begin{itemize}
\item en juin, il y a eu 45 photos prises dont 21 ratées.
\item en juillet, il y a eu 36 photos réussies et 41 ratées.
\item en août, il y a eu 54 photos dont une proportion de 0.26 de photos ratées.
\item en septembre, il y a eu 5 photos ratées, ce qui correspondait à 9.43\% des photos prises.
\item en juin, il y a eu 44 photos prises dont 24 ratées.
\item en juillet, il y a eu 32 photos réussies et 36 ratées.
\item en août, il y a eu 59 photos dont une proportion de 0.25 de photos ratées.
\item en septembre, il y a eu 12 photos ratées, ce qui correspondait à 21.82\% des photos prises.
\end{itemize}
\begin{enumerate}
@ -87,37 +87,35 @@
\hline
& Juin & Juillet & Août & Septembre & Total\\
\hline
Réussies & 24 & 36 & 40 & 48 & 148\\
Réussies & 20 & 32 & 44 & 43 & 139\\
\hline
Ratées & 21 & 41 & 14 & 5 & 81\\
Ratées & 24 & 36 & 15 & 12 & 87\\
\hline
Total & 45 & 77 & 54 & 53 & 229\\
Total & 44 & 68 & 59 & 55 & 226\\
\hline
\end{tabular}
\end{center}
\item Proportion de photos réussies
\[
\frac{148}{229} = 0.65 = 64\%
\frac{139}{226} = 0.62 = 61\%
\]
\item
\begin{itemize}
\item De juin à juillet
\[
\frac{41 - 21}{21} = \frac{20}{21} = 0.95 = 95\%
\frac{36 - 24}{24} = \frac{12}{24} = 0.5 = 50\%
\]
\item De juillet à août
\[
\frac{14 - 41}{41} = \frac{-27}{41} = -0.66 = -65\%
\frac{15 - 36}{36} = \frac{-21}{36} = -0.58 = -58\%
\]
\item De août à septembre
\[
\frac{5 - 14}{14} = \frac{-9}{14} = -0.64 = -64\%
\frac{12 - 15}{15} = \frac{-3}{15} = -0.2 = -20\%
\]
\end{itemize}
\end{enumerate}
\end{solution}
\printsolutionstype{exercise}
\end{document}

View File

@ -17,17 +17,17 @@
Détailler les calculs suivants et donner le résultat sous la forme d'une fraction irréductible.
\begin{multicols}{3}
\begin{enumerate}[label={\Alph*=}]
\item $\dfrac{8}{3} + \dfrac{4}{3}$
\item $\dfrac{8}{7} + 7$
\item $\dfrac{2}{8} + \dfrac{10}{8}$
\item $\dfrac{3}{4} + 4$
\item $\dfrac{7}{8} + \dfrac{6}{72}$
\item $\dfrac{8}{3} + \dfrac{10}{7}$
\item $\dfrac{6}{10} + \dfrac{8}{60}$
\item $\dfrac{6}{8} + \dfrac{9}{7}$
\item $\dfrac{2}{7} \times 9$
\item $\dfrac{4}{9} \times \dfrac{7}{9}$
\item $\dfrac{9}{10} \times 7$
\item $\dfrac{2}{3} \times \dfrac{8}{3}$
\item $\dfrac{- 7}{8} \times \dfrac{7}{56}$
\item $\dfrac{\dfrac{8}{10}}{\dfrac{1}{8}}$
\item $\dfrac{1}{9} \times \dfrac{- 7}{81}$
\item $\dfrac{\dfrac{7}{9}}{\dfrac{6}{9}}$
\end{enumerate}
\end{multicols}
\end{exercise}
@ -35,15 +35,15 @@
\begin{solution}
\begin{enumerate}[label={\Alph*=}]
\item $\dfrac{8}{3} + \dfrac{4}{3}=\dfrac{8 + 4}{3}=\dfrac{12}{3} = 4$
\item $\dfrac{8}{7} + 7=\dfrac{8}{7} + \dfrac{7}{1}=\dfrac{8}{7} + \dfrac{7 \times 7}{1 \times 7}=\dfrac{8}{7} + \dfrac{49}{7}=\dfrac{8 + 49}{7}=\dfrac{57}{7} = \dfrac{57}{7}$
\item $\dfrac{7}{8} + \dfrac{6}{72}=\dfrac{7 \times 9}{8 \times 9} + \dfrac{6}{72}=\dfrac{63}{72} + \dfrac{6}{72}=\dfrac{63 + 6}{72}=\dfrac{69}{72} = \dfrac{23}{24}$
\item $\dfrac{8}{3} + \dfrac{10}{7}=\dfrac{8 \times 7}{3 \times 7} + \dfrac{10 \times 3}{7 \times 3}=\dfrac{56}{21} + \dfrac{30}{21}=\dfrac{56 + 30}{21}=\dfrac{86}{21} = \dfrac{86}{21}$
\item $\dfrac{2}{8} + \dfrac{10}{8}=\dfrac{2 + 10}{8}=\dfrac{12}{8} = \dfrac{3}{2}$
\item $\dfrac{3}{4} + 4=\dfrac{3}{4} + \dfrac{4}{1}=\dfrac{3}{4} + \dfrac{4 \times 4}{1 \times 4}=\dfrac{3}{4} + \dfrac{16}{4}=\dfrac{3 + 16}{4}=\dfrac{19}{4} = \dfrac{19}{4}$
\item $\dfrac{6}{10} + \dfrac{8}{60}=\dfrac{6 \times 6}{10 \times 6} + \dfrac{8}{60}=\dfrac{36}{60} + \dfrac{8}{60}=\dfrac{36 + 8}{60}=\dfrac{44}{60} = \dfrac{11}{15}$
\item $\dfrac{6}{8} + \dfrac{9}{7}=\dfrac{6 \times 7}{8 \times 7} + \dfrac{9 \times 8}{7 \times 8}=\dfrac{42}{56} + \dfrac{72}{56}=\dfrac{42 + 72}{56}=\dfrac{114}{56} = \dfrac{57}{28}$
\item $\dfrac{2}{7} \times 9=\dfrac{2 \times 9}{7}=\dfrac{18}{7} = \dfrac{18}{7}$
\item $\dfrac{4}{9} \times \dfrac{7}{9}=\dfrac{4 \times 7}{9 \times 9}=\dfrac{28}{81} = \dfrac{28}{81}$
\item $\dfrac{- 7}{8} \times \dfrac{7}{56}=\dfrac{- 7 \times 7}{8 \times 56}=\dfrac{- 49}{448} = \dfrac{- 7}{64}$
\item $\dfrac{\dfrac{8}{10}}{\dfrac{1}{8}}=\dfrac{8}{10} \times \dfrac{8}{1}=\dfrac{8 \times 8}{10 \times 1}=\dfrac{64}{10} = \dfrac{32}{5}$
\item $\dfrac{9}{10} \times 7=\dfrac{9 \times 7}{10}=\dfrac{63}{10} = \dfrac{63}{10}$
\item $\dfrac{2}{3} \times \dfrac{8}{3}=\dfrac{2 \times 8}{3 \times 3}=\dfrac{16}{9} = \dfrac{16}{9}$
\item $\dfrac{1}{9} \times \dfrac{- 7}{81}=\dfrac{1(- 7)}{9 \times 81}=\dfrac{- 7}{729} = \dfrac{- 7}{729}$
\item $\dfrac{\dfrac{7}{9}}{\dfrac{6}{9}}=\dfrac{7}{9} \times \dfrac{9}{6}=\dfrac{7 \times 9}{9 \times 6}=\dfrac{63}{54} = \dfrac{7}{6}$
\end{enumerate}
\end{solution}
@ -52,10 +52,10 @@
Le radar a pris des photos pendant l'été:
\begin{itemize}
\item en juin, il y a eu 50 photos prises dont 23 ratées.
\item en juillet, il y a eu 31 photos réussies et 48 ratées.
\item en août, il y a eu 60 photos dont une proportion de 0.32 de photos ratées.
\item en septembre, il y a eu 11 photos ratées, ce qui correspondait à 20.75\% des photos prises.
\item en juin, il y a eu 41 photos prises dont 21 ratées.
\item en juillet, il y a eu 49 photos réussies et 42 ratées.
\item en août, il y a eu 52 photos dont une proportion de 0.19 de photos ratées.
\item en septembre, il y a eu 11 photos ratées, ce qui correspondait à 18.64\% des photos prises.
\end{itemize}
\begin{enumerate}
@ -87,37 +87,35 @@
\hline
& Juin & Juillet & Août & Septembre & Total\\
\hline
Réussies & 27 & 31 & 41 & 42 & 141\\
Réussies & 20 & 49 & 42 & 48 & 159\\
\hline
Ratées & 23 & 48 & 19 & 11 & 101\\
Ratées & 21 & 42 & 10 & 11 & 84\\
\hline
Total & 50 & 79 & 60 & 53 & 242\\
Total & 41 & 91 & 52 & 59 & 243\\
\hline
\end{tabular}
\end{center}
\item Proportion de photos réussies
\[
\frac{141}{242} = 0.58 = 58\%
\frac{159}{243} = 0.65 = 65\%
\]
\item
\begin{itemize}
\item De juin à juillet
\[
\frac{48 - 23}{23} = \frac{25}{23} = 1.09 = 108\%
\frac{42 - 21}{21} = \frac{21}{21} = 1.0 = 100\%
\]
\item De juillet à août
\[
\frac{19 - 48}{48} = \frac{-29}{48} = -0.6 = -60\%
\frac{10 - 42}{42} = \frac{-32}{42} = -0.76 = -76\%
\]
\item De août à septembre
\[
\frac{11 - 19}{19} = \frac{-8}{19} = -0.42 = -42\%
\frac{11 - 10}{10} = \frac{1}{10} = 0.1 = 10\%
\]
\end{itemize}
\end{enumerate}
\end{solution}
\printsolutionstype{exercise}
\end{document}

View File

@ -17,17 +17,17 @@
Détailler les calculs suivants et donner le résultat sous la forme d'une fraction irréductible.
\begin{multicols}{3}
\begin{enumerate}[label={\Alph*=}]
\item $\dfrac{6}{7} + \dfrac{8}{7}$
\item $\dfrac{6}{8} + 10$
\item $\dfrac{1}{9} + \dfrac{4}{9}$
\item $\dfrac{2}{5} + 1$
\item $\dfrac{1}{2} + \dfrac{3}{8}$
\item $\dfrac{6}{2} + \dfrac{5}{10}$
\item $\dfrac{10}{3} + \dfrac{2}{15}$
\item $\dfrac{5}{2} + \dfrac{10}{2}$
\item $\dfrac{6}{10} \times 1$
\item $\dfrac{3}{5} \times \dfrac{3}{5}$
\item $\dfrac{1}{7} \times 6$
\item $\dfrac{6}{9} \times \dfrac{10}{9}$
\item $\dfrac{- 1}{9} \times \dfrac{3}{63}$
\item $\dfrac{\dfrac{7}{3}}{\dfrac{4}{3}}$
\item $\dfrac{4}{5} \times \dfrac{- 10}{15}$
\item $\dfrac{\dfrac{10}{7}}{\dfrac{3}{8}}$
\end{enumerate}
\end{multicols}
\end{exercise}
@ -35,15 +35,15 @@
\begin{solution}
\begin{enumerate}[label={\Alph*=}]
\item $\dfrac{6}{7} + \dfrac{8}{7}=\dfrac{6 + 8}{7}=\dfrac{14}{7} = 2$
\item $\dfrac{6}{8} + 10=\dfrac{6}{8} + \dfrac{10}{1}=\dfrac{6}{8} + \dfrac{10 \times 8}{1 \times 8}=\dfrac{6}{8} + \dfrac{80}{8}=\dfrac{6 + 80}{8}=\dfrac{86}{8} = \dfrac{43}{4}$
\item $\dfrac{1}{2} + \dfrac{3}{8}=\dfrac{1 \times 4}{2 \times 4} + \dfrac{3}{8}=\dfrac{4}{8} + \dfrac{3}{8}=\dfrac{4 + 3}{8}=\dfrac{7}{8} = \dfrac{7}{8}$
\item $\dfrac{6}{2} + \dfrac{5}{10}=\dfrac{6 \times 5}{2 \times 5} + \dfrac{5}{10}=\dfrac{30}{10} + \dfrac{5}{10}=\dfrac{30 + 5}{10}=\dfrac{35}{10} = \dfrac{7}{2}$
\item $\dfrac{1}{9} + \dfrac{4}{9}=\dfrac{1 + 4}{9}=\dfrac{5}{9} = \dfrac{5}{9}$
\item $\dfrac{2}{5} + 1=\dfrac{2}{5} + \dfrac{1}{1}=\dfrac{2}{5} + \dfrac{1 \times 5}{1 \times 5}=\dfrac{2}{5} + \dfrac{5}{5}=\dfrac{2 + 5}{5}=\dfrac{7}{5} = \dfrac{7}{5}$
\item $\dfrac{10}{3} + \dfrac{2}{15}=\dfrac{10 \times 5}{3 \times 5} + \dfrac{2}{15}=\dfrac{50}{15} + \dfrac{2}{15}=\dfrac{50 + 2}{15}=\dfrac{52}{15} = \dfrac{52}{15}$
\item $\dfrac{5}{2} + \dfrac{10}{2}=\dfrac{5 + 10}{2}=\dfrac{15}{2} = \dfrac{15}{2}$
\item $\dfrac{6}{10} \times 1=\dfrac{6}{10} = \dfrac{3}{5}$
\item $\dfrac{3}{5} \times \dfrac{3}{5}=\dfrac{3 \times 3}{5 \times 5}=\dfrac{9}{25} = \dfrac{9}{25}$
\item $\dfrac{- 1}{9} \times \dfrac{3}{63}=\dfrac{- 1 \times 3}{9 \times 63}=\dfrac{- 3}{567} = \dfrac{- 1}{189}$
\item $\dfrac{\dfrac{7}{3}}{\dfrac{4}{3}}=\dfrac{7}{3} \times \dfrac{3}{4}=\dfrac{7 \times 3}{3 \times 4}=\dfrac{21}{12} = \dfrac{7}{4}$
\item $\dfrac{1}{7} \times 6=\dfrac{1 \times 6}{7}=\dfrac{6}{7} = \dfrac{6}{7}$
\item $\dfrac{6}{9} \times \dfrac{10}{9}=\dfrac{6 \times 10}{9 \times 9}=\dfrac{60}{81} = \dfrac{20}{27}$
\item $\dfrac{4}{5} \times \dfrac{- 10}{15}=\dfrac{4(- 10)}{5 \times 15}=\dfrac{- 40}{75} = \dfrac{- 8}{15}$
\item $\dfrac{\dfrac{10}{7}}{\dfrac{3}{8}}=\dfrac{10}{7} \times \dfrac{8}{3}=\dfrac{10 \times 8}{7 \times 3}=\dfrac{80}{21} = \dfrac{80}{21}$
\end{enumerate}
\end{solution}
@ -52,10 +52,10 @@
Le radar a pris des photos pendant l'été:
\begin{itemize}
\item en juin, il y a eu 52 photos prises dont 26 ratées.
\item en juillet, il y a eu 48 photos réussies et 35 ratées.
\item en août, il y a eu 69 photos dont une proportion de 0.28 de photos ratées.
\item en septembre, il y a eu 11 photos ratées, ce qui correspondait à 19.3\% des photos prises.
\item en juin, il y a eu 52 photos prises dont 28 ratées.
\item en juillet, il y a eu 49 photos réussies et 41 ratées.
\item en août, il y a eu 60 photos dont une proportion de 0.22 de photos ratées.
\item en septembre, il y a eu 5 photos ratées, ce qui correspondait à 9.43\% des photos prises.
\end{itemize}
\begin{enumerate}
@ -87,37 +87,35 @@
\hline
& Juin & Juillet & Août & Septembre & Total\\
\hline
Réussies & 26 & 48 & 50 & 46 & 170\\
Réussies & 24 & 49 & 47 & 48 & 168\\
\hline
Ratées & 26 & 35 & 19 & 11 & 91\\
Ratées & 28 & 41 & 13 & 5 & 87\\
\hline
Total & 52 & 83 & 69 & 57 & 261\\
Total & 52 & 90 & 60 & 53 & 255\\
\hline
\end{tabular}
\end{center}
\item Proportion de photos réussies
\[
\frac{170}{261} = 0.65 = 65\%
\frac{168}{255} = 0.66 = 65\%
\]
\item
\begin{itemize}
\item De juin à juillet
\[
\frac{35 - 26}{26} = \frac{9}{26} = 0.35 = 34\%
\frac{41 - 28}{28} = \frac{13}{28} = 0.46 = 46\%
\]
\item De juillet à août
\[
\frac{19 - 35}{35} = \frac{-16}{35} = -0.46 = -45\%
\frac{13 - 41}{41} = \frac{-28}{41} = -0.68 = -68\%
\]
\item De août à septembre
\[
\frac{11 - 19}{19} = \frac{-8}{19} = -0.42 = -42\%
\frac{5 - 13}{13} = \frac{-8}{13} = -0.62 = -61\%
\]
\end{itemize}
\end{enumerate}
\end{solution}
\printsolutionstype{exercise}
\end{document}

View File

@ -17,17 +17,17 @@
Détailler les calculs suivants et donner le résultat sous la forme d'une fraction irréductible.
\begin{multicols}{3}
\begin{enumerate}[label={\Alph*=}]
\item $\dfrac{7}{9} + \dfrac{1}{9}$
\item $\dfrac{9}{3} + 1$
\item $\dfrac{7}{9} + \dfrac{10}{9}$
\item $\dfrac{5}{10} + 3$
\item $\dfrac{1}{2} + \dfrac{9}{6}$
\item $\dfrac{6}{9} + \dfrac{3}{2}$
\item $\dfrac{8}{7} + \dfrac{4}{7}$
\item $\dfrac{10}{4} + \dfrac{5}{2}$
\item $\dfrac{4}{8} \times 3$
\item $\dfrac{5}{4} \times \dfrac{1}{4}$
\item $\dfrac{4}{3} \times 1$
\item $\dfrac{2}{10} \times \dfrac{8}{10}$
\item $\dfrac{- 2}{3} \times \dfrac{- 2}{30}$
\item $\dfrac{\dfrac{5}{8}}{\dfrac{7}{3}}$
\item $\dfrac{- 5}{9} \times \dfrac{7}{45}$
\item $\dfrac{\dfrac{9}{6}}{\dfrac{3}{7}}$
\end{enumerate}
\end{multicols}
\end{exercise}
@ -35,15 +35,15 @@
\begin{solution}
\begin{enumerate}[label={\Alph*=}]
\item $\dfrac{7}{9} + \dfrac{1}{9}=\dfrac{7 + 1}{9}=\dfrac{8}{9} = \dfrac{8}{9}$
\item $\dfrac{9}{3} + 1=\dfrac{9}{3} + \dfrac{1}{1}=\dfrac{9}{3} + \dfrac{1 \times 3}{1 \times 3}=\dfrac{9}{3} + \dfrac{3}{3}=\dfrac{9 + 3}{3}=\dfrac{12}{3} = 4$
\item $\dfrac{1}{2} + \dfrac{9}{6}=\dfrac{1 \times 3}{2 \times 3} + \dfrac{9}{6}=\dfrac{3}{6} + \dfrac{9}{6}=\dfrac{3 + 9}{6}=\dfrac{12}{6} = 2$
\item $\dfrac{6}{9} + \dfrac{3}{2}=\dfrac{6 \times 2}{9 \times 2} + \dfrac{3 \times 9}{2 \times 9}=\dfrac{12}{18} + \dfrac{27}{18}=\dfrac{12 + 27}{18}=\dfrac{39}{18} = \dfrac{13}{6}$
\item $\dfrac{7}{9} + \dfrac{10}{9}=\dfrac{7 + 10}{9}=\dfrac{17}{9} = \dfrac{17}{9}$
\item $\dfrac{5}{10} + 3=\dfrac{5}{10} + \dfrac{3}{1}=\dfrac{5}{10} + \dfrac{3 \times 10}{1 \times 10}=\dfrac{5}{10} + \dfrac{30}{10}=\dfrac{5 + 30}{10}=\dfrac{35}{10} = \dfrac{7}{2}$
\item $\dfrac{8}{7} + \dfrac{4}{7}=\dfrac{8 + 4}{7}=\dfrac{12}{7} = \dfrac{12}{7}$
\item $\dfrac{10}{4} + \dfrac{5}{2}=\dfrac{10}{4} + \dfrac{5 \times 2}{2 \times 2}=\dfrac{10}{4} + \dfrac{10}{4}=\dfrac{10 + 10}{4}=\dfrac{20}{4} = 5$
\item $\dfrac{4}{8} \times 3=\dfrac{4 \times 3}{8}=\dfrac{12}{8} = \dfrac{3}{2}$
\item $\dfrac{5}{4} \times \dfrac{1}{4}=\dfrac{5 \times 1}{4 \times 4}=\dfrac{5}{16} = \dfrac{5}{16}$
\item $\dfrac{- 2}{3} \times \dfrac{- 2}{30}=\dfrac{- 2(- 2)}{3 \times 30}=\dfrac{4}{90} = \dfrac{2}{45}$
\item $\dfrac{\dfrac{5}{8}}{\dfrac{7}{3}}=\dfrac{5}{8} \times \dfrac{3}{7}=\dfrac{5 \times 3}{8 \times 7}=\dfrac{15}{56} = \dfrac{15}{56}$
\item $\dfrac{4}{3} \times 1=\dfrac{4}{3} = \dfrac{4}{3}$
\item $\dfrac{2}{10} \times \dfrac{8}{10}=\dfrac{2 \times 8}{10 \times 10}=\dfrac{16}{100} = \dfrac{4}{25}$
\item $\dfrac{- 5}{9} \times \dfrac{7}{45}=\dfrac{- 5 \times 7}{9 \times 45}=\dfrac{- 35}{405} = \dfrac{- 7}{81}$
\item $\dfrac{\dfrac{9}{6}}{\dfrac{3}{7}}=\dfrac{9}{6} \times \dfrac{7}{3}=\dfrac{9 \times 7}{6 \times 3}=\dfrac{63}{18} = \dfrac{7}{2}$
\end{enumerate}
\end{solution}
@ -52,10 +52,10 @@
Le radar a pris des photos pendant l'été:
\begin{itemize}
\item en juin, il y a eu 49 photos prises dont 22 ratées.
\item en juillet, il y a eu 44 photos réussies et 43 ratées.
\item en août, il y a eu 65 photos dont une proportion de 0.28 de photos ratées.
\item en septembre, il y a eu 14 photos ratées, ce qui correspondait à 25.45\% des photos prises.
\item en juin, il y a eu 50 photos prises dont 28 ratées.
\item en juillet, il y a eu 48 photos réussies et 44 ratées.
\item en août, il y a eu 58 photos dont une proportion de 0.17 de photos ratées.
\item en septembre, il y a eu 6 photos ratées, ce qui correspondait à 10.71\% des photos prises.
\end{itemize}
\begin{enumerate}
@ -87,37 +87,35 @@
\hline
& Juin & Juillet & Août & Septembre & Total\\
\hline
Réussies & 27 & 44 & 47 & 41 & 159\\
Réussies & 22 & 48 & 48 & 50 & 168\\
\hline
Ratées & 22 & 43 & 18 & 14 & 97\\
Ratées & 28 & 44 & 10 & 6 & 88\\
\hline
Total & 49 & 87 & 65 & 55 & 256\\
Total & 50 & 92 & 58 & 56 & 256\\
\hline
\end{tabular}
\end{center}
\item Proportion de photos réussies
\[
\frac{159}{256} = 0.62 = 62\%
\frac{168}{256} = 0.66 = 65\%
\]
\item
\begin{itemize}
\item De juin à juillet
\[
\frac{43 - 22}{22} = \frac{21}{22} = 0.95 = 95\%
\frac{44 - 28}{28} = \frac{16}{28} = 0.57 = 57\%
\]
\item De juillet à août
\[
\frac{18 - 43}{43} = \frac{-25}{43} = -0.58 = -58\%
\frac{10 - 44}{44} = \frac{-34}{44} = -0.77 = -77\%
\]
\item De août à septembre
\[
\frac{14 - 18}{18} = \frac{-4}{18} = -0.22 = -22\%
\frac{6 - 10}{10} = \frac{-4}{10} = -0.4 = -40\%
\]
\end{itemize}
\end{enumerate}
\end{solution}
\printsolutionstype{exercise}
\end{document}

View File

@ -17,17 +17,17 @@
Détailler les calculs suivants et donner le résultat sous la forme d'une fraction irréductible.
\begin{multicols}{3}
\begin{enumerate}[label={\Alph*=}]
\item $\dfrac{6}{9} + \dfrac{1}{9}$
\item $\dfrac{3}{2} + 4$
\item $\dfrac{7}{6} + \dfrac{5}{6}$
\item $\dfrac{10}{3} + 8$
\item $\dfrac{10}{8} + \dfrac{5}{80}$
\item $\dfrac{8}{5} + \dfrac{3}{10}$
\item $\dfrac{7}{4} + \dfrac{2}{28}$
\item $\dfrac{6}{8} + \dfrac{5}{3}$
\item $\dfrac{7}{2} \times 7$
\item $\dfrac{5}{9} \times \dfrac{6}{9}$
\item $\dfrac{8}{5} \times 10$
\item $\dfrac{5}{7} \times \dfrac{4}{7}$
\item $\dfrac{8}{3} \times \dfrac{- 9}{9}$
\item $\dfrac{\dfrac{2}{4}}{\dfrac{1}{6}}$
\item $\dfrac{- 1}{9} \times \dfrac{10}{72}$
\item $\dfrac{\dfrac{7}{6}}{\dfrac{8}{2}}$
\end{enumerate}
\end{multicols}
\end{exercise}
@ -35,15 +35,15 @@
\begin{solution}
\begin{enumerate}[label={\Alph*=}]
\item $\dfrac{6}{9} + \dfrac{1}{9}=\dfrac{6 + 1}{9}=\dfrac{7}{9} = \dfrac{7}{9}$
\item $\dfrac{3}{2} + 4=\dfrac{3}{2} + \dfrac{4}{1}=\dfrac{3}{2} + \dfrac{4 \times 2}{1 \times 2}=\dfrac{3}{2} + \dfrac{8}{2}=\dfrac{3 + 8}{2}=\dfrac{11}{2} = \dfrac{11}{2}$
\item $\dfrac{10}{8} + \dfrac{5}{80}=\dfrac{10 \times 10}{8 \times 10} + \dfrac{5}{80}=\dfrac{100}{80} + \dfrac{5}{80}=\dfrac{100 + 5}{80}=\dfrac{105}{80} = \dfrac{21}{16}$
\item $\dfrac{8}{5} + \dfrac{3}{10}=\dfrac{8 \times 2}{5 \times 2} + \dfrac{3}{10}=\dfrac{16}{10} + \dfrac{3}{10}=\dfrac{16 + 3}{10}=\dfrac{19}{10} = \dfrac{19}{10}$
\item $\dfrac{7}{6} + \dfrac{5}{6}=\dfrac{7 + 5}{6}=\dfrac{12}{6} = 2$
\item $\dfrac{10}{3} + 8=\dfrac{10}{3} + \dfrac{8}{1}=\dfrac{10}{3} + \dfrac{8 \times 3}{1 \times 3}=\dfrac{10}{3} + \dfrac{24}{3}=\dfrac{10 + 24}{3}=\dfrac{34}{3} = \dfrac{34}{3}$
\item $\dfrac{7}{4} + \dfrac{2}{28}=\dfrac{7 \times 7}{4 \times 7} + \dfrac{2}{28}=\dfrac{49}{28} + \dfrac{2}{28}=\dfrac{49 + 2}{28}=\dfrac{51}{28} = \dfrac{51}{28}$
\item $\dfrac{6}{8} + \dfrac{5}{3}=\dfrac{6 \times 3}{8 \times 3} + \dfrac{5 \times 8}{3 \times 8}=\dfrac{18}{24} + \dfrac{40}{24}=\dfrac{18 + 40}{24}=\dfrac{58}{24} = \dfrac{29}{12}$
\item $\dfrac{7}{2} \times 7=\dfrac{7 \times 7}{2}=\dfrac{49}{2} = \dfrac{49}{2}$
\item $\dfrac{5}{9} \times \dfrac{6}{9}=\dfrac{5 \times 6}{9 \times 9}=\dfrac{30}{81} = \dfrac{10}{27}$
\item $\dfrac{8}{3} \times \dfrac{- 9}{9}=\dfrac{8(- 9)}{3 \times 9}=\dfrac{- 72}{27} = \dfrac{- 8}{3}$
\item $\dfrac{\dfrac{2}{4}}{\dfrac{1}{6}}=\dfrac{2}{4} \times \dfrac{6}{1}=\dfrac{2 \times 6}{4 \times 1}=\dfrac{12}{4} = 3$
\item $\dfrac{8}{5} \times 10=\dfrac{8 \times 10}{5}=\dfrac{80}{5} = 16$
\item $\dfrac{5}{7} \times \dfrac{4}{7}=\dfrac{5 \times 4}{7 \times 7}=\dfrac{20}{49} = \dfrac{20}{49}$
\item $\dfrac{- 1}{9} \times \dfrac{10}{72}=\dfrac{- 1 \times 10}{9 \times 72}=\dfrac{- 10}{648} = \dfrac{- 5}{324}$
\item $\dfrac{\dfrac{7}{6}}{\dfrac{8}{2}}=\dfrac{7}{6} \times \dfrac{2}{8}=\dfrac{7 \times 2}{6 \times 8}=\dfrac{14}{48} = \dfrac{7}{24}$
\end{enumerate}
\end{solution}
@ -52,10 +52,10 @@
Le radar a pris des photos pendant l'été:
\begin{itemize}
\item en juin, il y a eu 59 photos prises dont 30 ratées.
\item en juillet, il y a eu 49 photos réussies et 31 ratées.
\item en août, il y a eu 59 photos dont une proportion de 0.24 de photos ratées.
\item en septembre, il y a eu 13 photos ratées, ce qui correspondait à 20.63\% des photos prises.
\item en juin, il y a eu 54 photos prises dont 27 ratées.
\item en juillet, il y a eu 43 photos réussies et 34 ratées.
\item en août, il y a eu 64 photos dont une proportion de 0.31 de photos ratées.
\item en septembre, il y a eu 5 photos ratées, ce qui correspondait à 11.11\% des photos prises.
\end{itemize}
\begin{enumerate}
@ -87,37 +87,35 @@
\hline
& Juin & Juillet & Août & Septembre & Total\\
\hline
Réussies & 29 & 49 & 45 & 50 & 173\\
Réussies & 27 & 43 & 44 & 40 & 154\\
\hline
Ratées & 30 & 31 & 14 & 13 & 88\\
Ratées & 27 & 34 & 20 & 5 & 86\\
\hline
Total & 59 & 80 & 59 & 63 & 261\\
Total & 54 & 77 & 64 & 45 & 240\\
\hline
\end{tabular}
\end{center}
\item Proportion de photos réussies
\[
\frac{173}{261} = 0.66 = 66\%
\frac{154}{240} = 0.64 = 64\%
\]
\item
\begin{itemize}
\item De juin à juillet
\[
\frac{31 - 30}{30} = \frac{1}{30} = 0.03 = 3\%
\frac{34 - 27}{27} = \frac{7}{27} = 0.26 = 25\%
\]
\item De juillet à août
\[
\frac{14 - 31}{31} = \frac{-17}{31} = -0.55 = -54\%
\frac{20 - 34}{34} = \frac{-14}{34} = -0.41 = -41\%
\]
\item De août à septembre
\[
\frac{13 - 14}{14} = \frac{-1}{14} = -0.07 = -7\%
\frac{5 - 20}{20} = \frac{-15}{20} = -0.75 = -75\%
\]
\end{itemize}
\end{enumerate}
\end{solution}
\printsolutionstype{exercise}
\end{document}

View File

@ -17,17 +17,17 @@
Détailler les calculs suivants et donner le résultat sous la forme d'une fraction irréductible.
\begin{multicols}{3}
\begin{enumerate}[label={\Alph*=}]
\item $\dfrac{9}{7} + \dfrac{2}{7}$
\item $\dfrac{3}{9} + 5$
\item $\dfrac{2}{8} + \dfrac{1}{8}$
\item $\dfrac{3}{5} + 10$
\item $\dfrac{5}{2} + \dfrac{4}{12}$
\item $\dfrac{8}{6} + \dfrac{9}{7}$
\item $\dfrac{2}{4} + \dfrac{8}{24}$
\item $\dfrac{2}{9} + \dfrac{10}{7}$
\item $\dfrac{6}{10} \times 3$
\item $\dfrac{4}{6} \times \dfrac{9}{6}$
\item $\dfrac{4}{6} \times 4$
\item $\dfrac{2}{3} \times \dfrac{10}{3}$
\item $\dfrac{3}{10} \times \dfrac{- 9}{90}$
\item $\dfrac{\dfrac{9}{5}}{\dfrac{10}{3}}$
\item $\dfrac{- 9}{7} \times \dfrac{- 2}{14}$
\item $\dfrac{\dfrac{4}{6}}{\dfrac{3}{6}}$
\end{enumerate}
\end{multicols}
\end{exercise}
@ -35,15 +35,15 @@
\begin{solution}
\begin{enumerate}[label={\Alph*=}]
\item $\dfrac{9}{7} + \dfrac{2}{7}=\dfrac{9 + 2}{7}=\dfrac{11}{7} = \dfrac{11}{7}$
\item $\dfrac{3}{9} + 5=\dfrac{3}{9} + \dfrac{5}{1}=\dfrac{3}{9} + \dfrac{5 \times 9}{1 \times 9}=\dfrac{3}{9} + \dfrac{45}{9}=\dfrac{3 + 45}{9}=\dfrac{48}{9} = \dfrac{16}{3}$
\item $\dfrac{5}{2} + \dfrac{4}{12}=\dfrac{5 \times 6}{2 \times 6} + \dfrac{4}{12}=\dfrac{30}{12} + \dfrac{4}{12}=\dfrac{30 + 4}{12}=\dfrac{34}{12} = \dfrac{17}{6}$
\item $\dfrac{8}{6} + \dfrac{9}{7}=\dfrac{8 \times 7}{6 \times 7} + \dfrac{9 \times 6}{7 \times 6}=\dfrac{56}{42} + \dfrac{54}{42}=\dfrac{56 + 54}{42}=\dfrac{110}{42} = \dfrac{55}{21}$
\item $\dfrac{2}{8} + \dfrac{1}{8}=\dfrac{2 + 1}{8}=\dfrac{3}{8} = \dfrac{3}{8}$
\item $\dfrac{3}{5} + 10=\dfrac{3}{5} + \dfrac{10}{1}=\dfrac{3}{5} + \dfrac{10 \times 5}{1 \times 5}=\dfrac{3}{5} + \dfrac{50}{5}=\dfrac{3 + 50}{5}=\dfrac{53}{5} = \dfrac{53}{5}$
\item $\dfrac{2}{4} + \dfrac{8}{24}=\dfrac{2 \times 6}{4 \times 6} + \dfrac{8}{24}=\dfrac{12}{24} + \dfrac{8}{24}=\dfrac{12 + 8}{24}=\dfrac{20}{24} = \dfrac{5}{6}$
\item $\dfrac{2}{9} + \dfrac{10}{7}=\dfrac{2 \times 7}{9 \times 7} + \dfrac{10 \times 9}{7 \times 9}=\dfrac{14}{63} + \dfrac{90}{63}=\dfrac{14 + 90}{63}=\dfrac{104}{63} = \dfrac{104}{63}$
\item $\dfrac{6}{10} \times 3=\dfrac{6 \times 3}{10}=\dfrac{18}{10} = \dfrac{9}{5}$
\item $\dfrac{4}{6} \times \dfrac{9}{6}=\dfrac{4 \times 9}{6 \times 6}=\dfrac{36}{36} = 1$
\item $\dfrac{3}{10} \times \dfrac{- 9}{90}=\dfrac{3(- 9)}{10 \times 90}=\dfrac{- 27}{900} = \dfrac{- 3}{100}$
\item $\dfrac{\dfrac{9}{5}}{\dfrac{10}{3}}=\dfrac{9}{5} \times \dfrac{3}{10}=\dfrac{9 \times 3}{5 \times 10}=\dfrac{27}{50} = \dfrac{27}{50}$
\item $\dfrac{4}{6} \times 4=\dfrac{4 \times 4}{6}=\dfrac{16}{6} = \dfrac{8}{3}$
\item $\dfrac{2}{3} \times \dfrac{10}{3}=\dfrac{2 \times 10}{3 \times 3}=\dfrac{20}{9} = \dfrac{20}{9}$
\item $\dfrac{- 9}{7} \times \dfrac{- 2}{14}=\dfrac{- 9(- 2)}{7 \times 14}=\dfrac{18}{98} = \dfrac{9}{49}$
\item $\dfrac{\dfrac{4}{6}}{\dfrac{3}{6}}=\dfrac{4}{6} \times \dfrac{6}{3}=\dfrac{4 \times 6}{6 \times 3}=\dfrac{24}{18} = \dfrac{4}{3}$
\end{enumerate}
\end{solution}
@ -52,10 +52,10 @@
Le radar a pris des photos pendant l'été:
\begin{itemize}
\item en juin, il y a eu 54 photos prises dont 30 ratées.
\item en juillet, il y a eu 44 photos réussies et 48 ratées.
\item en août, il y a eu 53 photos dont une proportion de 0.23 de photos ratées.
\item en septembre, il y a eu 10 photos ratées, ce qui correspondait à 17.86\% des photos prises.
\item en juin, il y a eu 52 photos prises dont 26 ratées.
\item en juillet, il y a eu 47 photos réussies et 42 ratées.
\item en août, il y a eu 61 photos dont une proportion de 0.31 de photos ratées.
\item en septembre, il y a eu 11 photos ratées, ce qui correspondait à 21.57\% des photos prises.
\end{itemize}
\begin{enumerate}
@ -87,37 +87,35 @@
\hline
& Juin & Juillet & Août & Septembre & Total\\
\hline
Réussies & 24 & 44 & 41 & 46 & 155\\
Réussies & 26 & 47 & 42 & 40 & 155\\
\hline
Ratées & 30 & 48 & 12 & 10 & 100\\
Ratées & 26 & 42 & 19 & 11 & 98\\
\hline
Total & 54 & 92 & 53 & 56 & 255\\
Total & 52 & 89 & 61 & 51 & 253\\
\hline
\end{tabular}
\end{center}
\item Proportion de photos réussies
\[
\frac{155}{255} = 0.61 = 60\%
\frac{155}{253} = 0.61 = 61\%
\]
\item
\begin{itemize}
\item De juin à juillet
\[
\frac{48 - 30}{30} = \frac{18}{30} = 0.6 = 60\%
\frac{42 - 26}{26} = \frac{16}{26} = 0.62 = 61\%
\]
\item De juillet à août
\[
\frac{12 - 48}{48} = \frac{-36}{48} = -0.75 = -75\%
\frac{19 - 42}{42} = \frac{-23}{42} = -0.55 = -54\%
\]
\item De août à septembre
\[
\frac{10 - 12}{12} = \frac{-2}{12} = -0.17 = -16\%
\frac{11 - 19}{19} = \frac{-8}{19} = -0.42 = -42\%
\]
\end{itemize}
\end{enumerate}
\end{solution}
\printsolutionstype{exercise}
\end{document}

View File

@ -144,6 +144,4 @@
\end{solution}
\printsolutionstype{exercise}
\end{document}